Finance
Finance
Kenneth J Boudreaux
The courses are updated on a regular basis to take account of errors, omissions and recent
developments. If you'd like to suggest a change to this course, please contact
us: [email protected].
Finance
Kenneth J Boudreaux is Professor of Economics and Finance at the AB Freeman School of Business,
Tulane University, New Orleans, US.
Professor Boudreaux is an eminent scholar in finance, known widely for his ability to combine cutting-
edge knowledge of the field with understandable explanations to professionals. In addition to being a
successful researcher and university professor, he has for the past two decades lectured extensively to
executives on topics in finance, in all parts of the world. Professor Boudreaux is the co-author of The
Basic Theory of Corporate Finance, a widely used graduate-level text, and has published significant scholarly
research on issues in corporate finance, securities markets and corporate restructuring. His research is
frequently cited in journals of finance and economics world wide.
Professor Boudreaux is an active consultant to the business world, and regularly performs analyses
involving financial issues for firms across industries that include shipping, petroleum exploration and
production, airlines, consumer products and computers. Included on this list are: Atlantic Container
Lines, British Petroleum, Central Gulf Lines, Exxon, Hewlett-Packard, Petroleum Helicopters and Reckitt
& Colman.
Notwithstanding Professor Boudreaux’s wide experience, all organisations referred to in the worked
examples are for illustrative purposes only and are entirely fictitious.
First Published in Great Britain in 1996.
© Kenneth J Boudreaux 2002, 2003
The rights of Kenneth J Boudreaux to be identified as Author of this Work has been asserted in accord-
ance with the Copyright, Designs and Patents Act 1988.
All rights reserved; students may print and download these materials for their own private study only and
not for commercial use. Except for this permitted use, no materials may be used, copied, shared, lent,
hired or resold in any way, without the prior consent of Edinburgh Business School.
Contents
Index I/1
Learning Objectives
This module introduces the student to Finance as a subject area. It describes the
participants in financial markets, the decisions they must make, and the basic process-
es that are common to all such financial decisions. The module discusses the roles of
borrowers, lenders, equity, security issuers and purchasers, and the sources of value
for each. Because finance is inherently a quantitative and economic subject, this
introductory module devotes much effort to instructing the student in the essential
quantitative techniques of financial valuation, including discounting, present valuation,
determination of rates of return, and some important financial economics relating to
interest rates and security valuation. The module introduces some specialised financial
concepts such as ‘yield to maturity’ and the ‘term structure’ of interest rates. It
includes the first of several perspectives on the important company decision tools ‘net
present value’ and ‘internal rate of return’. The module finishes with an illustration of
the usefulness of even these basic financial techniques in understanding a market that
remains mysterious to many financial practitioners: forward and futures markets for
interest rates. In this module the student will learn the essence of the financial
environment, along with the basic quantitative tools of financial valuation that are
used throughout the course.
1.1 Introduction
In this first module of the finance course you will study the basic concepts and
techniques of analysis in finance. We shall investigate ideas of market value, of
investment decision-making, of interest rates, and of various kinds of financial
markets. It is always good to have some general appreciation of a subject before its
details are studied, and this is particularly true of finance, which is a very large and
complex field of study. This module will supply you with that essential understanding,
providing you with information on a number of fundamental concepts that can be
applied again and again to the solution of real financial problems.
Finance is the economics of allocating resources across time. This definition,
of course, is not particularly informative, but an example of a financial transaction
that is governed by it may help you to understand the definition. Thus, suppose a
new audio device, the digital audio tape player, has just come onto the market. Being
an audiophile, you must have one. Economic logic says that if you had the resources
to purchase it you would, because your satisfaction would increase by exchanging
cash for the digital device. But suppose that you had neither the cash nor other
assets that could be readily sold for enough cash to purchase the player. Would you
be able to buy it?
The answer is that you may or may not be able to, depending upon whether you
can convince someone to lend you the money. Whether you can is a function both
of the tangible assets you have and the expectation of developing more assets in the
future from which your creditor can expect to be paid. Because you do not have the
tangible financial resources now to buy what you want, by borrowing you can
shift some of your future resources back to the present so as to enable you to
buy what you desire. You are buying the player with resources that you do not yet
have in hand but are expected to get sometime in the future. And from the perspec-
tive of the lender, an exactly opposite transaction will take place: the lender gives up
some present resources in exchange for those that you are expected to provide in
the future as you pay off the loan. This shifting around, or reallocation of re-
sources in time, is the essence of finance.
This example is useful because it can also help us to understand why finance is an
important subject. Think how many transactions have this essence of shifting
resources around in time. We must include not only the borrowing and lending of
money by individuals, but by governments, corporations and other institutions. And
the borrowing and lending of money are not the only ways in which resources are
reallocated in time. When a company issues equity capital (in other words, raises
money from its owners) it is undertaking a financial transaction similar to your
borrowing to get the tape player; that is, it is accepting money now and giving in
exchange a promise to return money in the future (in the form of corporate
dividends). The owners of the company are engaging in a financial transaction with
the company that is very similar to the one that occurs between you and the lender
in the financing of your tape player.
Think how many purchases and sales of tangible assets are made possible by the
ability to shift resources across time. All personal credit purchases, much of
corporate asset acquisition, and a great deal of a government’s providing of assets
and services would not be possible without underlying financial transactions.
Understanding the finance characteristic of these activities is an important part of
being educated in business.
The above array of transactions that have important financial dimensions is im-
pressive in its breadth, but it can be intimidating in the complexity it implies for the
study of finance. We shall nowhere deny that finance is a large and complex field,
but its study need not be terribly intricate, at least at the outset. In this course our
approach will be first to create a very simple model of a financial market wherein
participants (the individuals, companies and governments in the market) can engage
in rudimentary financial transactions. That model is useful in acquainting ourselves
with the basic ideas common to all financial transactions. Once these have been
developed, we shall gradually include more and more realism in the model, until we
can deal with the characteristics of financial markets and transactions that we see in
actual practice.
CF1
F
£2640
B
£1870
E
£1540
A
£1320
P
£700
£1000
£1200
£2400
0 CF0
example, if all cash flows were transferred to t1, there would be £[1540 + (1000 ×
1.10)], or £2640 for t1, and nothing for t0 (point F).1
On the other hand, if all cash flows were shifted to t0, the participant would have
£1000 plus whatever he could borrow at t0 with a promise to repay £1540 at t1.
How much is this? For each £1 we borrow at t0, we must repay £1 × (1 + i) at t1, so
we can borrow (using CFt to mean ‘cash flow at time t’) where
1
1
Thus in our example:
£1540
1.10
£1400
The participant could borrow £1400 at t0 with the promise to pay £1540 at t1, the
£1540 comprising £1400 of principal and £140 of interest. The maximum amount
the participant could consume at t0 is thus £2400, consisting of the original £1000
cash flow, plus the £1400 which can be borrowed at t0 with the promise to repay
£1540 at t1. This is point P in Figure 1.1, £2400 at t0 and £0 at t1.
Believe it or not we have just done a calculation and arrived at a result that is of
the greatest importance and underlies many of the ideas of finance. Discovering that
£1540 at t1 is worth £1400 at t0 is called finding the present value of the £1540.
Present value is defined as the amount of money you must invest or lend at
the present time so as to end up with a particular amount of money in the
future. Here, you would necessarily invest £1400 at t0 at 10 per cent interest to end
up with £1540 at t1, so £1400 is the present (t0) value of £1540 at t1. The person or
institution that was willing to lend the participant £1400 must have done just that
type of calculation.
Finding the present value of a future cash flow is often called discounting the
cash flow. In the above example, £1400 is the ‘discounted value of the £1540’, or
the ‘present value of the t1 £1540, discounted at 10 per cent per period’.
From the calculations above you can see the type of information that the present
value gives us about the future cash flow it represents. For example, should the
participant’s expectation of receiving t1 cash flow increase to greater than £1540, he
could borrow more than £1400 at t0 (and vice versa for a smaller t1 expectation). Or if
the t1 cash flow expectation becomes risky, the lender will require a return higher than
10 per cent to compensate for the risk being borne. (The risk here is that when t1
arrives, the full amount of the £1540 expectation does not appear.) If the interest rate
1 The market interest rate is thus really an ‘exchange rate’ between present and future resources. It tells
us the price of t1 pounds in terms of t0 pounds. The exchange line in Figure 1.1 and the market
interest rate are thus giving us the same basic information. It should not surprise you to hear that the
steepness or slope of the exchange line (the ratio of exchanging t0 for t1 pounds) is determined by the
market interest rate. The higher is the market interest rate, the more steep is the slope of the exchange
line. In plain words, this says imply that the higher is the interest rate, the more pounds you must
promise at t1 in order to borrow a pound at t0 – which you doubtless knew already!
increases, you can see that the present value, and thus the amount that the participant
can borrow against it, declines. So the present value of a future cash flow is the
amount that a willing and informed lender would agree to lend, getting in return a
claim upon the future cash amount. The amount of the present value will depend
upon the expected size and risk of the cash flow, and when it is expected to occur.
How much you can borrow by promising to pay an expected future amount is an
important interpretation of present value, but by no means is it the only, or even the
most important, interpretation. Present value is also an accurate representation
of what the financial market does when it sets a price on a financial asset. For
example, suppose that our participant does not wish to borrow money, but instead
wishes to sell outright the expectation of receiving cash at t1. The participant can do
this by issuing a security that endows its owner with the legal right to claim the t1
cash flow. This security could be a simple piece of paper with the agreement written
upon it, or could be a very formal contract of the type issued by companies when
they borrow money or issue shares.
For how much do you think the participant might sell such a security? Everyone
thinking of buying will of course examine alternatives to buying this security.
(Economists call such alternatives opportunity costs because they represent the
‘costs’ of doing this instead of something else, in the sense of an opportunity
forgone.) They will discover that for each t0 £1 used to buy the security, £1.10 at t1
will be returned by other same-risk investments in the financial market (for example,
lending at 10 per cent). That being the case, the participant will be able to sell the
security for no more than £1400 (the present value of £1540 at t1 discounted at 10
per cent), because potential buyers need only lend £1400 to the financial market at
t0 in order to get £1540 at t1, exactly what the security promises. And because of the
competitive nature of financial markets, the security will not sell for less than £1400,
because if it did it would provide a cash return the same as other alternatives, but at
a lower present price. As potential buyers of the security begin bidding against each
other, the security’s price must increase or decline to the point where its expected
future cash flow costs the same as that future cash flow acquired by any other
means.
Present value is thus the market value of a security when market interest rates or
opportunity rates of return are used as discount rates. This is perhaps the most
important application of the notion of present value.
This brings us to another important application of the present-value idea. We
have seen that the present value of all of our participant’s present and future
resources (cash flows) is £2400. This amount also has a special name in finance: it is
known as present wealth. Present wealth is a useful concept in that it tells us the
total value of a participant’s entire time-specified resources with a single number. It
is even more important because it can be used as a benchmark or standard to judge
whether someone is going to be better or worse off because of a proposed financial
decision. But we shall need to introduce a few more ideas before we can develop
that point as completely as it deserves.
One thing we can now see readily from Figure 1.1 is that one cannot change
present wealth merely by transacting (borrowing and lending at the market rate) in
the financial market. Though borrowing and lending will move us up and down the
financial exchange line, and thereby allow us to choose the time allocation of our
present wealth that makes us most happy, such transactions cannot move the line
and therefore cannot change our wealth. The reason is easy enough to deduce. In
financial markets, by buying and selling securities (or borrowing and lending), the
total amount of wealth that is associated with those securities is unchanged. So if
one wishes to increase one’s wealth by buying and selling in those markets, one
would be forced to find another participant who, doubtless inadvertently, would
allow his wealth to be reduced. As we shall see soon, the odds of doing so are low.
1.3.2 Investing
If we cannot expect to change our wealth by transacting in financial markets, how
can we get richer? The answer is by investing in real assets. This type of financial
activity can change our present wealth because it is not necessary to find someone
else to give us part of their wealth in order for ours to increase. Investing in real
assets such as productive machinery, new production facilities, research, or a new
product line to be marketed, because it creates new future cash flows that did not
previously exist, can generate new wealth that was not there before.
Of course not all real-asset investments are wealth-increasing. Investments are
not free; we must give up some resources in order to undertake an investment. If
the present value of the amounts we give up is greater than the present value of
what we gain from the investment, the investment will decrease our present wealth.
Because that will allow us to consume less across time, it is a bad investment. Of
course a good investment would produce more wealth than it uses, and would
therefore be desirable.
Figure 1.2 shows how real-asset investments work in our simple financial market.
Suppose that our participant discovers an opportunity to invest £550 at t0 in a real
asset that is expected to return £770 at t1. In Figure 1.2 this appears as a move from
point E to point I. That investment would result in t1 resources of £2310 and t0
resources of £450. Should this opportunity be accepted or not? The answer is that it
depends upon the effect on the participant’s present wealth.
CF1
F'
£2640 F
I
£2310
£1540 E
P P'
0 £450 £1000 £2400 £2550 CF0
We can calculate the amount of this parallel shift by seeing how far the line’s
intercept has moved along the horizontal axis. As before, this means taking the
present value of any position on the new line. Since we know point I already, we can
use it:
1
£2310
£450
1.10
£2550
The outward shift in the exchange line is to £2550 at t0. But this is also the hori-
zontal intercept of the new exchange line, which (from what we know about the
discounted value of future resources) is our participant’s new present wealth. So we
have also discovered that his or her present wealth will increase from its original
level of £2400 to £2550 with the investment.
Remember that we are trying to connect the investment’s desirability to the par-
ticipant’s change in present wealth. The last step in that process is easy: since any
outward shift in the exchange line signals a good investment, and since any outward
shift is an increase in present wealth, any investment that increases present wealth is
a good one. That is simply another way of saying what we said earlier: investments
are desirable when they produce more present value than they cost.
the IRR has been calculated, we compare it to the rate of return that could be
earned on a comparable financial market opportunity of equal timing and equal risk.
If the investment earns a higher return than this opportunity cost, it is good and we
accept it; if it earns a lower rate of return, we reject it.
A more specific definition of the IRR is that it is the discount rate that equates
the present values of an investment’s cash inflows and outflows. From our
earlier discussion of NPV, this implies that IRR is the discount rate that causes
an investment’s NPV to be zero. We shall see shortly why the IRR can be defined
this way, but aside from simply broadening our education in finance these defini-
tions are useful in that they give us hints as to how we can calculate the IRR. In our
one-period financial market, calculating the IRR is easy. Returning to the original
example, and using the definitions immediately above, we have
£770
NPV 0 £550
1 IRR
£770
1 IRR
£550
1 IRR 1.4
IRR 0.4 40%
The internal rate of return of our participant’s investment is 40 per cent. Since the
opportunity cost as a rate of return is 10 per cent (from an investment of compara-
ble risk and timing in the financial market), the investment has a higher average per-
period earning rate than the best alternative, so it is acceptable.
By looking again at Figure 1.2 we can gain a valuable intuition about the things
that IRR and NPV are telling us. Remembering that the slope of an exchange line
on that graph reflects the interest or discount rate, we can interpret the line from
point E to point I as an ‘exchange line for this investment’ (i.e. giving up £550 at t0
for £770 at t1). Notice that the slope of the exchange line for the investment is
steeper than the exchange line for the financial market. This implies clearly that the
rate of return or earning rate on the investment is higher than the financial market’s.
Notice also that if the investment’s exchange line is steeper than the financial
market’s, the resulting resource location of the investment (point I) must lie outside
the original market exchange line. This means, as we saw in our discussion of NPV,
that our participant’s wealth would increase were he or she to accept the investment.
These observations about IRR in Figure 1.2 also imply that when IRR is greater
than the financial market rate, NPV is positive. So the two techniques are telling us
very similar things about the investment, but with slightly different perspectives.
NPV describes an investment by the amount of the wealth increase that would be
experienced by the participant who accepts it, whereas IRR tells us how the average
earning rate on the investment compares with the opportunity rate.2
2 We usually assume in finance that an investment at the financial market rate, as the opportunity cost of
the investment, is ‘the best alternative’ for any particular investment even though that might not be
strictly true. As we shall see eventually, if the investment decision is handled correctly and thoroughly,
we get the same answer as we would by actually using the true ‘best alternative’.
The IRR and NPV techniques usually give the same answers to the question of
whether or not an investment is acceptable. But they often give different answers to
the question of which of two acceptable investments is the better. This is one of the
major problems in finance, not so much because we do not know which one is
correct but because many people seem to like the technique that gives the wrong
answer! Obviously this deserves discussion, but we shall postpone that until we
make the financial market a more realistic place, so the reasons for the disagree-
ments between IRR and NPV can be explored more fully.
To review what we have discovered about the IRR technique:
1. IRR is the average per-period rate of return on the money invested in an
opportunity.
2. IRR is best calculated by finding the discount rate that would cause the NPV of
the investment to be zero.
3. To use IRR, we compare it with the return available on an equal-risk investment
of comparable cash-flow timing. If the IRR is greater than its opportunity cost,
the investment is good, and we accept it; if it is not, we reject the investment.
4. IRR and NPV usually give us the same answer as to whether an investment is
acceptable, but often different answers as to which of two investments is better.
As a review of your understanding of some of the points we have made so far,
look at the investment N in Figure 1.2. It requires an outlay of £550 at t0 and
returns £594 at t1. N’s NPV is given as follows:
£594
NPV £550
1.10
£10
Similarly, N has an IRR given by:
£594
0 £550
1 IRR
£594
1 IRR
£550
1.08
IRR 0.08 8%
The two techniques, NPV and IRR, give the same answer about N: it is not a
good investment. The NPV is −£10, which implies that a participant in this market
would lose £10 of present wealth if investment N was accepted. In Figure 1.2 the
resulting exchange line would shift back toward the origin and intercept the hori-
zontal axis at £2390 rather than £2400, making the participant less well off than
without the investment. N’s IRR is 8 per cent, which is a lower per-period earning
rate than the 10 per cent generally available in the financial market for investments
of equal risk and timing. Note that the exchange line for investment N in Figure 1.2
has a slope less steep than the market’s exchange line, FEP. This is a visual state-
ment that N’s earning rate is less than that of the market; so, again, N should be
rejected.
CF1
K"
L'
K'
J'
H'
G'
P'
0 K J H G P CF0
the investments up to and including the one tangential to the new financial exchange
line is the same thing as accepting investments until the IRR of the last one is just
equal to (or above) the market interest rate. This is the process that will create the
most wealth for shareholders, because it causes the company to accept all investments
with average per-period earnings rates (IRRs) greater than what the company’s
shareholders can earn on comparable investments in the financial market. (To accept
investments until the next has a negative or zero NPV is, of course, to do the same
thing.)
‘Not so fast,’ you say. ‘Suppose I were the type of person who preferred t0 to t1
consumption. If I were a shareholder of the company I would be happier if they
stopped at investment H or G or even made none at all. Then I would have the
highest capacity to consume at t0.’
That, of course, is not true. If the company undertakes no investment, your max-
imum t0 consumption is P of CF0. Whereas if the company accepts all of the
investments up to and including J, you can consume up to P' of CF0 simply by
selling your shares at t0 after the market discovers the astuteness of the company’s
investment decisions and adjusts the price of its shares. If you have an aversion to
selling, nothing would prevent you from borrowing against those shares at t0 in our
frictionless market and getting the same amount P' by that mechanism instead.
‘Fair enough,’ you say. ‘But my sister is also a shareholder of the same company,
and her consumption preferences are exactly the reverse of mine. She likes nothing
better than to increase her future consumption by reducing her current spending.
How is the company going to solve the problem of pleasing both of us?’
The answer is that in a market such as this one, companies face no such problem
because shareholders can easily solve it themselves. Your sister would simply avoid
selling shares, and reinvest any dividends that the company paid her, either in more
shares or in lending. The result would be that she delays present consumption until
the future. In essence we are saying that a company in this market need not worry
about its shareholders’ consumption preferences; the financial market will allow
them to make whatever transactions are necessary to be content with their time
pattern of resources. Shareholders with quite different preferences for patterns of
consumption can thus be content to own the same company’s shares, and the
company need not be concerned about the pattern it chooses in which to pay
dividends. The sole task of the company is to maximise the present wealth of its
shareholders. The shareholders can then adjust their individual patterns of re-
sources by dealing in the financial market.
Suppose that the company mistakenly undertook to please your sister by invest-
ing a greater amount at t0, and accepted all investments up to K′. From Figure 1.3
you can readily see that her t0 cash flow would decrease and her t1 increase, which is
her preferred pattern. But notice also that were the company to maximise her
present wealth by investing only to J′, she could in fact retain the same t0 consump-
tion (0K) and increase her t1 consumption to KK′′, thereby increasing her
satisfaction.
To review the important ideas we have discussed in this section:
1. We distinguished between financial and real asset investments, and argued that
because of the competitiveness of financial markets it is (usually) necessary to
choose real investments in order to expect wealth to increase by investing.
2. We developed the measure of investment desirability called net present value, as
the present value of the amounts by which an investment’s cash flows exceed
those of its opportunity cost. We also showed that NPV is equal to the change in
the wealth of the participant accepting the investment, and that NPV measures
the change in market value of the investor’s wealth.
3. We introduced the measure of investment desirability called internal rate of
return, the average per-period earning rate of the money invested. When IRR
exceeds the opportunity cost (as a rate) of an investment, the investment will
have a positive NPV, and therefore be acceptable.
4. We illustrated how these ideas could apply to the investment decisions of a
company in a simple financial market like the one described. The company
would accept investments up to the point where the next investment would have
a negative NPV or an IRR less than its opportunity cost. The company can ig-
nore its shareholders’ preferences for particular patterns of cash flow across time
because the financial market allows shareholders to reallocate those resources by
borrowing and lending as they see fit. This lets the company concentrate on
maximising the present wealth of its shareholders, the result of adhering to the
investment evaluation techniques of NPV and IRR in this market.
All of these ideas are important introductions to finance for people who will be
dealing with these decisions. As important is the general appreciation that we have
gained for what a financial market does:
1. It lets people reallocate resources across time, which provides money for real
investment.
2. It gives very important signals, in terms of market rates of return or interest
rates, about the opportunity costs faced by investors. These rates are used as
discount rates for making the real asset investment decisions that are so im-
portant to an economy.
returns spanning more than a single period: you can leave your money in a bank for
more than one interest period before taking it out; you can buy bonds that pay
interest for decades before they stop (or ‘mature’); and you can invest in corporate
equities (ordinary shares) that are expected to continue paying dividends for an
indeterminately long period into the future. (Some have been paying for a hundred
years or so already.) We must be able to address the questions of how such securi-
ties are valued, and how financial decision makers deal with real asset choices when
the returns from those real assets cover many periods.
With multiple-period assets generating returns across long periods of time, it
must seem at first that real financial markets are terribly complex things with which
to contend. And we would be telling less than the absolute truth if we said there are
no complications introduced by multiple-period assets in the financial market. But it
is true that these complexities introduce few new general concepts and are mostly
involved in the calculations that are necessary to describe and value the returns that
the assets produce.
Actually, there is one way of looking at multiple-period transactions in the finan-
cial market that is almost identical to the way we described the single-period market.
When we shifted resources across time in the single-period market, we multiplied by
1 i to move a period into the future (accruing interest), and divided by 1 i to
move one period into the past (discounting). The 1 i is effectively an ‘exchange
rate’ between t0 and t1 resources. In multiple-period transactions the same type of
exchange rate applies to shifting resources between any two time points.
Picture the financial market now covering the time points t0, t1 and t2. This
means simply that we have introduced another period after t1, the point at which
our single-period market stopped:
Period 1 Period 2
The financial market will now allow us to shift resources not only between t0 and t1
but also between t0 and t2 (or any pair of time points). The rate of exchange
between t0 and t1 resources is 1 i , but since we can now have another exchange
rate between t0 and t2, we must be able to distinguish between that rate and the rate
between t0 and t1. To do so we shall designate i1 as the interest rate between t0 and
t1, and i2, as the rate between and t0 and t2. Thus 1 i1 is the single-period
exchange rate.
To be able to write the two-period exchange rate, we must now deal with one of
the complexities of multi-period markets. Instead of writing the exchange rate
between t0 and t2 as 1 i2 , we usually write it as 1 i2 2. This may seem unnec-
essarily complicated, but it does serve a purpose: people are evidently more
comfortable in talking about interest rates per period than exchange rates over
more than one period, and this way of writing the exchange rate allows them to do
that.
An example might be useful here. Suppose that the same per-period exchange
rate existed between t0 and t2 as between t0 and t1, and that this rate per period was
our familiar 10 per cent. To shift resources either backward or forward between t0
and t1, the exchange rate 1.10 applies. But to shift resources between t0 and t2, we
must travel through two periods at the rate 1.10 per period.
Suppose that we wished to invest £100 in the financial market at t0, and leave it
there until t2, so as to then have an amount CF2. How much would CF2 be?
1 1
0 1
£100 1.21
£121
We would end up with £121 at t2. That is the result of earning 10 per cent per
period for two periods on an initial £100 investment. In finance, when we say that
the two-period interest rate is 10 per cent, we mean that to shift resources between
t0 and t2 the exchange rate is 1 10% 2, or 1.21.
Naturally, the present-value calculation works in exactly the opposite way. If we
expected to receive £121 at t2, and wished to know its present value (its market
price right now) we would calculate:
1
£121
1.10
£100
twice, three or even more times per period. If the interest rate stays the same, the
money amounts would be different because of the number of times interest was
compounded between time points.
The general arithmetic of interest compounding is not very complicated. The
amount of money you end up with by investing CF0 at compound interest is:
CF0 1 i/m mt
where i is the interest rate, m is the number of times per period that compounding
takes place, and t is the number of periods the investment covers. You can see that
this formula becomes our familiar CF0 1 i t when interest is compounded only
once per period.
Using the formula, were we to compound £100 twice per period at 10 per cent
interest, we would have at the end of the first period:
£100 1 0.10/2 2 £110.25
and at the end of the second:
£100 1 0.10/2 4 £121.55
continuing for as many periods as we choose.
You can see that these amounts are higher at each future time point than those
we figured earlier when compounding only once per period at the 10 per cent rate.
If your calculator can raise numbers to powers, see if you can use the compounding
formula to demonstrate to yourself that £100 invested at 10 per cent interest for
fifty years increases to £11 739.09 if compounded once per year, and £14 831.16 if
compounded daily (365 times per year).
Compounding of interest can be even more frequent than daily. The most fre-
quent type of compounding is called ‘continuous’. Continuous compounding
means that interest is calculated and added to begin earning interest on itself
without any passage of time between compoundings. In the general com-
pounding formula above, that means m is infinitely large and, without belabouring
the algebra, the formula reduces to:
CF0 eit
where e = 2.718 …, the base of the natural logarithm system.
If interest is compounded continuously, with 10 per cent interest, £100 increases,
for example, to £110.52 in one period, to £122.14 in two periods, and to £14 841.32
in fifty periods.
Financial institutions that borrow by accepting deposits from customers occa-
sionally use interest compounding as a marketing tactic in an attempt to lure
customers seeking high interest earnings. The advertisements are usually variants on
the above examples and, with the exception of an occasional inadvertent arithmetic
error, are correct in their implication that more frequent compounding produces
higher final amounts. Customers should, however, exercise care in choosing among
financial assets on the basis of compounding intervals. In a very competitive market
for deposits it is unlikely that one bank can afford to offer consistently larger
payments to customers than other banks. If the stated interest rate is only very
slightly lower than one compounded less frequently, the difference may nevertheless
offset any compounding benefit. Or some non-monetary dimension of service may
be different.
We must always remember that financial market participants consume money
resources, not interest rates or compounding intervals; they make their comparisons
of desirability on the basis of money-measured values. They cannot be fooled into
thinking, for example, that continuous compounding is necessarily better than no
compounding, unless they are also informed of the interest rates to be compounded.
Nor will they assume that lending money at a rate of 10.1 per cent is necessarily
better than lending at 10 per cent, if the two rates are not identically compounded.
For the remainder of the course we shall adopt the common convention that,
unless told otherwise, interest is compounded once per period.
1 1 1
£100 £100 £100
1.10 1.10 1.10
£100 £100 £100
1.10 1.21 1.331
£90.91 £82.65 £75.13
£248.69
The present value of the stream of cash flows is £248.69, being the sum of the
present values of the future cash flows in the stream. Though the arithmetic of this
example is quite rudimentary, the economic lesson it portrays is important. It tells us
that the correct way to view the value of an asset that generates a stream of future
cash flows is as the sum of the present values of each of the future cash flows
associated with the asset.
In terms of the mathematics involved, there is no general formula that will allow
us to solve all such IRR equations. Instead, the way we solve for the IRR of a
multiple-period cash flow stream is with a technique called ‘trial and error’. This
means we choose some arbitrary discount rate for IRR in the above equation, and
calculate NPV. We then examine the result to decide whether the rate we used was
too high or too low, choose another rate that appears to be better than the one we
just used, and again calculate NPV. We continue this process until we find the IRR
(or as close an approximation to it as seems necessary) that creates an NPV equal to
zero.
Suppose that we first try 15 per cent as a potential IRR:
£100 £100 £100
NPV £200
1.15 1.15 1.15
£28.32
A 15 per cent discount rate produces an NPV greater than zero, so 15 per cent is
not the IRR of the cash flows. Since the NPV is too large, we should probably
choose a higher discount rate (since increases in the discount rate, being in the
denominator, will cause a lower NPV for these cash flows). Let us try 25 per cent:
£100 £100 £100
NPV £200
1.25 1.25 1.25
£4.80
A 25 per cent discount rate yields a small but negative NPV, and so 25 per cent is
too large. Nevertheless, we have discovered that the IRR of the cash flows is
somewhere between 15 and 25 per cent because the former yields a positive and the
latter a negative NPV. To find the true IRR, we continue this search process until
we find the exact IRR, become convinced that the range we have found is suffi-
ciently accurate for the decision at hand, or run out of patience.
The actual IRR for this example is 23.4 per cent per period, implying that, be-
cause IRR exceeds the 10 per cent opportunity cost, the investment is acceptable.
Actually, we could have decided that as soon as we saw a positive NPV at a 15 per
cent discount rate, since we knew thereby that the IRR was greater than 15 per cent.
With an opportunity cost of 10 per cent and an IRR greater than 15 per cent we
have enough information to decide that the investment is desirable.
Figure 1.4 should be helpful in visualising the method of finding the IRR. Note
that the vertical axis of the figure records NPV, and the horizontal axis plots the
discount rates used to calculate NPV. The curve indicates that, in the example we
are examining, as the discount rate increases, NPV declines. (This is a very common
relationship between NPV and its discount rates. As long as cash outflows tend to
be closer to the present than cash inflows from an investment, we usually see a
curve that looks like this one.) The search for an IRR is easy to visualise in Fig-
ure 1.4. If your first try uses a rate less than 23.4 per cent, the NPV will be positive;
if more, it will be negative. If you get a positive NPV, you should try a rate higher
than the one you have used; if you get a negative NPV, you should try a lower one.
Eventually you will narrow the search to a rate that creates an NPV nearly equal to
zero, and that rate will be the IRR.
The process for estimating an IRR can be troublesome because NPV must be
recalculated in each pass of the search process. This can become quite tedious when
an investment is expected to generate cash flows for many periods. Fortunately for
those who like using the IRR, the wonders of modern technology have come to the
rescue with commonly available pocket calculators that have this search process
programmed into them. If you are faced with the prospect of calculating IRRs for
long-lived investments, you should consider one of these instruments or appropriate
software for your computer.
NPV
£48.69
IRR
£0
10% 23.4% Discount rate
﴾1.1﴿
1
When we wish to discount a stream of future cash flows to find its present value, we
simply find the sum of the present values as calculated above. The way to write the
instruction to calculate the present value of a stream of future cash flows is
∑
﴾1.2﴿
1
Though this Equation 1.2 may appear forbidding, it is telling you simply to find
the present value of each future cash flow and add up the results, which is exactly
what we did in the last section. (The ∑ sign is a symbol that says to sum everything
to the right of itself, beginning at t1, until you exhaust the cash flows at tT.) Occa-
sionally we shall use this equation (or a near relative of it) in our discussions. For
our purposes it is entirely appropriate to regard such equations as an efficient kind
of shorthand for a set of instructions that is telling you simply to calculate the
present value of a set of future cash flows.
When faced with a present-value calculation that has different cash flows across
the future and different discount rates for these cash flows, we have no choice but
to use the technique implied by Equation 1.2. Though that does happen, there are
common situations where we can find correct present values more easily than by
using that technique. One of the most often encountered simplifications is where
discount rates are constant across the future. This is rarely an accurate reflection of
what is really expected to happen, but it reduces the complexity of the calculations
so much that it is widely used.
Where discount rates are taken to be the same for all cash flows, Equation 1.2
becomes
∑
﴾1.3﴿
1
Equation 1.3 is the instruction to discount a stream of future cash flows to the
present using the same per-period discount rate for all cash flows. (Note that the
difference between Equation 1.2 and Equation 1.3 is that the discount rate i is not
time-subscripted.)
There are at least two reasonably straightforward ways of following the instruction
of Equation 1.3. To illustrate these, we can use the numerical example of £100 per
period for three future periods at a 10 per cent discount rate. The first technique
begins with the cash flow furthest into the future (in this case CF3 £100), and
divides it by 1 i : £100/ 1.10 £90.91. That number is the t2 value of the t3 cash
flow. To this is added the next-nearest cash flow, CF2 £100, and that sum, £190.91,
is divided by 1 i : £190.91/ 1.10 £173.55. That step produces the t1 value of
the t2 and t3 cash flows. To that is added the t1 cash flow itself, and that sum, £273.55,
is again divided by 1 i : £273.55/ 1.10 £248.69. We have seen this last result
before. It is the present value of the stream or, as we have seen from the foregoing
explanation, it is the t0 value of the t1, t2 and t3 cash flows.
The above technique is clumsy to explain but actually works quite neatly, and it is
better than anything else if you have a basic pocket calculator available. Put simply,
you start with the cash flow furthest into the future, discount it one period closer to
the present, add the cash flow from that closer time point, and discount that sum
one period nearer to the present; you continue that process until all cash flows are
included, and discounted back to t0.
Another commonly used technique of finding present values relies on present
value tables. Present value tables are simply lists of actual values of Equation 1.1
and Equation 1.3, with 1 as the cash flow in the numerators of the equations, and
with present values calculated for a wide range of time points, stream lengths and
discount rates. Present value tables therefore give the present values per pound of
future cash flow, either for a single cash flow (Equation 1.1) or a stream of constant
cash flows discounted at constant discount rates (Equation 1.3). We have included a
set of these tables in Appendix 1.
To illustrate the use of the tables, turn to Table A1.1 in Appendix 1, which shows
the present value of £1 to be received at time point t. Note that in the column for a
10 per cent discount, the factors for the first three time points are indicated as
0.9091, 0.8264, and 0.7513 respectively. To find the present value of our £100 per
period for three periods, we multiply each of these factors by the £100 cash flow
occurring at its time point, and add up the result. The answer is, of course, £248.69
(actually the result is £0.01 less because of rounding).
Using present value tables for such calculations is unnecessary since the calcula-
tor technique above has the same number of steps and does not use the tables.
There are, however, instances where the tables are efficient. One obvious example is
in calculating the present value of a single cash flow located far into the future when
your calculator cannot perform exponentiation (i.e. raising numbers to powers). You
would not enjoy dividing £100 by (1.10) twenty times if your calculator could not
figure 1.10 20 directly. Table A1.1 tells you that £1/ 1.10 20 is 0.1486, and there-
fore that the value of £100 to be received at t20 is £14.86 at t0 with a discount rate
of 10 per cent.
Another circumstance when the tables are useful is in finding the present value of
annuities. A constant annuity is a set of cash flows that are the same amounts
across future time points. Such a present value is calculated with our Equation 1.3,
but with no subscript on the cash flows since they are all the same. Since Table A1.2
in Appendix 1 gives the present value of £1 per period for t periods, it implies a
constant cash flow per period and is therefore an annuity table. To illustrate its use,
note that under the column for 10 per cent discount, the three-period annuity factor
is listed as 2.4869. It takes little effort to see that with £100 per period for three
periods, we arrive at our familiar answer, £248.69. When annuities run for many
periods, the use of this table rather than a calculator is wise (unless you own one of
the sophisticated devices programmed to do such calculations directly).3
3 Cash flows can, of course, be invested at interest to produce future amounts of cash. Table A1.3 and
Table A1.4 in Appendix 1 are the future value counterparts of Table A1.1 and Table A1.2 in the same
Appendix. In addition to the tables and pocket calculators with built-in financial functions, all current
personal computer spreadsheet programs have these (and many more) financial formulas able to be
called up as automatic functions. You should not hesitate to use such spreadsheets instead of the tables
or calculators if you feel comfortable doing so. Because of the increasingly common availability of
We mentioned earlier that some of the assets commonly valued in finance have
cash-flow expectations that extend very far into the future. When faced with cash
flows of that nature, there is yet another technique of present value calculation that
is often used in financial practice: the perpetuity. A perpetuity is a cash flow stream
that is assumed to continue for ever. Perpetuity present values are used because they
are so easy to calculate. The formula for the present value of a perpetuity is:
﴾1.4﴿
To find the present value of a perpetuity one merely divides the (constant) per-
period cash flow by the (constant) per-period discount rate. This means, for
example, that £100 per period for ever at a discount rate of 10 per cent has a
present value of £100/0.10 = £1000. (It is intuitively easy to see why this formula
works. Another way of saying the same thing is that if you put £1000 in the bank at
10 per cent interest, you can take out £100 per year for ever.)
The ease of calculation of perpetuities is attractive. Not so attractive is the as-
sumption that the cash-flow stream will continue for ever. Obviously, no asset is
actually likely to continue producing cash flows for ever, but before you become
offended that financial practitioners use a technique that makes such a ridiculous
assumption, let us test whether or not the answers it gives are ridiculous.
Suppose that you are faced with valuing a stream of £100 cash flows with 10 per
cent opportunity costs, and the stream is expected to continue very far into the
future but you are not exactly sure how far. Were you to use an admittedly incorrect
perpetuity value, it would be £100/0.10 = £1000. How large an error are you likely
to be making by using that value?
Suppose that the cash flows are unlikely to last much beyond forty periods. With
a discount rate of 10 per cent, £100 in the fortieth year has a present value of only
£2.21 £100/ 1.10 40. As a matter of fact, the total present value of all cash flows
from the fortieth year until the end of time is only £100/0.10 / 1.10 40 £22.10.
This means that by using a perpetuity valuation, even if the stream actually ends in
the fortieth year instead of continuing for ever, the value error is £22.10 out of
£1000, or 2.21 per cent. As we shall see in our study of uncertainty, other errors in
making cash-flow estimates are likely to be large enough so as to overshadow
mistakes of this magnitude in present-value calculations. Of course, the size of the
error is a function of when the actual cash flow would cease, and could be much
larger than 2 per cent. For example, if the cash flow were to cease in the twentieth
year, the error in using the £1000 perpetuity value would be £1000/ 1.10 20
£148.64, and a 14.9 per cent error may be too large to be tolerable.
Though perpetuity valuation can be convenient and may not err greatly in long-
lived assets, Equation 1.4 also assumes that the cash flows will be constant for each
future period. That is not very representative of actual patterns of cash flows that
we see. Fortunately there is a slight modification of Equation 1.4 which can make it
these spreadsheets (with the computers to use them) and their inherent power and flexibility, if you
have not made the decision to begin using one, now might be a good time to consider doing so.
a bit more useful without significantly altering its simplicity. If we assume that the
cash flows will continue for ever, but will grow or decline at a constant percentage
rate during each period, the perpetuity formula becomes
﴾1.5﴿
where g is the constant per-period growth rate of the cash flow, and is the next
cash flow.
For example, suppose that we must value a cash-flow stream that begins at the
end of this period with £100, but that will grow at a rate of 5 per cent per period
every period thereafter (such that there will be a cash inflow of £105 at t2, of
£110.25 at t3, and so forth, for ever). With a discount rate of 10 per cent, the value
of the stream is:
£100
£2000
0.10 0.05
(The ‘bank account’ intuition here is like that of the constant perpetuity, except that
your withdrawals grow each year at 5 per cent.)
This ‘growth perpetuity’ present-value calculation is widely used for several fi-
nancial applications, especially when investigating the values of long-lived
organisations like large modern corporations. One note of caution, however: the
equation obviously does not work when the discount rate i is less than or equal to
the growth rate g. The implication that a cash flow, growing for ever at a rate nearly
equal to its opportunity cost, has an infinitely high present value, is numerically
correct, but not economically useful, because it could not reasonably be expected to
happen.
Since this has been a rather long section, we should review the points we have
developed in it. In discussing the various techniques that finance uses to perform
discounting calculations we have discovered that:
1. There is a simple calculator-based technique that is very effective for valuing
cash-flow streams that run for only a few periods.
2. When the stream continues for several periods and has the same cash flow for
each period, annuity present-value tables (giving the present value of £1 per
period) can be used.
3. When cash flows are well into the future, and the calculator being used cannot
exponentiate, single-cash-flow present-value tables (‘present value of £1’) are
useful.
4. Some financial pocket calculators can do all of the above with pre-programmed
ease. The spreadsheet programs widely available on personal computers have
financial functions that are even easier to use, more flexible and more powerful
than sophisticated pocket calculators.
5. Perpetuities, either constant or growing (or even declining) by a constant
percentage per period, can often be used as reasonable approximations for cash-
flow streams from very long-lived assets.
With enough practice, some of which is provided by the exercises at the end of
Module 1, you will quickly come to recognise the particular situation in which each
technique is most efficient.
prices) of these securities, and they appear in the t0 column of Table 1.1. You might
find it a valuable exercise to see if you can arrive at the same result. (Remember that
the market price of a security is the sum of the present values of the cash flows
expected from it, discounted at the rates appropriate to those flows.)
Table 1.1 Bond cash flows and prices
Price Cash flows
Security t0 t1 t2 t3
A £1029 £1080
B £1037 £80 £1080
C £1029 £80 £80 £1080
D £923 £40 £40 £1040
E £1136 £120 £120 £1120
Hint: In case you are having some difficulty in arriving at the same prices as
shown in Table 1.1, the way we arrived at the price of security C is:
£80 £80 £1080
£1029
1.05 1.06 1.07
Securities such as those in Table 1.1 are not at all uncommon in financial mar-
kets. We have designed the cash-flow pattern of A, B, C, D and E to be similar to
that of coupon bonds, which are the type seen most often in bond markets. A
coupon bond has a face value that is used, along with its coupon rate, to figure the
pattern of cash flows promised by the bond. These cash flows comprise interest
payments each period (which are given by the face value of the bond multiplied by
its coupon rate). This continues until the final (maturity) period, when the face value
itself, as a ‘principal payment’ plus a final interest payment, is promised. All of the
bonds in Table 1.1 are £1000 face-value bonds; their coupon rates differ, however.
Bond E has a 12 per cent coupon rate, which means that E promises to pay 12 per
cent of £1000, or £120, each period (t1 and t2) until it matures, when it will pay 12
per cent plus £1000, or £1120 (at t3). Bond A, of course, is an 8 per cent coupon
bond maturing in one period. See if you can similarly describe the other bonds.4
4 The face value of a coupon bond is usually called the ‘principal’ and the coupon payment the
‘interest’. You should always keep in mind that, regardless of what they are called, these amounts are
simply the cash-flow promises from the bond issuer; the coupon rate of interest bears no necessary
relationship to market interest rates. The coupon rate is simply a contractual provision of the bond,
which determines the amounts and timings of cash-flow promises. (The separation into ‘interest’ and
‘principal’ may be important for tax purposes, but we are not yet ready to worry about taxes.) The
market uses the bond’s cash-flow promises or expectations, and the market’s own interest rates to set
bond prices.
discussion you should now be able to examine a newspaper’s table and see the
correspondence between it and Table 1.2.
The only piece of information in Table 1.2 that may not be familiar to you is the
column headed ‘Yield’. That column presents the yield to maturity of the bonds;
the yield to maturity (YTM) is something you have already seen in another guise. It
is the IRR of the bonds’ promised cash flows.5 In other words, if you used the
constant discount rate of 5.96 per cent on the cash flows of bond B in Table 1.1,
you would get a present value (or market price) of £1037. The yield to maturity is
the rate that discounts a bond’s promised cash flows to equal its market price, and
(from our knowledge of IRR) is the ‘average per-period earning rate on the money
invested in the bond’ (which money, of course, is the market price).
Before embarking upon our investigation of forward interest rates, we can pre-
pare ourselves by flexing our mental muscles briefly over the relationship between a
bond’s YTM and the set of spot rates that determine the bond’s price. Look at
bonds C, D and E in the two tables. All three have the same time until maturity, the
same number of interest payments, the same cash-flow risk (none), and are subject
to the same set of spot interest rates, which have produced the market prices we see.
But the YTMs of the bonds differ. How can that be? If the same interest or
discount rates have operated upon the bonds, how can their average per-period
earning rates differ? The answer is that the pattern of a bond’s cash flow across
time influences its YTM, and these three bonds have different cash-flow patterns.
Compare the cash flows of bonds D and E in Table 1.1. Bond D, with a 4 per
cent coupon, has interim interest (£40) payments that are smaller relative to its final
payment than does bond E with its 12 per cent (£120) coupon. In effect, bond D
has a greater proportion of its present value being generated by its t3 cash flow than
does bond E with its relatively larger interim interest payments. Remember that the
spot interest rate for t3 is 7 per cent, whereas the rates for t2 and t1 are lower at 6 per
cent and 5 per cent respectively. So relatively more of bond D’s value is being
generated with the higher interest rate than is the case for bond E, and we therefore
5 When seeing the word ‘yield’ in the newspaper you must be careful to read the newspaper’s footnote to
the table, because newspapers also use that term to stand for ‘current yield’ or ‘dividend yield’ instead
of yield to maturity. The other ‘yields’ are simply this period’s interest payment or dividend divided by
the current price of the security. That ratio is of little use in bond markets.
see a higher per-period earning rate or YTM for D than for E. See if you can
convince yourself that bond C’s yield can be similarly explained.
The phenomenon we have described above has a name in finance; it is called the
coupon effect on the yield to maturity. It gets this name because the size of the
coupon of a bond determines the pattern of its cash flows, and thus how its YTM
will reflect the set of spot rates that exists in the market. The YTM is mathematically
a very complex average of the spot rates of interest, as weighted by the pattern of
cash flows of a bond. Depending upon your attraction to matters mathematical, you
will be either pleased or disappointed to hear that at this point it would not benefit
us to pursue further this particular aspect of the YTM.
What we have learned about the yield to maturity should teach us to be very
careful how we use it. For example, it would be most unwise to make comparisons
among securities on the basis of their YTMs unless their patterns of cash flows
(coupons, for bonds) were identical. The YTM, being a complex constant per-
period average, might lead the uninitiated to think that the money invested in one
bond was earning a better or worse rate than another bond in any given period,
whereas it is clear that bonds of equal risk must earn the same rates during the same
periods. The YTMs are expressing not only the earning rates but also the amounts
invested in the bonds across time. In our example, bond E had a lower YTM than
bond D because E’s higher interim cash interest payments meant that relatively less
was invested at the later periods (which had the higher interest rates).
Occasionally you may see the yield curve referenced as synonymous with the
term structure of interest rates. The former is the set of YTMs that exists in the
market, usually for government coupon bonds. The latter is, as we know, the set of
spot interest rates. From what you now know about interest rates, you should be
uncomfortable with using the yield curve as a substitute for the term structure.
1 1
£80 £1080
1.05 1.06
£76 £961
£1037
6 For simplicity of presentation we have selectively rounded off the cash flows, values and rates in this
section. If you prefer numbers of higher accuracy, and if your calculator runs to several decimal places,
we encourage you to investigate the more accurate results. To have the results consistent, you can
assume that the spot rates, coupons and face amounts above are exact, and all other values and rates
produced herein are rounded.
Of the £1037 invested in the bond at t0, £76 of it produces £80 at t1 for a 5 per cent
return during the first period, and £961 of the t0 investment produces £1080 at t2, a
6 per cent return per period for two periods.
To this point we have done nothing more than we did when presenting the YTM
in terms of invested amounts and earning rates, with a bit more numerical detail.
But you recall that in pointing out the reasons why bond E had a lower YTM than
bond D, we said that bond E had ‘less invested in it’ during the later time periods
that contained the higher interest rates. Though that statement is entirely correct, we
were not specific about what it means to have money invested in an asset at some
future time. Understanding that will bring us a long way toward understanding
forward interest rates.
Let us return to bond B and investigate the amounts invested in it across time.
We know that there is £1037 invested at t0, and that after the final payment is made
at t2, the investment must be zero. So the only question is the amount invested at t1.
If there is £1037 invested at t0 and if the earning rate for the first period is 5 per
cent, the amount invested at t1 (before the t1 interest payment) must be £1037
1.05 £1089. After the £80 payment, the amount invested at t1 is thus £1089 – £80
£1009. We can find the amount invested in an asset across time by accruing past
invested amounts outward at the same rates that we discounted cash flows back-
ward in time.7
So £1009 is the amount invested in bond B at t1. That information is important
because it allows us to calculate an example of what we have been seeking: a forward
interest rate. (A forward interest rate is usually noted with the letter ƒ surrounded by a
left subscript indicating the rate’s beginning time point, and a right subscript indicating
the rate’s ending time point.) The £1009 invested in bond B at t1 produces a payment
of £1080 at t2. The implied earning or interest rate for bond B between t1 and t2
(noted as 2 ) must therefore be:
1
V 1
£1009 1 £1080
7%
Bond B is earning 7 per cent between t1 and t2: 2 7%. The implied forward
1
rate for bond B in the second time period is 7 per cent.
We can continue with this example to illustrate another important relationship:
that between spot rates and forward rates. We now know that the £1080 at t2 is
worth £1009 at t1 (discounted for one period at 2 ), and £961 at t0 (discounted for
1
two periods at i2). But we have seen that it is also correct to think of the t1 value of
bond B as being generated by an investment earning 5 per cent during the first
7 There is a financial market risk characteristic in addition to cash-flow risk that we are holding in
abeyance for the moment: the uncertainty of future interest rates. The examples with which we are
dealing in this module assume that the interest rates expected for future time periods will actually
occur. This assumption, of course, has little meaning to you until we discuss what future or ‘forward’
interest rates are. So until we do, you can regard this as a gratuitous comment designed to appease
picky academics who might be reading this.
period. Since an earning rate is just a discount rate in reverse, we can also think of
the £1080 at t2 being discounted to t0 with the appropriate forward rates:
1 1
And since is simply i1:
0 1
£1080
£961
1.05 1.07
So it is entirely correct to think of the present value of the t2 cash flow as being
arrived at either by discounting with the spot rate for two periods or by discounting
with the forward rates for one period each. This in turn implies that the relationship
between the rates is:
1 1 1
Generally, this type of relationship will hold for all spot rates compared with the
forward rates covering the same time. If the forward rates are known, the spot rate
of interest can be found by multiplying together 1 plus each of the intervening
forward rates, taking the nth root of that product (where n is the number of periods
covered), and subtracting 1. If the spot rates are known, the forward rates can be
found by a process of solving first for the forward rate nearest the present, and
successively working to rates further in the future, exactly as we did with bond B.
(Those of you with quantitative backgrounds will have recognised that (1 + spot
rates) are merely the geometric means of (1 + forward rates).)
As an exercise, let us calculate :
2 3
1 1 1 1
1.07 1.05 1.07 1
1 1.07 / 1.05 1.07 1.09
9%
(Note that the rate i3 = 7 per cent is, as we have shown, not the simple average of
the 5 per cent, 7 per cent and 9 per cent forward rates, but is the result of subtract-
ing 1 from the nth root of the product of 1 plus the intervening forward rates. This
is a geometric mean.)
To test your conceptual understanding of the various interest rate ideas that we
have produced, see if you can explain to yourself (or to anyone who is willing to
listen) these three different methods of calculating bond D’s present value, and how
they are related to each other:
£40 £40 £1040
£923
1.05 1.06 1.07
£40 £40 £1040
£923
1.05 1.05 1.07 1.05 1.07 1.09
£40 £40 £1040
£923
1.069 1.069 1.069
Your explanation should be to the effect that all three methods of calculating the
value of bond D are correct, the first using the spot rates, the second using the
forward interest rates, and the last using the bond’s YTM. The most accurate
portrayals of the financial market’s valuation process is given by the spot or forward
rates.
the risk of interest rate changes, only opportunity costs would have changed in this
example.)
The risk that interest rates might change unexpectedly is something that many
market participants would like to avoid. If you had decided to undertake a real asset
investment because it had a positive NPV and, after you had the investment well
under way, interest rates increased so as to cause NPV to be negative, you would be
disappointed. You need not have been: there are today available ‘financial futures’
markets that allow participants to guard against this kind of risk (and many other
related types) by buying and selling commitments to transact in financial securities at
future time points, at prices fixed as at the present. This would allow you, in the
situation described above, to ‘lock in’ or guarantee a set of discount rates for your
asset’s NPV by agreeing to sell some financial securities at set prices across the life
of the real asset. (You need not even own the securities you agree to sell, as long as
you can convince the market that your credit is good by posting what is called a
‘margin’ or amount of money that would make up any likely losses on the transac-
tion.)
To illustrate, suppose that you are about to undertake an investment that has a
positive NPV using the current set of interest rates of the market but that also
would be unlikely to be desirable if interest rates increased during the life of the
project. One tactic to insure against the detrimental effects of interest rate increases
would be to sell an interest rate futures contract in the approximate amounts and
timings of the cash inflows of the project. Let us examine a simple example of such
a transaction.
Suppose that you are about to undertake an investment with the following cash
flows:
t0 t1 t2
−£1700 £1000 £1000
and so the investment is acceptable. But suppose interest rates that were to occur in
the future were not known for certain, and this manifested itself in the risk that the
interest rate applicable between t1 and t2 (the forward rate ) might change from
1 2
the one now implied by the current term structure of interest rates. You recall that
we can find the implied by the current term structure by using the relationship
1 2
between spot and forward rates:
1 1 1
1 1 / 1
1 1.11 / 1.10
12.009%
So the forward rate between t1 and t2 implied by the current term structure is 12.009
per cent.
Now suppose there is risk that the 2 rate would increase to 15 per cent. If this
1
happens at t0, there will be a new i2 of:
1 1 1
1 1.10 1.15
1 1.265
12.4722%
And the present value of the investment becomes:
£1000 £1000
NPV £1700
1.10 1.124722
£0.40
The positive NPV of the investment has become negative due to the increased
interest rate applicable to the t2 cash flow. Were you to have committed any
resources to the investment prior to the interest rate change, you would doubtless
be distressed that an investment you expected to be good is now expected to be one
that will decrease your present wealth.
The futures market in interest rates can enable you to avoid (or, to use the termi-
nology of that market, hedge) the risk of such an occurrence. Suppose there were
such a market, and you were faced with the same investment, original term structure
and interest rate risk. This market would allow you to hedge the risk of a change in
by selling an interest rate futures contract. The interest rate applies
1 2 1 2
between t1 and t2, so your transaction will commit to sell at a fixed price a security at
t1 that has a single (£1000) cash flow at t2. If increases, the price of such a
1 2
security will decline. But since you will have a contract to sell that (now cheaper)
security at a fixed higher price, the value of your contract will increase. The increase
in the value of your contract will offset the decrease in the NPV of your investment,
and you will have avoided the risk of interest rate changes. Let us examine the
associated financial arithmetic.
Given the original term structure, the futures market will dictate a t1 price for the
one-period interest rate future of:
£1000
futures price
1
£1000
1.12009
£892.79
To hedge against interest rate risk you commit to sell a £1000 t2 cash flow at t1 for
£892.79; that is the essence of your futures contract. Now, still at t0, having under-
taken the investment and sold the futures contract (no cash actually changing hands
at t0, the contract being ‘sold’ meaning only that you have made a commitment to
sell the security at t1), increases to 15 per cent. We have already seen the
1 2
deleterious effect of that interest rate change upon the NPV of your investment:
NPV declines from +£20.71 with the original term structure to −£0.40 with the
new term structure, a wealth loss to you of £21.11. But what has happened to your
futures contract? The increase in causes a £1000 cash flow at t2 to decline in
1 2
value. The new t1 value of the t2 £1000 is:
£
value of cash flow
.
£869.57
This decline in value is actually good news to you because you own a contract
(the interest rate future you sold) that allows you to sell at t1 for £892.79 a security
that promises £1000 at t2. Even though the t2 cash flow is worth only £869.57 at the
new interest rate, you can sell it for £892.79. That is a valuable capability; the value
of your contract must therefore increase (remember that before the interest rate
change the contract was without value, because it was promising the same interest
rate as the market). Now the contract must obviously be worth £892.79 – £869.57
£23.22 at t1. That, however, is a t1 amount. If we discount that value to t0, with the
unchanged i1 = 10 per cent we get:
£23.22
Increase in contract value at
1.10
£21.11
We have seen that number before. In addition to being the increase in value of
your interest rate futures contract at t0, it is also the decrease in value of your
investment’s NPV. Thus the interest rate futures contract has hedged you against
the risk that interest rates will change so as to decrease the NPV of your investment.
Both the increase in futures contract value and the decrease in investment NPV are
caused by the change in interest rate. Because you are to receive a cash flow from
the investment that is discounted with that interest rate, the investment declines in
value; but because you are to sell a cash flow discounted with that interest rate
through the interest rate futures contract, the contract increases in value. Since the
amounts and timings of the cash flow are the same, the value changes are also the
same (and of course in opposite directions).
A word or two of caution about this illustration is necessary, however. First,
although the financial economics of the illustration is accurate, we have simplified
the contract and transaction somewhat for clarity of exposition (i.e. we have not
worried about margins, brokerage fees, the difficulties of finding an exact contractu-
al hedge, and other characteristics of real-life transactions). The analysis of hedging
in actual financial markets is best left to more advanced texts, and to professionals
in that marketplace.
Furthermore, you should keep in mind that hedging works in both directions to
remove the effects of interest rate changes. For example, if were to decrease,
1 2
the NPV of your investment would increase, but your futures contract value would
decrease to offset the NPV increase. Hedging means you lose both the bad and the
good surprises. Finally, note that the investment’s cash flow itself was unchanged
throughout the example. In real investments it is often the case that the same kinds
of event that cause interest rates to increase and decrease (inflation, for example)
will also cause cash-flow expectations to increase and decrease (in the same direc-
tion as interest rates). Were that to be a characteristic of your investment’s cash
flows, you would not be interested in hedging interest rate risks, because revisions in
your cash-flow expectations would effectively do it for you. Nevertheless, there are
many instances where such hedging is worthwhile, and it is ever becoming more
popular with sophisticated financial market participants.
This is a useful discussion for us, because it is a rather advanced illustration of
the ideas involved with forward interest rates, values, and the way financial markets
operate. But, for all that, we have introduced no really new basic concepts. A careful
review of the foregoing will show that we have essentially done nothing more than
discount future cash flows at market rates.
Forward interest rates, forward prices and futures contracts with their associated
transactions are some of the more sophisticated ideas that exist in finance. We do
not expect that you feel expert enough to participate in those markets at this point;
as a matter of fact, we would not be surprised if you were a little discouraged at the
apparent complexity of all these financial manipulations. But take heart. Our only
purpose in the illustrations above is to convince you that the interest rate ideas upon
which we have spent so much time are not mere academic exercises but valuable
concepts for the financial practitioner.
ments). But for whatever reason, such changes in interest rates imply changes in
value and therefore in wealth, which is important.
There is an interesting measure of the extent to which a particular bond with
specified interest payments is subject to interest rate risk. This measure is called
duration, and is a kind of index that tells us how much a particular bond value will
go up and down as interest rates change. It measures the ‘exposure’ of the value of a
bond to changes in interest rates. Rather than give a more detailed definition at this
point, let us look at an example.
Return to Table 1.1, and consider bonds C and D. Their prices are indicated as
£1029 and £923 respectively. Suppose that interest rates instantaneously increased,
such that the spot rates were 6%, 7% and 8%, instead of the 5%, 6% and 7% that
gave us the original values. The bonds’ values must, of course, decline, and if you do
your arithmetic correctly, you will now see that bond C is worth £1003 and bond D
is worth £898. But notice that the decline in value of bond C is less in percentage
terms than is the decline in the value of bond D – about 2.6% for bond C and 2.8%
for bond D. The same tendency would appear for reductions in interest rates: bond
D’s value would react more than bond C’s. Why is that? Why does bond D experi-
ence a greater percentage change in value than bond C? The answer is that bond D
has a greater duration than bond C.
We are now ready to see a more rigorous definition of duration. Duration is the
number of periods into the future where a bond’s value, on average, is generated.
The greater the duration of a bond, the farther into the future its average value is
generated, and the more its value will react to changes in interest rates. The reason
for this is not difficult to understand.
Consider two bonds, each of which have only one payment, but one of the
bonds will receive its payment after one year, and the other will receive its payment
after five years. (These types of bond are called zero coupon bonds, because in
effect they have only a final principal payment and no interim interest payments.)
No matter what the term structure of interest rates, the five-year bond’s value will
react more strongly to a given change in interest rates than will the one-year bond. If
interest rates go up or down, the five-year zero coupon bond’s value will decline or
increase in percentage terms more than the one-year bond’s. The reason of course is
that the five-year bond’s cash flow is discounted with an interest exponent of 5,
whereas the one-year bond’s cash flow is discounted with an interest exponent of
only 1. Notice also that the duration of the five-year zero coupon bond is simply 5,
because that is the time in the future that generates the bond’s entire value. Similar-
ly, the one-year zero coupon bond has a duration of 1. And so, here again, the
longer-duration bond is associated with a greater reaction to changes in interest
rates.
Let us return now to bonds C and D. Finding their durations is more complicat-
ed than finding durations for zero coupon bonds because bond C’s and bond D’s
values come from more than a single future time. We can calculate their durations
by ‘weighting’ the time points from which cash flows are generated, by the propor-
tion of total value generated at each time. One way to calculate bond C’s duration is:
80 80 1080
Duration C 1 /1029 2 /1029 3 /1029
1.05 1.06 1.07
Duration C 2.78
Similarly, bond D’s duration is:
40 40 1040
Duration D 1 /923 2 /923 3 /923
1.05 1.06 1.07
Duration D 2.88
So bond D has a longer duration than does bond C. Bond D’s average present
value is generated 2.88 periods into the future, whereas bond C’s average present
value comes sooner, 2.78 periods into the future. And with this longer duration,
bond D experiences a greater interest rate risk; its value will go up or down more
than bond C’s for a given change in interest rates.
The idea that duration is a measure of interest rate risk is particularly valuable for
coupon bonds, where comparative riskinesses might not be obvious simply by
inspection. For example, depending upon their coupon rates, a nine-year coupon
bond could have a longer duration than a ten-year coupon bond and therefore be
subject to greater interest rate risk. Further, duration is the starting point for an
important aspect of professional bond investing called immunisation which allows
certain portfolios of coupon bonds or other types of investments to be shielded
against unexpected changes in interest rates. Teaching you these techniques is
beyond the scope of this text, but it is important that you have heard that such a
thing is possible.
Learning Summary
One final note is of a more philosophic nature on interest rate structures. Amidst all
this discussion of complicated interest rate calculations, of forward rates, spot rates,
yields to maturity and so forth, you should remember that financial markets really
do only one thing: they set prices on financial securities. The array of interest rates
of various types that we see is merely the attempt of practitioners to make some
sense of the consistency among those market prices. As long as we keep in mind
that the market is made up of many wealth-maximising participants, each aware of
and concerned about opportunity costs, then the rest of the system almost invents
itself.
The section on realistic financial markets has introduced and developed several
important financial concepts and techniques, but at the same time it has been a long
and occasionally arduous one. While extending the single-period financial market of
the previous section to multiple periods, we studied present value tables, calculator
techniques of finding present values, compounding of interest, various perpetuity
formulations, yields to maturity, bond value ideas, the relationships among spot
rates, forward rates and yields, forward prices, and the workings of futures markets
in financial assets. This is a great deal of material, and it will take you some time and
effort to digest. You should not hesitate to reread sections; much of the material will
seem easier the second time around. Give very close attention also to the problems
that follow, for they are excellent learning devices.
Review Questions
1.1 Suppose that you are a participant in the single-period financial market described in this
module. Your certain expectation is to receive £3000 immediately, and an additional
£5328 at the end of the period. If the market rate of interest for riskless borrowing and
lending is 11 per cent, the maximum you can consume immediately is which of the
following?
A. £8328.00
B. £7800.00
C. £51 436.36
D. £7843.64
Questions 1.2 to 1.11 use the information in Question 1.1. You may find it
convenient to sketch a graph of your answers.
1.2 The maximum that you can consume at the end of the period is which of the following?
A. £8328.
B. £7800.
C. £5658.
D. £8658.
1.3 Suppose you wished to consume £5000 immediately. Which of the following amounts
could you then consume at the end of the period?
A. £3108.
B. £3128.
C. £8658.
D. £7548.
1.4 If you wished to consume £7548 at the end of the period, which of the following could
you consume now?
A. £3000.00
B. £1000.00
C. £981.82
D. £7800.00
1.5 Consider the present value of all four consumption combinations in Questions 1.1 to
1.4 above. You should expect them to be:
I. All different because the amounts consumed are all different.
II. All the same because they are all allocations of the same total wealth.
III. All the same because they are all available through borrowing and lending transac-
tions from the same initial expectations.
Which of the following is correct?
A. I alone.
B. II alone.
C. III alone.
D. Both II and III.
1.6 Suppose that you prefer a pattern of consumption that tends to emphasise present
consumption, while another participant with identical cash-flow expectations prefers
one that delays consumption into the future. Which of you would be the wealthiest?
A. You, because the other participant will have made investments that do not
produce income until the end of the period.
B. The other participant, because you will consume more of your income first.
C. Neither, because consumption patterns do not affect wealth.
D. Neither, because you will both eventually get to consume the same amounts.
1.7 Suppose that in the same financial market the following investments, also with riskless
cash flows, are available:
Investment t0 t1
1 −£1000 £1250
2 −£500 £650
3 −£1500 £1650
Using the NPV criterion, you would accept which of the following?
A. Investment 1 alone.
B. Investments 1 and 2.
C. Investments 1 and 3.
D. All three investments.
1.8 Using the IRR criterion for the investments in Question 1.7, you would accept which of
the following?
A. Investment 1 alone.
B. Investments 1 and 2.
C. Investments 1 and 3.
D. All three investments.
1.9 Suppose that the investments above were undertaken correctly by a company of which
you were the only shareholder. The change in your present wealth would be which of
the following?
A. £211.72
B. £198.21
C. −£2500.00
D. £227.27
1.10 Suppose that the company in Question 1.9 above informs you as its owner that it does
not have enough money to undertake the chosen investments, and requests that you
provide the necessary funds. Suppose also that your preferences for consuming your
wealth are such that you wish to spend £3000 at the present (t0), which is equal to your
initial resources at that time point (see Question 1.1). Your options are to:
I. Refuse the investment appeal because you would not be able to consume as you
wished at t0 if you provided the company with the money.
II. Provide the money as requested at t0 and borrow enough to consume as you wish.
III. Provide the money as requested at t0 and sell your shares so as to consume as you
wish.
IV. Refuse to invest further and suggest that the company borrows the money else-
where.
V. Refuse to invest further and suggest that the company sells shares to others so as to
raise the money.
A correct decision is to choose which of the following?
A. I.
B. Either of II or III.
C. Either of IV or V.
D. Either of II and III or IV and V.
1.11 Suppose that the three investments shown in Question 1.7 above are mutually
exclusive; that is, you could only accept one of the three. Which of the following should
you choose?
A. Accept the one with the highest NPV because it increases your wealth the
most.
B. Accept the one with the highest IRR because it increases your wealth the most.
C. Accept the one with the highest NPV because it earns the greatest return per
period.
D. Accept the one with the highest IRR because it earns the greatest return per
period.
Questions 1.12 to 1.18 refer to Module 1, Section 1.4 and Section 1.5.
1.12 You intend to open an ice cream stand and must choose a location for it. There are two
sites available, each of which requires that you make a present (t0) cash outlay of £2500.
You expect that the locations’ net cash inflows for the three-period life of the stand will
be:
t1 t2 t3
Location 1 £1200 £1300 £1450
Location 2 £1300 £1300 £1300
If your opportunity costs are constant at 10 per cent per period, you would, using the
NPV criterion, choose which of the following?
A. Location 1.
B. Location 2.
C. Either one, because you are as well off with either.
D. Neither, because they are both undesirable investments.
1.13 Suppose that your opportunity costs were 25 per cent per period rather than 10 per
cent in the scenario of Question 1.12. Would that change your answer?
A. Yes, you would choose the other location.
B. No, you would choose the same location.
C. No, you would still be indifferent between them.
D. No, you would still reject both of them.
1.14 You are faced with a choice between two investments that require the same outlay. The
first is expected to provide a stream of perpetual cash flows equal to £1000 per period
for ever. The second investment is also a perpetuity, and has a t1 cash flow of £800,
which will grow at a constant rate of increase each period for ever. If your opportunity
costs are constant at 10 per cent per period, what rate of increase in the second
investment’s cash flows is necessary to make it as desirable as the first?
A. 10%.
B. 2%.
C. 0%.
D. There is no rate which will make the second investment as desirable as the
first.
1.15 Suppose that the first investment in Question 1.14 was not a perpetuity, and the second
investment’s cash flows did not grow. For how many periods would you necessarily
expect the first investment to run so as to render it as desirable as the second?
A. About ten years.
B. About thirteen years.
C. About seventeen years.
D. About twenty years.
1.16 (To be completed easily, this problem requires that you be able to exponentiate. If your
calculator cannot do that, describe how you would solve the problem, and check your answer
against that provided.)
Suppose that you wish to purchase the digital tape device mentioned in the text, but
instead of borrowing money to buy it, you prefer to put aside enough in an interest-
bearing account to be able to pay cash for the machine. If the device costs £800, you are
willing to wait a year to get it, and the bank pays 10 per cent annual interest compound-
ed monthly, how much money must you put aside each month, beginning at the end of
this one, to purchase the machine at the end of the year?
A. £66.00.
B. £60.60.
C. £63.67.
D. £60.32.
1.17 Suppose the bank in Question 1.16 compounded interest continuously. How much
would you necessarily deposit in the bank at the beginning of the year in order to end
the year with £800? (Continue to assume a 10 per cent annual interest rate.)
A. £727.27
B. £723.87
C. £724.17
D. £738.16
1.18 You are now considering an investment opportunity that has the following certain cash-
flow expectations:
t0 t1 t2
−£15 000 +£7000 +£11 000
Market interest rates, and thus your opportunity costs, are 10 per cent per period. You
are concerned that, in the event that you decide to undertake the investment, there is a
chance that the actual interest rate occurring between t1 and t2 will not be 10 per cent
but 20 per cent instead. You should do which of the following?
A. Not undertake the investment because it has a negative NPV or an IRR less
than your opportunity cost.
B. Undertake the investment because its NPV is positive or IRR exceeds oppor-
tunity costs, and would continue to do so even with the increase in interest
rate described.
C. Undertake the investment and simultaneously sell an interest rate future for t1
in the amount of the value currently expected for the t2 cash flow at t1.
D. Undertake the investment and simultaneously buy an interest rate future for t1
in the amount of the value currently expected for the t2 cash flow at t1.
3 What is the one-period forward rate of interest for the second period ( )?
4 What is the one-period forward rate of interest for the third period ( )?
6 Without actually performing the calculation, would you expect the YTM of bond (A) to
be greater or smaller than that of bond (B)? Explain.
7 After its interest is paid at t1, what is the current expectation for the price of bond (B)
at t1 (its forward price at t1)?
8 Some investment bankers are now selling securities that they form by purchasing
coupon bonds and separately offering the coupons and principal payments to the
financial markets. In other words, it is now possible for you to purchase a security that
is a future claim upon a single interest payment from a coupon bond. If we assume that
interest rates in the future are known for certain, what would be the current price of
the t2 interest payment from bond (C)?
9 Suppose now that interest rates expected to occur in the future are uncertain – in
other words we can, if we wish, calculate a rate such as , but there is no guarantee
that when t2 actually arrives the existing i2 will equal that rate. As a matter of fact, there
is no guarantee that any forward rate will be the same in even the next instant of time.
If you were now about to undertake an investment that had cash-flow expectations
extending for the next few periods, outline the general characteristics of a strategy for
eliminating the risk that interest rates (and thus your NPV) would change during that
time. You may assume that any financial markets necessary to that strategy do exist.
10 Now assume that you are considering an investment, one of whose cash inflows is
£1000 at t3. Illustrate, with a quantitative example based upon your answer to Question
9 above, how you can hedge the risk that will change.
t0 t1 t2 t3
£12 000 £13 000 £14 000 £15 000
The rates of interest in the market are constant across the future at 8 per cent per
period.
2 At what current prices would you be able to sell each of your future cash flows?
3 At what price would you expect to be able to sell your t2 cash flow at t1?
5 Suppose that you wished to consume a constant amount at each time point, beginning
now. How much would you be able to consume as a maximum at each time? Demon-
strate with a specific set of financial market transactions (borrowing and lending) how
you could arrive at that consumption pattern.
Assume now, still at t0, market interest rates change such that the one-period spot rate
is 6 per cent, the two-period spot rate is 8 per cent, and the three-period spot rate is 9
per cent.
7 Describe how that effect occurred, with reference to the present value of each of your
expected cash flows. Have they all changed in the same direction as your present
wealth? Explain.
8 Is it still possible to consume in the pattern that your answer to Question 5 said was
your choice?
Assume now that interest rates have returned to their original levels (constant at 8 per
cent per period). An investment becomes available that requires a t0 outlay of £5060,
and returns £1500 at t1, £2000 at t2 and £2480 at t3.
10 Suppose that the interest rate structure was the one that applied to Question 6
immediately above. Would your answer to Question 9 be the same?
11 Suppose that the cash flows for the investment at t1 and t3 were swapped. Would the
relative desirability of the investment with the two interest rate structures be the same?
Explain.
Fundamentals of Company
Investment Decisions
Contents
2.1 Introduction.............................................................................................2/1
2.2 Investment Decisions and Shareholder Wealth..................................2/2
2.3 Investment Decisions in All-Equity Corporations ...............................2/6
2.4 Investment Decisions in Borrowing Corporations........................... 2/10
2.5 Share Values and Price/Earnings Ratios ............................................ 2/13
Learning Summary ......................................................................................... 2/17
Review Questions ........................................................................................... 2/17
Learning Objectives
Module 2 focuses the discussion of financial decision making upon the incorporated
firm, the ‘corporation’ or plc. It begins with a discussion of the corporate form of
economic organisation, with its important distinctions between debt and equity
capital and how these securities form a hierarchy of claims against the cash flows of
the firm. Next are the specific sources of value, based upon cash-flow expectations,
of those capital claims. The module then illustrates how real asset investment and
financing decisions taken by companies affect the wealth of shareholders and
bondholders, and this is shown by associating the internal financial decisions with
changes in values placed on the securities by stock and bond exchanges. The NPV
and IRR again form an important part of the analysis. Finally, this module provides
the student with some real market context in a discussion of the popular financial
measurement called the ‘price/earnings ratio’ and its correct application. Module 2
introduces students to the important distinctions between making financial deci-
sions as individuals compared with making such decisions in complex organisations
such as the modern corporation.
2.1 Introduction
In this module we begin to apply the ideas and techniques of the first module within
a familiar context: the organisations in which most of us work. Although this course
is a general introduction to finance, which is applicable to many different situations,
the richest and most varied applications are those that occur in complex organisa-
tions, especially the modern company organised as a corporation (we shall use the
terms ‘company’ and ‘corporation’ synonymously). The things we learn about
finance in companies will, for example, be transferable to the decisions that are
made in other organisations (such as governmental agencies) and to the level of our
own personal financial concerns.
Because we shall be studying finance from the company viewpoint, it is im-
portant that we understand from the perspective of finance exactly what a company
organised in the corporate form is. For our purposes we are interested in the
characteristics of corporations that distinguish them from other participants in
financial markets. In this context:
The corporation (plc) is an organisation that raises money from capital suppli-
ers by issuing contracts (we call them securities), invests that money in
productive assets, operates those assets (perhaps hiring other resources such
as management and labour), and distributes the money proceeds from operat-
ing those assets to all that have claims on those proceeds.
Those having claims upon the company’s cash flows include everyone holding
contracts, formal and informal, with the company. They include workers and
management (receiving cash flows as compensation for services), government
(receiving cash flows as taxes and fees), suppliers of raw materials (being paid for
materials used in the productive process), and finally, capital suppliers (being paid
their capital returns in the forms of interest, principal and dividends). This latter
group, the capital suppliers, are those who have purchased the securities that the
company has issued; the company issues these securities so as to raise money to
acquire productive assets. We usually know this group of capital suppliers as those
who own the equity and debt claims on a company.
Corporations are ‘creatures of the law’ in the countries within which they operate.
They may be called plcs, SAs, Incs, AGs, or any number of other titles depending
upon their country of location, but they will all have the essential characteristics
mentioned above and will all face the same basic financial decisions. These deci-
sions, as confronted by companies, are in economic dimension no different than
those that are faced by the simple participants in our Module 1, except that corpora-
tions are subject to many complicated taxation and other ‘real world’ phenomena
that make their analyses more challenging.
In this module we shall concentrate upon the investment decisions that compa-
nies make. In a sense, we shall be adding realism to the investment ideas that we
studied in the prior module. We shall show some of the elaborations that are
necessary for those techniques to work with real companies, and why those tech-
niques work well for those organisations. We shall connect the esoteric ‘textbook’
methods of finance to familiar measures of corporate performance such as earnings
per share, price/earnings ratios and share prices.
into a wealth increase of that amount for the shareholders of the company. The first
step in doing that is to learn more about the equity of corporations.
to stand last in queue for the corporate largesse, in exchange for everything that
is left over.
This module is concerned primarily about the investment decisions that compa-
nies make. We have already learned one important thing about those decisions:
companies seek to make investments that are in the best interests of their existing
shareholders. And we know from Module 1, shareholders are best off when they are
wealthiest. Consequently, companies, in making investment decisions, are attempt-
ing to make their shareholders as wealthy as possible. We also know from Module 1
that individual participants can maximise their wealth by choosing investments that
have the highest NPVs. We shall now demonstrate that companies maximise their
shareholders’ wealth by choosing corporate investments with the highest NPVs.
First we must become more specific about exactly what ‘shareholder wealth’ is.
Shareholder wealth is the market value of the common shares or equity that the
shareholders own. So if the company wishes to maximise its share-holders’ wealth, it
should seek to maximise the market value of the shareholders’ ordinary shares.
Div E
E
1
This seems to answer your question positively: the value of your common shares
in International Eugenics depends upon the price at which you can expect to sell at
t1. But there is something incomplete about this idea. What about the person to
whom you intend to sell the shares? What would induce that person to purchase the
shares from you for a price of E1? Naturally, he or she would be willing to pay only
the present value of what they expect to get back in the future, which is perhaps one
period of dividends, and then the market value of the shares when they sell them. So
they would be willing to pay, and you can reasonably expect to receive at t1:
Div E
E
1
This means that the value of your shares at t0 is really:
Div Div E
E
1 1
Compare this formula to the earlier one for E0 and notice the difference. The
price at which you can sell your shares at the end of the first period (E1 in the first
formula) has disappeared from the equation for E0 and has been replaced by the
dividend at the end of the next period plus the price at which the purchaser can
expect to sell the shares of International Eugenics at the end of that period.
You can probably see where this argument is heading. The person to whom you
sell your shares must in turn sell them to someone who expects to get some
dividends, and themselves sell to someone expecting dividends, and so forth. When
all is said and done, and all of the reasonable expectations of returns from holding
the shares are included, we are left with only future dividends as the basis for the
market value of a company’s equity.
A simple numerical example might be useful at this point. Suppose that Interna-
tional Eugenics has expectations of paying £100 in dividends for ever, and the
appropriate discount rate for the dividends is 10 per cent per period:
£100
E £1000
0.10
Your shares are thus worth £1000, based upon the set of future dividends expected.
You, however, having a finite life, cannot expect to receive all of those dividends.
In fact, you expect to sell your shares at the end of this period. At that time the
shares will still be expected to pay dividends of £100 per period for ever, and with
the same discount rate will still be worth £1000. So your expectation of cash flow
from holding the shares of International Eugenics is £100 of dividends and a £1000
sale price, both at the end of the period:
£100 £1000
E £1000
1.10
At a 10 per cent discount rate, the present value is £1000, the same price that the
market is quoting for your shares based upon an infinite stream of £100 dividends.
So even though you feel that the value of your shares is due in major part to the
price at which you can sell them, the market price actually depends upon all of the
future dividends that those shares are expected to receive.
This idea should actually be of some comfort to those of us who prefer the world
to be a place where, in most instances, intuitive common sense is a reasonable guide.
Most basically, this argument is saying that the value of the claims against a compa-
ny’s future cash flows must finally depend upon the amounts of cash that the
company is expected to pay to the claimholders, not upon what claimholders are
expected to pay each other when they buy and sell shares. The idea that the market
value of the shares of a company is the present value of its future dividends is
simply a manifestation of that common sense.
To review where our investigation has taken us to this point:
1. Corporations in making financial decisions attempt to maximise the wealth of
their existing shareholders.
2. Shareholder wealth consists essentially of the market value of the common
shares or equity of the company.
3. So the company, in maximising the wealth of its shareholders, must attempt to
maximise the market value of its common shares.
4. The market value of the common shares of a company is the present value of
the future dividends expected to be paid to the currently existing shares of the
company.1
1 You might be curious about our insistence upon always referring to the dividends expected for the
‘currently existing’ shares of the company as the basis for share value. The reason we talk about only
those dividends as the source of the value of the equity of a company is that the company will probably
issue some new shares at some time in the future, and will, naturally, pay dividends to those shares.
Since the dividends paid to new shares will not, of course, be paid to the current shares, those other
dividends cannot be part of the value of the company’s equity at the present time.
£50 000
NPV £100 000 £400 000
0.10
What has happened to the wealth of the shareholders? First, there will be an
increase in the market value of the company’s shares due to the increased future
amounts of cash that the Simple Corporation will be able to pay to its shareholders.
The £50 000 per year has a present value, with a 10 per cent discount rate, of
£500 000. The market value of the company’s shares will increase by that amount.
But that market value increase is larger than the NPV we calculated for the invest-
ment. If shareholder wealth increased by as much as the increase in the value of the
shares (£500 000), the corporate NPV (£400 000) would be less than the sharehold-
er wealth increase, and thus NPV would not be a good indicator of investment
desirability. What we have ignored to this point, and what is causing the difference,
is the £100 000 present cash outlay that the Simple Corporation makes when it
undertakes the investment.
Suppose that the Simple Corporation made the £100 000 investment outlay from
cash that would have otherwise been used to pay a shareholder dividend. Were that
the case, it is easy to see that the investment would increase the shareholders’ wealth
by £400 000, not £500 000, because the £500 000 increase in the market value of the
shares is partially offset by the £100 000 cash dividend that shareholders did not get.
The net difference is £400 000, as the increase in shareholder wealth and as the
corporate NPV.
‘Fine,’ you say. ‘But the Simple Corporation did not have to undertake the in-
vestment that way, by using the cash that would have otherwise been paid as a
dividend. Suppose the company paid the £100 000 dividend and at the same time
raised £100 000 from new equityholders for it, to be used for the investment outlay.
That sounds better for shareholders since they get both the current £100 000 cash
dividend and the market value increase in their shares.’
Clever, but wrong. The error is in underestimating the intelligence of the new
equityholders. The new equityholders are being asked to contribute £100 000 cash
to the Simple Corporation, and will naturally require something in exchange that has
a value at least as high. That something is, of course, shares. So the new sharehold-
ers must end up with at least £100 000 of the company’s share value or they would
not contribute that amount of cash. (And if the new shareholders end up with
significantly more than £100 000 of the value of the shares, the financial managers
of the Simple Corporation should be fired for not protecting the old shareholders’
interests.)
You can see where this leads us. If the Simple Corporation issues new equity to
raise the money for the investment, the market value of the company’s shares
increases by £500 000. But £100 000 of that must go to the new shareholders, which
leaves £400 000 for the original shareholders. Relative to the old financing plan that
used the dividend to make the investment, the original shareholders here do get the
£100 000 cash dividend, but lose £100 000 of the share value increase to the new
equityholders, so the original shareholder wealth increase remains at £400 000, the
corporate NPV of the investment. (Where does the £100 000 of value for the new
shares come from? Naturally, from the future dividends that old shareholders now
must share with the new shareholders – and that the old shareholders would not
have been forced to share had they contributed the £100 000 of cash themselves for
the investment.) So it does not matter from where the money for the investment
comes; from either our forgone dividends or outside equity, the investment’s
corporate NPV is the wealth increase of the existing or ‘old’ shareholders. Table 2.1
provides a useful and coherent display of the above ideas.
Table 2.1 The Simple Corporation: changes in shareholder wealth due to
investment
Cash Value Net
If dividend is not paid:
Old shareholders −£100 000 +£500 000 +£400 000
New shareholders 0 0 0
If dividend is paid:
Old shareholders 0 +£400 000 +£400 000
New shareholders −£100 000 +£100 000 0
The important things to notice about this example, and the things that will be
essentially true about all corporate NPVs, are:
1. The total value of the Simple Corporation increases by the NPV of the invest-
ment plus what it cost to undertake it. The market value of the company
increases £500 000, the investment costs £100 000, and the NPV is £400 000.
2. The shareholders of Simple Corporation, when the financial market discovers
that the company is going to undertake the investment, experience a wealth in-
crease equal to the investment’s NPV. Corporate NPV is defined as the
difference between how much the value of the company increases in the market
as a result of an investment, and how much its investment costs.
3. The existing, or ‘old’, shareholders of the company get a wealth increase equal to
the investment’s NPV regardless of who contributes the money necessary to
undertake the investment. We saw that the old shareholders get the same
£400 000 wealth increase whether the £100 000 investment cost comes from
new equityholders (with whom future dividends must be shared, and who there-
fore share market value), or from a dividend that would have been paid to the
old shareholders but is instead used to make the investment.
Note in Table 2.1 that the amounts listed are the changes that will occur when
the investment is accepted. This concentration upon only changes induced by the
decision is a very important attribute of investment analyses. One manifestation in
Table 2.1 of this reliance upon only changes that are caused by the investment is the
indication that there is no change in the cash position of the old shareholders if the
dividend is paid to them (see the 0 in the cash column in that part of the table). One
might be tempted to argue that a +£100 000 amount is appropriate there since that
is the amount of cash that the shareholders will receive as the dividend. Remember,
however, that in the absence of the investment, the old shareholders would have
received that dividend anyway. Thus if they continue to receive the dividend, there
is no change in their cash flow as a result of the investment. It is only when the
dividend is used instead to finance the investment that there is a change in share-
holder cash, and that is the −£100 000 in the upper half of the table.
Let us try another example to see how well we have learned the lesson of NPV
and shareholder wealth. Suppose that you own one share of common equity in
Cheetah Autos Ltd, the manufacturer of a luxury line of cars. Like the Simple
Corporation, Cheetah only has equity capital claims outstanding.
Cheetah has evaluated its prospective return to selling a convertible model as
having a positive NPV of £3 500 000. The necessary cash outlays associated with
assembly-line and other changes associated with the new model will total
£10 000 000. The company can either raise that money from new equityholders
outside the corporation or use the cash that it was going to pay as a dividend to its
old shareholders in this period. Cheetah currently has 1 000 000 shares outstanding,
and their market price immediately before the project is announced is £100 apiece.
Thus Cheetah before (or ‘without’) the investment is worth £100 000 000, and the
investment will add £13 500 000 to the market value of the company, but will cost
£10 000 000, producing an NPV of £3 500 000. If Cheetah undertakes the project
and if the market agrees with the opinion of profitability that Cheetah managers
hold, the share value of Cheetah will immediately go up by an amount that will
produce a total wealth increase of £3 500 000 for the existing shareholders of
Cheetah, and thus of £3.50 for you who own one of Cheetah’s 1 000 000 shares.
Suppose now that the company decides to raise the money for the project by
cancelling this period’s dividend. That means you will be losing the £10 per share
that you would have received as your dividend this period (the £10 000 000 cost of
the project divided by the 1 000 000 shares outstanding). But the market will value
Cheetah as being worth £113 500 000, £13 500 000 more than the £100 000 000 it
was worth without the project. And each of the 1 000 000 shares will therefore
increase in value from £100 to £113.50. Thus if the project is accepted and financed
this way, you lose £10 per share in dividends, but gain £13.50 in market value, a net
gain of £3.50 per share. With 1 000 000 shares, the £3 500 000 NPV of the project is
attained.
Should Cheetah decide to raise the money from new equityholders, the company
would sell them £10 000 000 of newly issued shares upon the announcement of the
project. All of the shares of the company will at that point have a total worth of
£113 500 000, so the new shareholders’ £10 000 000 comprises that amount of total
value. This means that the original shareholders will own the rest, which is
£113 500 000 − £10 000 000 = £103 500 000. With 1 000 000 old shares outstanding,
the share price of the old equity is £103.50, so the old equityholders again get a
wealth increase of £3.50 per share, which is NPV per share. (Since new equityhold-
ers provide the cash for the investment, the old equityholders simply receive the
dividend that they would get without the project. Thus though your share price is
£10 lower than had you forgone the dividend, your cash in hand is £10 greater, so
you are exactly as well off.) Again, regardless of the way the company gets the
The Cheetah example is admittedly slightly more complicated than the one im-
mediately before it, but it is useful because it shows that these ideas can also be used
in realistic contexts. To test your understanding of this example, suppose that
Cheetah decided to pay the dividend but instead of raising the cash for the project
from new shareholders it intended to raise it from the old shareholders. We can
assert in these circumstances that the old shareholders would again get the
£3 500 000 NPV, but here they would receive the dividend with one hand and with
the other immediately pay into the company the same amount of cash, so that their
change in cash is −£10 000 000 but they now own the entire value increase of
£13 500 000, so that NPV is again £3 500 000. If this assertion troubles you,
Table 2.3 may be helpful.
Here the old shareholders would simply hold the shares that new shareholders
would have held, and thus have the values of those shares, but would have paid
£10 000 000 for them. Again, however, the lesson is the same: the old shareholders
get the project’s NPV.
equity corporations also hold for companies that, in addition to having equityhold-
ers, borrow money from debt capital suppliers.
Suppose that Lynx Autos plc is a (fictitious) corporation identical to Cheetah
except that Lynx occasionally borrows money instead of raising all necessary cash
for investments from equityholders. Suppose, further, that Lynx faces an investment
identical to Cheetah’s in that the Lynx investment requires an outlay of £10 000 000
and, if accepted, will increase the total value of the company by £13 500 000 for a
corporate NPV of £3 500 000. But here the financing of the investment will be a bit
different from how it was with Cheetah: Lynx intends to borrow £5 400 000 of the
£10 000 000 necessary for the investment, and reduce its dividend by £4 600 000 to
get the rest of the cash.
What will happen to the wealth of the shareholders of Lynx if the company un-
dertakes the investment by borrowing in addition to delaying a dividend? The
shareholders will get the same wealth increase regardless. The only requirement is
that the new bondholders or creditors of Lynx are as intelligent as were the new
equityholders of Cheetah. You remember that Cheetah’s new shareholders would
give the £10 000 000 to the company only if they got £10 000 000 in share value as
compensation. The same idea applies to the new bondholders of Lynx: they will
provide the £5 400 000 in cash to the company if they, in return, receive £5 400 000
in bond value from Lynx.
Consequently, when the market value of Lynx increases by £13 500 000 as it
undertakes the investment, £5 400 000 of that market value increase is the value of
the new bonds that Lynx has issued to finance the investment. If the total value
increase of the company is £13 500 000, and the company’s debt value has increased
by £5 400 000, Lynx’s equity value must have increased by the difference,
£8 100 000. Thus the equityholders of Lynx are richer by the £8 100 000 share value
increase, but they have lost £4 600 000 in dividends that they would have had were
the project to have been rejected. The net of those two amounts is the change in the
wealth of Lynx’s equityholders. Of course that wealth increase is equal to
£3 500 000, the corporate NPV.
Table 2.4 illustrates these value and NPV results for Lynx. So again, corporate
NPV is equal to the change in the wealth of existing shareholders – even if some of
the money for the investment comes from creditors instead of equityholders.2
2 There is actually one other important condition for the Lynx example to work correctly, but we did not
wish to distract you with it during the discussion. If Lynx has old bonds outstanding, the investment
must not be allowed to increase their value. The reason is not too hard to see. If £5 400 000 of the
£13 500 000 value increase goes to new bondholders and £4 600 000 of equity dividends are invested,
any increase at all in the value of old bonds means that there will not be enough corporate value left to
generate £3 500 000 of NPV for the equity holders of Lynx. How could an investment cause the value
of old borrowings of the company to increase? Lots of ways: for example, if the investment made Lynx
a less risky company as a whole, the discount rates on the old bonds would decline and the market
prices of the old bonds would increase.
Table 2.4 Lynx Autos plc: changes in shareholder wealth due to in-
vestment
Cash Value Net
New bondholders −£5 400 000 +£5 400 000 0
Old shareholders −£4 600 000 +£8 100 000 +£3 500 000
Suppose now that Lynx borrowed the money as planned, but raised the remain-
ing £4 600 000 necessary for the investment by issuing new shares instead of
reducing its dividend. You should be able to work out that the old shareholders will
still gain the £3 500 000 NPV because the total value of the company would increase
by £13 500 000, of which £5 400 000 would belong to new bondholders, £4 600 000
to new shareholders, and thus £3 500 000 to old shareholders, who themselves put
up none of the cash for the investment. Thus their wealth has increased by the full
£3 500 000 NPV of the investment.
So the Cheetah and Lynx examples have shown us that equityholders get the
NPV of an investment regardless of the source of the money to undertake the
investment, be it forgone dividends, the selling of new shares, or the borrowing of
money from bondholders. The economics is easy to see: an investment will generate
£X of market value, and costs £ Y. If we raise £Y to undertake the investment, the
new capital suppliers will get £Y of the £ X value increase, leaving £X − £ Y = NPV
for the equityholders of the company. Why do the equityholders get to keep the
difference? Because, as we saw above, they are the residual capital claimants of the
company; they get to keep whatever is left after all other claimants’ contracts have
been honoured.
The lessons of Module 2 to this point are important ones because they allow us
to connect the actions of the company to the wealth of its shareholders. We have
found that:
1. The corporation is an organisation that has many potential claimants against its
future cash flows, all of these claimants being protected with contracts that spec-
ify timing and amounts of money to be paid, except for the claimants called
equity or shareholders. These equity capital suppliers have delegated their owner-
ship authority in the company to the managers and board of directors, retaining
only a residual claim and the capacity to vote for the directors. A residual claim is
simply the right to claim ownership of the resources of the company after all
other claims – capital and otherwise – have been satisfied.
2. Shareholders in a corporation usually have limited liability. This means that the
shareholders are at risk only for the value of their holdings in the company, and
even though they are owners in one sense, they are not personally liable for the
actions of the corporation.
3. When a company undertakes an investment, the investment will have an NPV
that can be calculated with a technique very similar to the one we used in Mod-
ule 1. Because of the residual nature of their claim, the shareholders of the
company will experience the corporate investment NPV as their increase in
wealth. Shareholders, being residual claimants, get nothing until the other capital
claimants’ contracts are fulfilled. But after those contracts are fulfilled, share-
holders get everything. A corporate investment NPV is the present value of the
amounts that are left over after all such contracts associated with the investment
have been fulfilled, and that NPV is thus the increase in wealth of the company’s
shareholders.
As a final note, you may have noticed we illustrated the investment analyses and
examples in this section with NPV instead of IRR techniques. Our reason for doing
that was not that we have forgotten about IRR; the simple truth is that the connec-
tions between investment decisions and shareholder wealth are easier to
demonstrate with NPV than with IRR. At this point we stand on the conclusion
that we can infer from our discussions in Module 1: a properly performed IRR
analysis will usually produce the same answers as a properly performed NPV
analysis, and so a company correctly using IRR to evaluate and accept an investment
will produce a wealth increase for its shareholders identical to that in a correctly
formulated NPV analysis.
Let us return to Cheetah Autos Ltd. As we saw in the previous section, Cheetah’s
market value (before the investment) is £100 per share, which we can take to be the
discounted present value of a perpetual stream of future cash dividends of £10. If
we assume that Cheetah’s earnings are also £10 per share, its earnings and its
dividends are the same. This implies that Cheetah is expected to pay out all of its
earnings as dividends. Using the Module 1 constant perpetuity formula for varying
Cheetah on a per-share basis, and re once again as the equity discount rate, we have:
Dividend per share
Price per share
£10
0.10
£100
If dividends are equal to earnings, we can manipulate these a bit to see that:
Earnings per share
Price per share
Price per share 1
Earnings per share
For Cheetah, the P/E ratio is the reciprocal of the equity discount rate. So there
is some type of relationship between the P/E ratio and the discounting procedure
that the market is using. But so far we have found this relationship only for compa-
nies like Cheetah, that are expected to pay out all of earnings as dividends and
produce a constant, perpetual stream of future dividends. What if we are interested
in a company that does not fit this mould? What does its P/E ratio tell us? Does it
bear a nice relationship to the company’s equity discount rate or not?
We can begin to answer these questions by supposing that conversation turns to
Ocelot Autos Ltd, a manufacturer of a broad range of medium-priced cars and
trucks. Ocelot is expected also to earn £10 per share this year, and has, like Cheetah,
a market price of £100 per share. Ocelot, however, is going to pay out only £7 of its
earnings in dividends. Ocelot thus has the same P/E ratio as Cheetah (Ocelot’s
earnings and market price are the same as Cheetah’s), but the market must be
valuing a different stream of future dividends for Ocelot, because it is paying out
only £7 per share in contrast to Cheetah’s £10 per share. Suppose that the market
expects Ocelot’s future dividends to grow at a constant rate of 3 per cent per period
for ever, and also considers Ocelot to have the same risk as Cheetah (so the same
value of re would apply). Remembering our lesson on growth perpetuities from
Module 1, we can state that Ocelot’s market valuation is being generated thus:
Dividend per share
Price per share
£7
0.10 0.03
£100
What about the relationship between Ocelot’s P/E ratio and its discount rate?
Since the discount rate applies only to dividends, and the P/E ratio to earnings, and
here the two are not the same, we must first connect earnings and dividends in
some manner. The way we usually do this in finance is by something called the
payout ratio. The payout ratio is simply the proportion of a company’s earnings
that it pays as dividends. If Ocelot is expected to pay a constant 70 per cent of its
earnings as dividends, we can find the relationship between the price/earnings ratio
and the discount rate by remembering that dividends are the result of multiplying
the earnings by the payout ratio:
Dividend per share
Price per share
Earnings per share Payout ratio
Price per share 1
Payout ratio
Earnings per share
As the above formula shows, the interpretation of Ocelot’s P/E ratio is not as
simple as the one for Cheetah. The P/E ratio here is influenced not only by
Ocelot’s discount rate, but also by the rate of growth expected for the company’s
future dividends and the proportion of its earnings that it will pay as dividends. (Of
course Cheetah’s P/E ratio is subject to the same general relationship, and happens
to be reflecting an expectation of a 100 per cent payout ratio and a dividend growth
rate of 0 per cent.)
Though Cheetah and Ocelot have the same initial earnings (£10), the same mar-
ket price per share (£100) and thus the same P/E ratio, they are quite different in
terms of the stream of dividends, associated rates of growth in dividends and
earnings, and the extent to which they pay out earnings as dividends. Obviously, one
could make a significant mistake about the implied discount rates, rates of growth in
income or dividends, or payout ratios of these two companies by examining only
their price/earnings ratios.
Even so, Cheetah and Ocelot are fairly similar companies compared with those
we might pick in a random draw among all companies. They are both perpetuities,
and they are equally risky, and thus have the same equity discount rates.
Suppose that the conversation now turned to the shares of Win-or-Lose Mining
Ltd. Win-or-Lose is expected to last for only a few more years, at which time its ore
will have been exhausted. Win-or-Lose is also substantially more risky than Cheetah
or Ocelot, such that the market requires a 20 per cent per period discount rate on
Win-or-Lose’s equity. Suppose that Win-or-Lose was expected to pay all of its
earnings as dividends of £10, £107, and £30 per share for the next three periods,
and then disappear. The market’s valuation of Win-or-Lose is thus:
£10 £107 £30
Price per share £100
1.20 1.20 1.20
Win-or-Lose has a market price of £100 per share, the same as Cheetah and
Ocelot, and Win-or-Lose also has the same P/E ratio, 10, as the two car companies.
But Win-or-Lose otherwise bears almost no similarity to the others. It is more risky,
it is expected to last only a few more periods, and it is a mining company rather than
a car manufacturer. Its P/E ratio has no simple interpretation and is a very strange
function of the discount rate, the number of periods in the remaining life of the
company, and the pattern of dividends that the market is expecting Win-or-Lose to
pay.
What are we to make of all this? The lesson is one of caution in using or attach-
ing much importance to the P/E ratio of a company. If the company is a constant
perpetuity, the interpretation of its P/E ratio is straightforward. But how many
companies do we encounter that are expected to pay the same dividend each year
for ever? Precious few. We do not see many constant growth perpetuity companies
either.
Does this mean that we can find no use for the P/E ratio of a company? No,
there are some uses to which that number can be put. For example, suppose that a
question arises as to which of two companies’ shares are riskier, and you are
reasonably sure that the companies are very similar in all other attributes. Take the
Lose-or-Win Mining Co., for instance. If it has the same dividend expectations as
Win-or-Lose but a P/E ratio of 5 instead of 10, that tells us that the market is
assigning a much higher discount rate to Lose-or-Win, and thus the company must
be more risky than Win-or-Lose. Or if Leopard Motors was seemingly identical to
Cheetah in risk and length of dividend stream and paid the same initial dividend as
Cheetah, but had a lower P/E ratio, the implication is that Leopard’s dividends are
expected to decline in the future.
The basic difficulty with the P/E ratio is that one of its components, the market
price of a company’s shares, is the net result of a great number of factors, only one
of which is the initial period’s earnings that appear in the numerator of the ratio.
Thus, unless we can feel comfortable that all but one of the factors influencing
market price are reasonably similar among companies being compared, the P/E
ratio can give little conclusive or useful information about the market’s relative
opinions about the prospects for such companies.
To summarise what we have learned about P/E ratios:
1. The P/E ratio is nothing more nor less than the ratio between the present value
of all the company’s future dividends (its market price) and its expected earnings
during the first period.
2. A company’s P/E ratio is a very complex number in terms of the information
that can influence it. It is affected by the pattern of dividends that a company
pays, its payout ratio, the riskiness of the company as evidenced by the discount
rate of its equity, and the stream of earnings that the company is expected to be
able to generate across the future.
3. For certain companies with very simple cash-flow patterns (such as constant or
constant growth perpetuities), we can derive a specific relationship between P/E
ratios and equity discount rates. But for most companies the relationship is much
too complex to make numerical estimation worthwhile.
4. If we are careful to limit the comparisons that we make (for example, staying
within the same industry so that the risk differences are minimised), we may be
able to examine P/E ratios so as to gain some qualitative information as to the
market’s opinion about the companies whose ratios we examine. For example,
we might infer that future growth rates expected for a company’s dividends and
earnings are above average if its P/E ratio is higher than companies in the same
industry (which companies therefore have the same risk and perhaps similar
payout ratios).
Learning Summary
This module has introduced you to the company investment decision process in the
following ways:
1. It has presented the general economic and legal basis for the corporate form of
organisation, the types of capital claims (debt and equity) which these entities can
issue, and some of the important characteristics of these claims.
2. It has shown that the prime purpose for financial decisions is the wealth of
currently existing shareholders, measured as the market value of their equity
claims.
3. It has argued that the best way to think of the market value of equity is as the
discounted value of all future dividends that current shareholders can expect to
receive.
4. It has demonstrated that calculating the NPV of an investment project is a good
way to estimate the effect upon shareholder wealth of the company accepting
the investment, regardless of the source of the funding for the investment.
5. It has discussed the weaknesses and strengths of the price/earnings ratio as an
indicator of the capital market’s opinion about the prospects for a company’s
future performance.
These ideas are important groundwork for the more applied investment analyses
of company projects that we shall present in following modules, and so the ideas
should be studied carefully. You will find that the review questions that follow are a
significant aid in your developing a good understanding of Module 2.
Review Questions
2.1 Financial decisions of companies should attempt to maximise the wealth of shareholders
because of which of the following?
A. All other claimants are protected by formal or informal contracts requiring
specific performance by the company.
B. Company financial managers usually own shares themselves, and would thereby
make themselves wealthy.
C. It is a way for companies to avoid their social responsibilities of looking to the
interests of consumers, the poor and the environment.
D. They are constrained to do so by the equity contract.
2.2 You are considering the purchase of a common share of Perpetual Payments plc which
is expected to continue to pay dividends for the indeterminate future. Those dividends
are expected to be £10 per share at the (end of the) first year, and you are convinced
that you will be able to sell the shares at that time for £210. If the company is expected
to pay a steadily increasing dividend each year, and its equity discount rate, re, is 10 per
cent, what rate of annual increase are you imputing to the dividends of Perpetual
Payments?
A. 10%.
B. No particular rate, simply the capacity to sell the shares at that price.
C. 5%.
D. The question is impossible to answer with the information given.
2.3 Suppose that Consumer Products SA is considering the introduction of a new line of
floor polish. The project will cost the company £2 000 000 in the first year, but is
expected to increase the total value of all of its capital claims by £3 000 000 at the same
time, as a result of the market’s evaluation of the alterations thereby caused in the
future cash flows of the company. Then:
I. The NPV of the project is £1 000 000 only if the project is financed totally by a
dividend reduction.
II. The NPV of the project is £1 000 000 if the project is financed either with new
equity or a dividend reduction.
III. The NPV of the project is £1 000 000 regardless of how it is financed, even with debt
in whole or part.
Which of the following is correct?
A. I only.
B. II only.
C. III only.
D. Both II and III.
2.4 ‘I notice that the market must have a very good opinion of our future prospects,’ says
your boss, glancing at the Stock Exchange quotations one morning. ‘Our price/earnings
ratio was 10 yesterday, whereas the average for all companies on the exchange was only
7. We must be doing something right – right?’
The correct response to the question is:
A. ‘Yes, because the ratio implies that our rate of increase in future cash flow is
above average.’
B. ‘No, because a higher ratio implies that the discount rate applicable to our
future cash flows is higher and thus riskier than the average.’
C. ‘Yes, because the higher ratio implies that the market thinks we are either less
risky or have higher increases expected for cash flow.’
D. ‘No, because such comparisons must be made with companies in essentially the
same line of business with ourselves, not market averages.’
Learning Objectives
Modules 1 and 2 have demonstrated that value calculations are the most basic and
important of processes in financial decision making, and that cash-flow expectations
are the most vital part of such value calculations. The corporate form of organisa-
tion presents complexities in estimation of cash flows for valuation processes. This
module instructs the student in the relationships between accounting measures of
performance (such as ‘profits’) and the financial cash flows necessary for valuation
exercises. Students learn here also the correct treatment and interpretation of certain
non-cash accounting numbers (such as ‘depreciation’) and the effect of such
numbers on company cash flows. The module contains several examples of the
correct interpretation of financial statement results of companies investigating
important investment decisions. Again, the context of these discussions is that for
company investment and financing decisions.
3.1 Introduction
We now return to the main concern, the investment-decision process of companies.
We have thus far learned certain things about this process:
1. Corporations should attempt to maximise the wealth of their existing sharehold-
ers.
2. Corporate investment NPV is the change that occurs in the wealth of existing
shareholders upon the corporation accepting an investment.
3. Corporate investment NPV is the difference between the increase in the market
value of all capital claims of the corporation (both equity and debt) minus the
present cost of the investment.
This section begins our more detailed study of exactly how corporations should
calculate investment NPVs (and IRRs) so as to make them consistent with these
points. And learning point (3) immediately above is an excellent clue to the proper
manner of calculating an NPV for a company’s investment. We must first calculate
the total corporate value change (defined as the increase or decrease in the market
value of all of a corporation’s capital claims) that would take place if the investment
were accepted. Next we subtract the cost of the investment. The result is the
investment NPV.
How do we arrive at the total value change in the capital claims of the corpora-
tion? However we do it, we must be consistent with the principles of value in
financial markets that we learned about in Module 1. You recall we established there
that market value is the discounted value of all future cash flows expected to be paid
to the security. The idea that the market value of a company’s equity is the present
value of its future dividends is merely an application of that principle.
The clear implication is that in order to find the changes that will occur in the
market values of all capital claims of a corporation as a result of an investment, we
must discount to the present all of the investment-induced changes in the cash that
capital suppliers can expect to get from the corporation in the future. In this module
we shall discuss and illustrate the cash flows that should be of interest in the
corporate investment decision.
Effort
Goods & services £ £ Taxes £ Subsidies
£
THE COMPANY
£ Invested Assets/services £
CAPITAL SUPPLIERS OF
SUPPLIERS ASSETS/SERVICES
£ Capital service
(dividends, interest, etc)
machines, land, expert advice, etc., which the company uses in its activities); and the
capital suppliers of the company who, as we know, give cash to the company in
exchange for dividends, interest and other future cash distributions.
You might question why we go to the trouble to show all of these other entities
when we should be interested only in the cash flows to the capital suppliers of the
corporation. The answer is that capital-supplier cash flows are residual to the other
groups. We would know very little about capital-supplier cash flows were we to
ignore those to whom the debt and equityholders must defer when it comes time to
distribute cash.
At this point the corporation should appear in your mind as an entity accepting
assets and services from employees and other vendors and paying for those with
cash, somehow or other operating those assets to produce goods or services that it
provides to customers who in turn pay for them with cash, paying taxes and
receiving subsidies from government, and raising cash from capital suppliers to
acquire productive capacity while servicing those capital claims with cash interest
payments, principal payments and dividends. The corporation relies upon the capital
suppliers to provide the cash necessary for it to accomplish its productive plans,
such as investment in assets to expand, enter a new line of business, or even simply
to keep its head above water for one more period.
‘Very nice,’ you say. ‘Figure 3.1 seems to have covered pretty well all of a compa-
ny’s transactions affecting the net amounts of cash that capital suppliers might
expect to get. But I do have a significant problem with your description of these
transactions being done for cash,’ you complain. ‘Take, for example, the company
buying raw materials for something it intends to produce. In my experience,
companies rarely or never actually pay cash when they get those materials. The usual
practice is to take some time to pay. And customers! Good heavens, in my company
we wait for what seems interminable periods to get cash for what we sell. Your
model of the company seems to be something less than realistic in that dimension.’
It is true that companies rarely pay cash ‘on the spot’ for the assets or services
that they buy, and customers often take some time to pay their bills. But that is not a
problem with the model of the corporation in Figure 3.1. Keep in mind that these
transactions are an ongoing process for corporations, and are continuously taking
place. Figure 3.1 should be taken to represent activity across time rather than at a
single time point.
Let us see how this works in terms of the concerns raised in your question. You
argue that something the company sells right now does not result in a cash payment
from a customer until next period. Fine. But remember that customers will be
paying this period for some of the things they bought last period. Similarly,
although the company may not be paying this period for its purchases this period, it
almost certainly will be expecting to be paying for what it bought earlier. Figure 3.1
makes no claim to be associating at a single time point particular cash flows with
particular services or assets provided. It simply says that companies receive cash
from, and pay cash out to, various groups during each period. And since our basic
concern is how much cash capital suppliers can expect to get across time, associa-
tions between particular flows in and out are at this point irrelevant.
‘I still do not feel comfortable with that explanation of corporate cash flows,’ you
declare. ‘I know for a fact that companies go to a lot of trouble to make exactly the
kind of associations between what they sell, who bought it, and when it is paid for,
that you say are irrelevant. Companies even have an asset called “accounts receiva-
ble” or “debtors” that appears in their books to record just such things. Are you
saying that companies are wasting their efforts in keeping track of such things?’ Not
at all. (As a matter of fact, companies even have an account for the services and
assets they have received but not yet paid for, called ‘creditors’ or ‘accounts paya-
ble’.) One of the very important managerial tasks of the company is to ensure
efficient handling of what it is owed and what it owes. Actually, the very infor-
mation in those accounts is one of the important inputs to estimating the amounts
of cash that a corporation can expect to pay and be paid across time.
But it would be wrong to record a cash transaction as taking place before the
cash was actually expected to change hands. Suppose that Cheetah Autos Ltd
recorded a cash inflow every time it sold a car to a dealer, even though the dealers
took a few time periods to remit the cash. If Cheetah tried to use that ‘cash’ to pay
interest or dividends to its capital suppliers it would likely soon run into trouble.
Similarly, if Cheetah recorded cash as having been paid whenever it bought a
machine or raw materials, even though the company was not expected to pay the
cash for some time, there would likely be money lying unspent that could have been
used to service capital claims (and increase the wealth of equity) that was not. So
financial cash flows are the cash amounts that are expected to occur at the times
for which the expectations are recorded. They are not to be confused with the
accounting numbers that are generated for quite different purposes.
It is now time for us to construct an example that illustrates these points about
corporate cash flows. Let us return to Cheetah Autos Ltd. Recall that in our
discussions of that company in Module 2 we mentioned an investment consisting of
Cheetah’s return to selling a convertible model. The investment cash outlay was to
be £10 000 000 and the investment had an NPV of £3 500 000. We shall now take a
closer look at that investment, paying particular attention to the cash flows that go
to make up its NPV.
Suppose that you are the managing director of Cheetah, and you ask to see more
detail from the finance department about the convertible project. In particular, you
have just seen Figure 3.1, and therefore ask that the data be presented in a format
consistent with that view of the corporation. After a bit of computer reprogram-
ming, the finance department sends you the information in Table 3.1.
Table 3.1 Cheetah Autos Ltd: cash flows (£000s) for the convertible
project
Now Yr 1 Yr 2 Yr 3
Customers 0 +£17 500 +£23 500 +£4 000
Opera- 0 −£7 000 −£3 830 −£5 200
tions
Assets −£10 000 −£4 000 −£2 000 0
Govern- 0 −£4 000 −£8 085 +£5 600
ment
Capital* −£10 000 +£2 500 +£9 585 +£4 400
* Free cash flow
$ $ $
NPV $10 000 000
. . .
$3 500 000
‘Very nice,’ you say. ‘But I have a few questions about the numbers here. The
first line says “Customers”, is that what I usually call “Sales” or “Revenues”?’
The finance department responds, ‘No, it is not. The amounts you see recorded
in the first line are the actual amounts of cash that we expect to take in from the
sales of cars in the first two years, and the third year includes both cash receipts
from car sales and also the selling of some of the machinery that would at that time
no longer be useful. The “Revenue” figures that our accountants will record for tax
and reporting purposes are a bit different because they recognise the sale of a car
when we ship it to a dealer, even though it has not yet been paid for.’
‘OK,’ you agree, ‘I remember having earlier heard something about that. How
about the duration of the cash flows across time? Why do they stop at the third
year?’
‘Well, Sir,’ the analysts say, ‘Our marketing department tells us that the converti-
ble will be quite popular when we first introduce it, but because it is a speciality car
its appeal will be quite finite, and by the end of the third year its market will have all
but disappeared. At that point we are planning to get into something else. That is
another reason why the third year’s cash flow includes some money from the sale of
machines in addition to cars.’
‘Let’s move on to the line called “Operations”. Can I assume that the cash out-
lays listed there are the expenses that the company will incur in the production and
sale of cars?’ you enquire.
‘In a sense that is true,’ reply your finance people. ‘But you must be careful about
the word “expenses”. That term has a very specific meaning to accountants and tax
experts. What we call “Operations” cash outlays is not exactly the same thing as
what the accountants call “expenses”. Actually, everything in our “Operations” line
would also be an expense to an accountant. There we record employee costs, fuel
and electricity, and so forth. Our rule for “Operations” cash flows is that they be:
1. paid in cash that year, and
2. deductible for taxes that year, and
‘To answer your second question, the reason we do not include interest payments
in “Operations” cash flow is because interest is part of what we are trying to show
as a net result: the amounts of cash that capital suppliers can extract without
disturbing Cheetah’s plans. If we were to include interest in “Operations” cash flow,
it would be subtracted from the total but in fact should be part of it. (As you are
aware, Cheetah currently has no borrowings, and we plan none for this project, so
we actually had no interest amounts to worry about anyway. But if we had, they
would not have been part of our “Operations” cash flow.)’
‘Tell me about the line called “Capital”,’ you ask. ‘It is obviously the net total of
the lines above it for each time point, but what is its meaning beyond that?’
‘The “Capital” line is the amounts of cash that could be taken out of the corpora-
tion by its capital suppliers and still have the convertible project run as planned,’ a
finance analyst responds. ‘It is the amounts of cash that are expected to be left over
after all necessary cash amounts have been paid. Note that it is negative at the first
time point when the big cash outlay for “Assets” is made, and then becomes
positive in future years. The negative £10 000 000 is the recognition that we plan to
cut our equity dividend by that amount at the point we begin the project, to pay for
the initial “Assets”. Because we are financed only by equity at Cheetah, you can
regard the “Capital” cash flows in future years as the increases in dividends that we
expect the project to allow.’
‘I think I am beginning to understand,’ you say. ‘Am I correct in interpreting the
NPV you have calculated in the report as the present value of all increases and
decreases in dividends that we expect to happen if we accept the convertible
project?’
‘Exactly!’ exclaim your finance people. ‘You can see that this is a good measure
of how that project will affect our equityholders’ wealth, because they will lose the
£10 000 000 dividend this period but at the same time their share values will increase
by the present value of the amounts that their future dividends are expected to
increase. The net of those two things is the investment’s NPV. The only caution
that we would give about this interpretation is that it works only for companies like
Cheetah, which are financed totally with equity and have issued no debt capital. For
companies with debt in their capital structure a similar, but not identical, explanation
of the genesis of NPV applies.’
‘It is nice to hear that I finally got something right,’ you declare. ‘Now if you tell
me that the discount rate of 10 per cent is our estimate of the rate of return that
would be required on the stream of “Capital” cash flows due to their risk, I shall be
doubly pleased.’
‘Correct again!’ the finance people congratulate. ‘As a matter of fact you can see
how we estimated that rate by reading Section 2.5 of Module 2 entitled “Share
Values and Price/Earnings Ratios” in a certain finance text that we shall provide
you. We figured the convertible project to have about the same risk as Cheetah in
general, so we used the rate that would apply to the company’s capital as a whole.’
‘I am feeling a lot better about what goes on in the finance department,’ you
offer. ‘Incidentally, what is that asterisk doing on the word “Capital” in the report?’
‘Well,’ an analyst replies, ‘you can see that it refers to a term entitled “Free cash
flow” as being synonymous with the “Capital” cash flow as we have defined and
calculated it. We put that reference there because it has a great deal of currency right
now, having been “discovered” by the investment bankers and security analysts who
make recommendations about companies’ share values. They think free cash flow is
some kind of new idea they have invented, but in fact we have been using that
calculation, without that name, for years. It is, as I have just said, the amount of
cash that can be taken by capital suppliers from the company as a result of
the investment while leaving all of the plans of the company unchanged.
“Free” in that sense means “unfettered” rather than “costless”; the free cash-flow
amounts do cost money (specifically, the negative free cash flow in the first period is
the “cost” of the positive ones that follow). The “free cash flow” term should stand
you in good stead in your many cocktail conversations with other managing
directors, and with the investment bankers, analysts and others who are always
seeking information about Cheetah. It stands us in good stead as the basis for
calculating NPVs of investment projects.’
‘Thank you,’ you congratulate. ‘You have helped me understand the convertible
project, and corporate investment analysis in general, much more deeply. I have one
more request. Would you send me another report on the convertible project, this
one arriving at the same free cash-flow figures but with the detail of taxes, deprecia-
tion and so forth? As long as I am being educated in finance, I suppose I should be
thorough.’
‘Right away, Sir,’ say your finance people as, with relief, they begin to exit.
‘Oh, and by the way,’ you catch them, ‘come up to my office about an hour after
you send the reports so that we can discuss them.’
‘We shall be delighted to,’ they lie.
Table 3.2 Cheetah Autos Ltd: income statements (£000s) for the
convertible project
Yr 1 Yr 2 Yr 3
Revenues £18 000 £25 000 £2 000
Expenses:
Operations 7 000 3 830 5 200
Depreciation 3 000 5 000 8 000
Interest 0 0 0
Total expenses 10 000 8 830 13 200
Profit before taxes 8 000 16 170 −11 200
Taxes (50%) 4 000 8 085 −5 600
Profit after taxes 4 000 8 085 −5 600
Table 3.3 Cheetah Autos Ltd: depreciation schedules (£000s) for the
convertible project
Yr 1 Yr 2 Yr 3
Due to asset outlay of:
£10 000 Yr 1 £3 000 £3 000 £4 000
£4 000 Yr 2 £2 000 £2 000
£2 000 Yr 3 £2 000
Total depreciation expense £3 000 £5 000 £8 000
To see where the items in the cash-flow statement came from, you begin com-
paring the items in the income and cash-flow statements. First, the differences
between the Revenue (Table 3.2) and Customer (Table 3.1) lines catch your atten-
tion. The Revenues in the first two years exceed the reported cash inflows from
customers in those years, but the opposite happens in the last year. Further, the total
of all Revenues equals the total of all Customer cash inflows. What you figure out is
this: in the first year, Cheetah expects to deliver £18 000 000 of cars to dealers but to
be paid for only £17 500 000 of the cars. The following year Cheetah sees itself
selling £25 000 000 of cars, and getting £23 500 000 of cash. Some of that cash will
be coming from the dealers who bought cars in the first year but are paying the cash
in year 2. Finally, in year 3, Cheetah sells £1 000 000 of cars, collects that plus the
£2 000 000 from prior-year sales that is still owed, and sells the assets of the
convertible project for £1 000 000, collecting the cash on the spot. For Cheetah, the
Revenue figures are not an inaccurate representation of the total amounts of cash
that the company will receive from the project (they would be inaccurate if, for
example, some dealers never did pay for their cars), but the Revenues do give a false
picture of the timings of the cash coming in from customers. And cash timing is
important to present values, and therefore to the present wealth of equityholders, as
you know.
Total expenses exceed total cash outlays for Operations by the amounts of De-
preciation expense, and they would also do so by the amount of interest if any were
expected to be paid. Total expenses are thus not themselves a cash measure, though
they do include some items of operating cash flow (those that are deductible).
Revenues and expenses are therefore different from cash flows, but by no means
are these accounting numbers unimportant. The source of their importance is that
Government uses the difference between them, called ‘taxable income’ or ‘profit
before tax’ to calculate the amount of tax that Cheetah must pay. And tax, of
course, is a cash flow. Thus you notice that the Government cash flow was figured
by your finance people as Cheetah’s corporate income tax rate (50 per cent)
multiplied by the amounts of profit before tax on the income statements for the
project. Profit after tax is simply the difference between profit before tax and the tax
amount.
Table 3.3 of depreciation calculations supplied by finance for the project next
draws your attention. You notice that the sum of all the depreciation expenses for
each asset is equal to the original cash outlay for the asset, as your discussion with
the analysts had predicted. A fast telephone conversation with the accounting
department informs you that for assets like these, Cheetah depreciates ‘straight line’
with ‘salvage value’ if applicable. This means that the original cash outlay is written
off as a depreciation expense in even amounts over the remaining life of the project,
except for the expected amounts that the asset may be sold for at the end of project
life. For the assets bought first, the original £10 000 000 is split into £9 000 000 of
‘depreciable value’ and £1 000 000 of ‘salvage value’. The £9 000 000 is written off at
£3 000 000 per year for three years, and the year-3 depreciation expense includes
that amount plus the writing-off of salvage value when the asset is expected to be
sold. The depreciation expenses for the other assets are easy to figure because they
have no salvage value. And the total depreciation expenses in each year are simply
the sum of those for each asset during that year.
Just then the finance group shows up for its meeting with you.
‘Welcome,’ you say. ‘This should be brief because, believe it or not, I seem to
understand almost everything about the genesis of the cash-flow figures for the
convertible project. I do have just a couple of questions, however. First, I notice
that in year 3 we are showing negative profits but positive cash flow. Now I can
understand how such a thing can happen if, as you predict, some of our customers
are not paying until that year even though the Revenue was recorded earlier; and I
do understand about Depreciation not being a cash flow even though it reduces
profits. But can you explain to me how we can convince the government to give us
£5 600 000 as negative taxes that year? I can see how the number is calculated, as
50 per cent of the negative “profit before taxes” number. But it is one thing to have
a number on a piece of paper, and quite another to get £5 600 000 in cash from the
government.’
‘The answer to that question points out one of the most important characteristics
of investment project cash flows,’ respond the finance people. ‘When we estimate
the cash flows of a proposal, we seek to include all of the changes that will take
place in the cash flows of the corporation as a whole. In this situation the cash flow
that we call “Government” is therefore the changes that we expect will occur in the
taxes that Cheetah will pay, should we accept the convertible project. The correct
interpretation of the negative tax amount in year 3 is that the company as a whole
will pay £5 600 000 less in corporate income taxes when the £11 200 000 negative
income number from the convertible project is combined with the other income
amounts that we expect Cheetah to earn that year.
‘In simpler words, if we accept the convertible project, Cheetah’s income for tax
purposes in year 3 will be lower by £11 200 000 than it would be if we reject the
project. That means that, at a 50 per cent tax rate, our corporate income taxes will
be less by one half the “loss”, or £5 600 000. This assumes that the company will
have enough taxable profits from other operations, or can take advantage of things
such as “carry-backs” of taxes (which set the negative income against some past
positive income) so as actually to get cash back from the government as a reduction
of taxes we paid earlier. There are all sorts of ways to get the “negative tax” cash
flow, but the important thing is the general idea about investment cash flows that it
illustrates: include all changes in the cash flows of the corporation as a whole were
it to accept the project, and include only those changes.’
‘That makes sense,’ you say. ‘Let me ask one last thing. I infer from the general
tone of your comments about accounting numbers such as depreciation expenses
and profits that you do not think very highly of the ideas that underlie accounting
estimates. Yet our entire information and management control system at Cheetah
(and most other companies) is based on accounting data. Is this something that I
should be concerned about?’
‘Now that,’ complains the senior finance analyst present, ‘is a tough question’. It
is undeniably true that for the types of decisions that we face in finance, cash flows
are more important than accounting numbers, which are not cash flows. But we
must always remember that particular cash flows are themselves functions of
accounting numbers. Our discussion of taxes a moment ago is a good example of
this. Taxes, a cash flow, are a function of “income” or “earnings” or “profit”, which
are accounting numbers. So, like it or not, we must use the data that the system of
“generally accepted accounting principles” produces.
‘That, however, is not the whole story. Without going into all of the reasoning
(and continuing argument within that profession), the information that accountants
produce in financial statements is by itself useful for certain other decisions that
companies make. Furthermore, accountants take very seriously their responsibility
of reliably reporting corporate results to the public and to government. That leads
them to use some measures that we in finance, not having to let outsiders see our
work, would not use. We are content to rely on our own “guesses” more than
accountants are.
‘You can see one illustration of this distinction between finance and accounting
by comparing our “free cash flow” figures for the convertible project to the “profit
after taxes” numbers in the income statements. The cash flows, as you know, are the
estimates of the amounts of money that the market will use to place a value on the
capital claims of Cheetah. And those amounts are significantly different at each time
point than the accounting profits. But notice that if we separately add up the free
cash flows and the profits after tax for all years of the project, they come to the
same amount. So the accountants are doing something similar to our cash-flow
calculation as they produce an earnings number, but the procedures they use cause
timing deviations from actual cash flows. In other situations there may also be
differences in the total amounts (particularly when companies have borrowed
money). You can doubtless figure the causes for the differences each period
between the accounting and cash-flow figures, but that is beside the point. Everyone
well educated in finance and accounting knows about those things and uses the
appropriate numbers for the decision at hand. For company investment decisions,
the correct numbers are the cash flows, not the accounting figures.
‘In sum, our answer to your question is yes, we do feel that accounting numbers
are not correct to use as substitutes for rigorous cash-flow estimates. However, that
is not to say that accounting numbers are useless. They serve an important function
in estimating particular cash flows, and are the primary way that corporations
communicate with the outside world. So in our earlier discussions we probably gave
you a misleadingly negative view of how finance people regard accounting numbers.
But make no mistake about our conviction that for the great range of financial
decisions it is cash flows, and only cash flows, that matter.’
‘Thank you again,’ you conclude, ‘You have, as usual, given me a lot to think
about.’
Learning Summary
When the finance group has exited, you summarise the major points that came up in
your discussion:
1. Investment NPV is correctly calculated by discounting to the present all of the
changes that will occur in the cash flows of the corporation as a whole, were it to
accept a project. The net of these amounts is called the free cash flow.
2. These corporate cash flows can be conveniently depicted at one level of generali-
ty by listing all of the outside groups with which the corporation transacts, as is
done in Figure 3.1.
3. Free cash flow is defined as the net amounts of cash that the company could pay
to its capital suppliers from the proceeds of a project at each time point without
upsetting the expectations associated with the project. This essentially means that
free cash flow is the amount that is expected to be left over (or ‘residual’) after
all commitments and contracts other than those to capital suppliers have been
satisfied according to the market’s expectations.
4. Cash flows are not the same as the numbers that appear in the financial state-
ments of corporations. Included in items such as Revenues and Expenses are
non-cash items that cause accounting numbers to differ from cash flows (chang-
es in debtors or accounts receivable, and creditors or accounts payable, and
depreciation are examples). There are also certain cash flows that are not includ-
ed in accounting figures at the times when they occur (such as outlays for
depreciable assets, and collections of receivables and payment of payables). If
accounting data are to be used as the basis for cash-flow estimates, all necessary
Review Questions
3.1 Figure 3.1 is a depiction of the basic idea underlying the calculation of what we have
called a company’s ‘free cash flow’. The transactions there closely resemble those
presented in a company’s profit and loss account or income statement generated by its
accountants. The income statement, however, is not generally applicable to the types of
financial decisions for which the ‘free cash flow’ concept is used. The reasons for that
are:
I. The numbers appearing in company income statements are not necessarily cash
flows, and the concept of present values (with associated market value and wealth
measures) requires that cash flows alone be used.
II. The numbers appearing in company income statements may be incorrectly located
with respect to the time occurrence of the actual cash flows associated with the
recorded transactions.
III. The above are too generous. The numbers appearing in company income statements
are wrong, and are developed under incorrect assumptions on the part of account-
ants as to what actually goes on in companies.
Which of the following is correct?
A. I only.
B. II only.
C. III only.
D. Both I and II.
3.2 ‘Our company sells office supplies, and it is often several weeks before we are paid by
customers for our sales to them. If, as you suggest in Module 3, we wait to record these
sales until the cash is received, we would, during that interim, be ignoring the fact that
we have a valuable asset, namely what our customers owe us.’ Which of the following is
correct in relation to this statement?
A. It misapplies the point of Module 3, that finance is more concerned with cash
flows than the recording of accounting numbers. Finance is unconcerned
whether you keep track of the amounts of money owed the company by
customers; it cares only about accurate recording of cash flows.
B. It mis-states the point of Module 3, that finance is more concerned with cash
flows than the recording of accounting numbers. Finance recognises that there
are quite legitimate reasons for keeping track of debtors or accounts receiva-
ble, and only insists that for financial decision-making purposes, the timings and
amounts of cash flows are the important measures.
C. It is a correct application of the point of Module 3.
D. It is an incorrect indictment of the point of Module 3, that cash flows are
important. Because financial cash flows will be presented simultaneously for
several periods of time, the eventual payment of receivables by customers will
appear in the cash-flow statements. Therefore it is unnecessary for the
company to worry about accounting numbers such as the assets mentioned.
3.3 ‘Your point about depreciation expenses in Module 3 continues to disturb me. I have
grown fond of the argument used by accountants to validate the use of depreciation
expenses, namely that the charging of such expenses across time, instead of instantane-
ously, recognises appropriately the deterioration in usefulness or value of the asset in
question. Your insistence in finance that all of the cash outlay be charged at the outset
of the investment seems to defeat this worthwhile perspective of depreciation expens-
es.’ As a result:
I. It is true that the cash-flow approach of finance does not attempt to recognise the
period-by-period deterioration of assets over time, but simply what is paid for them.
II. It is true that the cash-flow approach of finance does not attempt to recognise the
period-by-period deterioration of assets over time, but the complete set of cash
flows used by finance would include the future sale and any future cash maintenance
costs of the asset and thus accomplish the same effect, but in cash-flow terms, which
accountants produce with their depreciation expenses.
III. It is true that the cash-flow approach of finance does not attempt to recognise the
period-by-period deterioration of assets over time, because that is irrelevant to any
foreseeable financial decision.
Which of the following is correct?
A. I only.
B. II only.
C. III only.
D. Both II and III.
3.4 Suppose that your company was contemplating the introduction of a new product.
Furthermore, suppose sale of that new product would likely not produce positive
accounting income for a number of periods. During that time, what is the correct
treatment for cash-flow purposes of the taxes that will be associated with the new
product?
A. Taxes should be ignored, because the company will not be earning any profits
from the new product and therefore will not be paying any new taxes.
B. Taxes should be levied against the product, because even though it is causing
no tax liabilities now, it will in the future when it turns profitable. We must
therefore recognise the need to cover those taxes in the future.
C. Taxes should not be levied against the product, because the accounting losses
reduce the taxes the company must pay, thus offsetting the future tax liabilities
that the product will generate when it turns profitable.
D. Negative taxes should be recorded as cash inflows to the company during the
time when the product will be generating accounting losses.
3.5 Which of the following is the correct interpretation of the concept of ‘free cash flow’?
A. The amounts of money that the capital suppliers of the company can extract
from the company across time, without disturbing any of the plans or con-
straints under which the company is expected to operate.
B. The maximum amounts of money that the company can pay to its capital
suppliers at each future time.
C. The amounts of cash flow that can be considered to be free of legal encum-
brances.
D. The sum of expected interest, principal and dividends that the company is
expected to pay.
Learning Objectives
Module 4 extends the introduction to company investment decision making of
Module 3 to the more realistic context of companies with complex capital structures
(companies financed with both debt and equity capital). This is a two-step process:
first, the module describes necessary cash-flow estimates for such organisations (and
therein further explains the concept of ‘free cash flow’, the concept that has gained
such notoriety in merchant and investment banking realms). With complex capital
structures, good financial outcomes for companies imply adequate servicing of more
than a single claim, and the criterion by which financial judgments are made must
therefore be consistent with servicing all outstanding capital claims (both debt and
equity). The module next develops the weighted average cost of capital (WACC) as
such a criterion. Students will also encounter an important alternative to the WACC,
the adjusted present value (APV), and learn the applicabilities of, and relationships
between, APV and NPV.
4.1 Introduction
In this module we shall explore one of the more important concepts in finance, that
of a company’s weighted average cost of capital. It is important that you under-
stand this concept because it is a major component of the most widely used
techniques of investment decision making.
The weighted average cost of capital as an idea is straightforward. It is simply a
discount rate that combines the capital costs of all of the various types of capital
claims that a company issues. Thus it may include the costs of equity capital, debt
capital, and any other capital claims outstanding. The weighted average cost of
found that the £3 500 000 NPV of the Lynx project comes to equity as an
£8 100 000 increase in share value and a £4 600 000 decrease in dividends.
But upon exactly what cash flows is the financial market depending for its placing
of such values? Consistent with the principles we have developed, the market must
be discounting to the present the changes that will occur to the cash flows of all of
the capital suppliers of Lynx were it to accept the project.
As it turns out, we discover that the Lynx investment is also in a new model of
car, this one having a ‘dickey seat’ and intended to appeal to nostalgia buffs and
wealthy adolescents. The dickey-seat project is expected to produce exactly the same
cash flows and accounting results for Lynx as the convertible project does for
Cheetah, with the exception that Lynx must service its debt by paying interest.
Though the expected amounts are the same, the dickey-seat project’s cash-flow
expectations are riskier than the convertible project’s because of the uncertainty of
demand for the dickey seat’s anachronistic car-seating arrangement.
The borrowing that Lynx intends to use in the project financing has the following
characteristics:
1. Lynx borrows £5 400 000 now, and repays in instalments of £460 000 at the end
of the first year, £3 340 000 at the end of the second year and £1 600 000 at the
end of the third. This means that the amounts of debt outstanding during the
three periods are £5 400 000 for the first year, £4 940 000 for the second year,
and £1 600 000 during the third.
2. Lynx expects to pay 8 per cent interest on the amounts outstanding in each year.
Thus the interest payments are £432 000 for the first year, £395 200 for the sec-
ond and £128 000 for the third.
With this basic financing plan (along with the immediate one-time dividend re-
duction of £4 600 000 to make up the necessary £10 000 000 outlay), the income
statements for the project for Lynx appear in Table 4.1, along with the associated
cash flows to and from all entities with which Lynx is expected to transact in
Table 4.2.
Table 4.1 Lynx Autos plc: income statements (£000s) for the dickey-
seat project
Yr 1 Yr 2 Yr 3
Revenues £18 000 £25 000 £2 000
Expenses:
Operations 7 000 3 830 5 200
Depreciation 3 000 5 000 8 000
Interest 432 395.2 128
Total expenses 10 432 9 255.2 13 328
Profit before taxes 7 568 15 774.8 −11 328
Taxes (50%) 3 784 7 887.4 −5 664
Profit after taxes 3 784 7 887.4 −5 664
Table 4.2 Lynx Autos plc: cash flows (£000s) for the dickey-seat project
Now Yr 1 Yr 2 Yr 3
Customers 0 +£17 500 +£23 500 +£4 000
Operations 0 −£7 000 −£3 830 −£5 200
Assets −£10 000 −£4 000 −£2 000 0
Government 0 −£3 784 −£7 887.4 +£5 664
Capital* −£10 000 +£2 716 +£9 782.6 +£4 464
* Free cash flow
If we compare these statements with those of Cheetah in Table 3.1 and Table 3.2
in Module 3 you can see that the Revenue item on the income statements and the
Customer item in the cash flows hold no mystery because Lynx’s are identical to
those we discussed in Cheetah. The same holds true in comparing the Operations
items and Depreciation and Assets figures in the statements. But at that point
Lynx’s results begin to differ from those we saw for Cheetah.
Lynx’s income statements show an expense in each year for the interest that the
company expects to pay. This interest is deductible for taxes, so Lynx’s Profit before
taxes is less than Cheetah’s by the amount of the interest expectation in each year, as
are Lynx’s Tax item and Profit after tax item less than Cheetah’s. So, from the
perspective of Lynx’s income statements, the dickey-seat project shows lower
profits and taxes and higher expenses than does the convertible project for Cheetah,
and this is due solely to the interest payments that Lynx is expected to make.
The cash-flow statements for the two projects also begin to differ after the Cus-
tomer, Operations and Assets items. Those statements show that the essential cash-
flow difference between the convertible and dickey-seat projects is that Lynx is
expected to pay less taxes than Cheetah, and thus have more free cash flow to
distribute to capital suppliers. The reason for all of these differences is easy to see.
Tax laws allow corporations to reduce their taxes by deducting interest expense,
even though that expense is a payment to a capital supplier, just as are dividends.
But dividends are not a deductible expense. So companies that pay dividends to
equityholders instead of interest to debt suppliers pay more taxes than companies
that do pay interest.1
The Cheetah and Lynx projects are good examples of this. The two projects’
cash-flow amounts are identical to each other except that Lynx will distribute some
of its payment to capital as interest, and Cheetah, having no debt, will pay only
dividends to equityholders. Because Lynx’s interest is deductible. Lynx is expected
to have more cash to distribute to its capital suppliers than will Cheetah.
1 Some countries allow partial deductibility of dividends, but these deductibilities are generally not as
generous as those that are allowed for interest payments.
Table 4.3 Cheetah and Lynx projects: free cash flow (£000s)
Now Yr 1 Yr 2 Yr 3
Cheetah −£10 000 +£2 500 +£9 585 +£4 400
Lynx −£10 000 +£2 716 +£9 782.6 +£4 464
Difference 0 £216 £197.6 £64
The amounts by which Lynx’s free cash flow is expected to exceed Cheetah’s are
given in Table 4.3. These differences, as we have seen, are due to the lower amount
of taxes that Lynx is expected to pay. But these amounts also have a special name in
finance, signifying the source of the differences: the reduction in income taxes that a
corporation receives because of interest deductibility is called its ‘interest tax
shield’. Look at the set of Lynx’s interest tax shields in Table 4.4. We calculate these
shields by figuring the amounts of tax that Lynx would pay if it had no debt (like
Cheetah), and subtracting the amount of tax that Lynx will pay with the interest
deduction.
There is another way to calculate interest tax shields that is less cumbersome and
more to the point of the economic source of those shields. Since interest expense is
a reduction in taxable income, and since taxes are figured by taking a percentage of
the income number, interest tax shields can be found by simply multiplying the
expected interest payment in each period by the corporate income tax rate. For
example, Lynx is expected to pay £432 000 of interest in year 1. To find the interest
tax shield for that year we merely multiply £432 000 by the corporate income tax
rate of 50 per cent to arrive at the shield of £216 000. By multiplying Lynx’s interest
expenses from its income statement in Table 4.1 by the 50 per cent tax rate, you can
prove to yourself that the shields in Table 4.4 are correct.
‘All of that is most interesting, we suppose,’ you say. ‘But what significance does
it have? We already had the cash flows for the Lynx project in enough detail to find
its free cash flow in Table 4.2. And from what you told us before, that should be
enough to calculate the NPV of the project, which is our only concern. Why do we
need to be concerned about interest tax shields?’
The discussion of interest tax shields may at this point seem esoteric but its im-
portance to real-world financial decision making is undeniable. This concept is
intimately involved with the weighted average cost of capital.
Table 4.5 Lynx Autos plc: cash flows to debt and equity (£000s) for the
dickey-seat project
Now Yr 1 Yr 2 Yr 3
Free Cash −£10 000 +£2 716 +£9 782.6 +£4 464
Flow
Interest −432 −395.2 −128
Principal −5 400 −460 −3 340.0 −1 600
Equity −4 600 £1 824 £6 047.4 £2 736
Claim
Unlike the cash flows for Cheetah, which belonged to equity alone, Lynx’s free
cash flow is claimed by two types of capital suppliers: debt and equity. Since we
have been informed of the expectations for interest and principal payments to debt
suppliers, we can calculate the amounts of cash that equity can claim, as shown in
Table 4.5. With these cash flows, it is a simple matter to see the effect of the project
upon the wealth of the two types of claimholders. For debt, we know that an 8 per
cent return is required and so, by combining the interest and principal payments
from Table 4.5:
£892 000 £3 735 200 £1 728 000
Debt value
1.08 1.08 1.08
£5 400 000
The cash that debt suppliers expect to get in the future is equal in value to the
amount that is raised from them at present, namely £5 400 000, and so the debt
suppliers’ wealth is unaffected by the investment. From our study of the workings
of efficient financial markets in Module 1, this should not surprise you.
But how about equity? In order to figure its wealth change, we must compare the
investment-induced change in equity value to the forgone £4 600 000 dividend. To
calculate the value change we must have cash-flow expectations, which are given in
Table 4.5, and an appropriate discount rate, which is not. We obviously cannot use
the Cheetah convertible project’s equity rate, because we know that that project is of
lower risk and is purely equity financed. Let us assume that we refer the question to
our capital market experts, and they tell us that the operating and financing risks of
the dickey-seat project are such that the equity market would require a 14 per cent
return per period. That being the case, we can arrive at the change in Lynx’s equity
value due to the project:
£1 824 000 £6 047 400 £2 736 000
Equity value
1.14 1.14 1.14
£8 100 000
Thus Lynx’s shareholders will forego a £4 600 000 dividend and in return receive
an £8 100 000 increase in the market value of their shares as a result of the project.
The net of those two effects is the £3 500 000 NPV of the project.
Finding a project’s NPV by this process (estimating the changes in the amounts
of cash that equity will claim as a result of a corporate investment, and discounting
those to arrive at the expected present change in equity value) is a perfectly proper
technique. However, it is not one of the more commonly used techniques in the
modern company, for corporate financial decision makers are evidently more
comfortable with techniques that do not separate the cash flows into those going to
the various capital claims, but instead merely deal with the company’s cash flows as
a whole. These too can be perfectly acceptable techniques of investment analysis,
and will bring us finally to the notion of the weighted average cost of capital. We
shall now examine them.
Now Yr 1 Yr 2 Yr 3
−£10 000 +£2716 +£9782.6 +£4464
We wish to find the NPV of these cash flows. In order to do that correctly, we
must discount them with a rate that is commensurate with their risk. How do we
find such a rate? We want a rate that the market would apply to these cash flows
were they to be sold as a combined expectation, in other words, if the company sold
a security that had these cash flows as expectations, what rate would the market
apply to discount those flows?
We already know the discount rates that are necessary on these same cash flows
when they are separated into debt (8 per cent) and equity (14 per cent). To find the
discount rate appropriate for the combined cash flows we must therefore combine
the equity and debt discount rates. But careful here. We must combine the equity
and debt rates in the correct proportions, or the resulting corporate discount rate
will be incorrect.
What should the proportional combination be? We are seeking a market discount
rate for the total corporate cash flows by combining market discount rates for debt
and equity. Those rates for debt and equity are based on their proportional claims
upon the corporate cash flow. So we must combine the equity and debt rates in the
ratio of their claims on the corporate cash flow, and that ratio is given by the market
values of the two claims.
Think of the overall company rate this way: it is an average of the rates that all
capital claims require on the amounts they have invested in the project. And the
amounts they have invested are not necessarily the amounts of cash they paid into
the project but the market prices at which they could sell those claims. So the
overall project rate is a market-value weighted average of the rates required by the
various capital claims upon the investment.
For the Lynx project the rate is given as follows:
Debt market value
Overall rate Debt required rate
Total market value
Equity market value
Equity required rate
Total market value
£5 400 000 £8 100 000
0.08 014
£13 500 000 £13 500 000
0.116 11.6%
This tells us that the market would require an 11.6 per cent return on the dickey-seat
project’s free cash flow were it to be offered to the market as a single-security
expectation.
What would such a security be worth? Since we have its discount rate and its cash
flows, finding its value is child’s play:
Free cash flow
Value of the project ∑
1 Overall required rate
£2 716 000 £9 782 600 £4 464 000
1.116 1.116 1.116
£13 500 000
The investment is worth £13 500 000 if regarded as a single capital claim. It
should not surprise you that the project’s equity and debt values added together also
equal this amount. (If a participant for some reason wished to own such a claim as
this overall one, and it was not available, he could effectively create the claim by
buying both the debt and equity of the project. Therefore the total value of the
project must equal the sum of its constituent capital claim values.)
Of course the NPV of the investment is simply the difference between the mar-
ket value created by undertaking it (£13 500 000) and what it cost (£10 000 000), or
£3 500 000. So we now have another technique for finding the NPV of an invest-
ment, namely one that discounts the free cash flow of the project at a rate
commensurate with the risks of those cash flows.
single claims. The overall NPV method accomplishes that, but it is not the tech-
nique used by most companies. The preferred technique will require that we make
one small adjustment in the ‘overall’ technique we have just developed. Once that is
done, you will know as much (or more) about the NPV techniques that companies
use as do most of the corporate decision makers who use them.
The revision of the ‘overall’ technique of finding the NPV that we shall now
study has a long and distinguished history in finance. We should forewarn you,
however, that it will seem at first that we are taking some liberties with the basic
principles that we have emphasised so often in our discussions up to this point. The
truth is that we shall not be breaking any rules but rather at first merely extending
some basic relationships to be a bit more complex, and in the process we shall
produce a technique that is simpler but as accurate as the ‘overall’ one we already
have. Remember as you work through the scenario below that the net result will be
an easier, not a more difficult, method of finding a corporate NPV, and it will
involve the use of our long-sought weighted average cost of capital.
Remember, too, that you analysed the Cheetah investment from the perspective
of the managing director of that company. Suppose that Lynx has lured you away
from your position at Cheetah, and you are now the head of Lynx. As you did at
Cheetah, you ask to see all the details of pending investment proposals. If Lynx’s
financial people have been educated in reasonably modern times, they will send you
Lynx project data that looks like Table 4.6.
Table 4.6 Lynx Autos plc: cash flows (£000s) for the dickey-seat project
Now Yr 1 Yr 2 Yr 3
Customers 0 +£17 500 +£23 500 +£4 000
Operations 0 −£7 000 −£3 830 −£5 200
Assets −£10 000 −£4 000 −£2 000 0
Government 0 −£4 000 −£8 085 +£5 600
Capital* −£10 000 +£2 500 +£9 585 +£4 400
* Free cash flow
‘Now wait just one moment!’ you exclaim. ‘That is not fair. Those numbers are
not the same ones that we have just developed for Lynx following all of the
principles of value that you led us to believe were so important (Table 4.2). What
happened? Are Lynx’s financial managers stupid, or have you been leading us down
the wrong paths?’
Fortunately neither of the possibilities you fear is true. Lynx’s managers have
calculated the project’s cash flows in the way that most financial analysts would, and
this is not necessarily inconsistent with the principles we have laboured so hard to
develop. First, let us see what these new numbers are.
Even a brief examination of Table 4.6 shows that these Lynx project cash flows
differ from those in Table 4.2 only by the amounts of Lynx’s interest tax shields, as
shown in Table 4.7. So Lynx’s financial analysts have produced figures that are
exactly the same as we did in our earlier analyses for the Lynx project, except that
interest tax shields are not included. The common practice in financial analysis of
corporate investment is that, when estimating the cash flows of a project, its interest
tax shields are not included in the cash flows. In other words, the cash flows are
calculated as if the project will be financed totally with equity, even if the financing
plan is actually to use debt.
Table 4.7 Lynx Autos plc: free cash flows (£000s) for the dickey-seat
project
Now Yr 1 Yr 2 Yr 3
Table 4.2:
Free cash flow −£10 000 +£2 716 +£9 782.6 +£4 464
Interest tax shields 0 £216 £197.6 64
Table 4.6:
Free cash flow −£10 000 +£2 500 +£9 585 +£4 400
‘That cannot be correct,’ you argue. ‘You have told us consistently that in doing
present-value calculations we must discount the cash flows that a capital claim
expects to receive by the rate appropriate to those cash flows. Yet here the cash
flows are not the ones that the capital claimants of Lynx expect to receive. By
omitting the interest tax shields, we are ignoring money that the equity and debt
suppliers can expect to get, and therefore our answers will be wrong.’
You are entirely correct that the cash flows in Table 4.6 for the Lynx project are
not the ones that the capital suppliers of the company expect to receive. And you
are further correct in the implication of your comment that if we discounted these
cash flows with the overall rate for the project (11.6 per cent) as calculated in the
last section, we would get the wrong answer. But let us take the final step and see
how Lynx’s financial analysts would calculate the investment’s NPV:
£ £ £
NPV £10 000 000
. . .
£3 500 000
‘Well, we suppose that nothing should surprise us now,’ you say. ‘Would it be
impolite of us to ask the rationale for discounting these all-equity cash flows at a
rate that is neither the equity rate for Lynx (14 per cent), nor the overall rate (11.6
per cent), but is yet a third rate (10 per cent) whose origin is not at all clear. You
obviously get the correct NPV (£3 500 000), but it is not obvious why you chose to
get it that way.’
You are again entirely correct that we have not yet demonstrated how the 10 per
cent discount rate was derived, or the rationale behind it. That is exactly what we
shall now do. However, before we do so, let us point out that the 10 per cent rate is
in fact the goal of our long search in this module: it is the weighted average cost of
capital for the Lynx project.
How did Lynx’s financial managers arrive at this rate? The answer is in the cash-
flow alteration that took place when we moved from the overall flows to those that
ignore the tax shield benefits of debt. Think of the situation this way: for this
calculation to produce a correct NPV using the cash flows that exclude interest tax
shields, we must instead include the effect of interest tax shields in the discount rate.
And that is exactly what the weighted average cost of capital does. The weighted
average cost of capital that the Lynx financial analysts are using is 10 per cent
instead of the 11.6 per cent overall rate because this new rate is adjusted downward
for the deductibility of interest.
Intuitively speaking, the weighted average cost of capital (WACC) is the discount
rate that:
1. reflects the operating risks of the project;
2. reflects the project’s proportional debt and equity financing with attendant
financial risks; and
3. reflects the effect of interest deductibility for the debt-financed portion of the
project.
You can see that this list of attributes asks quite a lot of the WACC. It must
account for the basic operating risk of the project, much as the rate for the all-equity
financed Cheetah project did. It must be consistent with the proportional claims of
debt and equity, as does Lynx’s overall rate. And finally it must include the effect of
interest deductibility since the cash flows upon which it operates do not. That
sounds like a terribly complex number, and in a sense it is. But, fortunately, it is not
hard to calculate. (Actually, one of the main reasons why it is so widely used is that
it is relatively easy to derive.)
To see how to find a WACC, remember that Lynx’s overall (11.6 per cent) rate
has two of the three attributes of the WACC mentioned above; the overall rate is
adjusted for the operating risk of the project, and reflects its proportional debt and
equity financing. So the easiest way to find the WACC would be to alter the overall
rate to reflect interest deductibility on the investment’s debt-financed portion. The
overall rate, you recall is found by:
Debt market value
Overall rate Debt required rate
Total market value
Equity market value
Equity required rate
Total market value
£5 400 000 £8 100 000
0.08 014
£13 500 000 £13 500 000
0.116 11.6%
To find the WACC we must include the effect of interest deductibility in the
overall project discount rate. Interest deductibility can be regarded as an effective
reduction in the cost of borrowing for the investment. In other words, though the
debt suppliers of Lynx are requiring an 8 per cent return on their investment, Lynx
expects debt to cost less than 8 per cent. The reason is that the government, by
allowing interest deductibility, subsidises debt capital relative to other types whose
payments are not deductible. If interest is deductible at a 50 per cent tax rate, the
government is effectively subsidising 50 per cent of the interest payment that the
company makes. (Lynx’s taxes are lower by 50 per cent of the amount of its interest
payments because the payments are deductible at the 50 per cent tax rate.) This
appears, for example, as Lynx’s interest tax shield cash flows (which can be regarded
as governmental subsidies) being 50 per cent of its interest payments (see Table 4.1,
Table 4.3 and Table 4.4). So if we wish to reflect interest deductibility in the
discount rate (WACC) rather than the cash flows, the weighted average must use the
company’s after-tax cost of debt rather than the debt suppliers’ required rate.
The cost of debt to a company with deductible interest is simply debt’s required
return multiplied by the complement of the corporate income tax rate:
Debt cost Debt required return 1 – Corporate income tax rate
For the Lynx project:
Debt cost 0.08 1 – 0.5 0.04 or 4%
So the debt that Lynx will issue actually costs the company 4 per cent and not the 8
per cent return that the debt suppliers expect to get. The reason is that in our
scenario the government will provide a subsidy to the interest payment equal to the
reduction in taxes due to debt being in the company’s capital structure.2
Lynx’s WACC calculation is therefore exactly like its overall rate calculation,
except that we use the project’s debt cost instead of its debt required rate:
Debt market value
WACC Debt cost rate
Total market value
Equity market value
Equity required rate
Total market value
£5 400 000 £8 100 000
0.04 014
£13 500 000 £13 500 000
0.10 or 10%
So the financial analysts of Lynx were not trying to fool their new managing
director. The WACC–NPV analysis of an investment project is performed by
discounting the project’s free cash flow not including the interest tax shields
that the project’s financing will generate (these free cash flows are the same as
would be expected if the investment were financed totally by equity). The discount
rate used is the project’s weighted average cost of capital, defined as the market-
value weighted average of the project’s equity required rate and debt cost. Debt cost
is the debt suppliers’ required rate multiplied by the complement of the project’s
corporate income tax rate. This method adjusts for the deductibility of corporate
interest in the discount rate as opposed to the cash flows of the project, but yields
NPV answers identical to those of both the overall method and the method which
calculates the separate values of debt and equity generated by the investment.
This then accomplishes the primary goal of this module: displaying the WACC–
NPV technique used by companies as they evaluate investment projects. We have,
however, in a sense begged the question that you raised when we began discussion
2 We multiply by the complement of the tax rate instead of the tax rate itself because the corporate
benefit is the reduction in taxes rather than the taxes themselves. We multiplied by the tax rate rather
than its complement in the cash-flow form of the benefit (the interest tax shields) because interest
payments are expenses that reduce taxes. Remember our example uses a 50 per cent tax rate and we
therefore get the same answer for debt cost whether we multiply the required rate by the tax rate itself
or by its complement. That, of course, would not be the case for any other tax rate, and the comple-
ment of the tax rate is the only generally correct multiplier.
of the WACC–NPV method. That question was why companies used this evalua-
tion technique instead of any of the others that we have discussed. The answer lies
in examining the information each of the methods requires to work correctly.
The overall technique and the one we discussed first, namely the separated debt
and equity value process, both require that we know exactly how much debt
financing will be issued by the company for the project. That is because those
methods use the actual cash taxes expected for the project in their free cash-flow
estimates, including the interest tax shields in the project’s cash flows. The WACC–
NPV, on the other hand, does not require that the exact amount of debt to be used
in the project be known in order to calculate the project’s NPV. The cash flows
used in the WACC–NPV are those that would be expected if the investment were
all-equity financed. (The interest tax shields are not included in the free cash flows
of the project.) So in terms of the data necessary to estimate free cash flow, the
WACC–NPV technique is less demanding.
‘Aha!’ you exclaim. ‘We have caught you now. How seriously can you expect us
to take all of this talk about real applications and information requirements of these
techniques when your own example does not make sense from that perspective?
Lynx’s financial analysts came up with the correct answer using the WACC method
as you described it, and the rationale of WACC–NPV seems to make sense. But
look at the calculation that they did to arrive at the WACC:
£ £
WACC 0.04 014
£ £
0.10 10%
‘The thing that disturbs us about this calculation is that the project’s total present
value (£13 500 000) is used to find the proportions of debt and equity financing.
Now, we agree that the proportions are necessary for the technique to work
correctly. But if you have the £13 500 000 value for the project as a whole, why go
further? At that point, merely subtract the present cash cost of the investment
(£10 000 000), and you get the NPV directly! The whole example makes no sense if
we can get the final answer that easily.’
We wish it were that simple. Remember where we got the £13 500 000 value:
from the earlier and quite exhausting NPV calculations before we arrived at the
WACC method. So the number was not arrived at by any ‘easy’ shortcut. But your
question does raise another important point about why and how companies use the
WACC–NPV method. It is true that the analysis we presented used the total market
value of the project as one of the pieces of information in the analysis. And if that
piece of information were required in order to complete a WACC–NPV calculation,
the method would be redundant. But it is not required. The £13 500 000 was used
to conclude that the debt financed proportion of the project is expected to be:
£5 400 000
Debt financing ratio
£13 500 000
40%
The equity ratio is, of course, 60 per cent. This implies that information about
the market values of the capital claims of a project are not necessary to find its
WACC, as long as the expected ratios or proportions of those financings are known.
Companies using the WACC–NPV are those willing to specify the expected
proportions of debt and equity in terms of their market values, but they do not
know exactly what the claims will be worth until after the analysis is complete.
You can now see why many corporations prefer the WACC–NPV method over
the others we have seen. It does not require that the amounts of debt to be issued
are known, nor does it require that the market values of any of the resulting claims
be estimated beforehand. The only information necessary comprises estimates of
the required rate for equity, debt’s after-tax cost rate, the all-equity free cash flows,
and the proportions intended for debt and equity financing. This is less demanding
an information set than the other techniques require, and is probably the reason that
companies prefer this NPV method.
interest tax shields to the all-equity financed value. So the APV finds the NPV by
first finding the value of an investment as if it were financed only by equity, and
then adds the present value of the project’s interest tax shields.
Suppose that the Lynx corporation used the APV method and wished to analyse
the dickey-seat project. The APV technique begins by finding the value of a project
as if it were financed wholly through equity and so our first task is to find that value
for the dickey-seat project. The all-equity cash flows have, as we know, already been
estimated, because those are the ones we used in the WACC–NPV analysis
(Table 4.6 and Table 4.7). The only question is what discount rate to use.
‘That is also easy,’ you say. ‘We already have an equity rate for the dickey-seat pro-
ject, the 14 per cent rate that we used to value the equity claim in the “separated”
NPV that we calculated first of all for Lynx.’ Careful here. If you were to use that 14
per cent equity rate you would be making a mistake. Remember that this calculation is
to be as if the project were financed wholly through equity. And the 14 per cent rate,
though an equity rate, is not an ‘all-equity’ rate. The 14 per cent equity discount rate is
associated with a project that is partly financed with debt. Since that equity claim is
residual to the debt claim, the 14 per cent rate is in part recognising the risk of the
higher-priority debt claim upon the cash flows of the equityholders. So you cannot use
the 14 per cent rate; it is too high for all-equity financing.
‘Well, then, how about the 10 per cent true “all-equity” rate that we used for the
Cheetah project?’ you ask. That too would be wrong. The 10 per cent rate is an all-
equity rate, but it applies to a different investment, namely one that we have been
told has a lower operating risk than the dickey-seat project. So the 10 per cent rate is
too low.
‘We give up. How do we find the correct all-equity rate for this investment?’ you
sigh. Since it will be a rate we have not seen before, we must confer with our capital
market experts, who consider the cash-flow operating risks of the project and advise
us that the all-equity rate for the Lynx project is approximately 11.69 per cent. (If
you happen to be checking our numbers as you go along, use 11.690475 per cent as
the exact all-equity rate necessary to reproduce our results.) You can see that this
rate is greater than the lower-risk all-equity Cheetah project rate of 10 per cent, and
is lower than the 14 per cent residual equity rate that this investment requires when
£5 400 000 of debt (40 per cent of the project value) is used. So the Lynx project, if
totally equity-financed, would be worth:
£2 500 000 £9 585 000 £4 400 000
All‐ equity value
1.11690475 1.11690475 1.11690475
£13 079 785
But of course the Lynx project will not be financed only with equity. The debt
that will be used with the project will reduce its taxes, and cause more cash to be
available to service its capital claims. The amounts of cash that will be added to the
all-equity flows are the interest tax shields, and they will cause the actual value of the
investment to be higher than its all-equity value. The APV technique adds the value
of the interest tax shields to the all-equity value to find the investment’s total value.
What is the value of the interest tax shields? To find it, we follow the same value
rules that we always have: discount the cash flows at the appropriately risk-adjusted
discount rate. And what rate will thereby apply to the interest tax shields of the
investment? Since the interest tax shields are contingent upon the interest payments,
their risk must be the same as the debt cash flows, which require an 8 per cent
return. So 8 per cent is the correct risk-adjusted rate to use for valuing the interest
tax shields of the project,3 and the interest tax shields (see Table 4.7) for the Lynx
project are thus worth:
£216 000 £197 600 £64 000
Interest tax shield value
1.08 1.08 1.08
£420 215
So the APV technique says that the Lynx project’s present value comprises
£420 215 of interest tax shield value and £13 079 785 of ‘all-equity’ value. The APV
calculation of the dickey-seat project’s effect upon Lynx shareholder wealth is thus:
APV All‐ equity value Interest tax shield value Present cost
£13 079 785 £420 215 £10 000 000
£13 500 000 £10 000 000
£3 500 000
As we promised, the APV method gives us exactly the same result as did the
several NPV techniques applied earlier to this example. Note particularly that the
APV method tells us that the sum of the all-equity and interest tax shield present
values is £13 500 000, which is the value that we earlier discovered for the capital
claims on the project. The APV arrives at the capital claim value not by discounting
the claims’ cash flows separately, as we did at first, nor does it combine them as did
the overall and WACC–NPV methods. The APV finds present values by splitting
up the cash flows into a basic operating set (which we have called the all-equity cash
flows) and a set of cash flows caused by the way the project is financed (the interest
tax shield cash flows). These are each then discounted separately at rates appropriate
to their individual risks.4
3 Actually, there are a few more hidden assumptions in this statement which the picky will appreciate.
For example, since the interest tax shields are tax reductions, this rate implicitly assumes that if interest
is paid, taxes will be reduced. In situations where accounting income is nil and taxes are as well, interest
obviously cannot reduce taxes. Companies recognising the potential value of these reductions use tax
‘carry-backs’ and ‘carry-forwards’, leasing agreements, mergers, and all manner of other tactics to
reduce someone’s taxes with their interest deduction (and of course be compensated for doing that).
4 This demonstration of the equality of valuations using WACC and APV is actually labouring under
some restrictive conditions that are probably not obvious to the first-time reader. Amongst these
conditions are that the project’s debt repayment schedule is designed to maintain a constant market
value captial structure over the life of the project. For more sophisticated extensions of this idea, see J.
Miles and R. Ezzell, ‘The Weighted Average Cost of Capital, Perfect Capital Markets and Project Life:
A Clarification’ in the Journal of Financial and Quantitative Analysis (September 1980).
NPV method seemed to be justifiably the most popular when compared with the
‘separated claim’ and ‘overall’ techniques, because it required either the same or less
information, was a bit easier to calculate, and gave exactly the same answers as the
other two. Making the same comparisons between the WACC–NPV and the APV,
it is not clear that one would automatically be preferred to the other.
The reason for this ambiguity is that they require different information for their
result. Because it does not require that actual amounts of debt to be issued are
known, the WACC–NPV is less demanding than APV, which does. On the other
hand, the APV does not require that the market value proportions of debt and
equity claims resulting from the investment be known, as does the WACC–NPV.
Though both, possessed of full information about a project, would arrive at the
same result, on many occasions companies simply do not have all of the infor-
mation necessary to use one or the other of the techniques. Or companies may be
of the opinion that they are not comfortable in forecasting the type of information
that one or other of the techniques requires. In those situations the choice of
WACC–NPV versus APV is easier.
There is one characteristic of the APV method that fosters its particular use in
complex investment situations. We have concentrated on the financing cash-flow
effect called interest tax shields to illustrate the value effect of how a project is
financed. Interest deductibility, however, is not the only effect that a project’s
financing may have upon its value. For example, some governments give tax credits
of various types; sometimes the deductibilities for interest may conflict with other
deductions that the company may take for tax purposes; and it is also possible that
borrowing may create cash costs in addition to interest and its deductions (for
example, those costs involved in bankruptcy proceedings). When complexities such
as these appear relevant to a company’s financial decisions, the APV approach to
investment analysis may be easier to use than WACC–NPV.
The reason for APV’s desirability where there are several financing-induced cash-
flow effects is the way that APV treats each of these separately, by estimating the
cash flows of each, and discounting them at rates appropriate to their unique risks.
In other words, an APV analysis in a real company may produce a final calculation
that includes separate values for interest tax shields, investment tax credits, litigation
costs with creditors, etc. The APV method is set up to handle each of these sepa-
rately, and they are best understood in that way. The WACC–NPV, on the other
hand, seeks to include no financing-induced cash flows in the analysis, but to
capture their effect by altering the discount rate (the WACC). It would perhaps be
possible to adjust the WACC to include several effects in addition to interest
deductibility, but because of its complexity the task has never been seriously
attempted.
On the negative side of APV, it does not have the automatic characteristic of
being consistent with maintaining an intended ratio between the various kinds of
financing a company uses in its investments. If it is important to a company, for
example, to have debt claims to be about x per cent of the corporation’s market
value, the WACC, because it is a function of these ratios, can display such a result.
The APV, however, being determined by an amount rather than a ratio of debt,
cannot easily do this.
Learning Summary
The information that you should have gleaned from this module is some of the
most important in the finance course. The investment evaluation techniques used by
corporations, and the reasons why they use the ones they do, is central to a good
education in finance.
At this point it is time to bring together in a convenient summary all of the basic
ideas we have introduced about corporate investments. Rather than give you a long
and tedious narrative of the concepts you have covered in this module we have
chosen to kill two birds with one stone and structure the summary in a shorthand
system of notation that should help you to remember these ideas efficiently. In
other words, we are going to show you the formulas for the important forms of
NPV and APV that we have discussed above. Because we did not wish you to be
distracted by these formulas when we first introduced the ideas behind them, we
have delayed their introduction until now. Actually there was no good reason to
produce them for our earlier purposes, because the best uses to which such formu-
las can be put are as aids in remembering to include all of the things that should be
included, and to exclude all that should be excluded in investment analyses.
Henceforth we shall use the notation that follows to signify these ideas, all of
which you have already studied.
Cash flows
FCF = Free cash flow: the amount of cash that the corporation can distribute
to its capital suppliers at time t due to an investment, consistent with
the company’s contractual and operating expectations. This can be a
negative amount if the investment is expected to raise more cash than it
pays to capital suppliers at a given time. FCF is the net amount of the
cash amounts to be transacted with customers (as cash receipts),
government (as taxes and subsidies), and suppliers of labour and assets
(as their cash costs).
I = Interest cash flow at time t.
T = Corporate income tax rate.
ITS = Interest tax shield cash flow: the reduction in corporate income taxes at
time t caused by interest deductibility of the debt issued for the invest-
ment; equal to It Tc.
FCF ∗ = Unleveraged (ungeared) free cash flow: the amount of free cash flow
that the company is expected to generate at time t due to a project, not
including interest tax shields; equal to FCFt – ITSt.
Market values
E = Market value of the equity of the investment at time t.
D = Market value of the debt of the investment at time t.
V = Market value of the investment at time t; equal to Et Dt.
Discount rates
= Required return on the equity of the investment; required returns are not
usually time-subscripted, but can be.
= Required return on the debt of the investment.
∗ = Cost of debt as a rate to the investment; equal to rd 1 – Tc .
= Overall weighted average return on the capital claims of the investment;
equal to
D E
V V
∗ = The weighted average cost of capital (WACC) of the investment; equal
to:
D ∗
E
V V
= All-equity or unleveraged (ungeared) required return on the investment;
the rate that would be required on the investment were it to be financed
purely with equity.
you from what you should be doing, namely learning how to use the relationships
and techniques to make financial decisions. If it makes you feel better, we promise
never to ask you to reproduce any formula given above in an examination. You will
surely, however, find it useful to refer to that compilation as you continue this
course in order to refresh your memory about specific definitions. Believe it or not,
before long you will have internalised the formulas and definitions enough so that it
is unnecessary.
To review briefly the summary above, you can see that it has four main sections:
Cash flows, Market values, Discount rates, and Investment evaluation techniques.
The section on cash-flow definitions lists those for free cash flow, interest tax
shield, and FCF* (the notation that we use for the cash flow necessary for the
WACC–NPV method), along with brief definitions of each.
In the market value section, we simply introduce notation for debt and equity
market values, and define the total value of the investment as the sum of its debt
and equity market values. The discount rate section lists the notation for debt and
equity required rates of return, shows how the overall investment required rate, rv,
is calculated, defines the cost of debt, rd*, and shows how its use generates the
WACC, rv*. That section also lists the all-equity discount rate, ru, for the investment
(the rate that would be required were it to be financed totally by equity). The ru rate
is used in the APV calculation.
Finally, the investment evaluation section shows the WACC–NPV and APV
techniques in the form of equations. Though these equations are correct, and the
mathematician–economists among you might be able to master the techniques by
simply studying the formulas and related definitions, we remind you that our major
purpose in showing them to you is so that you will recognise them as we use them
in the rest of the course as a convenient method of referring to the components of
the calculations.
Notice that we have not included the formulas for finding investment values with
either the ‘overall’ or ‘separated’ techniques. The reason we have not included those
methods is simply that they are not regularly used by corporations in making
investment decisions. They were useful in developing the ideas behind WACC–
NPV and APV in the text, but once the latter are understood, the others are
unnecessary. As a matter of fact, the others are so little used that from now on,
whenever we refer to NPV, you should take that to indicate that we are talking
about the NPV found by the WACC–NPV method, or the method itself. That is
the usual convention in corporate finance.
We must also say that you should be able to recognise, for example, that the
NPV formula is telling you to do exactly the same thing that earlier took several
pages of text to accomplish: discount the FCF* cash flow with the rv*, or WACC.
Note also for example that, unlike the text discussion, there is no separate indication
in the NPV formula above to subtract the present cost of the investment (e.g. the
£10 000 000 for the Lynx project). The reason for that is that the formula begins its
summation of FCF* present values at t 0, which would automatically include
initial costs as part of the FCF* summation (e.g. if you follow exactly the instruc-
tions of the formula, you will have an FCF∗0 £10 000 000 for the Lynx project,
which is discounted at 1 rv* 0 1, just what you wanted to do). See if you can
similarly decipher the APV formula given above.
In following modules we shall investigate some of the important uses, infor-
mation sources, competitors, and interpretations of these techniques. Before you
plunge into that material you should feel reasonably comfortable with the material in
this and the preceding module. We suggest not only thorough reading but also a
significant attempt at the problems given at the ends of these modules.
Review Questions
All of the Review Questions for this module are based on the following data for an
investment proposal being considered by Weir Fishing Tackle plc to purchase and place
into service a robot reel fabricator. The investment is expected to have only a three-
year life because of technological advances. The basic marketing, operational, taxation
and asset cash flows of the project (the changes expected in the company’s cash
flows were it to accept the investment) are as set out in Table 4.8.
Table 4.8 Weir Fishing Tackle plc: robot reel fabricator (£s)
Now Yr 1 Yr 2 Yr 3
Customers +10 000 +14 000 +9 000
Operations −3 000 −6 000 −4 000
Assets −7 000 +1 000
Government nil −4 122.47 −5 095.84 −1 065.22
(Note: Do not attempt to derive the taxes shown by applying Weir’s income tax
rate, which is 50 per cent, to the displayed cash-flow estimates. As you are aware, taxes
are based upon accounting ‘income’, numbers which are likely to be quite different from
cash flows.)
Weir’s finance department has decided to finance the project with both debt and
equity capital. The company’s intention is to keep Weir’s market value ratios of debt
and equity at 25 per cent debt and 75 per cent equity. If this is done, Weir thinks that
equity’s required rate re will be 16 per cent whilst debt’s ( rd will be 12 per cent.
4.1 Using the appropriate formulations for the WACC–NPV (see the cash flows and
discount rates in Learning Summary above), make a recommendation to Weir as to the
desirability of the robot reel fabricator project.
4.2 Is it proper in answering Question 4.1 to include the cash-flow effects upon the
company of the income tax reductions produced by the debt used to finance the
investment?
A. Yes, these must be included, because they are legitimate cash flows caused by
the project, as we have defined such cash flows.
B. No, because the effect of interest deductibility has been captured in the
formulation of the WACC for the project, and to include it in the cash flows
would be double counting.
C. Yes and no, because you can get the same answers for NPV if you appropriate-
ly adjust the cash flows to include the effect of interest deductibility while
excluding that effect from the discount rate, or vice versa.
D. No, because we have not yet been told how much interest Weir will be paying,
and thus interest tax shields cannot be known for the purposes of Question
4.1.
4.3 Suppose that you were asked to explain to the uninitiated in general terms what
economic information is contained within the WACC. Choose from among the
responses below the one that which you consider to be the best:
A. The WACC is the discount rate that will tell you how much an investment will
be worth.
B. The WACC tells you the minimum acceptable return that must be earned on
the funds invested in a project.
C. The WACC takes account of the returns required by capital suppliers, and the
tax effects of financing.
D. The WACC applies to the return required by shareholders were the invest-
ment to be all-equity financed, and separately discounts the tax effects of
interest deductibility of debt (interest tax shields).
4.5 Suppose that you are now informed that the interest and principal payments for the
debt of the robot reel fabricator are expected to be (£s):
Now Yr 1 Yr 2 Yr 3
Interest – 244.94 191.68 130.44
Principal – 443.83 510.39 1086.96
If Weir were to finance this project totally with equity, the project’s required return
(ru) would be 15.07 per cent.
With this information, calculate the APV of the investment, and compare it with the
NPV calculated for Question 4.1.
4.6 The APV is preferred to the NPV by some financial practitioners because:
A. The APV is more easily amenable to inclusion of financing-induced effects upon
shareholder wealth (other than interest deductibility) than is NPV.
B. They do not know any better.
C. The APV requires only that proportions as opposed to actual amounts of
borrowing be pre-specified.
D. There is a seemingly psycho-social bias toward presenting numbers as rates of
return as opposed to cash-stated wealth changes.
Learning Objectives
This module is devoted entirely to a detailed example of how a company would go
about deciding on a major proposal to enter a new line of business. In it, a firm is
considering introducing a new product line, similar to its current business, but with
major implications for the future cash flows of the company. The central issues in
the example deal with which of a company’s cash flows are relevant to an invest-
ment decision and which are not. In particular, the module gives significant
attention to the issues of estimating appropriate cash flows based upon the types of
accounting data normally available within companies for decisions of this type. The
student should pay special attention to solving the case at the end of the module,
which gives the opportunity for a full analysis.
5.1 Introduction
In Module 3 and Module 4 our main concern was to study the basic techniques of
company investment decision making, to understand why they take the forms that
they do, and how they are consistent with the primary corporate goal of maximising
shareholder wealth. Now that you have a good appreciation of these important
characteristics of the techniques, we shall begin studying an equally important set of
topics: how these techniques are implemented in real companies. This module will
concentrate upon the process of estimating the cash flows to be evaluated by the
techniques.
You recall that we introduced the cash flows to be included in a corporate in-
vestment analysis by examining Figure 3.1 in Module 3, which shows the general
outline of the transactions that corporations undertake with outside entities. Those
transactions are: with customers; with suppliers of labour, materials, services and
assets; with government; and with suppliers of capital. In the Cheetah and Lynx
projects we examined examples of these cash flows, and how they related to some
1 Note that we use the term ‘expectations’ or ‘expected’ to characterise the cash-flow estimates used in
evaluating projects. Our use of such terms is a signal that the actual amounts of these cash flows that
will occur in the future are not known for certain. Thus you should recognise that such estimates are
not deterministic but are more in the nature of ‘best guesses’ at the time such estimates are made. We
shall have more to say about these estimates and their uncertainties in a succeeding module of the
course.
the other product. Often, when a company is about to introduce a new product
its competitors are not far behind; if the other product’s cash flows are about to
decline anyway because of competitive pressures, it would not be correct to
charge those declines to the new product. In estimating investment cash flows,
analysts must ask the right questions of the marketing people in a company, and
listen to their answers very carefully. Remember, the correct cash flows are the
changes in the company’s cash flows if this project is accepted.
3. ‘Inclusion of all relevant cash flows’ also means that analysts must know what
things should be omitted from the investment’s cash flows. For example, one
must never forget the doctrine that ‘sunk costs’ are to be ignored. ‘Sunk costs’
are cash outlays that have already been made. The fact that a company has mil-
lions ‘invested’ in a project is often argued as a reason that the project should be
continued or even augmented. This is simply bad economics; whether or not an
investment deserves to be increased or continued must depend totally upon
whether it will generate enough future cash flow to make any necessary addition-
al investment worthwhile. Said another way, an investment should be
discontinued if its future cash flows’ present value is less than what the company
would obtain by selling or even abandoning the project, now or later.
4. ‘Inclusion of all relevant cash flows’ means, thirdly, that analysts must be very
careful that the accounting numbers provided for a project are interpreted cor-
rectly. For example, analysts are usually given figures by the accounting
department for ‘overhead’ that a project will incur. It would almost certainly be a
mistake to regard those amounts as cash outflows that the company will experi-
ence if the project is accepted. There are two main reasons for this. The first is
that overhead can include non-cash expenses, such as depreciation, in addition
to overall corporate costs for things such as the electricity and gas bills, and head
office functions such as financial analysts’ salaries. The second reason is that
accountants ‘allocate’ overhead expenses on the basis of arbitrary rules which
have no necessary correspondence to the additional or incremental cash flows
that a project will require. Accountants perform many such overhead allocations
on the basis of measures such as the amount of plant floorspace devoted to the
manufacture of the product, or to some arbitrary activity measure. Neither of
these will generate an accurate estimate of the changes in the company’s ‘over-
head’ cash flows that will occur if the project is accepted.
By no means should you interpret this to say that overhead expenses are irrele-
vant to corporations; accountants doubtless have reasons for figuring the
numbers the way they do. Our point is that we are interested only in cash flows,
and only in those that will be caused by the investment. If the project will cause
the company to have higher electricity bills, it is correct to include as cash out-
flows the increments to overall corporate electricity bills caused by the
acceptance of the project. If new managerial talent will be hired because of the
project, its cost is appropriately included (or if old managers’ effectiveness in
their other efforts deteriorates because of the demands of the new project, the
cost reckoned for that deterioration is an incremental cash outflow of the new
project). These costs will probably be quite different from the overhead alloca-
tions that accountants will charge against the project, but are nevertheless the
correct ones for an investment analysis.
This list is not, of course, exhaustive of all the cash-flow estimates about which
analysts must be careful, but it is instructive. Note that all of the examples have a
common thread: the only amounts of relevance to an investment analysis are the
changes or increments that are expected to take place in the cash flows of the
corporation were it to accept the investment. We urge you to take a most literal
interpretation of this instruction: if an amount is offered as ‘relevant to an invest-
ment’, it must pass that test to be included as a cash flow for analytical purposes.
There are many corporate cash flows that should not be included because they
are not incremental; they would not be affected by the project’s acceptance or
rejection (managerial salaries often fall into this category). There are also many
expenses and revenues that should not be included because they are not cash
amounts (depreciation, inventory cost allocations, and changes in receivables or
payables are all examples). There are other cash flows that are relevant but are
always in danger of being overlooked (like interactive effects of a project with the
other activities of the company, or increments to the amounts of cash that the
company will need to keep in the bank to facilitate the transactions that the invest-
ment will generate across its lifetime). As you can see, it would be impossible for us
to give you a complete checklist or template of cash flows to accept as relevant or to
reject as irrelevant. The best analysts are those who understand the basic principles
of cash-flow estimation, and apply those consistently to the projects they are
evaluating.
t0 t1 t2 t3
Fixed assets:
Gross plant and equipment 1 300 1 500 1 500 1 500
Accumulated depreciation 0 300 600 1 500
Net fixed assets 1 300 1 200 900 0
Total assets £2 400 £2 950 £2 220 £150
Current liabilities (working capital):
Accounts payable (creditors) 0 300 800 150
Total current liabilities 0 300 800 150
Long-term liabilities and equity:
Debt 1 300 1 000 800 0
Equity:
Paid-in equity capital 1 100 1 475 620 0
Retained earnings 0 175 0 0
Total equity 1 100 1 650 620 0
Total long-term liabilities and equity 2 400 2 650 1 420 0
Total liabilities and equity £2 400 £2 950 £2 220 £150
Though your boss trusts that these are good estimates, she is informed enough
about modern finance to know that the accounting numbers in these financial
statements cannot be directly applied to the decision at hand, namely whether
Remote Systems should embark upon the WalkPhone project. She would like you to
manipulate the above information, collecting more as necessary, to produce a
correct analysis.
t0 t1 t2 t3
Profit before tax 0 320 220 370
Income tax 0 160 110 185
Profit after tax* 0 160 110 185
* Not shown is a t4 negative £60 000 profit before tax due to an expected bad debts
write-off from the project at that time.
The first data that claim your attention are the estimates of operating revenues
made by the Marketing Department (see Table 5.2). You are fully aware that operat-
ing revenues are not necessarily the same thing as cash inflows from product sales,
and so you decide to grapple with that problem first. You go to the Marketing
Department to enquire about the extent to which the revenue figures for
WalkPhone are really cash amounts.
‘Our responsibility is to sell, not to be collection agents. Go and ask the Credit
Department,’ responds the Marketing Department head, indignantly. Tail between
legs, you hustle to the Credit Department. The credit manager indicates to you that
cash receipts can be figured by adjusting the operating revenue numbers for the
changes in accounts receivable (debtors). Any increase in an account receivable from
one period to another means that net operating revenues overestimate cash receipts
by that amount, and vice versa.
‘May I ask a stupid question?’ you enquire of the credit manager. ‘I assume that
the accounts receivable in the balance sheets are the total amounts of money that
Remote Systems’ customers will owe on their WalkPhone purchases at the times
indicated. If that is so, I do not understand why we show £150 000 as accounts
receivable as at t3 [see Table 5.1] when all of the other asset estimates are finished at
that point. The amount owed at t3 implies that there will be another cash receipt at
t4 when those people finally pay up.’
‘Very perceptive,’ offers the credit manager. ‘It is in fact true that some of the
sales made in the last period that we intend to sell the WalkPhone will not result in
receipt of cash until the next period. The only thing I would add is that, of the
£150 000 in accounts receivable at t3, we will probably never collect £60 000. That
amount is what we expect in bad debts coming from sales in all periods. We
probably should have included a negative profit figure for t4 in the pro forma
income statements, but we put it in as a footnote instead. You might also notice that
we expect to owe £150 000 to our suppliers at t3 from the WalkPhone project [see
Accounts payable in Table 5.1]. Of course we intend to pay that in full by t4.’
Returning to your office, you recall that operating revenues overestimate receipts in
any period by the amount of the increase in accounts receivable during that period,
and vice versa. With that idea in mind, you start to do the necessary adjustments to
the sales revenues estimates of WalkPhone, but immediately another thought strikes
you: if revenues are potentially misestimating cash inflows, will not expenses also do
the same for cash outflows? So you return to the Accounting Department.
‘Can you answer a few questions about the expense figures for the WalkPhone
project?’ you ask. ‘I am particularly interested in the numbers for operating expens-
es.’
‘Certainly,’ answer the accountants. ‘We estimate operating expenses as follows.’
(At this point the accountant launches into a completely confusing lecture about
how these expenses are estimated, the interaction with inventory (stocks) valuation
methods called ‘LIFO’ and ‘FIFO’, the crediting and debiting of accounts payable,
and the company’s overhead allocation procedures.)
You listen politely to the explanation, and ask what you should have asked in the
first place: ‘Suppose that I wanted to change each of your expense numbers into the
actual amounts of cash that the company was going to spend for operations due to
the WalkPhone project, and would not otherwise have spent. How would I do that?’
‘Oh,’ say the accountants. ‘Why didn’t you ask that in the first place? To take care
of the payable and inventory adjustments, we would suggest that you subtract from
our expense figure the increase in payables, and add to it the increase in inventories
and cash, and vice versa for decreases in payables and decreases in inventories and
cash. For overhead, you can subtract the overhead expenses, and add estimates of
cash payments in the company’s overhead accounts that are caused uniquely by the
project. We have all of the necessary information for those adjustments right here,
but we would not recommend that you make the adjustments that way.’
‘Why not?’ you say. ‘It sounds like a pretty complicated process, but I suppose
that I must go through all of those adjustments if I wish to estimate actual cash
flows.’
‘It would indeed be a very complicated process separately to adjust each operat-
ing expense (and revenue) item to be a cash flow, and it would also be unnecessary,’
says the Accounting Department representative. ‘There is a much more straightfor-
ward process, which accomplishes the same thing. Recall that the balance sheet for
the WalkPhone project [Table 5.1] contains current asset and liability items which
sum as follows (£000s):
The difference between those has a name: “net working capital”, and is:
If you simply examine the changes that occur in net working capital from period to
period, you will adjust for almost everything about which you are concerned [see
Table 5.3].
Table 5.3 WalkPhone project: net cash from sales and after outlays
other than fixed assets, taxes and capital (easy calculation:
£000s)
t0 t1 t2 t3 t4
Operating profit 0 750 620 400 −60
Less change in
net working capital −1 100 −350 +930 +520 0
Plus overhead changes 0 +70 +80 +80 0
Operating cash flow −£1 100 +£470 +£1 630 +£1 000 −£60
‘This is indeed a correct and much simpler method of estimating operating cash
flow. The reason it works is that almost all of the adjustments you would have to
make so laboriously to the separate operating revenue and expense items on the
income statement in fact come from changes in working capital accounts (receiva-
bles, payables, inventory and cash). Well, by going directly to the total change in all
of those amounts, the change in net working capital, we are simply saving a lot of
detailed work, and accomplishing the same thing. The reason why we subtract that
from the operating profits is that operating profits is the difference between net
operating revenue and operating expenses, the two numbers upon which you began
making all of the adjustments in the first place.
‘Finally, the reason why we must add the overhead adjustment separately is that
the overhead problem was not a working-capital consideration, so it would not have
appeared in the changes in those accounts. The numbers you see as cash flows for
changes in the company’s overhead due to the project are figures that we got from
our records. You must trust us to have estimated those correctly.’
‘What you say has a ring of truth,’ you agree. ‘But I shall have to think about it
for a while. Meantime, I take it that the negative £60 000 operating profit figure at t4
is the write-off of bad debts that the Credit Department mentioned to me earlier?’
‘Yes, and if you had read the footnote to the income statements you would have
seen that for yourself,’ scolds the accountant.
At this point you pause for a moment to consider what you have discovered and
what has yet to be done. You have arrived at good figures for the cash flows that
the company will experience from operating the WalkPhone project, including cash
inflows from sales, and outflows for operating expenditures, inventory and cash and
marketable securities. The only items yet to be included are the taxes that Remote
Systems will pay due to the project, and the cash flows from originally purchasing
and subsequently selling the necessary fixed assets.
Taking a deep breath, you begin to consider the cash flows for fixed assets. A
glance at the project’s balance sheets (Table 5.1) indicates that the company would
invest £1 300 000 in such assets at t0, and that this investment would increase to
£1 500 000 at t1. A quick call to the Production Department verifies that these
amounts are indeed a cash outlay of £1 300 000 at t0 and an additional £200 000 at
t1. The Production people indicate that these are the only such outlays for the
WalkPhone project. In response to your question about what will happen to those
assets at the end of the WalkPhone, they say that the plan is to sell them, and that a
£950 000 inflow is expected at t3 from that source. You refer to the income state-
ments (Table 5.2) and spot an item of other revenue for t3 that corresponds to that
cash inflow.
So it turns out that the fixed-asset cash flows are quite straightforward.2 The
fixed-asset cash flows you calculate are given in Table 5.4.
The only remaining cash flow to be dealt with is taxes. And that one is the easiest
yet. WalkPhone’s income statements (Table 5.2) indicate that the project’s taxes are
expected to be as in Table 5.5 (remembering the £60 000 t4 bad debt write-off and
tax at 50 per cent).
Because they have been so helpful in the past, you call the Accounting Depart-
ment with the only question that comes to mind about the tax calculation. ‘What
about these fixed asset sales that we plan at t3? I see that we have included the cash
amounts as other revenues, but are such things taxed in the same way as any
operational revenues? I thought that asset sales were somehow different. What is the
term “capital gains” that I hear so often? Does it have anything to do with asset
sales?’
‘You’ve asked a lot of questions,’ respond the accountants. ‘To be absolutely
honest, the best answer we can give you in general about tax questions is simply to
trust we have figured them correctly, and go on to some more productive pursuit.
The reason is that tax laws vary so much, depending upon which government is
doing the taxing, and they also change too much across time, so that it takes many
full-time tax experts to keep up with the tax rules. We do have some tax people
(they are part of the overhead that you were so concerned about earlier) who do
nothing but that.
2 Actually they were straightforward anyway because the accumulated depreciation item on the balance
sheet was shown separately, before the net fixed assets accounting number was calculated. Many
companies show just the latter, especially on their publicly released statements. If the income
statements are not detailed enough to show the depreciation expense as a separate item, figuring out
the fixed-asset cash flows can be a nightmare.
‘The specific question that you have asked is about the taxation of fixed-asset
sales. The way our laws work right now is that we are allowed to write off as an
expense anything that is still on the books (undepreciated book value) for an asset
when we sell it. The only time that capital gains is important is when we sell an asset
for more than our original purchase price, and we do not plan that here. So the
other revenue from asset sales is taxed at the normal corporate income tax rate, and
we are allowed to reduce our taxes on that to some extent by a “shield” created by
the remaining book value of the asset sold.
‘WalkPhone assets are to be depreciated “straight line” over the life of the pro-
ject, should it proceed, and the original £1 300 000 t0 outlay has a £400 000 “salvage
value” for depreciation purposes, leaving £900 000 to be depreciated. A deprecia-
tion-based salvage value is not really a value in the sense that you understand the
word; it is an amount that we keep on the books for an asset and do not expense as
depreciation until we sell the asset in question. The £200 000 t1 asset outlay is not
regularly depreciable at all, and is to be simply written off against other revenue at t3.
You can thus see the origin of the depreciation expenses in the income statements
[Table 5.2]. The £300 000 expenses at t1 and t2 are the regular straight-line charges
for a three-period depreciation of the £900 000 of depreciable value of the t0 asset
purchase. The £900 000 t3 depreciation is the sum of the last £300 000 of that
expense, the £400 000 book salvage value of that asset, and the £200 000 on the
books for the t1 asset outlay. Actually, we plan to sell the t0 asset for £800 000 (more
than its book value at that time), and the t1 asset for £150 000 (less than its book
value), but that makes no difference. They will both be subject to the same income
tax rates (our taxes will be a bit higher because of the first asset’s sale, and a bit
reduced because of the “loss” on the sale of the second asset), but it makes no
difference if we combine the revenues and depreciations from them or not. The
taxes will be the same.
‘Now if you really want to hear something complicated, let us tell you about what
would happen if we had taken an investment tax credit on either of those assets, and
had sold them before the prescribed holding period!’ chuckles the accountant.
‘I would love to, but I have a bit of a headache,’ you reply. ‘I shall be back soon
to hear your story about that. It sounds enthralling,’ you lie.
Again returning to your office, you consider what you now know about the ex-
pected cash flows of the WalkPhone project. With the information about the taxes
that Remote Systems will experience from the investment, you have covered all of
WalkPhone’s transactions other than capital cash flows. You know what the project
is expected to generate in terms of cash flows, both inflows and outflows, with
labour and asset suppliers, and with government. The summary of those cash flows
is given in Table 5.6.
Proudly you carry the results of your labours to your boss, who eagerly peruses
the numbers.
‘Well, I shall say this for you. You certainly came up with results that look differ-
ent from the profit figures that appeared in the pro forma financial statements for
the project. Without going into all of the detail of where you got them, can you
explain to me what these figures are, and how we should use them?’ she asks.
‘I shall be delighted to,’ you respond. ‘Let me call your attention first to the “bot-
tom line” number in this type of analysis, namely the figures labelled FCF in the
summary of cash flows [Table 5.6]. The amounts that you see listed are our expecta-
tions for the changes that will occur in Remote Systems’s cash flows if we accept the
WalkPhone project. The FCF cash flows are the amounts of cash that the company
can distribute to the debt and equity capital suppliers of WalkPhone without
disturbing the currently held expectations and plans for the project.’
‘Interesting, and well explained,’ says your boss. ‘But I know all that. What I
really would like is for you to explain the general steps by which you arrived at the
numerical results you have in the summary.’
‘OK,’ you answer. ‘You see that the first line in the summary is the “Operating
cash flow”. These are the amounts of cash that we will take in and pay out from
operating the assets that we use in the project. It includes the cash from selling
WalkPhones, and the costs of producing them including the amounts of cash that
we use as working capital. The easiest way to come up with this number is to
subtract the changes in net working capital from operating profit, and then include
any non-working capital adjustments to cash flows that are necessary to turn the
operating profit number into cash. One example is the difference between overhead
allocations and the actual changes in cash overhead that the project will induce at
Remote Systems.
‘The next line is “Fixed-asset cash flow”. That is simply the amounts of cash that
we expect to spend on assets such as machines, plant and equipment, and the
amounts of cash that we expect to get when we sell them. Or if we do not expect to
sell the assets at the end of the project, we would include the benefit that the
company would expect to receive from using the assets in whatever operations they
are intended for (and that, of course, will be more than what we could sell them for,
or we would have sold them).
‘The cash flows for taxes appear next. Those amounts come from the taxes that
will be due to be paid, based upon the income statements for WalkPhone. The taxes
are based on the classic accounting definitions of income or profit, and cannot
generally be deduced without having calculated those numbers. Naturally, taxes
include not only those from the operations of WalkPhone, but also the effect of
asset depreciation and sales of assets. FCF is simply the result of adding operating
cash flow to fixed asset cash flow, and subtracting taxes.’
3 If taxes had originally been calculated ignoring the interest deduction, this step would of course be
unnecessary. Many companies estimate project taxes in just that manner.
pointed at that outcome. So we had better have all of our arguments in line for the
questions that will be asked. I have heard of a couple of alternative techniques of
making investment decisions that are supposedly consistent with modern finance. I
believe they were called the adjusted present value and the internal rate of return. Is
it a lot of trouble to see how the WalkPhone would look under them?’
‘Not at all,’ you say. ‘Let me see if that report from your capital market experts
says anything helpful.’ As it turns out, it does. The report says: ‘In our opinion the
unleveraged required return ru for the WalkPhone project is 21.08 per cent, and
the required return to debtholders rd is 10 per cent.
With this information you can calculate the APV for WalkPhone:
FCF ∗ ITS
APV ∑
1 1
£45 £1470 £1725 £30
£2400
1.2108 1.2108 1.2108 1.2108
£65 £50 £40
1.1 1.1 1.1
£402 £130
£272
So the NPV and APV techniques give the same result, as they should. (Note the two
different discount rates in the APV analysis.)
The calculation of an investment’s internal rate of return, you recall from the first
module of your Finance course, requires that you find the discount rate that will
cause NPV to be zero. (Or the IRR can be calculated by finding the rate that causes
the present value of an investment’s cash outflows to be equal to the present value
of its inflows, which is to say the same thing.) By trial and error, you solve the
following for the IRR:
£45 £1470 £1725 £30
0 £2400
1 IRR 1 IRR 1 IRR 1 IRR
IRR 12.39%
The WalkPhone project has an internal rate of return of 12.39 per cent, which can
be interpreted as the average per-period rate of return on the resources that would
be invested. Since the financial market consultants have advised that the project’s
cost of capital is 18 per cent, our comparison of the IRR with the returns that
capital suppliers can earn on equal-risk investments is:
IRR 12.39% return on WalkPhone project
rv* 18% return available on equal‐risk investments
So the WalkPhone, if accepted, would yield a return substantially less than that
which is available on comparable investments in the financial market. (This is often
called the ‘hurdle rate’, a graphic description of what the IRR must do to be
acceptable; the rv* is the IRR’s hurdle rate if the cash flows used to calculate the
IRR are those that would have been used for an NPV analysis.)
Here the IRR tells us the same thing as the NPV and APV do: reject the
WalkPhone project. The NPV and APV tell us exactly how much wealth will be lost
by the shareholders of Remote Systems if the project is accepted, whereas the IRR
simply says the project’s return is too low.
‘Excellent,’ says your boss. ‘The message is clear. We should recommend that the
WalkPhone be rejected.’
Learning Summary
The example in the previous section should serve at least two purposes in your
Finance education. First, the example shows several ‘tricks of the trade’ that are
valuable for financial analysts to know when attempting to estimate project cash
flows. Below we remind you of the highlights of those procedures. As important –
perhaps more so – is the example’s consistent application of the general principles
of investment cash-flow estimation. By this we mean that the major importance of
an illustration like the WalkPhone example is not that you learn specific procedures
(though they are useful, indeed necessary), but that a correct investment analysis
requires adherence to certain principles of cash-flow estimation. In their most
succinct form, these can be distilled to the following:
1. Include all changes that will occur in the cash flows of the corporation were it to
accept the project, at the time points when those cash-flow changes are expected
to take place. Those cash flows comprise all transactions that the corporation
would undertake with suppliers of labour and management skills, with suppliers
of materials, services and assets, and with government. Operational opportunity
costs are legitimately included in a project’s cash flows, but cash flows to and
from capital suppliers are not.
2. When the cash flows are being estimated for an NPV or IRR analysis, it is not
necessary to estimate the interest tax shields for the project; the cash flows are
all-equity flows. When an APV analysis is to be done, interest tax shields must
also be estimated, based upon the debt issued to finance the project.
These principles seem easy enough to comprehend, but as with so many straight-
forward ideas, implementation is more complex. The WalkPhone example is
reasonably rich in applying the general principles of cash-flow estimation to the
specifics of a realistic investment decision. In the WalkPhone example, the follow-
ing cash-flow considerations were important:
1. The project’s cash inflows from selling WalkPhones to customers were expected
to occur across time in a pattern different from the way revenues were recorded
on the accounting records of the company. Use of revenues as the cash inflows
from that source would produce erroneous results.
2. Similarly, the costs of the project in terms of the cash outlays for operational
activities (labour costs, management salaries, utilities costs, raw materials costs,
and so forth) also had a pattern different from the accounting costs registered
across time as operational expenses.
3. The accounting expenses for overhead, depreciation and interest generally
differed from cash flows, though the transactions that they represented may have
had cash-flow importance. For example, we found depreciation to be the ac-
countant’s way of recognising the original cash outlay for acquiring the asset,
whereas in the cash flows of the investment it was important to recognise the
cash outlay for the asset at the time it was expected to occur. Overhead account-
ing expenses allocated to the project were not the actual changes in the
overhead-type costs that the company was expected to experience should the
project be accepted. The accounting overhead numbers needed to be thrown out
and the correct cash-flow changes substituted. Interest, of course, was not a cash
outflow at all if the purpose was to estimate the cash that is to be available to
service capital claims. Interest goes to a capital supplier, and therefore was not
subtracted from the total that we sought.
In the WalkPhone example we presented these complications in the context of a
situation where the only data available were from accounting pro forma (expected)
income statements and balance sheets. In that example we adjusted operating profits
to be the cash flows that they represented by subtracting the changes in working
capital and adding the overhead adjustments. We found the timing and amounts of
cash that the company would be paying for fixed assets, and the money that would
be received upon their sale by examining the changes in book values of the fixed
asset accounts and other revenue items.
All of this was reasonable to do in the context of the WalkPhone problem in
order to get as close as possible to actual cash-flow expectations. Your accounting
friends, however, in reading the WalkPhone example, will doubtless point out to
you that we were making several optimistic assumptions about the way the account-
ing numbers were formed, so as to be able to claim that our adjustments created
cash flows from accounting data. We would be the first to agree, but also to argue
that, if the question at hand is whether or not the WalkPhone project should be
undertaken and the only data available are financial statements, the adjustments we
made are by far preferable to using the unadjusted accounting numbers to decide
about the project.
One corollary is that financial analysts find their jobs much easier to perform
correctly if the company’s information system is designed to produce cash-flow
estimates in addition to accounting data. In other words, the pro forma financial
statements themselves are developed from the same basic information that would
be used to form cash-flow figures. The basic information in the WalkPhone
example was first ‘coded’ into accounting numbers, and then ‘decoded’ into cash
flows. Imagine how much easier the analysis would have been had Remote Systems’
information system been set up, when queried about the WalkPhone proposal, to
produce a report such as Table 5.8. There is nothing that would prohibit such a
system being implemented in a company; indeed, many sophisticated corporations
have done just that.
Even if your company has such a system, it is nevertheless quite important that
you have the capacity to translate between accounting numbers and cash flows. The
main reason is that accounting numbers are still the primary ‘language’ in which the
financial results and expectations of companies are displayed. Not only must you be
able to function with cash flows as your decision-making vehicle, but you must
convince those who are interested in ‘profits’ that your numbers are meaningful.
Often that will mean performing the type of translations that we showed in the
WalkPhone example.
Though our example is fairly rich in realistic detail, there are several common
cash-flow considerations that did not appear in it. We have mentioned most of these
in the introduction to this module, but let us try to put them in the context of
WalkPhone. Suppose, during your investigations of the cash flows, you discovered
that:
1. Remote Systems manufactures another telephone product, the StayPhone, which
is similar to the WalkPhone but must remain connected by a cord to a telephone
outlet. The marketing people are unanimous in their opinion that the introduc-
tion of the WalkPhone would cause the sales of StayPhone to decline.
2. The WalkPhone manufacturing process will use a piece of equipment that
Remote Systems already owns, and thus is not forced to purchase. The account-
ing system of the company has not included that equipment in the assets of
WalkPhone’s balance sheets.
3. One of the assets that Remote Systems will purchase if it accepts the WalkPhone
project is actually a replacement for an asset that the company is currently using
in other operations. If the project is rejected, the company will stick with the old
asset. If, however, the WalkPhone is accepted, the increased volume of
WalkPhone production will make replacing the old asset worthwhile, and the
efficiency of its other applications will improve.
We could make the list as long as our imaginations permitted, but the above are
sufficient to make the point: there are often indirect effects of an investment, which
are as legitimate in their effect on corporate cash flows as the direct inflows and
outflows from the sales of the product.
Review Questions
cash on hand, and inventories amounting to 10 per cent of revenues each year, and
accounts payable (creditors) being itself 10 per cent of revenues. Cash on hand and
inventories will necessarily be increased at the beginning of the years in which sales
based upon them are expected to occur.
Additional repairmen will cost £20 000 in the first year, £40 000 in the second, and
will increase at 15 per cent per year each year thereafter. Other direct costs of
operating the service (fuel, insurance, training, etc.) are expected to amount to 15 per
cent of revenues each year.
PC Problems’ accountants have informed the study team that administrative over-
head for the project (consisting of management salaries allocated on the basis of
revenues) will be 20 per cent of revenues each year. Independent of overhead consider-
ations, it will be necessary to hire a full-time counter-service manager at an annual salary
of £10 000 to replace the person who will now be devoted full-time to on-site servicing.
The latter’s remuneration is £15 000 per annum. Management remuneration is expected
to increase by 10 per cent per annum.
Marketing outlays consist of a £25 000 market survey recently completed by the
company, and annual expenses of £15 000 in the first year and £10 000 in the second
and following years. No increase in marketing outlays is predicted past that point.
Assets
PC Problems must acquire service vans, which will cost a total of £54 000. These vans
will be depreciated by the straight-line method over a three-year life with a book
(accounting) salvage value of £9000. It is expected that the vans could be sold in year
five for £15 000. The communications equipment of choice appears to be the new
portable cellular telephones, which will cost £4000 in total (their operating charges
being included in the direct costs described above). The phones will be depreciated by
the straight-line method on a four-year life with no book salvage value. They will likely
become obsolete by the end of the five-year project life.
It is assumed that any other assets (or liabilities) held by PC Problems as a result of
this project can be liquidated at the end of year five for their book values.
Government
PC Problems pays taxes on accounting income at the flat rate of 52 per cent. Interest
and depreciation are deductible expenses, as of course are all typically deductible
operating costs.
Capital suppliers
The company is planning to borrow enough money to pay for the vans and cellular
telephones. The interest and principal payment expectations for these are (£000s):
t1 t2 t3 t4 t5
Interest 6.96 5.22 3.48 1.74 0
Principal 14.50 14.50 14.50 14.50 0
2 PC Problems also must decide whether the adoption of the cash flows you have
estimated above is desirable. Please advise the company about this, using the WACC–
NPV criterion.
3 What do you think of the borrowing plans described in the information above? Is this
useful information? What relevance does it have for this investment analysis? Is there
any particular technique of investment analysis that would utilise that information
differently from the way that you have?
(Hint: A company’s WACC is in part a function of the proportion of debt it uses in
financing itself. To test this for PC Problems, recall that the new value increment will
be NPV + FCF ∗ and the borrowing is £58 000.)
4 Upon the presentation of your report, a marketing manager of the company raises the
point that the analysis has not considered the possible erosion of carry-in service
revenues due to the new service being offered. How should you respond to such a
question?
Learning Objectives
Module 6 presents a wide range of tools for companies making real-asset investment
decisions (called ‘capital budgeting’). The module begins with a review of the set of
techniques that are commonly used, and this set includes, in addition to the NPV
and IRR techniques already studied, the ‘payback period’, ‘return on investment’,
‘profitability index’ and ‘cost–benefit ratio’. The module discusses each of these,
pointing out situations when each is worthwhile and when use could produce
incorrect results. Module 6 also illustrates investment mutual-exclusivity situations
where IRR is potentially misleading as an evaluation technique, and develops a
variant of IRR that corrects that difficulty. Other topics include deciding upon
investments when capital is in limited supply (i.e. rationed), and dealing with causal
economic interrelatedness among investment cash flows, with ‘repeatable’ invest-
ments, and with inflation. Students will emerge from studying this module with a
much better ‘toolkit’ of specific techniques to apply in various realistic environments
for company decision making.
6.1 Introduction
In the preceding modules we have dealt at some length with the NPV technique,
and its close cousin APV, in corporate investment decisions. Our extensive atten-
tion to these two approaches is warranted because they are the methods of
investment analysis that hold the greatest general promise of producing shareholder
wealth-increasing decisions. But by no means are NPV and APV the only tech-
niques used by companies in their investment decisions. The most widely used
competitors of NPV and APV are:
1. Payback period
2. Average (accounting) return on investment (AROI)
3. Internal rate of return (IRR)
4. Cost–Benefit Ratio (CBR) and Profitability index (PI)
We shall examine each of these investment evaluation methods, contrasting them
with our preferred techniques. (Henceforth in this section we shall refer to NPV
and APV collectively as NPV because the comparisons apply to both, and it saves
space.) The end result of these comparisons will be that all of the alternatives to
NPV have significant shortcomings, and should not generally be used unless an
NPV analysis is performed concurrently.
t0 t1 t2 t3 t4
A −£1000 +£600 +£500 +£300 +£200
B −£1000 +£500 +£400 +£500 +£400
Suppose that the company had decided that a two-period payback period was the
maximum acceptable for a project. A glance at the above shows that investment A is
acceptable and B is not. But suppose also that the appropriate discount rate to apply
to the above projects is 10 per cent. Project A’s NPV is £321, while B’s is £434. So
it is clearly possible that payback period and NPV can yield different answers. And
if NPV decisions maximise shareholder wealth, payback period must be doing
something other than that.
Upon seeing the above example, you might be tempted to argue that we have
designed it to make payback look bad, by choosing a minimum time that conflicts
with the 10 per cent discount rate. Actually, we did. But that is not to say that
payback period can generally be made to work correctly (i.e. to give the same
decisions as NPV) by carefully choosing a minimum time period. Suppose, for
instance, that we had specified a four-period payback. Then project B would have
been allowed under payback, and preferred under NPV. But A nevertheless still has
the shorter payback, and if the technique is applied both relatively and absolutely, as
it often is, A remains the investment of choice regardless of the fact that sharehold-
ers would be better off with B.
We could continue with examples, but to little gain. The problems with payback
period are:
1. It ignores all cash flows beyond the minimum acceptable payback period, even
though there are often likely to be cash flows of importance beyond that time.
2. It does not discount the cash flows within the minimum acceptable period,
thereby effectively giving equal weight to all cash flows within those periods.
This is inconsistent with shareholder opportunity costs on the monies invested.
These characteristics of the technique will in many cases cause investments to be
erroneously accepted and rejected.
In recent years, bowing to the pressure of modern times, some companies have
altered their payback period techniques to be ‘discounted payback period’. This
procedure calculates the present values of cash flows, and dictates a minimum
acceptable period until the discounted cash flows accumulate to equal the initial
outlay. This alteration does away with our second complaint about payback period,
but leaves the fact that cash flows beyond the accumulation point are ignored. In
case you might be interested, the discounted payback periods of A and B are 2.18
and 2.57 periods respectively. (Both projects require all of their first two discounted
cash inflows plus the indicated percentage – 18 per cent for A and 57 per cent for B
– of the discounted third period inflow to total the initial outlay.)
Why do some companies still use payback period with all of its faults? A survey
of companies using this investment technique would probably indicate their
preference for it depends upon its ease of calculation, simplicity and ‘understanda-
bility’, along with the capacity to adjust for project riskiness by decreasing the
required payback period. It is difficult to take seriously statements about ease of
computation with the aids available to that end today; and simplicity alone is also
difficult to accept because there are lots of even simpler techniques (like flipping a
coin, or consulting a fortune teller). We think that the payback-period method is an
artefact of the times before discounting techniques were well understood, and when
concepts of risk were quite rudimentary. If you think about it a little, you can see
that the shorter the minimum payback period, the ‘safer’ will be the acceptable
projects, because of the bias toward near-term cash payout. This attribute of
payback period is of course not generally to be desired above the ability to discover
the wealth-changing promise of investments, but was probably used to ensure that
some minimum standard of cash-flow return safety resided in investments. There
are superior techniques available today to accomplish this end, as we shall see in a
later module.
Should a company feel it must use a payback criterion, the maximum allowable
period itself should be set with some basis in reason. To this end, a rudimentary
estimate of such a maximum can be found by:
1 1
Payback ∗ ∗ ∗
1
where n is the number of periods in the project’s total lifetime. Among other serious
restrictions, this payback is accurate only for projects with fairly constant cash flows
each period. Nevertheless, within these restrictions, the above will indicate the
number of periods across which, if the original outlay is not returned (in FCF*), the
investment will have a negative NPV.
1 If something begins at a value of £9000 and steadily declines to £0, its average must be £4500 across
time.
Investment t1 t2 t3
Profits
C £3000 £2000 £1000
D £2000 £2000 £2000
E £1000 £2000 £3000
Free cash flow
C £6000 £5000 £4000
D £5000 £5000 £5000
E £4000 £5000 £6000
It is (or should be) clear to you that regardless of the discount rate used with
NPV as the criterion (as long as the rate is greater than zero) project C is the best
project. But AROI would rank all projects as equally acceptable, since the AROI of
each is their average profit divided by average assets: £2000/£4500 0.44444 …
Whether or not these investments would be acceptable under the AROI method
depends upon the standard chosen. It is not generally possible to choose a standard
minimum AROI that will generate the same answers as NPV.
Why do many companies continue to use the AROI technique? The answer is
probably that the habit of judging the result of an endeavour by looking at its
accounting profits has been erroneously carried over to prospective investments.
Nevertheless, we can also report that the use of AROI as an investment evaluative
method is very much on the decline, and justifiably so. It is the rare company that
relies solely upon AROI to decide on the projects it will accept and reject.
AROI is, however, often used as an evaluative or control device to check the
progress of an ongoing project on a period-by-period basis. This use of AROI is
more defensible in that such single-period deviations from targets are less subject to
the time-based errors of AROI than are decisions about prospective investments.
This situation produces a necessity for well educated finance students to be con-
versant with both techniques. So another purpose of this section is to show you
how to use IRR in a manner that is most likely to produce ‘correct’ investment
decisions (that is, decisions identical with those via NPV).
In Module 1 you learned certain things about IRR that we have no intention of
reteaching you here. These introductory matters are covered in Section 1.3.4 and
Section 1.4.4 of that module, if you need a review. Further comparison between IRR
and NPV occurred in our worked example of the WalkPhone in Section 5.3.
In those sections:
1. The IRR was defined as the average per-period earnings rate on the amounts
(market values) invested in the project across time.
2. The IRR calculation process was described as finding the discount rate that
causes NPV to equal zero, and we said that the only way to do that in most cases
is by a ‘trial and error’ search method of using various discount rates until you
find one that creates a zero NPV.
3. We indicated that IRR is comparable with a ‘hurdle rate’ or minimally acceptable
rate of return to judge an investment’s desirability. In the case of a corporate
investment project wherein the FCF* free cash flows are used, the appropriate
hurdle rate is naturally the project’s cost of capital, rv*.
If you are not comfortable with these ideas, you should review the appropriate
sections of Module 1 and Module 5.
Before beginning our discussion of IRR vs. NPV, we should mention one bad
argument that is often made in favour of using IRR instead of NPV. This assertion
is that the IRR is preferable because it requires less information: in order to calculate
an NPV there needs to be known both a discount rate and the relevant cash flows,
whereas in order to calculate an IRR only cash flows are necessary. This of course is
true in one sense, but begs the question of what is done with the IRR once it is
calculated: the IRR must be compared with a hurdle rate – and the correct hurdle
rate to use for the IRR is the discount rate that an NPV analysis would use. So the
information necessary to accomplish the two analyses is identical, and not a basis
upon which to choose either.
In a sense our introductory remarks about IRR vs. NPV at the beginning of this
section were a bit overstated, because in many instances the two techniques will
yield the same decision. But it is important that you are able to recognise the
situations in which that is not true, because using the IRR in those decisions can be
a serious error. We shall now start our discussion of those instances where NPV and
IRR do indeed give different answers.
Most of the time, when the decision to be made is simply whether or not a par-
ticular project should be accepted (so that the choice does not involve finding the
best among a set of competing projects), IRR gives the correct answers. There is,
however, one exception to this rule.
Our earlier discussions of IRR in Module 1 ended with an example that generated
Figure 1.4, which we reproduce here in a more generalised form as Figure 6.1. You
recall that the outcome of the discussion in Module 1 was to show that IRR and
NPV give the same answer when the relationship between a project’s NPV and its
discount rate is a smooth, negatively sloped curve like the one in Figure 6.1. As long
as the IRR is greater than the hurdle rate (the discount rate used to find NPV), the
NPV will be positive. And we accept projects when the NPV is positive, or when
the IRR is greater than its hurdle rate. We have good news and bad news about the
idea shown in Figure 6.1.
NPV
IRR
0 Discount rate
t0 t1 t2
−£12 000 +£75 000 −£75 000
If you were to calculate the IRR of this project you would find that it had no less
than two IRRs, 25 per cent and 400 per cent. Figure 6.2 shows the relationship
among discount rates, IRR and NPV for this project.
NPV
0 25% 400%
Discount rate
and turn to NPV. The danger with IRR is that even though you may get answers,
the economic interpretation of them is either difficult or impossible. It is within the
capacity of finance to find specific ‘fixes’ for the IRR in such situations, but not
within the scope of an introductory course. If you are in a company that uses IRR,
and you are occasionally faced with such projects, you can make a valuable contribu-
tion by making certain that everyone is aware of these attributes of the IRR
technique.
What is the good news? It is simply that there are not too many individual pro-
jects that exhibit cash-flow patterns that produce anomalous IRRs. So the initial
statement we made about the IRR’s usefulness still holds: when the only question is
whether an individual investment should be accepted or rejected, the IRR usually
works well. But watch out if the cash-flow pattern of the project is not our classic
‘early outlays followed by subsequent inflows’ – if you use IRR in such situations
you will sooner or later make a bad decision.
There is one other situation in which IRR can cause problems not encountered
by NPV. We have been assuming throughout that multiple-period cash-flow
investments can be evaluated using the same discount rate for every cash flow
across time. If that is a correct procedure for NPV, it must be acceptable for IRR,
because the discount rate for NPV is the hurdle rate for IRR. But suppose that a
single rate could not be correctly applied, and different rates were necessary for each
cash flow (as we illustrated with bonds in Module 1). This presents no problem for
NPV; we merely use the different rates on each cash flow, as we did in Module 1.
But IRR has a problem here. IRR is by its nature a single rate calculated from a
group of cash flows. You can find the cash flows’ IRR, but to which of their
discount rates would you compare the IRR? It would not be correct to compare it
to any of their rates. To find a correct hurdle rate, you would have to find the yield
to maturity of a security with the same risk and cash-flow pattern as the investment
in question. The likelihood of regularly finding such securities when necessary is
low. We do not at this time, however, wish to make too much of this indictment of
IRR. The reason is that almost all companies use only a single discount rate for the
cash flows of a project when calculating its NPV. That being the case, until compa-
nies start using the term structure of interest rates to estimate individual period
discount rates for NPVs, it is not really fair to criticise IRR alone for this shortcom-
ing.
therefore preferred to the others. Unfortunately the same test will not in general
work for the IRR.
Consider the investments between which you decided in Review Questions 1.12
and 1.13 at the end of Module 1. The cash flows of the projects are:
t0 t1 t2 t3
No. 1 −£2500 +£1200 +£1300 +£1450
No. 2 −£2500 +£1300 +£1300 +£1300
If you did the problems correctly, you discovered that investment in location No.
1 has a higher NPV than investment in location No. 2 at a discount rate of 10 per
cent, but No. 2’s NPV is greater than No. 1’s at a discount rate of 25 per cent. How
can such a thing happen? In the answers given we show how through Figure A4.3,
which we reproduce here in slightly modified form as Figure 6.3.
NPV
Investment 1
Investment 2
Investment 2
Investment 1
serious problem with IRR, because the situation we illustrate here is not at all
uncommon. Competing projects’ cash-flow patterns are not always such that they
‘cross-over’ at some discount rate less than the IRR as do No. 1 and No. 2, but it
happens often enough that you should be concerned about it. And particularly
insidious is that, unlike the multiple-rates problem with IRR (when sign changes
signal a problem with IRR use), there is no simple way to examine the projects’ cash
flows and know beforehand that this is going to happen.
What should be done? In the best of all worlds, our advice is to forget the IRR
and use the NPV for such comparisons because it is not subject to this problem.
But sometimes you do not have that luxury; your company may have as a policy that
IRR is the technique of choice, or your boss may not want to hear anything but
comparisons of rates of return. What then? We shall now teach you a technique that
can be used in such situations (indeed should be used in all comparisons of
investments where IRR is the evaluation method of choice).
This addition to the IRR technique is known in finance as incremental cash-
flow analysis. As we said above, it should be used whenever attempting to choose
among investments on the basis of their IRRs. The technique is not very complicat-
ed, but there are enough steps in it so that it is easier to remember if they are in a
list. (A list of steps to take is called an algorithm, so we are giving you an algorithm
to use when choosing the best of mutually exclusive projects on the basis of IRR.)
Here are the steps:
1. If you have a lot of projects to decide among, think of them as in a pot, and pick
any two from the pot.
2. Find the one that has the highest net positive cash-flow total (the simple sum of
all of its FCF*s, which can be thought of as an NPV calculated with a discount
rate of zero). Call the investment with the highest net positive cash-flow total the
‘defender’, and the other one the ‘challenger’.
3. At each time point, subtract the cash flows of the challenger from those of the
defender, and call the resulting stream the ‘incremental cash flows’.
4. Find the IRR of the incremental cash flows.
5. If the IRR of the incremental cash flows is greater than the appropriate hurdle
rate, keep the defender, and throw out the challenger. If the IRR of the incre-
mental cash flows is less than the appropriate hurdle rate, keep the challenger,
and throw out the defender.
6. Omitting the thrown-out project(s) and keeping the survivor of the procedure to
this point, return to the pot, pick another investment, and begin the algorithm at
step 2. When all available projects have been tested, you will have a single survi-
vor that has not been thrown out. Call that one the ‘winner’.
7. Calculate the IRR of the winner. If it exceeds the hurdle rate, accept the winner;
if the winner’s IRR is lower than the hurdle rate, reject the winner and all other
projects that were in the pot.
We shall explain to you why this works in just a moment. First, you should see
how to use the technique with an example, and the cash flows of the investments in
locations No. 1 and No. 2 used earlier will serve that purpose well. Let us assume
that the appropriate discount (hurdle) rate for these projects is 10 per cent. Taking
the steps of the algorithm in order, we have the following:
1. We choose No. 1 and No. 2 because they are the only ones in the pot.
2. Adding up the cash flows of each, we see that No. 1’s equal £1450, and No. 2’s
are £1400, so No. 1 is the defender and No. 2 is the challenger.
3. Subtracting the challenger’s cash flows from the defender’s, we arrive at the
incremental cash flows:
t0 t1 t2 t3
No. 1 −£2500 +£1200 +£1300 +£1450
No. 2 +£2500 −£1300 −£1300 −£1300
Incremental cash flows 0 −£100 0 +£150
4. Through trial and error, we discover that the IRR of the incremental cash flows
is about 22.5 per cent.
5. Since the IRR of the incremental cash flows is greater than the hurdle rate
(22.5% > 10%), we keep the defender (No. 1) and throw out the challenger (No.
2).
6. We return to the pot, and discover that we have exhausted its contents, so the
last defender (No. 1) becomes the winner.
7. We calculate, by trial and error, that the winner’s IRR is greater than the hurdle
rate (No. 1’s IRR is slightly less than 25.9 per cent, and the hurdle rate is 10 per
cent), so we accept the winner, No. 1, as the better of the two projects.
After working through the algorithm in this way you might be tempted to think
that this is a great deal of trouble to go to when we could have simply calculated the
IRR of No. 1 in the first place, and found that it was acceptable. A glance at
Figure 6.3 reminds us how wrong that attitude would be. Since our task is to choose
between No. 1 and No. 2 on the basis of IRR, No. 2 (with an IRR of slightly less
than 26 per cent) having the higher IRR seems to be the logical choice. But with a
discount rate of 10 per cent, No. 1 is the better project. Only by dealing with
incremental cash flows are we able to use the IRR to choose No. 1, the correct
answer.
Why does this algorithm work? One way to see why it does is to consider the
economic interpretation of the IRRs of incremental cash flows, since that is the
main decision variable in the algorithm. Look again at steps 3, 4 and 5 of the
calculation above. The incremental cash flows (ICF) are the differences between the
cash flows of No. 1 and No. 2, or:
ICF CF No.1 ― CF No.2
By calculating the IRR of the incremental cash flows we are testing whether
investing in the difference between No. 1 and No. 2 is worthwhile. If the IRR of
the incremental cash flows is greater than the hurdle rate, that tells us it would be
and that would be the correct choice if all of both No. 1’s and No. 2’s interim cash
flows could be reinvested at the IRRs of the respective projects. In other words, if
we could expect to invest the stream of No. 1’s interim cash flows at a rate of
slightly less than 25.9 per cent, and those of No. 2 at slightly less than 26.0 per cent
(their IRRs), No. 2 would have a higher NPV than No. 1, including the reinvest-
ment outflows and inflows.
On the other hand, using the NPV technique, if interim reinvestments are not
explicitly included (which is the usual procedure) this implicitly assumes that interim
cash flows can be invested at the discount rate used in the NPV analysis. Most
finance people who worry about this question think that the NPV assumption of
investing interim cash flows at the opportunity costs of capital suppliers (rv*) is
more viable than the IRR’s assumption of interim investments being available that
return the same rate as the one being analysed. We agree with them. Of course,
because it gives the same answers as NPV, the incremental cash-flow algorithm
causes the IRR to make the same interim cash-flow investment assumption as the
NPV.
With the incremental cash-flow algorithm, the IRR technique can be made to
operate in many corporate investment situations where a ranking or choice among
alternatives must be made. Without the use of the algorithm or an equivalent
method, the IRR will not as a rule give correct indications of relative investment
desirability. Even with the incremental cash-flow method of IRR, however, there
can be problems. These are sufficiently significant that we recommend that, should
you encounter any of the situations listed below, you abandon the IRR method and
use only the NPV technique. Educating your boss on the reason why you think this
is the correct thing to do will give you some feeling for the challenges in writing a
finance course such as this.
The situations in which the incremental cash-flow method of choosing among
investments on the basis of IRR should never be used are:
1. When the incremental cash flows that you calculate have more than one change
of sign across time. This is the same problem as we discussed in the first part of
this section. The algorithm does not fix it.
2. When the projects differ in risk or financing, so that they require different hurdle
rates. Rereading the paragraph above on the graphical explanation of the algo-
rithm will show why this presents problems.
Even when the above situations appear, the algorithm can be made to work
correctly. But the task of doing so will be ridiculously complex in comparison
simply to using the NPV, which always works well.
and the cash outflows in the denominator. If the NPV would have been positive,
the CBR will be greater than 1; a negative NPV is the same as a CBR less than 1. It
is clear that the NPV and CBR will produce the same investment recommendations
if the same cash flow and discount rate information is used in each, when faced with
the question of an investment’s desirability.
The CBR, however, has some of the same problems as the IRR when asked to
choose among mutually exclusive investments. Just as the IRR errs in choosing
high-return investments over those with high wealth-increasing expectations, the
CBR is attracted to those investments that have the greatest ratio differences
between inflow and outflow present values instead of actual cash or value differ-
ences.
Consider the two investments, M and N, below:
t0 t1 t2
M −£100 +£200 +£400
N −£1000 +£1500 +£3500
At a discount rate of 10 per cent, the CBRs of both are greater than 1, so both
are acceptable. If they are mutually exclusive, however, the CBR indicates that M is
preferred (CBR = 5.12) to N (CBR = 4.26). The NPV of M is 412 whereas the NPV
of N is £3256.
The CBR is giving an incorrect signal as to which of the two investments should
be preferred if they are mutually exclusive. The reason is that it is a ratio of values
rather than a value measure itself. Capital claimants are concerned about their
absolute wealth – which in the final analysis is measured in value amounts not ratios.
If the NPV is available, it should always be used in preference to the CBR.
t0 t1 t2 t3
No. 3 −£1000 +£500 +£600 +£700
No. 4 −£2000 +£900 +£1000 +£1300
The PI of No. 3, using a 10 per cent discount rate, is 1.476, whereas the PI of
No. 4 is 1.311 at the same discount. The PI tells us (correctly) that both investments
are acceptable. But suppose that the two are mutually exclusive. The PI would seem
to say that No. 3 is better than No. 4 because the former has the higher PI ratio.
Were you to choose No. 3 over No. 4 on that basis, you would be making a mistake.
The NPV of No. 3 is £476.33, while No. 4’s is £621.34. No. 4 is the better choice if
the projects are mutually exclusive.
The reason that the PI is unsuitable for ranking investments is because it displays
another relative measure, namely the wealth increase per pound of initial outlay
instead of the wealth increase itself. One can easily run into a situation where the
investment that would promise the greatest wealth increase has the lower wealth
increase per pound of initial outlay (or PI) because its initial outlay is higher.
Though you may not earn as high a return on each pound invested in No. 4, it has
the dual virtues of returning more than its opportunity cost, and also allowing you
to invest a greater amount of money at that desirable rate than does the alternative
investment. (Naturally, if you could have invested £2000 in No. 3, and thereby
doubled its future cash-flow expectations, it would be the more desirable. We
would, however, maintain that such opportunities to expand outlay without reduc-
ing rate of return are rare to the point of non existence in real asset investments.)
This explanation sounds very much like the one offered for the CBR above. In
fact, with minor substitutions, we could have simply duplicated the CBR explana-
tion, and saved some space. The astute reader may have already noticed that the
CBR measures for No. 3 and No. 4 are identical to their PIs and the same thing
holds for M and N. See if you can figure out why, and when that would not happen.
A final postscript to our discussion of alternatives to the NPV must deal with
nomenclature. We have used names for the various techniques that we believe are
the most commonly accepted ones. You should be aware, however, that these same
techniques are regularly found lurking in corporate finance departments under a
wide range of aliases. The IRR is often called the ‘discounted cash-flow rate of
return’; the PI is sometimes called the CBR, and vice versa; the accounting return on
investment on occasions goes under the name of the ‘average return on book value’,
the ‘accounting rate of return’, or one of a number of other titles. Because one can
rarely be certain of exactly what a particular name signifies, one should without
embarrassment ask for the particular definition of any investment technique offered
for consideration.
2. The external capital markets disagree with the corporation as to the desirability
of the investment. In this situation the company may find itself unable to raise
the amounts of cash at the rates of return or interest that its analyses of projects
indicate should be available from the market. When that happens, and if the
market cannot be convinced of the company’s opinion, there will be an effective
limit on the money that can be invested in ‘acceptable’ projects.
One can doubtless uncover other situations in which a corporation with desirable
projects cannot find the money to undertake all of them, but the above are common
enough to require that financial analysts have some method for choosing the group
of projects that will maximise shareholder wealth while using no more than the
funding available.
In finance we use the term capital rationing techniques for the set of methods
designed to deal with this situation. The goal of the financial manager is, as ever, to
maximise shareholder wealth. But now the constraint on the total funds available to
finance desirable investments requires that we accept the set of desirable projects
that both is within the ‘budget constraint’ and yields the highest NPV. There are
many techniques that financial analysts use to accomplish this goal, ranging from the
quite straightforward to the very sophisticated. We shall illustrate the former and
briefly discuss how you can gain access to the latter.
Suppose that your company had the following opportunities to invest in assets,
with the associated expected results:
Outlay at NPV
t0
E £1000 +£300
F £2000 +£560
G £5000 +£870
Though the question can only arise within a capital-rationing situation, the an-
swer has less to do with the fact that outlays are constrained than it does with the
simple economics of the NPV calculation. Anyone who is troubled by the above
solution needs a brief refresher course in NPV: remember that the NPV is the
present value of the amounts by which the future cash flows of a project exceed
those that a comparable-risk outlay in the capital market would yield. This means
that the additional £2000 invested in G is expected to return not only the £10
greater NPV, but also the required return on the £2000 itself. To see this, suppose
that the three investments above were single-period ones and that the appropriate
discount rate for all was 10 per cent. The cash flows of the investments can then be
deduced to be:
With this data we can see that an additional £2000 invested in G produces an
additional £6457 – £2816 – £1430 £2211, a return in excess of its opportunity
cost. Note that accepting E and F along with investing the unused £2000 at the
opportunity cost of 10 per cent is not as good as accepting G, because the return on
this £2000 investment would be £2200 whereas G returns £2211 on the ‘extra’
£2000 invested in it relative to E and F. Were these investments to be multiple-
period, perhaps even of different number of time periods, the illustration would be
more complex but the economics of the argument identical.
In the illustration we have seen to this point, the capital-rationing situation pre-
sents no particular analytical problem. We simply look at all possible combinations
of investments that lie within the budget constraint and choose the package that has
the greatest NPV. That kind of procedure is called ‘exhaustive enumeration’, and
works well as long as there are not too many potentially acceptable projects. But it is
extremely cumbersome when there are many projects to be considered. In that
situation, exhaustive enumeration, if you remember your elementary school arithme-
tic of combinations, can be exhausting indeed. When there are many projects to be
considered, in order to find which combination fits within the budget constraint and
has the highest NPV, finance turns to more sophisticated search techniques.
Suppose that you were the financial analyst in a company that had the investment
opportunities below:
Outlay at t0 NPV PI
H £5 000 +£3 000 1.60
I £10 000 +£5 000 1.50
J £20 000 +£6 000 1.30
K £1 000 +£200 1.20
L £1 000 +£150 1.15
M £4 000 +£560 1.14
N £1 000 +£100 1.10
O £3 000 +£150 1.05
One of the commonly used methods of dealing with the capital-rationing prob-
lem when there are many projects is to calculate the profitability indices for the
investments, and to list them in declining order of PI. The investments are then
accepted in order of PI, until the budget has been exhausted. A project may be
‘skipped’ because its outlay is too large, and the next one having a small enough
outlay taken as you work down the list.
You can see that this works nicely in the list above for budgets of £35 000
(choose H, I and J), £36 000 (choose H, I, J and K), £37 000 (choose H, I, J, K and
L) and £38 000 (choose H, I, J, K, L and N). But look what happens with a budget
of £39 000. The company must abandon the PI procedure and choose H, I, J and
M, the last one having a PI less than the rejected projects K and L. So the PI
technique must be used with some caution in ranking investments to be chosen
under capital rationing. The problems tend to occur when the highest PI projects do
not use up the entire budget (such as H, I, J, K, L and N under a £39 000 con-
straint).
For most companies of even modest size today, access to computer facilities
(personal computers included) means that they need not rely upon hit-or-miss
methods like the PI procedure when faced with a capital-rationing problem. There
exist inexpensive software packages that contain the sophisticated techniques
necessary to solve these problems correctly and quickly. (Look for ‘integer pro-
gramming’ as the key words, and ask someone who understands optimisation
techniques – an ‘operational research’ or ‘management science’ person – to tell you
how to use it, if you cannot understand the instructions.) These sophisticated
programs are little more than efficient search methods that go on within the
computer to find the package of investments that has the highest total NPV, and
that is within the budget constraint.
Capital-rationing situations do occur and require that appropriate analytical pro-
cedures be brought to bear in order to arrive at the best solutions. But you should
remember that being under capital rationing in the first place is an undesirable
situation. It means either that you have not been able to solve your internal organi-
sation and communication problems, or that you have been unconvincing to the
capital market about your prospects – both failures of some significance. They imply
that you will be forced to forego investments that would have increased the wealth
of your shareholders.
One last warning. By no means should you ever interpret the existence of high
market required rates as a capital-rationing situation. When interest rates are high, it
is simply a signal that your capital costs are also high. The market stands ready to
provide capital, but at a rate that may cause you to arrive at a negative NPV for
projects under consideration. The capital-rationing situation implies that financing
beyond the budget constraint carries not a high but an infinite cost. The distinction
is important, especially when arguing the politics of capital budgets within compa-
nies.
NPV
Funeral parlour £265 000
Rock-concert arena £327 000
NPV
Funeral parlour, no rock-concert arena £265 000
Rock-concert arena, no funeral parlour £327 000
Funeral parlour, rock-concert arena next door −£126 000
Rock-concert arena, funeral parlour next door £327 000
NPV
1. Build nothing 0
2. Funeral parlour £265 000
3. Rock-concert arena £327 000
4. Funeral parlour and rock-concert arena £201 000
NPV
1. Build nothing 0
2. Funeral parlour £265 000
3. Rock-concert arena £327 000
4. Funeral parlour and rock-concert arena £201 000
Notice that if all possible combinations have been listed, the combinations must be
mutually exclusive; in other words, if Global accepts any one of the above four, it
automatically rejects the other three. Whether one has successfully listed all possible
combinations can be tested by their being mutually exclusive. As an exercise, see if
you can list the combinations that Global must decide among, were the record shop
to be included in the analysis. Your answer should be:
build nothing
funeral parlour
rock-concert arena
record shop
funeral parlour and rock-concert arena
funeral parlour and record shop
rock-concert arena and record shop
funeral parlour, record shop and rock-concert arena.
Next, once all possible combinations of interrelated investments have been speci-
fied, being mutually exclusive, the combination with the highest NPV can be
chosen. (In the initial Global example, it is the rock-concert arena alone. In the
expanded group, it is probably the arena and the record shop.)
More generally, when facing a set of investments whose cash flows are interde-
pendent, the financial manager would do well to consider the various
combinations that are available to be separate investments themselves. In other
words, as far as the Global analysis is concerned, undertaking the funeral parlour
and the rock-concert arena is a separate investment that has characteristics that can
be very different from those implied by undertaking either of the two alone.
t0 t1 t2 t3
Largemouth −14 −3 −3 −3
Lunker −7 −5 −5
If 10 per cent is an appropriate discount rate for these cash flows, the Largemouth’s
NPV is −£21 461, while the Lunker’s is −£15 678. Since these are ‘cost minimising’
investments, the Lunker would seem the best, because it has the least negative NPV.
But the cash flows listed above for the two machines are not really descriptive of
those that the company expects to incur from choosing one over the other. Choos-
ing the Lunker implies purchasing another Lunker at t2, t4, and so forth. The
Largemouth must be replaced at t3, t6 etc. Of course, each continues to incur its
annual cash outlays during those times. Actually, Bayou Software has no particular
plans ever to stop using whichever machine it adopts, so the stream of cash flows
for each can be regarded as continuing indeterminately far into the future.
Thus the actual expectations for the machines are quite complex, being very long,
non-constant cash flows across the future. This would seem to present a formidable
problem in present-value calculation, and can in fact be exactly that. The most
obvious logical answer to setting up a manageable analysis is to extend the cash
flows as described above far enough into the future until the machines require
replacement at the same time point. If we then calculate the NPVs, the numbers will
not be the present values of the entire stream of costs that the machines will incur
but will instead be of the same relative magnitudes of the entire stream’s NPVs.
This is because all cash flows to the future of the point where their replacements
coincide will be multiples of those up to that time point.
Since the Largemouth’s life is three years and the Lunker’s is two, we need a total
of six years until their replacements will coincide (again in £000s):
t0 t1 t2 t3 t4 t5 t6
Largemouth −14 −3 −3 −17 −3 −3 −3
Lunker −7 −5 −12 −5 −12 −5 −5
This display is the cash flows of the two machines up to the point that they first
expire at the same time. (Note that the £17 000 outflow for the Largemouth at t3
and the £12 000 outflows for the Lunker at t2 and t4 represent the sum of a new
machine’s purchase cost and an operating outlay, as would be the case if Bayou
wished to avoid downtime in replacing machines.) We can now calculate NPVs that
have some comparability. Under the replacement cycle above and with a 10 per cent
discount rate, the Largemouth’s NPV is −£37 584, and the Lunker’s is −£39 343.
This result contrasts with the NPVs for a single lifetime for each machine, which
indicated that the Lunker was the lower-cost alternative. In fact, if the machines are
to replace themselves over time, the Largemouth is cheaper, because its somewhat
higher initial outlays are spread further apart than the Lunker’s, and the Large-
mouth’s lower annual operating costs are now compared to the Lunker’s for an
equal number of periods, as opposed to the single-life procedure, which used fewer
operating periods for the Lunker than for the Largemouth. So Bayou has narrowly
missed accepting the wrong machine, and will correctly choose the Largemouth
Special diskette copier.
The procedure above will work for all problems of this type, but can itself often
become cumbersome. That happens when the common replacement of alternatives
will happen very far into the future. For example, suppose that the Largemouth had
a seven-year and the Lunker a nine-year interval. One must shudder at the sixty-
three-year discounting period that would be necessary in that situation! There are
other types of asset investments that can generate even worse combinations (try a
new building that could go for either twenty-one or thirty-one years). Fortunately,
finance has discovered a technique that can deal with such problems, and which we
can also recommend for cases like our original Largemouth–Lunker example. This
procedure is a rather elegant application of an idea with which we are already
familiar: annuities, as discussed in Module 1.
An annuity, you will recall, is a stream of cash flows which is constant across time
(these are the ‘£1 per period’ present value tables in Appendix 1). Think of the
situation this way: if the alternatives each had absolutely constant cash flows across
time for ever, there would be no necessity to find their present values. We could
decide which was preferred by simple inspection of any period’s cash flow. In the
case of cost-minimising investments, we would simply choose the alternative that
had the lesser constant outflow in perpetuity, and we could be confident that we
had chosen the least negative NPV.
The application of that idea to the Largemouth–Lunker type of problem is not
obvious, however, because the investments do not have constant cash flows over
time. But with a little effort we can make the analysis behave as if they do. The idea
is to turn the non-constant perpetual streams into constant perpetual streams of the
same present values. What we do is find the NPV of a single cycle for each asset,
and divide that number by the annuity present-value factor for the number of years
in each asset’s replacement cycle at the appropriate discount rate. The result is the
constant annuity outlay per period that has the same NPV as the asset. We then
need only compare the per-period equivalent annuity outlay for each asset, and
choose the one with the (equivalent constant) lower cost per period.
This sounds more difficult than it really is. Let us return to Bayou Software and
apply this new procedure. First, recall that we have already calculated the single-
cycle NPVs for the alternatives; the Largemouth’s is −£21 461, and the Lunker has a
single-cycle NPV of −£15 678. The present value tables in Appendix 1 (Table A1.2)
indicate that the annuity present value factor for the Largemouth is 2.4869 (three
periods, 10 per cent), and for the Lunker is 1.7355 (two periods, 10 per cent).
Following the instructions above, we divide the single-cycle NPVs by the annuity
factors, and arrive at constant annual equivalent cash flows of −£8630 for the
Largemouth and −£9034 for the Lunker. This tells us that if we accept the Large-
mouth it is the same thing (i.e. has the same NPV) as accepting a constant cash
outlay of £8630 per period, whereas the Lunker implies the equivalent of a £9034
outflow per period. So we would be led by this technique to accept the Largemouth
and reject the Lunker, which we already know is the correct decision.
The application of this equivalent annual cost technique to this particular ex-
ample requires about the same amount of effort as simply stringing out the cash
flows until they end at the same time point. But that is not the case for most such
comparisons – we designed the Largemouth–Lunker comparison to finish in only
six periods. If you are not convinced that this is an easier way, try doing the example
again for the seven- and nine-year cycles mentioned above. You will quickly come to
appreciate this approach.
In summary, when a company faces a decision between renewable assets that are
of different lifetimes, the simplest way to make a correct decision is to find the
constant periodic cash flow having the same present value as a single lifetime or
cycle for each asset. Analysts can then simply compare the periodic amounts and be
certain that the correctly calculated NPVs would have the same rank-ordering. It is
generally not appropriate to compare NPVs of single cycles for alternative assets in
this situation.
inflation, and that investment analyses, to be done correctly, must account for this
phenomenon.
Our first task is to understand enough about the economics of inflation to man-
age its corporate financial aspects. Fortunately, this will not require that we become
macroeconomists, not only because it is not clear how much they really understand
about inflation, but also because much of their concern is why inflation occurs and
how to control it. We cannot be concerned with those issues in this context. Our
perspective is simply: given that inflation is an important dimension of investment
decisions, how is it best treated?
Inflation is an increase in price unaccompanied by any other changes (such as
quantity, quality, and so forth). With inflation, we must pay more money across time
to acquire the same goods and services. If the inflation rate is 5 per cent in a period,
something that cost £100 at the beginning of the period will cost £105 at the end.
The prices in this example are stated in what economists call nominal terms.
‘Nominal’ means the actual number of £s (or whatever currency) that would change
hands at the time the purchase is made.
There is another way to measure the amounts that are paid in inflationary times.
Economists call this alternative measure a real price. A real price is the number of
£s (or whatever currency) that would have been exchanged to purchase some-
thing before the inflation took place. In the above example the real price of the
item, if inflation is 5 per cent, has not changed between the beginning and the end
of the period. In terms of ‘beginning-of-period-money’, the real price is still £100,
even though the nominal price is £105.
In corporate finance we are interested in the cash flows that investments will
generate across the future. One of the important decisions that analysts must make
is whether to use nominal or real measures of cash flow in their estimates for
investments. Before we can answer which is best, we must deal with the influence of
inflation on the other part of the investment analysis, the discount rates.
Economists are more or less unanimous in their opinion that inflation influences
the rates of return that people require on their investments. In a very simple world
where inflation is perfectly predictable, inflation affects required rates by merely
adding enough to an inflation-free required return to compensate investors for their
loss of purchasing power during the time of the investment. For example, suppose
that when there is no inflation a particular investment requires a return of 10 per
cent per period. If inflation is expected to be 5 per cent per period, the project
would have to return enough to offset both the required ‘inflation-free’ return and
the inflation rate itself in order to be acceptable. We call the ‘inflation-free’ return
the real rate, and the return that compensates for both that and inflation, the
nominal rate. The nominal return is thus given by:
1 Nominal return 1 Real return 1 Inflation
1 15.5% 1 10% 1 5%
In other words, in our example, the investment must return £15.50 each period for
each £100 invested in order to be acceptable. If £100 is invested in a single-period
project at t0, it will be required to produce at least £115.50 at t1. That is its nominal
return. To find the real value of that t1 nominal amount, we divide by (1 plus) the
inflation rate expected, £115.50/(1.05) = £110, the required real cash return based
upon a 10 per cent real required rate.
If we assume that the information for each is available, correct investment deci-
sions can be made using either real or nominal cash flows and discount rates. For
example, suppose that Channel Petroleum is evaluating a proposal to begin operat-
ing at a new location, and the expected real free cash flows are (in £millions):
t0 t1 t2 t3
−2000 +500 +900 +1300
If the real cost of capital of the project is 10 per cent, the NPV is (in £millions):
£500 £900 £1300
NPV £2000
1.10 1.10 1.10
£175
Suppose that the expectations are that inflation will certainly be 5 per cent per
period. The nominal cash-flow expectations, if they are uniformly affected by that
rate, will thus be (in £millions):
t0 t1 t2 t3
−2000 +525 +992 +1505
The nominal cash flows are simply the real cash flows for each time point multi-
plied by 1 inflation rate t, where t is the time point at which the cash flow is
expected. The result is the actual (nominal) amount of cash that is expected to be
available at that time, as opposed to the real cash flow, which is the t0 purchasing
power that the investment is expected to provide at that time point.
Recalculating the NPV of Channel Petroleum’s project using the nominal cash
flows and a nominal capital cost of 15.5 per cent (we know that to be the nominal
rate because we have already found such a rate with a 10 per cent real return and 5
per cent inflation earlier in this section), we have the following (in £millions):
£525 £992 £1505
NPV £2000
1.155 1.155 1.155
£175
The NPV of the Channel Petroluem project is the same regardless of whether we
use real or nominal calculations.
‘Fine,’ you say. ‘That makes some sense, though it is not clear yet what the im-
portance of all this is. But we have been dealing with cash flows and required
returns all through the text to this point. What were those, real or nominal?’
Good question. You are correct in the inference that we have not been explicit as
to which type of measure we were using; the reason was because it was not neces-
sary until the whole question of inflation was raised. The values, cash flows and
required rates we have been using all along have been nominal rates, and for a very
good reason: nominal measures are the only ones that we can have much confidence
in estimating. The prices and interest rates for financial securities that you see
quoted in the financial press are nominal amounts, as are almost all of the financial
results that companies release to the public in their financial statements.
The truth of the matter is that no one has ever really seen a real required rate.
The reason is that such measures are economic concepts that are designed to help
understand inflation’s influence upon nominal rates, and not things that can be
looked up in the newspaper. Remember that the real rate is the difference between
the nominal rate and the expected influence of inflation on required rates for some
time in the future. Because there is no way to measure such expectations’ effects,
the real rate is not measurable. Economists have attempted to estimate the real rate
that actually occurred over past periods, but have not really reached agreement over
the right way to do even that. One of the big problems in measurement is that
inflation itself is not predictable. Many economists feel that nominal rates include
not only the effects of expected inflation but also the uncertainty about it. So far, no
one has been able to separate the two in a way that is widely accepted.
Does this mean that real measures of cash flow, value and required return are not
useful? The answer is, as usual, both ‘yes’ and ‘no’. For making financial decisions
and calculating investment values, the answer is that real measures are not typically
useful. Almost no companies attempt to estimate and use real rates and cash flows
in their decisions, for the reasons of estimation difficulty mentioned above. But the
real measures can be useful in helping us understand how inflation is correctly
included in investment decisions, as we are now in the process of discovering.
What, then, is the answer to the best way to deal with inflation in corporate in-
vestment decisions? The answer is to use nominal cash flows (the actual amounts of
money that are expected, at the time points that they are expected) and nominal
discount rates. If the analysis is performed carefully, the impact of inflation on the
investment, and on shareholder wealth, will appear in the NPV.
The most common error that investment analysts make in dealing with inflation
appears in estimating cash flows. It is quite unusual for analysts to specify in their
requests for cash-flow estimates that the estimates be ‘inflated’ by the estimator’s
belief about the effect of inflation on that amount by the time it is to be actually
paid or received. Not being told otherwise, the marketing, production or other
department is likely to estimate cash flows in the amounts that they are most
comfortable with, namely those based upon the prices that exist today. The result is
something like real cash flows being used in the analysis. If nominal discount rates
(the ‘observable’ ones, which include inflation’s influence and are therefore higher
than the associated real rates) are used with real cash flows, there will be a bias
toward rejecting good projects along with the bad.
To see how this can happen, suppose that Channel Petroleum’s policy is to calcu-
late NPVs using nominal rates and cash flows. If the company’s analysts are not
explicit in their requests for inflated future cash-flow estimates, they are likely to get
the uninflated or real cash flows from the operating divisions in charge of coming
up with the estimates. The NPV that they find is thus likely to be (in £millions):
inflation. You are likely to win or lose something, depending on the side you
choose, but it is impossible to know beforehand whether that will happen to the
borrower or the lender.
That situation is exactly what we would expect to see in a very competitive mar-
ket (such as the financial market) attempting to predict the unpredictable. No one
can reasonably expect to win something from someone else possessed of the same
information.
What have we learned about inflation and corporate investment decisions? Actu-
ally, the lesson itself is quite simple: discount estimated nominal cash flows with
nominal market discount rates. Do not concern yourself with real rates and cash
flows; they are the grist of theoretical economists. While the economists are
observable, real rates are not. Be certain that all involved in the estimation of cash
flows understand that the analysis requires cash flows including the effect of
inflation. And do not choose to finance your investment in any particular way due
to inflation, unless you can find someone foolish enough to accept a rate less than
the market rate. In reasonably efficient economies, the odds of finding such an
investor (other than governments) is somewhat less than finding a wallet full of £50
notes on the pavement.
6.11 Leasing
Our discussion of applications in company investment analysis would not be
complete without some exposure to leasing. ‘Leasing’ is a contractual arrangement
between an asset owner (the lessor) and a company that will actually operate the
asset without owning it (the lessee); in essential economic terms the lessee rents the
asset from the lessor. In exchange for allowing the lessee to operate the asset, the
lessee must pay the lessor lease payments during the period that the lessee has the
use of the asset.
There are many types of lease contracts that companies can pursue. The most
commonly encountered one, and the one that is described above, is a financial or
capital lease. In these kinds of leases the lessor is usually in the business of leasing
assets, and not in the business of actually operating the asset that is leased. There are
many large companies around the world that are in the business of leasing assets to
other companies that actually operate the assets. Such assets include aeroplanes,
ships, shipping containers, computers, buildings, and many others (some reasonably
exotic, such as sewerage treatment facilities, jet fighters and nuclear fuel).
First, here are some things that are often mistakenly taken to be benefits of leas-
ing:
Leasing saves money for the lessee because the lessor is paying for the asset and the lessee
therefore does not have to make the large initial purchase outlay for the asset.
Wrong. It is true that the lessee does not have to make a large initial outlay as
the lessee would do if it purchased the asset, but the lessee of course must con-
tract to make a stream in lease payments, which carry the same essential promise
as a debt claim. On the face of it, there is no reason to think that the present
value of the lease payments is either larger or smaller than the present value of
the purchase price of the asset.
Leasing helps the lessee’s balance sheet, because the lessee does not need to borrow money to buy
the asset, and so the lessee’s debt capacity is higher than it would otherwise be and therefore the
company is less risky.
Wrong. Financial market participants who are interested in a company’s balance
sheet relationships and financing abilities are unlikely to be fooled into thinking
that a lease contract is not roughly similar to a debt contract (most lease con-
tracts have to be at least described in footnotes to the lessee’s financial
statements, and many actually appear as liabilities on the financing side of the
balance sheet).
If the above are not good reasons for leasing to exist, what are the valid reasons
for this to be such a popular financing or investment vehicle? Among the correct
reasons are:
1. Leasing allows for higher tax benefits than the alternative of borrowing and purchasing an
asset.
Right. Consider the situation of a lessee firm having low or uncertain taxable
income, and the wish to operate some particular asset. As you are aware, the
deductions for depreciation and interest on such a transaction are potentially
valuable to the firm operating the asset. But these benefits are contingent on the
company having enough taxable income so that its taxes would be reduced. If
the lessee does not have the prospect of such income over the life of the asset, it
makes economic sense for another firm with such income tax liabilities (the les-
sor) to purchase the asset, take advantage of the tax benefits of depreciation and
interest on money borrowed to purchase the asset, and set lease terms with the
lessee such that these benefits are shared between the two. A similar benefit
derives from a lease transaction if the lessor’s income tax rate is simply higher
than the lessee’s.
2. ‘Information asymmetries’ exist on certain types of assets, and leasing can serve to lower the
costs of such information problems.
Right. Certain assets, particularly expensive high-tech equipment such as large
computers and complex medical devices, are subject to the frequent and uncer-
tain timing of obsolescence. (Who among us has not purchased the ‘latest and
greatest’ personal computer, only to hear within a very short time that its succes-
sor is twice as good and half the cost?) Sellers of such assets usually know more
about the expected schedule of obsolescence than do buyers. A lease can be
written in such a manner as to allow the lessee to take advantage of ‘new and
improved’ or upgraded assets, and thus avoid the risk of costly obsolescence.
And the lessor/seller can probably purvey more of the asset through a leasing
mechanism, given the high uncertainties of outright purchase by users.
3. There are ‘economies of scale’ in the management of specialised asset leasing.
Right. As we have seen, leasing from the perspective of the lessee is economi-
cally similar to purchasing an asset and borrowing the money to do so. The
transaction of ‘borrow and purchase’ requires that the lender evaluate the lessee,
the asset, and other dimensions of the transaction. This can be a costly proce-
dure, particularly if it is done only sporadically. A company in the business of
leasing a particular kind of asset (say, airliners) can be more efficient than a po-
tential lender (say, a commercial bank) because the leasing company is intimately
familiar with the asset to be leased, and may well do several such transactions per
day.
There are other good reasons for leasing, but these are sufficient to give you the
flavour of why this transaction exists. These reasons have the common characteris-
tic of providing a benefit to one or other party to the transaction, which benefit
would not be available without the lease.
2 The cash flows from operating the asset are typically not any different when the asset is leased as when
it is purchased, although some leases provide for asset maintenance. You can consider the situation as
being one where the potential lessee has calculated a positive NPV for the asset if purchased, and is
now considering a lease alternative. It is possible also that an asset would not be an acceptable
investment if purchased, but would be if leased. This implies that companies, when evaluating the
acquisition of assets, should always investigate the alternative of leasing.
Notice that:
There are no operating cash flows from the aeroplane in the analysis because
there is no difference for Topsy-Turvy in either alternative.
Initially, Topsy-Turvy actually gains £838 000, through avoiding the £1 000 000
purchase outlay but paying the £270 000 lease payment and getting the deduc-
tion for making that payment, which is a tax saving of 0.4 × £270 000 =
£108 000.
Topsy-Turvy, after the initial cash flow, continues making the lease payment, and
loses the depreciation tax shields of the aeroplane (which are taken by Brigadier
Electric and are worth £100 000 per year, the £250 000 annual depreciation mul-
tiplied by Topsy-Turvy’s tax rate of 40%).3
In essence, leasing compared with purchasing gives Topsy-Turvy an initial
£838 000, but requires that the company make effective payments of £262 000 per
year for four years.
What then, should Topsy-Turvy do? The answer of course depends on the NPV
of the lease vs. borrow-and-purchase cash flows. The only issue that remains is the
appropriate discount rate to use. What rate is appropriate? We are now familiar with
the idea that the discount rate must be consistent with the riskiness of the cash
flows upon which it operates. And the riskiness of the cash flows at issue here,
because of the contractual nature of the lease, is essentially similar to those of
Topsy-Turvy making comparable payments of interest and principal. So the correct
discount rate is the after-tax interest rate (or what we have earlier called rd*).
Suppose that Topsy-Turvy can borrow at 14%. The after-tax rate is thus 14% 1 –
0.4 , or 8.4%.
Applying that discount rate to the lease cash flows above produces −£22 106.68,
a negative NPV. Topsy-Turvy would be better off purchasing the aeroplane than
leasing it. The present value of after-tax cash flows from leasing are more costly
than those from purchasing.
Notice that if Topsy-Turvy can negotiate a lease payment less than approximately
£255 000 per annum, the lease is better than purchasing. This is often a valuable
3 These are designed to be very straightforward cash flows. More realistic ones might have a sale of the
asset for some positive value and associated tax deduction in the purchase option, and perhaps a
purchase option in the lease alternative. The analysis is nevertheless similar, though more complex.
piece of information for a potential lessee to have, because many such leases are
individual negotiations between lessee and lessor.4
4 The lessor’s cash flows are essentially the mirror-image of the lessee’s, so the negative present value to
Topsy-Turvy is a positive NPV to Brigadier Electric. However, if Brigadier Electric’s tax rate is higher
than Topsy-Turvy’s, both can potentially have positive NPVs (which is, as we know, one of the
reasons for leases to exist).
These methods are increasingly common, and because they directly relate man-
agement compensation to shareholder wealth outcomes, do work effectively at
higher levels of management where decisions are company-wide and therefore have
effects that are more obviously associated with stock market valuations of the
company. But these methods too have shortcomings. For one thing, many factors
other than the decisions made by its managers influence a company’s stock price.
For example, changes in overall economic activity and in the fortunes of related
industries can all have very large effects on the value of a particular company’s
shares, and are essentially outside of the control of management.
Highly paid compensation consultants can perhaps design schemes for a compa-
ny to pay managers so as to filter out such ‘noise’ in the value of the company’s
shares. But even this does not solve the problem of evaluating and creating appro-
priate incentives for middle and lower level managers whose decisions affect only
one part of a larger firm, and therefore are not clearly connected to the company’s
overall share value. The decisions of these managers are in effect bound together in
the company’s share value, which likely makes that value an inappropriate vehicle to
use for compensating individual managers, some of whose decisions may be good
and some bad.
5 These methods have been refined and marketed by consulting firms such as Stern-Stewart (of
Economic Value Added or ‘EVA©’ fame) and McKinsey & Company (selling Economic Profit as
‘EP’).
Revenues £1 000
Expenses (other than interest) 500
Profit Before Tax 500
Taxes 200
Profit After Tax £300
Net Assets (including working capital, and after depreciation) £2 000
Now suppose that the Slacker division’s WACC is 18%. This implies that the net
investment of £2000 requires a break-even economic opportunity cost return of
18% × £2000 = £360. Yet Slacker has produced a return of only £300, so share-
holders have actually experienced a negative £60 outcome, despite the division’s
showing £300 of accounting profit.
Though the actual implementations of EVA© and similar methods are much
more sophisticated than this illustration (for example they use more refined
measures of investment, and more cash-flow oriented definitions of income) they
are consistent with its essence. And the appeal of such measures to those schooled
in WACC–NPV techniques should be obvious. In a sense these economic profit
measures are nothing more than ‘period-by-period’ applications of WACC–NPV.
This tells us much about both their strengths and weaknesses.
The strength of economic profit measures is that they explicitly recognise that all
capital suppliers, not just creditors, require adequate returns: the total of invested
capital must be compensated for its opportunity costs. And as we know, the WACC
is the best measure of the rate of such necessary compensation. Economic profit
measures have the strength of uncovering company operations that are profitable in
an accounting sense, but not in an economic sense. The company’s activities that
produce positive economic profit have positive effects on share value, the activities
that do not generate positive economic profit cause share value to decline.
That economic profit is tied directly to shareholder wealth makes it an excellent
candidate for use as a management performance measure. And it has become very
popular for that reason.
This is not, however, to say that economic profit measures are without significant
impediments to being used effectively. From our study of the differences between
accounting numbers and cash flows, you know of some potential pitfalls in the
application of economic profit. For example, how should the differences between
accounting measures of depreciation and economic depreciation be dealt with in
determining ‘investment?’ And even more importantly, recall that the economic
profit performance measure is applied ‘period-by-period.’ How can this be used to
evaluate a new division that can reasonably be expected to produce negative cash
flows in its earlier years, which are only compensated later? Economic income
measures are not present values of streams of income, as is NPV.
These reservations having been expressed, it is only fair to observe that economic
profit performance measures continue to be implemented at a very fast pace at
companies around the world, and have secured many devoted adherents. That the
Learning Summary
In this module we have examined several miscellaneous issues in applying the
investment-decision techniques developed in earlier modules. We have looked at the
potential competitors to NPV, discussed their weaknesses and strengths, why many
companies continue to use them, and some ways in which certain of them can be
made to work correctly (that is, give the same answers as NPV would).
In addition, we have dealt with several complexities of company investment
analysis, including capital rationing, investment relatedness, renewable investments,
inflation, leasing and the implementation of economic profit performance measures.
As with so many topics in an introductory course such as this, the list of complexi-
ties is not exhaustive of all that an analyst may encounter. But the list is instructive
of many such complexities and – as important – is illustrative of the economic logic
that underlies finance’s approach to all situations of that type. Remember that
investment analyses attempt to calculate the effects upon shareholder wealth
promised by accepting the cash flows of a project. These cash flows are best
regarded as the changes that are expected to occur in the cash flows of the company
as a whole if the project is accepted. The goal of such analyses is to accept the
project or group of projects that has the highest possible increase in shareholder
wealth. This is best measured by calculating the NPVs of the various investment
proposals at hand.
Application of these principles cannot be one of strict rigidity in all situations,
particularly ones that the analyst has not before encountered. For example, suppose
that you are asked to evaluate a decision to abandon or liquidate an investment. At
first glance it would seem that the correct analytical approach would be to estimate
the changes that would take place in the company’s cash flows were the investment
to be ended, then calculate the NPV implied by those cash flows, and abandon the
investment if the NPV so found is positive.
Actually, however, that procedure is not a good one for abandonment decisions.
What it ignores is that, although abandonment this period is superior to operating
the investment over the remainder of its expected lifetime, abandonment of the
investment at some future time may be superior to both. This means that the astute
analyst will produce NPVs for abandoning the investment not only in this period
but also for all future periods during which it is expected to continue operating. If
any of those NPVs is greater than that for abandoning now, the project is not
abandoned now. (Actually, it may not be abandoned even at the future time with the
currently highest NPV, because after the passage of time expectations as to the
abandonment’s desirability may change. For example, next period’s analysis may
indicate that abandonment is best then, at some other point, or never.)
These lessons in investment applications will not make you an instant expert in
financial analysis, but sufficient exposure to the basic logic of the approach will go a
long way toward developing the correct intuitions within you when you are faced
with previously unexperienced analytical situations. Pay close attention to the
Review Questions that follow, and the explanations associated with each in the
answers provided.
Review Questions
6.1 Payback period as an investment evaluation technique for companies:
I. Is not a preferred technique because it is inconsistent with the discounting processes
involved in valuing cash flows.
II. Is reasonably good most of the time because it can be made to work if used carefully
in most situations.
III. Can be used to produce correct answers, but only in limited situations.
Which of the following is correct?
A. I only.
B. II only.
C. III only.
D. Both I and III.
6.2 The average return on investment (AROI or ROI) as an investment evaluation technique
can best be described through which of the following?
A. It is not preferred because, like the payback-period method, it does not
discount, but can be made to work correctly in limited circumstances.
B. It will work essentially correctly because the accounting numbers’ accrual basis
does essentially the same thing as a discounting process.
C. It will occasionally produce different answers from using NPV because of
certain mathematical complexities of the discounting in the AROI technique.
D. It is essentially unsuitable because it uses accounting numbers instead of cash
flows and does not discount.
6.3 Suppose that you must choose at most one of the two investments whose free cash
flows are listed below, and the appropriate discount rate to be used for both is 8 per
cent (this is the investments’ rv*). Calculate the NPV and IRR for each investment, and
decide which you would choose.
t0 t1 t2
Investment 1 −£1500 +£180 +£1680
Investment 2 −£1000 +£130 +£1130
6.4 Suppose your company requires that all investment analyses should be performed with
the IRR technique. Return to the examples of Question 6.3 and produce an analysis that
is both correct and in keeping with your company’s policy.
6.7 In capital-rationing situations the object is, as ever, to maximise the wealth of currently
existing shareholders. In order to do this the decision maker must do which of the
following?
A. Use the IRR to rank investments in order so as to find which give the highest
return per £ invested, and accept in order of declining IRR until the budget is
exhausted.
B. Use the PI to rank investments in order so as to find which give the highest
NPV per £ of outlay, and accept in order of declining PI until the budget is
exhausted.
C. Use the CBR to rank investments in order so as to find which give the highest
ratio of present-value inflow to outflow, and accept in order of declining CBR
until the budget is exhausted.
D. By whatever method find the set of investments that has the highest NPV and
is within the budget constraint.
6.8 The Happy Pooch Dog Food Company is considering three projects.
Project S is a proposal to increase the output of Happy Pooch Dog Food by investing in
some new equipment at a cost of £100 000 and some plant modifications that are
expected to cost £40 000. Because Happy Pooch has been quite successful, its managers
expect that the company’s future cash flows will increase as a result of this project in
the amount of £210 000 on a present-value basis, thus creating a £210 000 − £100 000 −
£40 000 = £70 000 NPV for the proposal.
Project H is the introduction of a new Healthy Pooch Dog Food line, with a somewhat
higher vegetable and fibre mix, which can be marketed to dog owners greatly concerned
about their pet’s health. Project H will require an outlay of only £25 000 for some
fancier labelling equipment, and is expected to produce present value future FCF* of
£55 000, thus creating an NPV of £30 000.
Project F is a proposal for Happy Pooch to begin marketing a line of dog ‘accessories’
such as leashes, coats, playtoys and so forth, in order to capitalise upon the company’s
good name with dog owners. The equipment necessary to manufacture, package and
market these items will cost £210 000, but the expected future net cash flows have a
present value of £305 000, yielding an NPV of £95 000.
It is clear that all three proposals have positive NPVs. However, these numbers have
been presented under the assumption that each project is undertaken independently of
the others. Actually, there are several interactions among the projects of which Happy
Pooch is aware:
I. If S and H are both undertaken (but F not), because of the volumes of output, there
is the chance to buy a somewhat more efficient production process, which would
add £15 000 to their total combined initial costs but would increase the present
values of their combined future net cash flows by £30 000.
II. If H and F are both undertaken (but S not), the combined total initial outlay will be
£15 000 less than their independently considered outlays, because a particular piece
of labelling machinery can be used in both product lines and has enough capacity to
service both.
III. If S and F are both undertaken (but H not), there are no foreseeable interactions of
outlay or future cash flow.
IV. If all three projects are undertaken, all of the above interactions are expected to be
present; and, in addition, there will necessarily be a plant expansion that will cost
£100 000 but will alleviate the plant modification outlays necessary for S were it
undertaken alone or with either of the other two projects.
Advise Happy Pooch what it should do, and why.
6.9 Icky Fishing Supplies must get a new aerator for its maggot and worm production
facility. There are two models being considered by the financial managers of Icky. The
Echh Aerator can be bought for £53 000, will have operating costs of £13 250 per year,
and will require replacement at the end of year three. The Ugh Aerator costs £66 300
to purchase, £10 600 per year to operate, and should last a year longer than the Echh
before needing replacement. Once purchased, the installation requirements dictate that
a machine be replaced with itself rather than another model. Icky believes that its capital
costs for these machines is 12 per cent.
Which of the following should Icky undertake?
A. Purchase the Echh because the NPV of its cash flows is less than those of the
Ugh.
B. Purchase the Ugh because the NPV of its cash flows is greater than the Echh.
C. Purchase the Ugh because the company will end up paying less in present value
considering future replacements.
D. Purchase the Echh because its lower NPV costs in the first cycle of ownership
means that it will always be the cheapest alternative.
6.10 ‘Inflation is no problem for us to handle in our analyses,’ says your financial department.
‘We are aware that market interest rates and other capital rates already include
compensation for expected future inflation, and also that we have no way of knowing
exactly what part of such rates is due to inflation. So the use of real or uninflated
discount rates and cash flows is not a good option for our analyses. Instead we accept
the market rates with their inflation premium, and apply our own inflation estimates to
the cash flows given us by operating units as they submit proposals.’
Which of the following should encapsulate your opinion of this statement?
A. It sounds correct.
B. It appears as if the finance department does not fully understand the workings
of inflation and its interaction with capital rates and project cash flows. If they
inflate the proposal cash flows instead of deflating the market rates, they are
simply exchanging one set of uncertainties for another, to no benefit.
C. The finance department is definitely incorrect in its approach to adjusting NPV
analyses for inflation. The problem is that the manner of accepting the market
rates, and in addition adjusting the cash flows upward for inflation, actually
double-counts the influence of inflation in the analyses: once in the discount
rate, and then again in the cash flows.
D. The finance department seems to be doing the correct thing, given all dimen-
sions of the problem, but it should be certain that the cash flows coming from
the operating units are not already inflated, and that better inflation estimates
might not be available from the operating units themselves.
Learning Objectives
Module 7 is the first rigorous exposure for you to modern ideas of risk measure-
ment in finance. These concepts are important because they underlie the set of
returns and prices that exist in real financial markets for securities and assets of
differing risk. The module begins with illustrations of appropriate risk measures for
an individual investor’s entire wealth position, and departs from that basis to
notions of portfolios of assets, diversification and risk of individual assets within
portfolios. The culmination of this process is the module’s presentation of the
capital asset pricing model (CAPM), which depicts the market relationship between
the relevant risk of an individual asset and the rates of return or discount rates
applied to the cash flows of that asset. In other words, the CAPM tells how securi-
ties markets set prices on risky securities. The module then illustrates how
companies can use the CAPM’s risky-discount-rate estimation process to make
better investment decisions. Several detailed applications illustrate how these ideas
are best applied. In this module you will discover one of the most important sets of
ideas in modern finance, namely the correct treatment of risk in cash-flow expecta-
tions, and how that translates into required returns for decision making. This notion
is one of the cornerstones of the course.
7.1 Introduction
Through the finance course to this point we have mentioned the concept of risk
with some regularity, and have emphasised that the correct treatment of risk in
financial decisions is most important. But we have not yet attempted to deal with
risk in a measurable way. It has been sufficient to our purposes until now to
describe risk in merely an ordinal manner, e.g. that an investment, a security or a
cash flow is of greater or lesser risk than another, and thus requires a greater or
lesser return.
It is now time to introduce the notion of risk as a measurable or quantitative
characteristic and illustrate how financial managers can make good decisions using
this risk measure.
What we seek is reasonably easy to describe. Consider Figure 7.1. The graph
plots risk on the horizontal axis and required return on the vertical. You can see that
the line (called the security market line or SML) describes the relationship
between risk and return as being positive: the higher the risk, the higher the required
return.
Return
SML
Return1
Risk1 Risk
Figure 7.1 Relationship between risk and return – the security market
line
Figure 7.1 also indicates how financial managers would be able to use a quantita-
tive risk–return relationship to make financial decisions. Suppose that we were
facing a decision to undertake an investment I, and all of the necessary cash flows
had been estimated but the appropriate discount rate to apply to those cash flows
was not known. If the risk of the cash flows could be measured and found along the
horizontal axis (see RiskI in Figure 7.1), then one could find the appropriate discount
rate simply by using the SML (see ReturnI in Figure 7.1).
Figure 7.1 indicates to us exactly what our main task will be in this module: to
specify in a measurable fashion exactly what the financial markets regard as ‘risk’
(the horizontal axis of Figure 7.1), and to specify the location of the SML so that it
can be used to generate risk-adjusted discount rates to be used in financial decisions.
Before we finish the module we shall have redrawn Figure 7.1 with the appropriate
quantitatively useful risk and return measures.
Let us assume that the probability distribution of rates of return on your entire
asset holdings are as shown in Table 7.1. This information can be used to find the
riskiness of this set of holdings by calculating the standard deviation of its probabil-
ity distribution of returns. The calculation is not very difficult; it requires that we
find the ‘mean’ or average expected return. To calculate the mean or average
expected return we multiply the rates of potential return by the probability of their
occurrence. The sum of these products provides the mean. We then subtract the
mean from each potential outcome, square those differences, multiply these
differences by the likelihoods of their occurring, add them up and take the square
root. That probably sounds more intricate than it really is; here is the maths for our
example:
Mean return: 0.085 0.35 0.02975
plus 0.11 0.10 0.01100
plus 0.135 0.30 0.04050
plus 0.16 0.25 0.04000
Sum 0.12125
1 Actually, there is quite a bit of intricate economic and statistical thought of a theoretical nature behind
the standard deviation as a risk measure, going all the way back to Daniel Bernoulli in the eighteenth
century (and purists will not be pleased with the ‘non-normality’ of the distribution example above, but
for our purposes at this point the illustration will serve well).
Return
Sharpe and Lintner then offered a simple but elegant solution to the problem of a
risk measure for individual assets by merely asking and answering the question of
the relationship between risk and return for such assets when financial market
participants understand the phenomenon of diversification in Markowitz’s sense.
The result is a measure of risk for individual assets that bears the requisite positive
empirical relationship with returns earned, and that measures the risk that is not
diversified away when individual assets are held in portfolios.3
Suppose that the portfolio we studied in Section 7.2 comprises equal investments
in two individual assets with probability distributions of return as given in Table 7.2.
The expected returns for the two assets are:
Asset A: 0.10 0.45 0.20 0.55 0.155 or 15.5%
Asset B: 0.07 0.65 0.12 0.35 0.0875 or 8.75%
and the standard deviations of return are:
Asset A: 0.10 – 0.155 2 0.45 0.00136
0.20 – 0.155 2 0.55 0.00111
0.00247
the square root of which is 0.0497 or 4.97%, and:
Asset B: 0.07 – 0.0875 2 0.65 0.00020
0.12 – 0.875 2 0.35 0.00037
0.00057
3 There are a couple of promising new advances in theory, which may supersede Sharpe’s and Lintner’s
work. The conclusion to the module outlines the most prominent.
A joint probability distribution is simply more detail about the individual asset
probability distributions (as given in Table 7.2) with which we are already familiar.
To understand the joint distribution in the table, note that there are four cells inside
the box, and that each cell describes the probability of a particular set of returns
being simultaneously earned by both Assets A and B. This is called a joint probabil-
ity. For example, the upper left-hand cell says that the joint probability of asset A
producing a 10 per cent return while asset B produces a 7 per cent return is 35 per
cent. You can see that to be fully specified (to cover all of the eventualities) the total
of the joint probabilities of a 10 per cent return occurring in asset A must be the 45
per cent we saw for this outcome earlier. The joint probability distribution now is
telling us that the 45 per cent chance of a 10 per cent return on asset A is itself
associated with a 35 per cent probability of it simultaneously occurring with asset
B’s 7 per cent return, and a 10 per cent chance of it occurring with asset B’s 12 per
cent return. The other cells are interpreted similarly, and the interior (joint) probabil-
ities must sum in rows and columns to equal the original probabilities of the
individual security returns, while the sum of all of the cells must equal a 100 per cent
(1.0) probability.
Though displaying it may seem like a lot of trouble, the information contained in
a joint probability distribution is necessary to find the likelihoods of various returns
being earned on a portfolio. Without the probabilities of joint events being known,
there would be no way to construct the portfolio’s return distribution, because it is
obviously determined by the simultaneous occurrences of various returns on the
assets in the portfolio.
Let us see what all of this means for the portfolio formed from equal parts of
your assets A and B. The joint probability distribution indicates that there are four
possible events that can happen with the portfolio (see Table 7.4).
The separate events in Table 7.4 are the joint occurrences of the pairs of returns
from the joint probability distribution for the two assets, and the column headed
‘Portfolio’ is simply the equally weighted average of the returns indicated for the
assets in the respective events. The returns are equally weighted because the
portfolio is formed of equal parts of assets A and B. The probabilities in the last
column are merely those for the particular event, taken from the joint probability
distribution.
Note the last two columns of Table 7.4. That information is identical to that in
Table 7.1, the probability distribution of returns originally provided for the portfo-
lio. So we now know the source of portfolio return probability distributions. Such a
distribution is the specification of the likelihoods for joint events from the joint
probability distribution of the assets in the portfolio.
We now also know where we went wrong in trying to calculate the standard
deviation of the portfolio’s return by merely taking the average standard deviation of
the securities in the portfolio. The problem with that procedure is that it ignores the
interactions of returns represented by the joint probability distribution. When we
take those into account, we end up with the distribution for the portfolio in either
Table 7.1 or Table 7.4, and we already have discovered that the standard deviation
of that distribution is 3.008 per cent as opposed to the individual security average of
3.68 per cent.
Why does the whole portfolio have less risk than the average risk of the securities
in it? The answer is diversification. Take another look at Table 7.3. Note that there
are significant chances that when asset A is generating a high (20 per cent) return,
asset B will be generating a low (7 per cent) return: a 30 per cent chance. And the
reverse also holds: there is a 10 per cent chance of a low return on A while B
provides a high return. This means that the risks of the two securities, not being
perfectly positively related to each other, will to some extent cancel each other out.
Much of the time things are good or bad for both simultaneously. But fully 40 per
cent (30 per cent plus 10 per cent) of the time one security is expected to do
relatively well while the other does relatively poorly. The reason why their joint risk
(3.008 per cent) is less than their average risk (3.68 per cent) is just that.
‘What a lot of trouble!’ you exclaim. ‘Does that mean we shall be forced to deal
with complicated joint probability distributions whenever we attempt to find the
risk of a portfolio?’
No, fortunately you will not. One of the important contributions of Markowitz
was to inform us about easier ways to find portfolio risks, by dealing directly with
the interrelatednesses of asset returns. One of statisticians’ favourite measures of the
extent to which two things are related to each other is called a correlation coeffi-
cient. This is just a number that can take on values from −1 to +1, with the former
indicating perfect negative relatedness and the latter indicating perfect positive
relatedness, while values in between are various degrees of positive or negative
relatedness, short of perfection. For example, the joint probability distribution of
returns in Table 7.3 produces a correlation coefficient between the returns for assets
A and B of +0.2423. A correlation coefficient of that magnitude and sign indicates
that the two securities’ returns are positively related, but not very strongly so.
Though it is unnecessary for us to reproduce it here, there is a complex formula
embodying the correlation coefficient (or its statistical counterpart, the covariance)
which can be used to find a portfolio’s risk directly, without the onerous specifica-
tion of the joint probability distribution as we did it.4 The important point for our
purposes, however, is not the best way to find a portfolio’s risk. The main lesson to
be learned from this discussion is that portfolio risk is not the same as average
individual security risk, and that risk reduction through diversification is the reason.
The more positively related are securities’ returns within a portfolio, the less there
4 For example, the answer to the portfolio risk above would be:
0.03008 2 0.5 2 0.0497 2 0.5 2 0.0239 2 2 0.5 0.5 0.0497 0.0239 0.2423
which is more generally given by the expression:
∑ ∑
will be to gain from diversification. The less positively related are the returns from
assets in a portfolio, the greater will be the reduction in risk relative to average
individual asset risk.
Figure 7.3 shows this argument graphically. The lines emanating from assets C
and D show the various risk and return combinations available from combining C
and D in various proportions. Each line is associated with an assumed correlation of
returns between C and D. Note that as the correlation between the two assets
declines, the potential for reducing risk through diversification increases.
We have now finished discussing what we need to know about portfolios and
their risks. We have found that:
1. When individual assets are held in a portfolio, the risk associated with the
portfolio is not likely to be simply the average risk of the assets of the portfolio.
2. The detailed interactions of individual asset returns, their return distributions,
and the return distributions of a portfolio formed from them can be seen by
developing the joint probability distribution of the assets in question.
3. The risk of individual assets, depending upon how their returns are related, will
to some extent cancel each other out when a portfolio of these assets is formed.
4. The extent to which that risk will cancel depends upon how positively the
returns of the constituent individual assets are related. One way to measure that
relatedness is by the correlation coefficient of paired returns of the assets or
securities within the portfolio.
Return
Corr.= +0.25
Corr.= – 0.25
Corr.= – 1.0 D
Corr.= +1.0
C
measure for that asset when it is held by itself, and not in a portfolio. When includ-
ed in a portfolio, it is likely that some of that asset’s (standard deviation) risk will be
diversified away.
The next great step in understanding the riskiness of individual assets – and
therefore the capacity to estimate their risk-adjusted discount rates – was, as with so
many important discoveries, a simple one. William Sharpe and John Lintner simply
asked (and answered) the question: ‘What is an appropriate risk measure for
individual assets when the market behaves as if everyone understands the benefits of
diversification?’5
The answer, as you could probably have guessed yourself, is that when the bene-
fits of diversification are understood by the market and its participants, the only risk
that is relevant is the risk that cannot be diversified away, that which remains after
diversification has taken place. In finance that is called the undiversifiable or
systematic risk of an asset. We shall investigate how to measure the systematic risk
of assets. Once we have understood it, we can easily specify exactly how to use the
security market line relationship to find discount rates for company investment
decisions.
Our first task is to understand the notion of undiversifiable or systematic risk.
Perhaps the best way to do this is to examine the results of an experiment that has
been performed several times in many different financial marketplaces. The re-
searchers set up the following experiment: they collected data on the returns,
standard deviation of returns and correlations of returns for many (usually over a
thousand) securities in a certain stock market for a reasonable period of time (a
number of years). Then they formed many hypothetical portfolios of those securi-
ties, gradually increasing the number of securities in them, and measured the
riskiness (standard deviation of return) of the portfolios as more and more securities
were added (i.e. as diversification became greater and greater). A summary of the
results in all these studies would appear as the graph in Figure 7.4.
5 Sharpe, W.F. (1964) ‘Capital Asset Prices: A Theory of Market Equilibrium under Conditions of Risk’,
Journal of Finance, 19 (September), 425–42 and Lintner, J. (1965) ‘The Valuation of Risk Assets and the
Selection of Risky Investments in Stock Portfolios and Capital Budgets’, Review of Economics and Statistics,
47 (February), 13–37.
RiskM
where βj is the beta coefficient for j; σjm is the covariance of j and the market; and
2
is the variance of the market. This expression for a ‘beta’ coefficient is also
known as a ‘regression’ coefficient. It is telling us the same information as the
previous systematic risk measure did, but scaled to the risk of the market as a whole.
For example, a security having a β equal to 1.0 implies that the market’s influence on
that security is such that an x per cent increase or decrease in the return on the
‘market’ is associated with an x per cent increase or decrease in the return on that
security. On the other hand, a β of 1.5 signals that an x per cent change in market
return implies a 1.5x per cent change in the return on that security. Numerically, a
security currently expected to have a 15 per cent return would, with a β of 1.3 and
with the market return increasing from 12 per cent to 14 per cent, experience an
increase in its expected return from 15 per cent to (15% + 1.3 (14% − 12%)) = 17.6
per cent.
Return of security j
Line of best fit
Slope = ßj
E(rj)
SML
M
E(rm )
rf
1.0 ßj
We can use the graphical presentation of the SML in Figure 7.6 to specify the
quantitative relationship between risk and return in terms of the now famous
equation:
E ﴾7.1﴿
This formula is simply the graphical SML expressed in terms of the line’s inter-
cept rf and its slope E rm ‐rf . The economies of the SML expressed this way
indicate that the required return on a particular security E rj comprises a compensa-
tion for the passage of time rf , plus compensation for risk-bearing, E rm ‐rf βj.
This latter is formed of a ‘market price of risk’, E rm ‐rf , multiplied by an amount
of risk βj. As we now know, that amount of risk is the part of the security’s total risk
that is undiversifiable.
Equation 7.1 is a very important one. It will allow us to estimate discount rates
and opportunity costs for company investment projects in a manner consistent with
the way that the company’s capital suppliers actually hold the company’s securities.
In effect, by using the SML-based discount rates, we will be judging whether the
company’s capital suppliers would be willing to hold the proposed investments in
their portfolios.
Though debate continues in both academic and practitioner circles about the
correct specification of the SML, exactly how to measure systematic risk, and several
other important issues surrounding its use, the SML has proved itself to be a very
powerful idea with significant capacity to explain how rates of return align them-
selves in the market. Recall that we mentioned that researchers in finance found that
their ability to explain the set of returns offered by the market, based upon total
(standard deviation) risk was embarrassingly absent (see Figure 7.2, and the discus-
sion of it). Although there continues to be debate as to the validity of the findings, a
similar plot of actual returns and systematic risk, i.e. an empirical test of the SML,
would show a tight fit of actual returns and systematic risk, with a positive linear
slope, much like the theoretical Figure 7.6.
Before embarking upon an exploration of how financial managers can actually
use the SML rates as investment criteria, we should consolidate the many ideas we
have developed in this section to this point:
1. The total risk of an individual asset or security can be separated into two types of
risk, that which can be diversified away, and that which cannot.
2. The risk that cannot be diversified away is related to an underlying ‘market
factor’ that is common to all assets and securities, and is thus a common correla-
tion limiting the amount of risk reduction through diversification that is possible
by including a security in a portfolio.
3. This undiversifiable or systematic risk can be measured by the β coefficient
(standard deviation times correlation with the market) of the security in question.
4. If the financial market sets securities’ returns based upon their risks when held in
well diversified portfolios, systematic risk will be the appropriate measure of risk
for individual assets and securities, and the SML as depicted in Figure 7.6 and
Equation 7.1 will dictate the set of risk-adjusted returns available in the market.
5. These SML-based returns are the opportunity costs of capital suppliers of
companies, and thus can form the basis for evaluating internal company invest-
ments. These investments must offer returns in excess of capital suppliers’
opportunity costs in order to be acceptable.
E(rj)
SML
ReturnA A
WACC WACC
ReturnB B
ßB RiskWACC ßA ßj
risk, such as some ‘cost-saving’ investments (where savings are based upon known
engineering as opposed to product market estimates) are automatically granted an x
per cent reduction in discount rate (where x is set by the company). Scale increasing
investments, which are likely to be much the same as the average risk of the
company, are evaluated with the company’s WACC, whereas riskier projects,
including new product lines or research and development, are judged with y or z per
cent additions to the company’s WACC (again, variables y and z being set by the
company).
Such approaches are an improvement over complete ignorance of investment
risk differentials, but do not completely solve the problem of evaluating investments
whose risk differs from that of the company as a whole. Remember that the
market’s valuation of an investment’s desirability will depend upon the invest-
ment’s expected return and systematic risk. Unless the adjustments to the WACC of
the company described above are related to the systematic risks of projects, the
criterion will not be identical to the valuation process felt by the capital suppliers of
the company. An SML-based criterion is designed to provide such adjustments.
E D
V V
where βe and βd are observed equity and debt β coefficients respectively, E and D
are their observed market values, and V is the sum of E and D. (This illustration is a
bit simplified because it ignores tax effects, but the essential points are intact.)
Solving for the ungeared β of EAR, we get:
250 100
1.6 0.44 1.27
350 350
This would be EAR’s β coefficient if it had no debt outstanding. We now must
adjust for revenue and operational gearing differences between airframe part
manufacturing and government consulting. EAR’s marketing people inform us that
the company’s revenues have tended to fluctuate about 1.2 times those of overall
economic activity, whereas consulting is expected to have a comparable ratio of only
0.95. To adjust EAR’s now ungeared β coefficient for revenue risk differentials, we
simply multiply it by the ratio of the investment’s revenue volatility to that of the
company:
Project revenue volatility
Revenue‐ adjusted
Company revenue volatility
0.95
1.27
1.2
1.01
Next, we are informed by EAR’s accountants that the fixed costs of operation in
the airframe parts business comprise about 25 per cent of net cash flow, whereas in
the consulting business they are expected to be only 15 per cent. Once we adjust for
this consideration we shall have arrived at the ungeared β for the consulting
business:
1 Project fixed cost %
Project Revenue‐ adjusted
1 Company fixed cost %
1.15
1.01
1.25
0.93
The final step that remains is to readjust the reconstructed and ungeared β coeffi-
cient for any financial gearing planned for the project. In order to do so, we must
know the β coefficient for the debt that will be issued by EAR for the project.
Suppose that the financial experts at EAR have decided that the consulting opera-
tion can sustain a 20 per cent gearing ratio (this is a market value of debt to total
debt-plus-equity ratio, D/V). Rather than going through an intricate and similar
adjustment illustration for the β coefficient of the debt of EAR, let us assume that
investment bankers have advised EAR that debt issued solely to finance this project,
and in the 20 per cent gearing ratio, would have a βd of 0.22. (We shall show you
where this figure comes from in the next section.)
We now know the ungeared β coefficient of the project (0.93), the β coefficient
of the debt of the project (0.22), and the gearing ratio of the project (20 per cent).
With this information we can calculate the equity β coefficient of the project:
E D
V V
0.93 0.80 0.22 0.20
0.93 0.22 0.20 /0.80
1.11
So the β coefficient of the shares of the consulting project of EAR, were such
shares to be sold independently in the market, would be 1.11, in contrast to EAR’s
βe of 1.6. Government consulting is evidently less risky than building airframe parts,
including the effect of financing.
The foregoing has shown how a β coefficient for a project can be constructed in
situations where a market β coefficient is unavailable. Of course it would have been
much easier for EAR to arrive at the investment’s β coefficient if there were a
company exclusively engaged in government consulting and whose shares were
traded on an exchange. (Remember that even in that situation there would be an
adjustment necessary for financial gearing if EAR intended to finance the project
with a different debt proportion.) It is also true that the financial gearing adjustment
could have been done in just one step, by using a similar ratio calculation as that
used for operational gearing and revenue volatility. We presented the ungeared β
coefficient as an interim step because the notion of an ungeared return is familiar
and will appear again.
E(rj)
SML
reProject
ßProject equity ßj
been 9.1 per cent above the risk-free rate, whereas in the US a somewhat broader
index indicates an 8.8 per cent market portfolio ‘excess’ average return, both for
periods of time of more than fifty years. Such averages are by no means constant
over time, but they are more so than E rm itself and are thus used until something
better presents itself.6
We now have enough information to illustrate the use of the SML to find a re-
quired return. The EAR example from the previous section will serve this purpose
well. We have discovered that, adjusted for all important operating and financial
characteristics, a reasonable estimate of the β coefficient of the equity of the EAR
consulting project is 1.11. Using 10 per cent as an estimate of the existing risk-free
rate rf and the 9.1 per cent excess market return of the UK (where EAR is assumed
to be based), the SML tells us that:
E
E 0.10 0.091 1.11
0.201 20.1%
To capture the opportunity costs of its shareholders, EAR must use an equity
required return of 20.1 per cent in evaluating its government consulting project.
This rate will compensate shareholders for the undiversifiable risk that the project
will add to their asset portfolios.
Should EAR then use 20.1 per cent as the discount rate for the project’s free cash
flows? Of course not. Remember that the appropriate discount rate for a project’s
FCF* must reflect the necessary returns (or capital costs) to the full financing of the
project. The EAR consulting project does have equity financing, but is also partially
to be financed with debt. We can recall from Module 4 that the project will require
that EAR use a WACC (or rv*) to evaluate the project. And this WACC is not the
WACC for the entire company but that which would uniquely apply to the consult-
ing project were it to exist as a separate company. This is because the capital
suppliers of EAR are effectively being asked to add the consulting project to their
portfolio of holdings, and thus it must stand or fall on that basis.
Let us find the WACC that EAR should apply to the cash flows of the consulting
project. You recall from Module 4 that the WACC is defined as:
∗
D ∗
E
V V
We have all of the necessary information to calculate rv* except for rd*, the after-tax
cost of debt capital. Fortunately, we also have enough information to calculate it,
too. Remember that we have been informed that the βd for the project is 0.22.
Thus:7
6 There are reporting services which sell such estimates, many of which are numerically lower than those
quoted above.
7 Recall that the βd of 0.22 was assumed to have been supplied to us by investment bankers. In practice,
the investment bankers would have probably supplied us with the 12 per cent debt rate itself, and we
could have thence deduced from Equation 7.1 the βd.
E
E 0.10 0.091 0.22
0.12 12%
If we assume a 50 per cent company income tax rate, the WACC of the project is:
∗
0.20 0.12 1 0.50 0.80 0.201
0.173 17.3%
EAR should use a discount rate rv* of 17.3 per cent to evaluate the consulting
project’s free cash flows, FCF*.
We have already seen in Figure 7.7 the kinds of errors that a company can make
by evaluating projects with the company’s WACC. It is instructive to see whether
EAR would have likely made an error in its evaluation of the consulting project by
using the company’s WACC. Of course, in order to shed light on that issue, we
must calculate the WACC for EAR. Fortunately, we have sufficient information to
do that quite easily.
EAR’s equity β coefficient, we were earlier informed, is 1.6. Therefore we can
find the company’s re by again invoking the SML:
E
E 0.10 0.091 1.6
0.2456 24.56%
EAR’s equityholders are requiring a 24.56 per cent return. If we assume that the
company’s debt requires a 14 per cent return (higher than the project because of the
company’s higher operating risks and financial gearing relative to the project), and
using the known debt and equity proportions of 100/350 and 250/350 respectively,
we find EAR’s WACC to be:
∗
D ∗
E
V V
0.29 0.14 1 0.50 0.71 0.2456
0.1947 19.47%
EAR’s WACC is 19.47 per cent, whereas the correct WACC for the project is 17.3
per cent.
E(rj )
SML
19.47%
17.3%
rf
certainty-equivalent cash flow, and to discount that (now ‘certain’) cash flow at
the risk-free rate.
It is possible to create such certainty equivalents in a manner consistent with the
SML. Essentially, the question asked is: ‘How much money would I now agree to
accept in the future for certain in exchange for the risky cash flow of the invest-
ment?’
To use the SML to answer the question we must change the Equation 7.1 a little.
Instead of stating it in terms of discount rates, if we restate it in terms of cash flows,
the SML adjustment becomes:
E ﴾7.2﴿
Covariance CF,
Variance
Equation 7.2 is thus the SML, but stated in terms of cash flows instead of dis-
count rates. The certainty-equivalent cash flow CFce is found by subtracting from
the expected risky cash flow (CF) an adjustment for its systematic risk. That
adjustment uses a variant of the ‘market price of risk’, E rm ‐rf /Var. rm , multi-
plied by a measure of the systematic risk of the cash flow, Covar. (CF, rm . The only
new terms in the Equation are Var. rm and Covar. (CF, rm ; they are the variance
(the squared standard deviation) of the market return, and the covariance (the cash
flow’s β coefficient multiplied by the variance of the market return) of the cash flow
with the overall market.
Equation 7.2 is no better nor worse than Equation 7.1. The same information is
required for each of them to function, and the answers obtained by using them are
the same. The only difference is that instead of learning from Equation 7.1 what
risk-adjusted discount rate to use in a valuation of cash flows, you learn from
Equation 7.2 what risk-adjusted cash flow to discount with the risk-free rate. The
present-value answers are identical if the equations are used correctly.
Let us briefly examine an example to test this incongruity. Suppose your compa-
ny is evaluating an investment wherein the following information is known:
E 0.18
Var. 0.01
£150 000
Covar. , £2400
0.10
Substituting into Equation 7.2, we have:
0.18 0.10
£150 000 £2400
0.01
£130 800
This result tells us that the market would be willing to exchange an absolutely
certain £130 800 for the uncertain £150 000 cash flow of the investment, adjusting
for its systematic risk. This exchange, however, would be made at the future time
point t1, so the present value of the cash flow is the discounted value of the certain
£130 800, which using the risk-free rate is:
£130 800
PV CF
1 0.10
£118 909
So the present value of the risky future cash flow is £118 909, found by the certain-
ty-equivalent method. Were we to have used the risk-adjusted discount rate
Equation 7.1, we would, instead of the covariance of the cash flow, have needed to
know the β coefficient of the investment’s return. Without going into a lot of
algebra and statistical theory, take our word for it that the relationship is given by:
Covar. , £2400
2.01835
Var. 0.01 £118 909
Substituting the above into Equation 7.1 for this example, we obtain:
E 0.10 0.18 0.10 2.01835 0.261468
So the cash flow would require slightly over a 26 per cent return (we are using a lot
of decimal places to make the answers come out the same). Applying this discount
rate to the cash-flow expectation:
£150 000
£118 909
1.261468
This, of course, is exactly the same value we found using the certainty equivalent
method.
Is there anything to choose between the two techniques? Not really. Some may
feel more comfortable using one as opposed to the other for psychological reasons,
but the answers we get are the same. Nevertheless, your education as a finance
person would be incomplete if you had never heard of certainty equivalents.
Learning Summary
This module has introduced us to a quantitative method of adjusting discount rates
for the risk inherent in the cash flows of a project. The basis for such adjustment is
a ‘market model’ of how such rates are determined. We call that model the security
market line (SML) or capital asset pricing model (CAPM). The essential characteris-
tic of the model is that it recognises that capital suppliers to a company themselves
hold portfolios of assets that are well diversified (or where the market prices their
securities as if they do). With this characteristic, markets will only compensate (with
higher rates of return) investors for the amount of risk that cannot be diversified
away. We call this risk ‘systematic risk’ and it is measured by the β coefficient that
appears in the risk-adjusted return Equation 7.1.
This idea of the market rewarding only a particular type of risk bearing is a very
important one. Not only has it produced a theory of risk-adjusted discount rates
superior to anything yet available – and with impressive empirical validity – but it
allows for certain nice intuitions about company investments and risk in general,
which would be impossible without it.
For example, one of the lessons of the SML is that company diversification for
the purposes of reducing risk is not destined to be a desirable activity from the
shareholders’ perspective. A company may indeed reduce the risk of its cash flows
by undertaking projects whose cash flows are less than perfectly correlated with
those of the company. Similarly, two firms with less than perfectly correlated cash
flows can merge, thus reducing the risk of their combined cash flows. But think
whether that activity is of benefit to shareholders. If the shareholders already hold
well diversified portfolios, there is nothing to be gained in their portfolios by the
company reducing only its diversifiable risk, which would be the sole result of such
investments and mergers. On the other hand, if the shareholders were not well
diversified, they could have simply themselves bought the shares of the two
unmerged firms and thereby produced the risk-reduction benefits of the merger in
their own portfolios, and at doubtless lower costs than those entailed in a company
merger. The lesson is that if shareholders are already well diversified, diversification
at the company level is irrelevant (and probably costly) to them.
One counter-argument offered to the above statements is that company risk
reduction through diversification is beneficial not because shareholder risk is
reduced but because, with debt in its capital structure, the likelihood of a company
defaulting on its debt and the attendant unpleasantries is reduced. This plausible-
sounding argument is also incorrect.
Take, for example, the situation of two companies merging and thereby reducing
the risk of their debt. It is true that the debt of the combined company will likely be
of lower risk than the average risk of the debt of the two companies uncombined.
But think for a minute how that risk reduction came to pass. The imperfect correla-
tion of the companies’ cash flows is the cause. And the fact that debt is now less
risky implies that there was at least some chance before the merger that a default on
debt would have taken place in one or the other (or both) of the companies. Now,
however, such risk has been reduced, if the basic cash-flow expectations of the
companies themselves have not been affected by the merger but merely combined,
the only place the debt risk reduction could have arisen is from equityholders. In
essence, the equityholders of one firm are now providing insurance to the debt
suppliers of the other firm, and vice versa. (Thus the term ‘coinsurance’ is used to
describe this phenomenon.)
What does this mean? Simply that the result of such a merger will be to increase
the value of the debt of the companies, but reduce the value of the shares of the
companies by the same amount. What about any benefits of diversification? The
debt-holders of the company are better off, but the shareholders are worse off, and
by the same amounts; thus the only ‘benefits’ of risk reduction through diversifica-
tion have been to shift some shareholder wealth to debt-holders. Financial managers
who do that sort of thing are not destined to have long, prosperous careers.
What is the lesson of this? Simply that companies should not try to provide
something to shareholders which the shareholders can do perfectly well on their
own. Since shareholders can perfectly well provide to themselves the benefits of
diversification (by holding the shares and bonds of many companies, by buying
diversified mutual funds, by belonging to retirement or pension arrangements that
hold diversified portfolios, and many other low-cost arrangements) companies
should spend their time trying to find high NPV investments as opposed to
diversifying so as to reduce risk.
We have presented the CAPM and SML as if these ideas are completely settled
and agreed upon by all concerned. You should be aware that there still exists much
debate in the finance world, both academic and practitioner, as to the validity of the
ideas and measures we have herein presented. But one fact stands out: the ideas we
have studied in this chapter are at this point the only serious contenders for estimat-
ing risk-adjusted discount rates based upon some consistent economic model of
compensation for risk bearing.
There are at least one or two other promising approaches,8 not inconsistent with
this, that embody less stringent assumptions as to investor behaviour, where risk-
adjusted rates are deduced from groups of securities, including options of various
types, which promise exactly the same risks as the opportunity in question. But such
approaches have not yet been made as convincingly applicable to company invest-
ments as has been the SML.
To review what we have learned in this module:
1. Diversification reduces risk. The total risk inherent in an investment can be
thought of as comprising diversifiable and undiversifiable risk, the former of
which disappears when the investment is included in a well diversified portfolio.
2. Capital suppliers require higher rates of return for bearing higher risk, but the
risk that they bear is only that which is undiversifiable in the portfolios they
hold.
3. The market, in setting the required returns on assets and securities, depends only
upon their undiversifiable systematic risks. It ‘prices securities as if they were
held in well diversified portfolios’.
4. This pricing process of financial markets can be captured in an equation that
specifies required return as a function of systematic risk, or β coefficient. The
equation is Equation 7.1, and that for the SML is depicted in Figure 7.6.
8 One promising alternative to the CAPM is the APT (or arbitrage pricing theory), which posits a
somewhat more complex set of factors – inflation, the shape of the yield a curve, economic activity,
etc. – each of which has a β coefficient for each company.
Review Questions
7.1 Even though it was generally accepted in financial circles that the standard deviation of
rates of return is an appropriate measure of risk, few empirical studies in finance could
find a good relationship between that risk measure and the actual rates of return being
earned by holders of securities across time. Which of the following gives the most
important reason for that?
A. The level of sophistication available in statistical analysis was not high enough to
detect the subtleties of such a relationship.
B. Investors in securities are not really risk-averse in the sense that they require
higher rates of return for bearing risk; they are only interested in becoming
wealthy, and are willing to take risks to do that.
C. Actually, any such tests are destined to produce questionable results because
they are attempting to measure the unmeasurable: the feelings and expecta-
tions held by investors as they buy and sell securities.
D. The risk measure was incorrect. People only regard as risk that part of return
standard deviation that cannot be diversified away when they include the
security in a well diversified portfolio. So an undiversifiable risk measure was
required.
7.2 Suppose that you already hold many securities and are considering investing in the
following four additional ones. You are convinced that the data provided are correct:
The risk-free rate is 10% per cent, and the excess of the market return over the risk-
free rate, E rm ‐rf , is 8 per cent. You would:
I. Invest in either X or Y or some combination of the two because they are the only
securities that offer enough return for the risk they contain.
II. Invest in any or all four because the returns and standard deviations of return are
ordered the same; that is, the higher-return securities have the higher risk, and vice
versa.
III. Invest in only the highest return security if not risk-averse, only in the lowest risk
security if very risk-averse, or in one of the others if somewhere in between in your
preference for risk bearing.
Which of the following is correct?
A. I only.
B. II only.
C. III only.
D. Both I and III.
7.3 Anglo-Auto plc, whose shares are traded on the stock exchange and are widely held, is
considering an investment in a gold mining operation. The gold mined will have no
particular use in Anglo-Auto’s operations, and will simply be sold on the market. Anglo-
Auto is expecting to pay a fair market price for the mine. The stated purpose behind the
venture, according to the managing director of Anglo-Auto, is diversification: ‘We at
Anglo have found that the company’s fortunes are too much dictated by the state of the
overall economy, which has been quite volatile in the last decade. By investing in the
gold mine we are providing a nice offset to the risks of operating in the economy as a
car manufacturer, because we have discovered that gold prices tend to do well when
our company does poorly, and vice versa.’
Which of the following is correct?
A. Anglo-Auto is making a mistake by thinking that an investment in a gold mining
operation is beneficial in reducing the risks of the company. Gold mining is itself
a very risky operation, and adding two high-risk operations together is simply
bad strategy.
B. The plan is a good one, if a bit extreme. Since the cash flows of car manufactur-
ing and gold mining are less than perfectly correlated, combining them into a
single stream will likely offer to the market a less risky stream than either
would provide individually. This should result in an increase in the price of
Anglo-Auto’s shares.
C. It is true that Anglo’s risk will likely be reduced, but the risk reduction is in
total, not systematic, risk. So the benefits of such diversification will appear
only at the corporate level, not for shareholders.
D. Either B or C could be correct, depending upon whether the shareholders of
Anglo-Auto are well diversified.
7.4 Consider the risks and returns available from combining into a portfolio the shares of
the companies listed below.
I. General Automobile Manufacturers
II. General Steel Company
III. General Pawnbrokers plc
IV. General Bakeries.
Which pair of shares is likely to produce the greatest reduction in risk due to diversifi-
cation effects?
A. General Automobile Manufacturers and General Pawnbrokers plc because the
two would be expected to have opposite relationships with overall economic
activity.
B. General Automobile and General Steel, because these two companies’ shares
would produce a beneficial source of raw materials for the car manufacturing
component, and a ready market for the steel produced.
C. General Bakeries and General Steel, because the former will not be affected
much by overall economic activity, whereas the latter would.
D. Either General Auto and General Pawnbrokers or General Steel and General
Pawnbrokers for the basic reason stated in A for General Auto and General
Pawnbrokers.
7.5 Examine the graphs in Figure 7.10. Each shows a scatter diagram of the historical
relationship between the returns of a particular security and an overall market portfolio
or ‘market index’.
Security A Security B
ReturnA ReturnB
ReturnM ReturnM
Security C Security D
ReturnC ReturnD
ReturnM ReturnM
7.6 Within the context of the scatter diagrams in Figure 7.10, describe the manner in which
the total risk of securities is transformed into systematic risk when these securities are
held in well diversified portfolios.
7.7 Engineered Genetics (EG) is considering a foray into the development and manufacture
of an organism that, when combined with red vin ordinaire, would transform cheap wine
to taste exactly like 1945 Chateau Mouton Rothschild. The odds of such a development
being successful in an engineering sense are about one in four, would take three years to
resolve, and cost £3 000 000. EG figures that such project risks would require a 25 per
cent discount rate per period. On the other hand, were the research to be successful,
EG feels that the product would generate free cash flow of £3 000 000 for ever, after an
initial additional outlay of £5 000 000 for a production facility. Alternatively, EG could at
that point sell the patent on the process to a certain French company (to remain
unnamed) for £25 000 000. Given the popularity of such a product, success is virtually
assured, and thus EG is comfortable with a 10 per cent discount in that phase. The risk-
free rate is 8 per cent. Should EG begin developing the product or not?
1 Advise NOSE as to the correct WACC for it to use in evaluating ‘Cyrano’, and contrast
this with NOSE’s WACC.
Learning Objectives
A company’s shareholders are the most important capital suppliers to the firm, and
financial decision making is aimed toward maximising their wealth. One of the
important financial decisions made within a company is its dividend policy; that is,
the strategy the company uses to transfer cash to its shareholders. The module
begins with some basic economics of dividends, showing how dividends can
simultaneously be both the basis for share value and an irrelevancy. Actually, the
argument is that the dividend decision, under certain idealised conditions, makes no
difference to shareholder wealth, as long as other financial decisions (such as real
asset investments and capital structure) are given. When those idealised conditions
are removed, however, dividend policy may become important. When taxes,
brokerage fees, consumption patterns of investors, and information frictions are
considered, dividend policy has the potential to be quite important. The module
goes to some lengths to display what is known and not known about the impact of
the choice of dividend policies on shareholders’ wealth.
8.1 Introduction
This module addresses company dividend policy. Most simply, the question of
dividend policy is how much money the company should pay to shareholders across
time. Dividends are the amounts of cash that a company distributes to its share-
holders as the servicing of that type of capital. Shareholders have invested in the
company by purchasing shares, and dividends are the company’s direct compensa-
tion to shareholders for their investment.
Dividends, unlike interest and principal payments to debtholders, are not a con-
tractual right of shareholders. There is no requirement that a company pay its
shareholders any particular amount of dividend at any particular time. However,
since shareholders are the residual claimants of the company’s cash flows, they do in
a sense ‘own’ the amounts of cash that the company produces net of all other
contractual requirements (other claims being the costs of operations, taxes, interest,
and so forth). Financial managers of a company must decide how much of its
residual cash should be paid as dividends to shareholders. Since any residual cash
not paid as dividends is still ‘owned’ by shareholders, this retained cash is reinvested
in the company on behalf of shareholders. The dividend decision is thus also, in
mirror image, a cash retention or reinvestment decision. Any reasonable discus-
sion of dividend policy questions must treat both aspects of this decision.
As we promised for all of our discussions of company financial decisions, our
analysis of company dividend policy will assume that managers of companies try to
make shareholders as wealthy as possible. So the real question of dividend policy
boils down to this: with amounts of cash that could be distributed to shareholders
across time, and given that any amounts not so distributed would be reinvested by
the company in the name of the shareholders, is there a particular strategy of
dividend distribution that would produce more wealth for shareholders than any
other? The task of this module is to investigate possible answers to that question.
If dividend is paid:
Old shareholders 0 +£400 000 +£400 000
New shareholders −£100 000 +£100 000 0
The second important point about this example is the connection it illustrates
about the company’s dividend and investment decision. This is not unique to the
Simple Corporation’s dividend decision, but is common to all dividend policy
questions. What is the connection between dividends and investment? Figure 8.1
may help you to visualise the relationship. The figure shows the company’s residual
cash available to be allocated between dividends and retentions, along with the
company’s plans as regards investments. The investments can be financed with
retentions of residual cash, or by new share issuances (for simplicity we again
assume an all-equity company).
nd
ide
Div
SHAREHOLDERS
Ne
we Retention
qu
ity
Investment
‘bird in the hand’ view of dividend policy. It argues that dividends are preferable to
capital gains because the former is actual cash in hand, and the latter is based upon
future dividends not yet received. Thus a policy that substitutes future uncertain
dividends for current certain dividends is by its very nature designed to increase the
riskiness of the company’s shares. This sounds plausible, but is wrong.
Remember that one of the primary functions of financial markets is to place
current (certain) values upon future (uncertain) cash flows. That is exactly what has
been done in the setting of the prices of companies’ shares. So the current market
share price includes an adjustment for the riskiness of future dividends. In no sense
is that an incorrect price, regardless of the dividend policy of the company.
Now consider the situation of a shareholder whose company adopts a policy of
reducing dividends (and also, of course, of reducing the sale of new shares). That
shareholder will have the same total wealth, but the wealth will have a different
composition: it will comprise more wealth in share value and less in cash from
dividends. That is indeed a riskier situation for the shareholder. But if the share-
holder were unhappy with this portfolio composition, there is nothing to prevent
the shareholder from selling enough shares to duplicate the cash dividend that was
not received. Were that to be done, the shareholder’s wealth would have the
identical composition as it would if the company had not adopted the lower
dividend payout.
For example, suppose that the company’s dividend policy had produced £1000
less cash because of a dividend reduction, but this had caused the company to issue
£1000 less new equity. The shareholder’s wealth is exactly the same total, but
differently comprised. There is now £1000 more in value and £1000 less in cash. If
the shareholder was happier with the £1000 in cash rather than share value, shares
in that amount can be sold, and the shareholder will end up with the same cash and
value holding that would have resulted from the company paying the dividend and
issuing the new shares itself. The shareholder has effectively issued the shares
himself, by selling some of his shares to new shareholders. Thus up to this point
there is no reason to think that dividend policy makes any difference to sharehold-
ers.
There is actually a deeper lesson in this illustration, which will serve us well in
discussions of all company financial decisions. In the above illustration the share-
holder has in effect ‘undone’ the company-level dividend decision in the
shareholder’s own portfolio. When the effect of company financial decisions upon
shareholders’ portfolios can be undone by offsetting actions of shareholders, the
company financial decision is irrelevant. This is a very powerful insight, and will
eventually allow us to understand the importance of various financial decisions
much more deeply.
phenomena are we talking about here? Essentially three: taxes, transaction costs and
flotation costs. Let us examine each of these in turn.
In either case, the shares of the company will increase by £500, but should cur-
rent shareholders receive the dividend, they cannot also receive the share price
increase. It will go to the new shareholders who contribute the £500 necessary for
the investment outlay. If the shareholders do not receive the dividend, they do
receive the £500 share value increase because the company was not forced to raise
cash from new shareholders, and because the money for the investment was
retained instead of being used to pay the dividend.
The important thing to understand about this illustration is that total taxes paid
by the company and its shareholders are higher when dividends are paid than when
they are not. Shareholders receiving the dividend have only £350 cash in hand after
tax whereas they would have had £500 in share value had dividends not been paid.
The company needs £500 for investment, and can raise that money from new or old
shareholders. But the damage has already been done: the £150 of taxes is lost to
government.
When dividends are not paid, shareholders receive no cash, but neither do they
pay taxes. Such shareholders paying no cash into the company, nor having others
contribute cash, retain their full claim, and have a value increase of £500. The
difference between the final wealth of shareholders who receive dividends and those
who do not is thus £500‐£350 £150, the difference in the taxes paid. (If and when
shareholders realise ‘capital gains’ by selling shares, there may be some tax paid, but
it is usually later and sometimes less than that paid upon the receipt of dividends.)
In such a tax system where ‘double taxation’ of dividends is unavoidable, there is
a strong tax incentive against the payment of dividends for companies seeking to
please their shareholders. (Interestingly, the empirical evidence as to whether high
dividend-paying companies’ shares are adjusted in price for the taxability of the
dividends is mixed, or at least not agreed by consensus of researchers. Some think
that in these economies there are enough dividend tax-avoidance transactions
available so that this tax detriment of dividends is really unimportant.)
In some countries (such as the UK), dividends are not taxed as heavily. These
‘imputation’ systems make some attempt to alleviate the double taxation of divi-
dends by imputing an amount of company taxes to shareholders based upon the
dividends that companies pay, and then giving shareholders a credit on their taxes
for that amount. The effect of such systems is to cause less of a bias against
dividends than systems that tax both company profits and shareholder dividends.
Using the above example, assume that the tax system is such that dividends re-
ceived by shareholders are imputed by tax authorities as having had 30 per cent tax
already paid on their behalf by the company. Further, companies paying dividends
pay the same total tax as companies not paying dividends, even though shareholders
are given tax credits for the imputed tax on their dividends. Our example now
becomes as in Table 8.3.
Table 8.3 Results of the dividend payment decision under the imputa-
tion tax system
Pay Do not pay
dividends dividends
Profit before tax £1000 £1000
Company income tax −500 −500
Profit after company income tax 500 500
Dividends 500 0
Shareholder income tax 214.29 0
Tax credit 214.29 0
Net after-tax cash to shareholders 500 0
Increase in share value to existing share- 0 500
holders
Note two things about this example. First, shareholders receiving dividends pay
no net tax on them, so that there is no wealth difference between the position of
those who receive dividends and those who do not. The tax credit exactly matches
the tax liability. (The reason that the liability and the credit are more than 30 per
cent of the dividend paid is the nature of the ‘imputation’ calculation itself, wherein
the dividends received are assumed to have been after the 30 per cent tax is levied,
so that the dividend received is actually 70 per cent of the imputed dividend. The
latter is taxed at the 30 per cent rate, meaning that the actual dividend generates a
tax liability and credit of 30/70 of the cash amount paid.)
Secondly, note that there actually is a tax liability on the part of the shareholder,
against which there is a fixed credit. In this example, the tax and the credit are the
same amount, so there is no net tax paid on the dividend and shareholders would be
indifferent between receiving them or not. But suppose that there is a personal tax
liability that is higher than the tax credit received (because of higher personal tax
brackets by shareholders). Here the credit would not cover the tax liability, and the
shareholders would be worse off receiving dividends than not. Suppose also that
shareholders’ personal income tax rates were 40 per cent. The situation now
becomes as set out in Table 8.4.
Pay Do not
dividends pay dividends
Net after-tax cash to shareholders 428.58 0
Increase in share value to existing share- 0 500
holders
Here again there would be a bias against the payment of dividends, even in an
imputation system. (In the UK there are further aspects of the company-versus-
personal dividend imputation system that produce a bias against dividend payments,
having to do with the fact that the imputed shareholder dividend tax is paid in
advance. Advance corporation tax (ACT) must be paid by the company even if
there is no company income tax. The company can set this off against its income tax
– mainstream corporation tax (MCT) – if and when this is eventually paid. You
are familiar enough with the time value of money to see the penalty involved when a
company with no income tax liability pays dividends.)
called flotation costs, and they can be significant for the issuance of new shares,
depending upon the mechanism of sale. If intermediaries such as investment
bankers are used, the costs can be as high as 5 to 25 per cent of the total value of
issued shares.
The recognition is important that shareholders are not all alike in the exposure
they have to dividend and capital gains taxation and that preferences for consump-
tion of wealth across time differ. Intuitively it should be easy for you to see that one
type of shareholder, say those in high personal tax brackets, would prefer one kind
of dividend policy (low cash payout), whereas another kind of shareholder in a low
tax bracket might well prefer high cash payout. Such different kinds of shareholders
have come to be called clienteles in finance. The interpretation is that they com-
prise groups that would be willing to pay extra to get the type of dividend policy
that is best suited to their own tax and consumption proclivity. Said another way,
they have probably been attracted to the shares of a company that pursues a policy
that to them is attractive.
What does this mean for the dividend decisions of companies? Consider, for
example, the Complex Corporation. Its shares are traded in a financial market
wherein the companies and shareholders are both subject to the full range of
‘frictions’ we discussed in the previous section. Complex Corporation could
undertake to study the average tax situation of all shareholders, brokerage fees,
proclivities to consume wealth, and so forth, so as to reach an opinion as to the
dividend policy that would be of most appeal to the average of all shareholders, or
even to its own average shareholder. In a market having only one firm or just a few
firms providing wealth to shareholders, that might make sense. The best policy for
shareholders could be designed, and might produce a wealth increase for sharehold-
ers.
But there are not just a few companies providing wealth disbursements to share-
holders; there are many. And those companies provide a wide range of dividend
strategies to the market. Given the number and types of dividend payout patterns
available, we can raise questions as to whether anything a particular company can do
to change its dividend policy is likely to give its shareholders something that they
could not acquire elsewhere. As a matter of fact, this idea is one of the underpin-
nings of current thinking about company dividend policy, and it brings us back to
the original ‘irrelevancy’ conclusion, even in realistic financial markets.
Think of the situation this way. As we have argued exhaustively above, there are
many potential different shareholder ‘clienteles’ for various dividend policies by
companies. Some shareholders (commonly called ‘fat cats’) want low payouts
because of high personal income tax brackets. Others (‘widows and orphans’) want
high payouts because of low personal taxes and preferences to consume their wealth
in the form of cash payments. And there is a continuum in between of possible
variations on these themes. For example, managers of institutional share portfolios
(pension fund and mutual fund managers) might well prefer low dividend payout if
their salaries depend upon the total value of the shares they manage, because cash
dividends in some instances must be paid out to those who are serviced by the
institution. Some other institutions may be legally constrained to hold only shares
that maintain a relatively high payout. These clienteles will each tend to be attracted
to the shares that provide the dividend policy they desire. Companies are thus in a
competitive market for dividend policies as well as for their other goods and
services.
seem to be loath to pay a dividend unless they think it can be sustained for some
period of time by the expected cash flows of the firm. This seems to be true even
when the company would be retaining cash beyond its current need for investment
funds. And in other instances, companies simultaneously pay a cash dividend and
sell new shares, an obviously expensive combination.
How can this be explained? One obvious explanation is that company financial
managers do not understand the arguments about company dividend policy that we
have made up to this point in the text. In a competitive market for financial
managers, however, that would not be a very convincing argument. There must be
something else afoot.
One explanation for the ‘smoothing’ across time of company dividends that
seems more reasonable is the signalling value of dividends. The argument for
‘signalling’ goes as follows. Real financial markets have frictions not only in the
form of transaction costs, brokerage fees and taxes, but also in the free flow of
information. Companies find themselves operating under various constraints in
informing shareholders about the future prospects of the company. And it is in the
interest of shareholders that shares may be bought and sold at prices that fully
reflect this information. These constraints take the form of restrictions on public
forecasts of cash flows by managers – by the accounting profession through its
‘generally accepted accounting principles’, through government regulatory provi-
sions in company security issuances, through the fear of litigation being brought by
disappointed share purchasers should the forecasts prove incorrect, or through the
fear that competitors will receive valuable information about the company’s plans,
thus reducing the value of those plans.
Yet it is obviously in the interests of managers and shareholders to have share
prices reflect new information as quickly as possible. (This is true even when it is
bad news, because you are likely to fool the market only once or twice by hiding or
delaying bad news, and thereafter would have a difficult time getting good news
believed.)
How then might shareholders be informed of events that cannot be explicitly
broadcast? This can take place through subtle signals that the company gives – by
alterations, for example, in its dividend payment. There is evidence, though by no
means conclusive, that companies do just that when changing dividends.
It is now easier to see why dividends must be reasonably smooth over time. If,
for example, a company pursued a policy of paying as much cash dividend as
possible (a ‘widows and orphans’ policy), the time pattern of dividends would be
driven by the occurrence of ‘residual’ cash across time. For most companies this is
likely to be a rather random process. And it is difficult to the point of impossibility
to signal information by changing dividends if the base pattern from which the
signal is to be interpreted is random. The same would hold true for any other
clientele-based policy.
So one reason for the smoothing of dividends across time is that the dividend
announcement can be made to be a ‘surprise’ (either good or bad) to the market.
Based upon the past pattern of dividends (either with respect to time or relative to
other measures such as earnings), the market will have developed an expectation for
What of the claim of ‘investing in itself’? A company can no more invest in itself
by repurchasing its shares than a snake can successfully nourish itself by chewing
upon its tail. Eventually truth will out. There will always be a 100 per cent equity
claim outstanding as long as there is one share not repurchased, and since that single
share would claim the entire firm, it would take the entire equity value of the
company to repurchase it. This exposes share repurchase for what it truly is: a
payment of cash dividend (or, in the extreme, a liquidation of the company).
Does this mean that company share repurchases are to be scorned and avoided as
duplicitous? Not at all. Remember, the market is never fooled by such transparent
statements. Why then are such actions taken? One reason is that in some countries
the money received by shareholders in share repurchase transactions is taxed more
lightly (or even not at all) in comparison with cash dividends declared by the
company. Obviously it is bad public relations for a company to announce that by
share repurchase it is seeking to help its shareholders avoid paying their income
taxes on dividends. So the announcement of ‘investing in itself’ is made. Share
repurchases on the open market also show some signs of being signalling attempts
that receive a positive response from shareholders.
There is one type of share repurchase that is not so positive for shareholders,
however. In some countries a company can undertake a ‘targeted’ share repurchase.
This is a transaction wherein a company offers to repurchase only particular shares
(usually held by an individual or group which the company’s managers are fright-
ened will take over the company and make things less pleasant for existing
management). The repurchase price is usually at a significant premium above the
existing market price of the company’s shares. And the shares of the company on
the open market usually decline in price even more than would have been expected
by the loss of the cash premium paid.
Evidently the market thinks that targeted share repurchases are bad news for
shareholders, in that existing managers will now be left to make decisions about the
company without the implied oversight of the external ‘market for managerial talent’
evidenced by the now bought-out shareholdings.
Learning Summary
The discussion of dividend policy in this module has run the full gamut from a clear,
simply analysed decision in perfect financial markets to a very complex process with
many different alternatives and tactics available to the company in its wish to
maximise shareholder wealth. We have seen that in ‘frictionless’ financial markets
(without taxes and transaction costs, and with the free flow of information) divi-
dends make no difference. We have also seen that no markets are really so
frictionless, and that there is at least the possibility that shareholders may prefer one
dividend policy to another because of real frictions that are experienced.
Companies, however, in seeking to maximise shareholder wealth can be expected
in aggregate to have offered the market the mix of dividend policies that ‘clears the
market’ of potential share premiums for any particular dividend policy, even in the
presence of dividend clienteles. That leaves us with the notion that dividends are not
really irrelevant, but that marginal gains from switching dividend policies are
unlikely, and may even be costly to shareholders.
Finally, we saw that dividends may be used as signals by managers to sharehold-
ers for information that would be expensive to disseminate otherwise. That may
cause companies to ‘smooth’ their dividends across time more than a basic wish to
serve a particular clientele would suggest. Such signals could also be accomplished
by many other techniques, such as share dividends instead of cash dividends.
Though these are the main ideas about company dividend policy, they are not
exhaustive of all the implications of company dividends for shareholder wealth. For
example, one effect on the dividend a company declares might be that the debt the
company has issued has a legal restriction on the amount of dividend that a compa-
ny is allowed to pay. (And companies may pay less than the maximum allowable
dividend in some years so as to build up a reservoir of allowable dividends should
they face a time of plentiful residual cash and few positive NPV investments.)
There are those who think that dividends also play a role in the problem of con-
flicts of interest between shareholders, managers and creditors. These are called
agency considerations. The payment of cash dividends can be regarded as a
shifting of control of these assets from managers to shareholders, the latter then
having the option of whether or not to allow managers to regain operating control
over such assets. This may constrain managers to behave more in the interests of
shareholders.
Dividends can be used to shift assets out of the company and therefore from the
potential claim of creditors. Dividend payments can also change the overall riskiness
of the company’s asset base. Both of these can be detrimental to creditor wealth,
and creditors will doubtless take pricing or contractual actions to offset these
potential uses of dividends.
Company dividend decisions are thus not the simple process of deciding how
much cash is left after all commitments and plans have been executed, and paying
that amount to shareholders. The considerations of signalling, agency and the
effects of market imperfections upon optimal dividends are important dimensions
about which financial managers must be aware.
Review Questions
8.1 Suppose that ABT plc is a company in a financially frictionless, taxless, information-
efficient world. ABT’s next dividend is a propsed cash dividend of £10 per share, and the
market expects this dividend to continue for ever, increasing at an annual rate of 5 per
cent. ABT’s equity discount rate is 15 per cent per annum. Using the perpetuity-growth
valuation technique:
Dividend £10
Price/share £100
0.15 0.05
ABT’s shares thus now sell for £100 apiece. ABT is considering an alteration in its
dividend payout from the existing 30 per cent of available cash to 60 per cent of that
amount, a doubling of the cash payment, to begin at the next dividend payment date.
Any shortfall of cash retention will be made up with new share issuances. Which of the
following will be the effect upon ABT’s share prices of the dividend change?
A. The shares will double in value, because the share price is a function of the
expected future dividends, which are now expected to be twice as much.
B. The shares will have the same price, because the increase in annual dividend for
the current shareholders will be exactly offset by a decrease in the rate of
growth of their dividend.
C. The shares will decline in value because the existing shareholders are effectively
liquidating the company by taking such a large dividend.
D. The shares will have the same price, because shareholders will find half of the
dividend now taxed away.
8.2 You are a fledgling financial manager in a company operating in a taxless, frictionless,
information-efficient world. Your boss comes into your office and says, ‘What’s all this I
hear about dividends being irrelevant? I remember very clearly from my courses in
finance that dividends are the very basis for share value! Does that imply that a compa-
ny’s share value is irrelevant? Explain things to me.’
Which of the following should be your response?
A. Explain that dividends themselves are not at all irrelevant, and are in fact the
basis for share prices, which are themselves representative of shareholder
wealth, but dividend policy irrelevancy is a useful pedagogical device that
finance texts employ to illustrate how real companies begin thinking about the
dividend decision.
B. Explain that dividends themselves are not at all irrelevant, and are in fact the
basis for share prices, which are themselves representative of shareholder
wealth, but dividend policy irrelevancy can happen even with complex taxation
and preferences for consuming wealth by shareholders as long as there are
many companies competing in offering dividend policies to the market. This is
because the forces of demand and supply for particular dividend policies would
cause their ‘prices’ (or benefits for switching from one policy to another) to be
equal.
C. Explain that dividends themselves are not at all irrelevant, and are in fact the
basis for share prices, which are themselves representative of shareholder
wealth, but dividend policy is irrelevant because shareholders do not care how
they receive their wealth, either in cash or in capital gains, and the dividend
decision merely allocates a constant amount of wealth differentially between
those two forms.
D. Explain that dividends themselves are not at all irrelevant, and are in fact the
basis for share prices, which are themselves representative of shareholder
wealth. Further explain that modern finance theory is not that dividend policy is
irrelevant. Irrelevancy only occurs when all real-market frictions such as taxes
and transactions costs are assumed away.
8.3 Cellular Telesystems plc pays, on average, a high percentage of its income as dividends
to its shareholders. This dividend is not a constant percentage of earnings but tends to
be reasonably stable in value, increasing in absolute cash amount over time. Cellular,
being a high-technology growth company, has large needs for investment outlays over
time, and thus often goes to the capital market to raise money for investment purposes.
Which of the following best describes Cellular’s dividend policy?
A. A very poor one, because it is engaging in unnecessary transaction costs of
raising new equity capital across time. If the dividend payout were lowered, less
outside cash would be necessary from new shareholders and the attendant
costs would be reduced, to the benefit of existing shareholders.
B. Excellent, because the company is maximising its share value by paying as high a
cash dividend as it can, because dividends are the basis for share value.
C. Good, because the shareholders of the company are ‘fat cats’ in high tax
brackets, and thus prefer the policy that the company has chosen.
D. Reasonable, because the stable pattern of dividends allows the company to
‘signal’ events to the market, and the split between capital gains and cash
dividends has likely attracted a shareholder group that prefers it.
8.4 A company finds itself with cash from operations in excess of its profitable investment
needs, and well in excess of its dividend payment in the prior period. It must decide
what to do with the money. Which of the following would be a reasonable choice?
A. Declare an ‘extra’ dividend to shareholders, with the understanding that the
cash amount is not to be taken as a ‘good news’ signal about future earnings
prospects.
B. Declare a cash dividend for the excess amount of cash along with the usual
dividend.
C. Retain the cash and invest it in government securities.
D. Retire (i.e. pay off) some debt.
Learning Objectives
Companies must decide how they are to be financed, and in particular what mix of
debt and equity claims are appropriate for the firm to issue. Module 9 develops the
set of ideas and techniques by which companies can decide which mix offers the
greatest promise of enhancing shareholder wealth. First the module shows how debt
in a company’s capital renders its equity claims more risky. The ‘EBIT-EPS’ chart
illustrates this risk dimension of gearing or leverage (the existence of debt). As with
dividend policy, the module then shows an idealised set of conditions under which
shareholders would be indifferent about capital structure, followed by an explana-
tion of the effects of real market phenomena such as taxes, competing tax benefits
and bankruptcy costs on optimal capital structures of firms. The module concludes
with some specific advice to companies as to how capital structure decisions can be
made. You will learn from the module not only valuable techniques for deciding
how much debt and equity a firm should issue but also some important financial
economics of such decisions, which can readily be extended to making decisions in
less familiar situations.
9.1 Introduction
Suppose that you are the financial manager of a company. While about your tasks
one day, a proposal to undertake a new investment project comes across your desk.
The proposal has been thoroughly evaluated by your junior staff and clearly has a
positive NPV (or IRR exceeding its opportunity cost), and so it should be accepted
by the company. After examining the company’s financial resources, however, it is
also clear to you that the firm does not have enough money in the bank (or in other
liquid assets) to pay the costs of the project, nor are operating revenues large
enough to cover the investment outlay.
Obviously, the company will be forced to raise new capital so as to be able to
undertake the project. At this point the major question to be addressed in this
module makes its appearance: should the money be borrowed, or should it be raised
from shareholders?
This situation and the question it presents is one way of thinking about the capi-
tal structure decision companies face. A company’s capital structure is the extent
to which it is financed with each of its capital sources (debt and equity). Actually, a
company’s capital structure is the entire mix of financing it uses, which may include
types other than pure debt and equity (such as preference shares, warrants and so
forth), but for the purposes of this discussion the above definition will serve well.
In this module we shall explore the company capital structure decision. As with
the financial decisions we have examined in prior modules, our concern will
primarily be the decision’s potential effects upon shareholder wealth, here due to the
choice of financing type: debt or equity.
interactions will be instructive of the fallacy of the argument, and will also advance
our understanding of company capital structure decisions.
E SML
re
U
ru
Required return
D
rd
Figure 9.1 The security market line and required returns to capital
claims
Figure 9.1 illustrates this point with the help of the SML. Here you see three
points plotted on the SML. Point U is the amount of risk and required return
inherent in the ongoing operations of the firm itself, independent of its capital
structure. (The letter ‘U’ appears because it signifies the amount of risk that would
exist for the shareholders of the company if the firm were ‘ungeared’, or had no
debt in its capital structure. Here the only risk borne by shareholders is from the
operations of the company.) Points D and E in Figure 9.1 indicate the amounts of
risk and return that exist for the company’s capital claims if the firm is partially
financed with debt (D) as well as equity (E). Note that, because of debt’s higher
position in the hierarchy of capital claims, it gets ‘first chance’ at the company’s
operating cash flows. And because the company is only partially financed with debt,
debt does not bear the full operating risk βu of the company. Equity, on the other
hand, now has a residual claim, with debt taking precedence. So the equity of a
company that uses debt as part of its financing must be riskier than ungeared equity.
Thus point E, the geared equity risk–return location, must be higher up the SML
than point U and require a higher return.
The rationale for the questioner’s assertion that debt’s return is always lower than
equity now becomes clear. What is not yet clear is why we can counter-assert that it
is not correct to think that debt financing is therefore ‘cheaper’ than equity.
Perhaps the simplest way to understand why debt is not cheaper is to return to
Figure 9.1 with a slightly different perspective. Suppose that there is an ungeared
company with the risk–return location U. The corresponding value of ru will be its
required return, an ungeared equity return. Suppose further that there is another
company, identical in operations, but with some debt in its capital structure, as
represented by points D and E in Figure 9.1. If the questioner above were correct,
we would see the average capital costs of the geared firm being lower than those of
the ungeared firm. Do we see that in Figure 9.1? It is not clear that we do.
Examine the relative risk and return locations of U, D and E. It is true that D,
debt’s risk and return, is less than U. This is consistent with the questioner’s point
about debt being less costly than equity. But note the relative locations of U and E.
The existence of the debt’s prior claim has increased the risk (and thus the required
return) to equity. Geared equity (E) is riskier than ungeared equity (U). Thus debt is
‘cheaper’ in the sense of carrying an interest rate or required rate of return less than
equity, but the very existence of debt’s prior claim serves to increase the risk of
equity to be higher than it would be without debt. This means that debt is not in the
fullest sense necessarily a ‘cheap’ capital source when the interactions between debt
and equity financing are considered.
A more careful examination of the economics of company capital structure as
evidenced by Figure 9.1 shows that it is entirely possible that whatever gains are had
from issuing low-interest debt might well be offset by increases in the risk and
attendant returns required by shareholders. (Another perspective on this same
question is to ask yourself which company, the geared or the ungeared, has the
lower WACC. You should be able to tell from Figure 9.1 that it is not at all clear
which is lower, if the ‘average’ required rate is used to judge the WACC.)
amount of money available to service both debt and equity (of course taxes must be
subtracted, but again, for simplicity, let us assume taxes away).
Ungeared plc is financed totally by an equity claim that has a market value of
£1 000 000. So the company’s shareholders expect a perpetual £120 000 per annum
and value that at £1 000 000.
Geared plc is a firm identical to Ungeared except that Geared uses some borrow-
ing in its capital structure. Geared has borrowed £500 000 and has agreed to pay
interest of £40 000 per annum on the loan. Thus Geared bondholders expect to get
£40 000 per annum and its shareholders look forward to a perpetual £80 000 per
annum, based upon the operating EBIT of £120 000 with a £40 000 prior claim due
to the lenders.
Geared shareholders obviously have a smaller investment in their company than
do Ungeared shareholders. On a value basis, Geared is using 50 per cent debt, so let
us assume that it has only half the number of shares outstanding that Ungeared has.
(That would make the value per share the same for each company.) Specifically, let
us assume that Ungeared has 10 000 shares outstanding worth £1 000 000 (or £100
per share), whereas Geared has 5000 shares worth £500 000 (thus also worth £100
per share).
To see how the existence of debt in a company’s capital structure causes its equi-
ty to be riskier, we shall now illustrate that Geared’s equity is riskier than
Ungeared’s. This will be sufficient to prove our point because the companies are
identical in all ways save capital structure.
In Module 7 we showed that risk is best measured by the β coefficient, or the
extent to which a capital claim’s returns vary with the market. The β coefficient, you
recall, can be thought of as the standard deviation of a security’s returns multiplied
by its usual relationship with the market. Other things held the same (as they are in
this example), we have that the higher the variability of a security’s returns, the
higher its risk. So we can test whether Ungeared’s or Geared’s equity is the riskier by
seeing which has the higher variability of returns.
But the only information we have about the returns of the two companies’ shares
is the expected returns. How about the potential variability of their returns? To
answer this, remember that companies’ operating cash flows (or EBIT in this case)
are not known for certain. We have said that each here expects EBIT of £120 000,
but that is merely an expectation. Let us further specify the probability distribution
of EBITs by assuming that for both companies it is as in Table 9.1.
Here we have been more specific about the EBITs that might actually occur each
year for the companies, and the likelihoods of those occurrences. Note that the
expectation or mean of the probability distribution is in fact the £120 000 that we
have earlier portrayed. (Multiplying each outcome by its probability of occurrence
and adding yields £120 000.)
With this new information about their operating or EBIT risks, let us now turn
our attention to the risks of the two companies’ equity claims. Ungeared plc has
only equity outstanding, so the entire EBIT goes to shareholders. (Again, for
simplicity we assume away complexities of depreciation, retention, investments, and
so forth.) Therefore the risk of Ungeared plc equity is exactly the same as its EBIT,
given above.
To be able to make reasonable comparisons between the two companies, howev-
er, we must somehow recognise that the shareholders of Ungeared plc have invested
a different amount (£1 000 000) than those of Geared plc (£500 000). The easiest
way to adjust for this difference is to compare the returns and risks on a ‘per share’
basis (remembering that the shares are selling for the same £100 price per share).
Thus for Ungeared, we can find the distribution of returns to shareholders on a
per-share basis by dividing the outcomes by the number of shares outstanding
(10 000). Table 9.2 does that, and shows the probability distribution of returns to
shareholders of Ungeared on a per-share basis.
Geared plc in contrast has debt in its capital structure so as to require only 5000
equity shares of financing. On a per-share basis, the risk to Geared’s shareholders
comprises both the company’s operating (EBIT) risk and also the risk inherent in
the existence of higher-priority (often called ‘senior’) interest payments to the debt
capital. Table 9.3 shows the probability distribution of returns for Geared’s equity
and debt.
The lesson about the risk of equity in the presence of company borrowing is
easily seen by comparing the EPS outcomes and probabilities of Table 9.2 and
Table 9.3. Table 9.2 shows what happens to the per-share income of equityholders
in Ungeared plc when operating income (EBIT) varies from £15 000 to £150 000.
Note that the range of this variability is from £1.50 to £15.00 per share. On the
other hand, the shareholders of Geared plc experience a range of per-share incomes
of −£5.00 to £22.00 when the company’s operating income has the same variability
as Ungeared’s. The reason that Geared shareholder returns are more risky is obvious
from Table 9.3: the £40 000 interest payments stand ‘first in the queue’ to receive
any cash forthcoming from the operation so geared. The variability of Geared’s
operating income is thus amplified (or ‘leveraged’) upwards to shareholders by the
existence of company borrowing.
Comparison of Table 9.2 and Table 9.3 indicate that the range of possible returns
to shareholders is much greater (with the same probabilities) for Geared than for
Ungeared. Geared’s equity must therefore be more risky. Geared shareholders are
obviously in the more risky situation, yet the only difference between the two
companies is that Geared plc has to some extent used debt in its capital structure,
whereas Ungeared plc is ungeared.1
Figure 9.2 uses the operating income and shareholder return information in Ta-
ble 9.2 and Table 9.3 and plots these in a graph. Note that operating income (EBIT)
and shareholder return exhibit a much more volatile relationship for Geared than
for Ungeared. A given change in operating income for Geared produces a much
greater change in shareholder result than for Ungeared. (Incidentally, this graph has
a name in finance. It, predictably, is called an EBIT-EPS chart. Such displays are
used to illustrate graphically the risk to shareholders inherent in using particular
amounts of borrowing in a company’s capital structure. And the terms ‘gearing’ or
‘leverage’ are verbal descriptions of the differences in steepness of the Geared and
Ungeared lines in EBIT-EPS graphs.)
EPS £ Geared
22.00
16.00 Ungeared
15.00
12.00
1.50
0
120 000 150 000
EBIT £
–5.00
Figure 9.2 An EBIT-EPS chart for Geared plc and Ungeared plc
It should also be clear that the more borrowing a company does, the steeper will
be the line depicting the EBIT-EPS relationship, and, also, the higher the equity
risk. This type of risk is often called financial risk to distinguish it from the
underlying ‘line of business’ or operating risk (also known as ‘operating leverage’)
that resides in EBIT.
The above discussion should be sufficient to convince you that company bor-
rowing causes the returns to shareholders to be more risky than it would otherwise
be. One or two other issues may, however, have caught your attention in that
discussion, and we will now move on to deal with them.
First, you may have been curious how Geared could generate a negative EPS
(−£5.00) when the £15 000 operating income outcome results. The arithmetic is, of
course, easy: £15 000 of income minus £40 000 of interest is a negative £25 000,
which divided by the company’s 5000 shares yields −£5.00 per share. The bother-
some aspect of the result is the seeming contradiction with shareholders’ limited
liability. In other words, one way in which the negative EPS would have been
produced would be for the company to raise the £25 000 from shareholders so as to
pay interest to debt. And shareholders under limited liability are under no legal
obligation to make good on company debts. (Another way to raise the money would
be for Geared to sell some assets.)
Only if such cash is paid to creditors will the £40 000 interest be legitimately
included in the company results, and shareholders need not pay it. The question is:
would they? The answer is that they probably would, given that unless they do,
creditors can foreclose on the debt and take over the assets (and future expecta-
tions) of the company. A glance at the probability distribution of operating results
and EPS for Geared plc would indicate that the odds are that shareholders will do
much better on the next period’s ‘roll of the dice’, and therefore would be wise to
pay the interest even though they need not.
Of course a company can default on an interest payment whenever it chooses,
not just when operating income is less than the interest due. One can thus in general
regard the payment of interest by companies as essentially a transaction wherein
shareholders repurchase the assets of the firm from creditors, the price being the
interest payment. The counterpart to this view of the transaction is that when
companies originally borrow, they sell their assets to creditors but withhold an
option to repurchase the assets by making interest payments. Keep this thought
somewhere in the back of your mind as we continue to discuss company capital
structure decisions. It will serve us well in discussions to come.
Finally, notice that there is a nice consistency in the required returns of the vari-
ous capital claims of Ungeared and Geared. Turn your attention to Figure 9.3,
remembering that the two companies are operationally identical. Ungeared, with
only equity claims outstanding, is worth £1 000 000. That £1 000 000 is generated by
a perpetuity expectation of £120 000 per annum, implying a required return re of 12
per cent. This is plotted as point U in Figure 9.3.
SML
E
re = 16%
U
ru = 12%
Required return
D
rd = 8%
0
RiskD RiskU RiskE Risk
increase in the (generalised) geared shareholder required return exactly offsets the
lower cost of borrowing.
As you would expect, this numerical result was designed into the arithmetic of
the example. But there is an important set of economic arguments that produces the
same result, not by artificial construction but by financial market actions. There are
still other important capital structure arguments that yield quite different predictions
as to what happens to company capital costs when debt is issued. We now have
enough of an introduction to such a company financial decision as to look at these
debates and learn what is known about the best way to decide how much a company
should borrow.
2 Williams, J.B. (1938) The Theory of Investment Value, Harvard University Press, Cambridge, Mass.
3 Modigliani, F. and Miller, M. (1958) ‘The Cost of Capital, Corporation Finance and the Theory of
Investment’, American Economic Review, 48 (June), 261–97.
does not discriminate among the various types of claims upon which it sets prices.
This condition means simply that two claims promising the same probability
distribution of future cash flow will sell for the same price.
Now suppose that there are many firms like Ungeared plc in the market and
there is no company like Geared plc. Ungeared is thinking of becoming geared by
borrowing some money and is interested in the effect of that capital structure
alteration upon its shareholders. To become geared, Ungeared would borrow
£500 000 at an interest rate of 8 per cent, and the market is prepared to lend money
to the company at that rate. The question is thus: ‘What will happen to the wealth of
Ungeared shareholders if the company alters its capital structure in that fashion?’
The answer to this question will require that we discover what happens to Un-
geared’s equity value when it becomes geared. To see this, recall that prior to the
borrowing, Ungeared equity is worth £1 000 000 (based upon claiming a risky
£120 000 perpetuity at a 12 per cent required return). The £500 000 cash that the
company borrows as it becomes geared must somehow be given to its shareholders
(either through a dividend, share repurchase, or cancellation of a new equity
flotation), or else there would also be a company asset alteration.
So the shareholders of Ungeared will immediately get £500 000 more cash, but
have given up something in return. They have effectively sold to lenders a £40 000
prior claim on the future operating cash flows of Ungeared, receiving the £500 000
in compensation. Of course, the loss of first priority claim on £40 000 of future
operating income in perpetuity must cause the equity value of Ungeared to fall as it
becomes geared. The question is how much it will fall.
Table 9.4 illustrates the situation of shareholders. If equity value decreases less
than £500 000, shareholders of Ungeared will gain wealth from becoming geared (at
the same time renaming the company Geared plc). If equity value declines more
than £500 000 shareholders will be worse off, and if it drops exactly £500 000 they
will experience no wealth change as a function of the company’s alteration of its
capital structure.
Table 9.4 Ungeared plc to Geared plc: Shareholder wealth and capital
structure
Equity value + Cash = Wealth
Ungeared £1 000 000 + 0 = £1 000 000
Geared ? + £500 000 = ?+£500 000
What will be the equity value of the renamed company? Here is where M&M make
their important contribution. They argue that the equity value of Ungeared must
change to equal exactly £500 000 as Geared equity, and leave unchanged the wealth of
shareholders. Their argument is as follows. Suppose the equity value of Ungeared
changes to £600 000. That equity now will be claiming a future cash flow of the
company’s operating EBIT of £120 000 minus the £40 000 company interest pay-
ment, namely £80 000 per year in perpetuity. Ungeared shareholders will have
experienced a wealth increase of £100 000 because they get £500 000 in cash from the
debt issuance and retain £600 000 of equity value, totalling £1 100 000. Prior to the
capital structure alteration their wealth was £1 000 000 of ungeared equity.
But is this likely to happen? Can Geared’s equity value actually be £600 000?
M&M would say no. Again, think of the situation of the Geared shareholders. They
are holding shares worth £600 000 which entitle them to £80 000 in perpetuity
coming from the company’s operating cash flow of £120 000 with a £40 000 prior
claim by lenders. That £600 000 of equity is not, however, a viable value in the
capital market. Why not? Because the identical future cash-flow expectation can be
achieved in a different manner, and less expensively. Let us now see how.
Suppose that Mr Wrench wishes to acquire an expectation of £80 000 in perpetu-
ity, with the same risk characteristics as Geared’s equity cash flow. This could be
accomplished by simply buying the shares of Geared for £600 000, or by buying the
shares of Ungeared but borrowing personally so as to arrive at the same £80 000
expectation (£120 000 from the company, with a £40 000 personal interest pay-
ment).
Suppose that Mr Wrench personally borrows £500 000, the same amount as
Geared did, and puts up enough cash to purchase the shares of Ungeared. At what
interest rate could he borrow? If the shares of Ungeared plc are offered as collateral,
Mr Wrench would be able to borrow at the same 8 per cent interest rate that the
renamed Geared plc does. This is because the company and personal loans would
be identical. Geared borrows by promising first claim on its future operating cash
flows, and Mr Wrench will be borrowing to purchase Ungeared by offering identical
(Ungeared operating cash flow) collateral.
After this transaction, Mr Wrench now finds himself owning all of the shares of
Ungeared, which entitles him to an operating cash-flow expectation of £120 000 per
annum. He has personally borrowed £500 000, offering only the shares of Ungeared
as collateral, and has acquired an 8 per cent interest rate in the non-discriminatory
capital market. That creates a £40 000 prior claim upon his personal cash-flow
expectation from the Ungeared shares. So Mr Wrench now has an £80 000 net
personal cash-flow expectation arising from a £120 000 risky Ungeared operating
cash flow with a £40 000 prior claim to lenders. Table 9.5 illustrates this compari-
son.
Table 9.5 Shareholder cash-flow expectations: Personal and company
borrowing
Company borrowing Personal borrowing (Mr
(Geared shareholders) Wrench borrowing and
buying Ungeared)
Company operating cash £120 000 £120 000
flow
Company interest payment £40 000 0
Net cash from company £80 000 £120 000
Personal interest 0 £40 000
Net personal cash flow £80 000 £80 000
Note that in terms of future cash flows, having borrowed personally, Mr Wrench
has the same expectations and risks as do the shareholders of Geared plc. So in
terms of their personal portfolio of holdings, they have identical future cash-flow
expectations. Note also, however, that the portfolio of personal borrowing and
Ungeared shares has cost Mr Wrench only £500 000 out of his pocket (the
£1 000 000 Ungeared share value minus the £500 000 personal borrowing), whereas
the Geared shares are valued in the market at £600 000.
This is an untenable situation. The Geared shareholders would quickly sell their
shares and borrow personally to purchase Ungeared shares (getting the same future
cash flows and netting themselves a nice £100 000 in the bargain). In fact, everyone
would avoid the overpriced Geared shares. (Said another way, who would be silly
enough to buy the shares of Geared for £600 000 when the identical future cash
flow is available through Ungeared and personal borrowing for only £500 000? The
answer is, of course, no one.) In an efficient market the shares of Geared must sell
for just what it would otherwise cost to acquire the same future cash-flow expecta-
tions. The Geared shares must sell for £500 000 – no more and no less. (We could
illustrate a similar mispricing if Geared shares sold for less than £500 000, based
upon others willing to pay £500 000 for Geared’s future cash flows, but the intuition
that such an argument can be made is sufficient.)
What does this example of company and personal gearing tell us about company
capital structure decisions? It says something that seems surprising at first: company
capital structure decisions do not matter. When Ungeared became Geared, with
Geared shares necessarily worth £500 000, shareholder wealth is unchanged.
(Substitute this result in Table 9.4 to see the truth of the statement.) Because
shareholder wealth will be the same regardless of company capital structure, the
latter is irrelevant.
This statement must, however, be carefully interpreted. Company capital struc-
ture is irrelevant because it has no effect upon shareholder wealth in the market we
have herein designed. Because shareholders can borrow and lend on the same basis
as companies, any benefit (or detriment) residing in company borrowing can be
duplicated (or cancelled) by shareholder borrowing or lending transactions in their
own personal portfolios. So company capital structure does not matter.
Company capital structure in this illustration does affect the risk and return of
equity. Recall that Ungeared shareholders’ required returns increase when the
company becomes Geared, recognising the increased risk of their holdings in the
company. But the shareholders’ wealth did not change (the Geared shareholders
have riskier shares and riskless cash rather than the lower-risk shares of Ungeared).
And if the now-Geared shareholders are not pleased with their £500 000 of cash
and £500 000 of riskier shares, they need merely sell their Geared shares and use the
resulting total £1 000 000 of cash to purchase all of the shares of a company such as
Ungeared plc. Then the shareholders would be right back where they were before
Ungeared became Geared. So company capital structure is completely irrelevant in
the world of Geared and Ungeared.
Parenthetically, we also note that Geared’s cost of capital is the same as Un-
geared’s (you can find the numerical demonstration at the end of the previous
section), which makes the same point about irrelevancy but more succinctly.
One thing that often bothers those who hear this M&M argument for the first
time is that it seems to depend upon very artificial assumptions, to the extent that
the real market applicability of it is not clear. As we shall see very soon, there are
indeed complications that must be addressed in real markets. But the basic econom-
ics of the M&M argument will remain intact and are very important.
There is one criticism of M&M that we should address directly at this point,
however. Many, upon hearing this discussion, are disturbed that it seems to require
several operating companies, all identical to our Ungeared plc, so as to work
correctly. This is not at all the case. The assumption that there were many identical
operating companies available to shareholders for their personal portfolio realloca-
tions made the illustration simpler to understand, but is not necessary for its correct
functioning. All that is necessary is that shareholders have other opportunities to
acquire the same risks, from whatever source. And such risk-altering opportunities
are widely available in real markets. For example, there are non-identical companies
that have the same operating risks; or there are non-identical companies that can be
bought with personal borrowing or lending to duplicate the ungeared operating risks
of any other company. Or there are any of a number of other non-share invest-
ments (such as real estate, precious metals, and so forth) that shareholders could use
so as to realign the risks of their portfolios and accomplish the M&M result. M&M
were perfectly aware of this, but doubtless felt that pedagogical clarity required them
to keep the illustration as simple as possible.
Think of what this means. It tells us that the market is ever vigilant for mispric-
ings of single securities and all of the combinations that can be formed from
securities. Even if two portfolios have wildly different securities in them, if the
portfolios’ aggregate future expectations are identical, the portfolios must be equally
valuable, and the individual security prices within them will adjust to accom-
plish that. If the prices do not, there will be an arbitrage opportunity, which would
cause investors to leap at the chance to increase their wealth at no risk. The forces
of demand and supply will quickly cause prices to adjust so as to destroy the
arbitrage opportunity. Arbitrage opportunities thus disappear when such portfolios
have the same values.
From the perspective of company capital structure decisions, remembering that
company and personal borrowing and lending are types of securities, we now know
that the market is capable of pricing these as well as any other security or portfolio.
This will ensure that the same consistency among future cash-flow expectations and
market prices reigns with respect to capital structure decisions, as it does with any
other portfolio-altering action.
1 200 000
1 000 000
V
800 000
£
D
600 000
400 000
E
200 000
0
0.00% 16.67% 33.33% 50.00% 66.67% 83.33% 100.00%
'Ungeared' 'Geared' D/V Proportion
25%
20%
re
15%
ru rv
10%
rd
5%
0%
0.00% 16.67% 33.33% 50.00% 66.67% 83.33% 100.00%
'Ungeared' 'Geared' D/V Proportion
Notice, however, that with both the equity and debt rates increasing with D/V,
there is no increase in the company’s overall capital cost or required return, rv, their
value-weighed average, which remains steady at 12%. How can this happen? How
can it be that rv does not increase even though its components re and rd do? The
answer is best understood by returning to the value relationships in Figure 9.4.
There we see that V, total company value, does not change as D/V increases. Since
company value V must be simply its cash flows discounted by its overall required
returns, or
FCF
V
we must have that FCF as well as V is left unchanged as D/V increases. Therefore
from the above simple value relationship, the company’s overall capital cost (rv)
must also be left unaffected by alterations in the company’s capital structure.4
With respect to the specific weighted average relationships determining rv, from
Module 4, we have
which implies
1
Even though both the equity re rate and the debt rd rate increase with in-
creases in D/V, notice that the value weight [1 − (D/V)] on the (higher) equity rate
re will steadily decline, and the value weight D/V on the (lower) debt rate rd
will steadily increase as D/V increases. The higher proportion of lower-cost debt
exactly offsets the lower proportion of higher-cost equity, such that their
weighted average is unchanged. This is not magic. It is dictated by the constant-
value relationship above and, more directly, reflects simply the forces of the capital
market at work. By the arbitrage examples we have studied, it would be impossible
for any other result to occur in the financial markets as we have constructed them
up to this point.
4 That a company’s value is the present value of its future free cash flow should be clear from our work
in Module 4.
for income tax purposes, the companies’ taxes are lower by an amount called the
interest tax shield.
For example, let us assume that there is a company income tax of 50 per cent and
that this applies to the world of Ungeared and Geared from the previous section. In
the absence of any other kinds of taxes (such as taxes on personal income), the cash
flows to the shareholders would change from the taxless world of Table 9.5 to those
of Table 9.6.
Note that the geared company pays £20 000 less in taxes than the ungeared.
From the company cash-flow perspective of Module 4, the interest tax shield in
this case is £20 000, the company interest payment of £40 000 multiplied by its
income tax rate of 50 per cent. This cash benefit accrues directly to shareholders of
Geared plc, and is unavailable to the personally geared shareholders of Ungeared
plc. (Because of different company tax liabilities, the risk attributes of the two
shareholder portfolios are no longer the same, but the basic cash-flow effect is
sufficient to make this point about tax benefits of company borrowing.)
The deductibility of interest payments by companies should cause there to exist a
bias in company capital structures toward the use of borrowing instead of equity
capital. The net result of company borrowing in an economy where there is a
company income tax with interest deductibility is to cause there to be more after-tax
cash available to the capital suppliers of the company than if the company did not
borrow. (Note in Table 9.6 that the bondholders and shareholders of Geared plc
each get £40 000, netting £80 000; on the other hand, the lenders to the personally
geared shareholders of Ungeared plc get £40 000, but the shareholders only net
£20 000, the difference going to government in higher taxes on Ungeared.) This is
an obvious incentive for companies to borrow, and we can expect that the aggregate
value (of both bonds and shares) of borrowing companies will be higher than the
shares of companies that do not.
Interestingly, this general phenomenon can still hold true even when there is a
personal tax on shareholders, and when shareholder personal interest payments are
deductible on their personal taxes. Table 9.7 illustrates how this occurs, using a
personal tax rate the same as the company rate. (That it would also be true with a
lower personal tax rate should be clear, since Table 9.6 in effect uses one equal to
zero.)
Table 9.7 shows that even when both companies and shareholders are taxed on
their income, and with both being able to deduct interest payments on borrowings,
shareholders are still better off with the gearing on the company level as opposed to
the personal level. The reason is that the company tax benefit occurs with respect to
a tax that, once paid, cannot be recouped by shareholders through deductible
personal borrowing. Under the system of both company and personal income
taxation, in the absence of any other tax provisions there will thus remain an
incentive for companies to use borrowing instead of equity, though the relative
benefit is smaller, as the comparison of Table 9.6 and Table 9.7 indicates.
The sentence above contains what has been appropriately called a ‘weasel phrase’
in finance. The phrase is: ‘… in the absence of any other tax provisions …’. As we
are all aware, governments are endlessly inventive in their capacities to generate
complex tax rules. One of the more interesting of these ‘other tax provisions’
appears in the UK tax system, wherein there is an ‘imputation’ of shareholder
personal income tax payment to companies as the companies pay their own income
taxes. We have discussed this system in Module 8; if you have forgotten that
discussion, you should review it (see Section 8.3.1).
The UK ‘imputation’ system does partially eliminate the tax benefit of company
borrowing. Because the imputed shareholder tax rate is less than the company
income tax rate, there is still some tax benefit to the issuance of debt by companies,
even in the UK. Only if the imputed personal rate is equal to the company rate is
the advantage of debt neutralised by an imputation system.
What does this discussion of the interaction of company capital structure deci-
sions and taxes mean? From what we have said so far, it would seem the answer is
straightforward: when taxes exist, and when interest is deductible by companies,
companies will tend to use debt as their primary source of capital. The reason is
simply that debt is ‘cheaper’ in the sense that the total of taxes paid by companies
and their shareholders will be lower than if the companies were to issue equity.
So if we have told the complete story about capital structure, companies would
tend to be financed primarily by debt. But that is not what we see. There is in fact a
wide range of actual capital structures that exists in real companies. Either the
financial managers of those companies are not aware of the tax benefits of borrow-
ing, or there is more to the story than what we have told so far. The latter is of
course the case.
same as the company’s interest payments.5 And the interest payments require a rate
of return equal to rd. So the value of the tax benefit must be equal to:6
ITS
VITS
This formula should be somewhat familiar to you. Look in Section 4.5. There you
see a similar expression in the APV discussion. Actually, this way of thinking of the
effect of tax deductibility on a company’s value is very similar to the idea of the
APV technique. As a matter of fact, we can profitably think of the entire firm as an
APV-type valuation where:
V VITS
In other words, instead of thinking of the company’s value as the sum of its debt
and its equity values, we can just as well think of the company’s value as the sum of
its value if it were unleveraged plus the value of its leveraging-based tax benefits. We
would get the same total value using either our familiar ‘debt plus equity’ or this
APV-type valuation, but this latter view helps us understand the workings of capital
structure much more easily.
How does this value behave as a company increases the proportion of debt in its
capital structure? Figure 9.6 shows the relationships between changes in capital
structure and changes in capital claim and company values, with the rules of the
capital market as we have developed them to this point.
5 This assumes that the tax benefit will be available only contingent on the payment of interest. Since it is
possible that a company could pay interest and not get a tax benefit (for example, if it had no income
to report for tax purposes), we are implicitly assuming the existence of ways to obtain the tax benefit
regardless of the existence of taxable profits. In most countries’ tax systems there are ways to obtain
these tax benefits in such situations, with tactics such as (perfectly legal) tax ‘carry backs’, ‘carry
forwards’ and mergers.
6 The illustration that follows is consistent with our exploration of the WACC in Module 4, and
specifically makes the same implicit assumptions about the riskiness of Interest Tax Shields as that
module. As we mentioned earlier, Miles and Ezzel (op. cit.) argue that these conditions are too
restrictive in many contexts, and suggest that adjustments be made to the ITS valuation formula.
Specifically, they suggest that because of uncertain future operating outcomes, even if D/V is held
unchanged, the variation in V will cause more uncertainty in the amount of interest payments across
time. This in turn implies that the ITS should be discounted using rv rather than rd, (because of the
indirect influence of operating risk on the amount of interest paid), and the resulting ITS value
multiplied by 1 rv / 1 rd to recognize that the amount of expected interest is knowable one
period in advance.
1 200 000
1 000 000
800 000
V
£
600 000 VU
D
VITS
400 000 E
200 000
0
0.00% 16.67% 33.33% 50.00% 66.67% 83.33% 100.00%
'Ungeared' 'Geared' D/V Proportion
7 Notice that the values of the Geared and Ungeared firms are lower than the values we found when the
companies were not subject to income taxes. This seems logical, particularly that Ungeared is worth
exactly half of its ‘tax free’ value, since the company income tax rate is 50%. Where did this value go?
Into government coffers, of course. Actually a corporate income tax is something like a forced equity
participation in the company on the part of the government.
8 Of course, governments do not typically allow companies to do this. There are usually taxation rules
that effectively require some minimum amount of equity (and therefore taxes).
20%
re
15%
rv
10% ru
rd
5%
rv*
0%
0.00% 16.67% 33.33% 50.00% 66.67% 83.33% 100.00%
'Ungeared' 'Geared' D/V Proportion
from Module 4, if V is increasing and FCF* is constant, rv* must be declining as D/V
increases. This is exactly as depicted in Figure 9.7.
What is the importance of this? The essential importance is the illustration that a
company’s cost of capital is affected by the amount of debt in its capital structure,
and if interest is deductible for company income tax purposes (again with some
caveats to come), the company’s cost of capital will be lower the more debt it uses.
The reason for this is simply that interest deductibility is a kind of tax subsidy for
corporate borrowing that is not available for equity financings.
9.4.3 Capital Structure Irrelevancy II: Taxes
One reason why companies do not use debt exclusively is that it may not be as
cheap as it seems from the discussions in the previous section, even with taxation of
the type therein discussed. The reasoning for this was offered by Merton Miller,9
when he asked us to remember that tax benefits of borrowing can only be finally
calculated by finding the amount of cash that companies can get into all capital
suppliers’ pockets after all taxes at the company and personal level have been
deducted. For example, we must remember that bondholders also pay personal
taxes on the income that they get from company interest payments. He further
cautioned us not to forget that most tax systems have personal tax rates that are
‘progressive’ (i.e. are higher with higher income).
This latter observation caused Miller to argue that as companies issued more and
more debt, they would be forced to pay higher and higher interest rates in order to
induce those in higher tax brackets to purchase their bonds. This is because bond-
holders would be paying higher and higher personal income taxes on interest
income from companies. Eventually, the increase in interest rates on company
borrowing would offset the benefit of deducting interest at the company level, and
there would be no particular reason to issue debt instead of equity. In other words,
the tax benefits of company deductibility would still exist, but would be erased at
the personal level by the high personal tax rates that bondholders would be forced
to pay on their interest income.
Competition would cause companies to continue issuing bonds until interest
rates had risen enough to cause all net (company combined with personal) tax
benefits of company borrowing to disappear. Then there would be no reason for
borrowing as a source of capital to be preferred to the issuance of equity.
There is still much controversy surrounding Miller’s assertion that company
capital structure is irrelevant even under taxation and interest deductibility. Various
reservations and counter-arguments have been offered, some with apparent merit,
but it is probably fair to say that the tax benefits of borrowing are not as high as the
simple arithmetic of interest tax shields would imply.
There are yet other considerations within the question of tax benefits of compa-
ny borrowing. For example, interest is not the only thing that companies can deduct
from income so as to reduce income taxes. As we are aware from our cash-flow
discussions in Module 4, various depreciation, tax credits and other tax-reducing
mechanisms are available to companies. Now consider the situation where a
company, because of all of these other tax write-offs, has no ‘income’ for tax
purposes. There is obviously no reason here to choose borrowing for tax reasons,
because all tax benefits thereby potentially available have already been exhausted by
the company through other means.
The existence of ways other than borrowing to reduce taxes would tend to make
borrowing less attractive, but still, for companies without enough of these other
write-offs, debt would remain attractive. When we also recall that the amount of
‘income’ a company will earn in any given year is uncertain, whereas the commit-
ment to pay interest is not, there is a further lessening of potential tax benefits in a
probabilistic sense. This implies that borrowing to obtain tax benefits is less
desirable in an uncertain world than when future ‘income’ streams are known with
certainty.
thereby reduce its taxes rather than use shareholders’ capital, the servicing of which
does not generate tax benefits.
This tax benefit of borrowing is common to almost all developed economies, and
persists even when shareholders themselves pay personal income taxes on their
receipts of cash from companies, and can deduct personal interest from their own
borrowing.
There are several considerations in real financial markets that tend to diminish
this tax benefit of company borrowing, however. In the UK and certain other
economies, there is something like deductibility of equity dividends. Such ‘imputa-
tion’ systems, making equity more attractive, tend to diminish debt’s relative tax
benefits.
In addition, Miller has argued that as more and more borrowing is undertaken by
companies in economies with progressive personal taxes, the interest rates necessary
to sell bonds to high personal-tax investors will cause the benefits of company
borrowing to disappear.
Further, the tax benefits of company borrowing compete with other mechanisms
used to reduce taxes (depreciation, credits, etc.). This tends to reduce debt’s
advantages, particularly when the amounts of ‘income’ that require shelter from
taxes is uncertain.
At this time we have only a quite rudimentary understanding of how these com-
plex relationships between capital structure and taxes interact to produce a final
result. Most who think seriously about this question seem of the opinion that there
is still some net tax advantage to company borrowing relative to equity issuance, but
that gross generalities are dangerous.
of that decision in the fondly recalled days of no income taxes. But that too was not
the case. Companies’ capital structure actions were no more random then than they
are tax-shield-maximising now. Either company financial managers are not behaving
rationally, or there are important influences other than taxes on the amount of
borrowing that a company undertakes. In competitive markets for financial manag-
ers, we would bet upon the latter.
What is the nature of these other influences? Agency considerations seem to
hold the most promise of being the other important factors in a company’s capital
structure decision.
The study of ‘agency problems’ has a long and distinguished history in econom-
ics. ‘Agency’ deals with situations where the decision-making authority of a
‘principal’ (such as a shareholder or bondholder) is delegated to an ‘agent’ (such as
the managers of a company). Agency considerations concern themselves with the
instances where conflicts of interest may arise among principals and agents, and
how those conflicts of interest are resolved. This may sound esoteric, but it has
great practical moment in the study of finance, particularly in the capital structure
decisions of companies.
Consider the following situation. A company has two mutually exclusive single-
period projects, both with positive NPVs of which it can only choose one:
Suppose that Project A is the higher-NPV project, and the company would fi-
nance its (assumed) outlay with borrowing, promising an end-of-period £8000
payment collateralised with the project itself.
The lender asked to provide such financing has an agency problem. The lender
will recognise that the company has an incentive to switch from Project A to Project
B (with the same outlay) once the money has been lent. Why would the company do
that? Think of its shareholders as the residual claimants of either Project A or B.
Each project has an £8000 debt claim against it, so to find the returns to sharehold-
ers we merely subtract the £8000 outlay from each outcome, (remembering that
equity has limited liability):
Returns to shareholders
Project A Project B
End of period Probability End of period Probability
outcome outcome
£0 50% £0 50%
£6000 50% £12 000 50%
If the company has the interests of its shareholders foremost in its decisions,
there will be an incentive to accept Project B even if it has the lower NPV as
calculated using the project’s WACC.
What difference does this make to lenders? Let us examine their payoffs if Pro-
ject B is selected instead of A:
Returns to lenders
Project A Project B
End of period Probability End of period Probability
outcome outcome
£8000 50% £2 000 50%
£8000 50% £8 000 50%
possible if there are creditors. But it is not true that the possibility of bankruptcy is a
drawback of borrowing. As a matter of fact, a logical argument can be made that the
chance of going bankrupt is a positive, not a negative attribute of borrowing.
This surprises many who hear it for the first time, but the logic is inescapable.
One key to understanding this important point about borrowing is to remember
that the unfortunate economic circumstances that precipitate bankruptcy would
exist even if the company had not borrowed. The other key is that bankruptcy’s
essential effect is merely to change the legal ownership of the company’s assets from
shareholders to bondholders. It will be easiest for us to understand the issues of
default and bankruptcy with an example.
Suppose that Crewboats Limited, a petroleum-exploration company, having
borrowed money to purchase vessels, goes bankrupt because oil prices decline and
there is no demand for the company’s services. The value of the vessels of the
company is less than the face value of the debt issued by the company to purchase
them. This condition is most unfortunate for the company, but the question of
bankruptcy’s deleterious effects must be judged from the perspective of whether,
given the decline in demand and the resulting effect on asset values the
company (and its shareholders) are worse off having borrowed than they would
have been had they instead issued share capital. It is obvious that shareholders have
experienced an unpleasant, wealth-reducing episode. What is not clear is whether or
not the type of company financing undertaken (debt) has exacerbated the untoward
economic event (the decline in oil prices).
The real problem here is that the boats owned by Crewboats Limited have de-
clined in value due to the decline in demand for what they do. If the company has
borrowed such that the amounts of cash flow available from running the boats is
insufficient to service the debt, there is little choice but to inform creditors that the
debt payments will be defaulted. And if the boats cannot be sold for enough to pay
the debt’s promised amounts, bankruptcy will probably ensue. But suppose that the
company had instead financed itself with equity capital. There is no reason to think
that the decline in oil prices would have affected the boats’ operating cash flows any
differently if the vessels had been financed with equity instead of debt. So the
decline in value due to oil price reductions would be felt regardless of financing.
What will happen to the assets of the bankrupt company? The odds are that they
will either be taken by creditors and sold, or operated by the company under the
supervision of the courts. Either way, the assets will be operated if it is profitable to
do so, given their current market values, or sold instead if more wealth is thereby
generated. It should be easy to see that the same thing would have happened had
the company financed itself with equity capital, given the decline in demand.
What then is the importance of bankruptcy? It is not that the assets have de-
clined in value due to economic conditions independent of financing; that event
would have occurred anyway. The importance is that there may be extra costs
incurred when a company goes bankrupt (or is otherwise close to financial distress),
which would not appear in the absence of borrowing.
The most obvious of these costs are those of litigation, which can be expensive
and consuming of management time (better spent in operating assets, developing
markets, and so forth). In addition, there are implicit costs such as the delay in using
or selling assets caused by being tied up in the legal system. And there are almost
surely operational constraints (e.g. suppliers unwilling to extend credit, customers
concerned about warranties on products and services to be purchased) that are
costly. As important are the conflict-of-interest situations illustrated in the prior
section, which are now more than ever likely to be burdensome to the company,
and keep it from making optimal least-cost, highest-profit decisions.
So the importance of bankruptcy is perhaps different from what a superficial
analysis would indicate. Bankruptcy costs are not the declines in value that precipi-
tate bankruptcy; those are independent of capital structure. The true costs of
bankruptcy or financial distress are:
1. the costs involved in pursuing the legal process of realigning the claims on the
assets of the company from those specified in the original borrowing contract;
and
2. the implicit and opportunity costs incurred in this effort relative to what would
have happened had the company been financed instead by equity capital.
We can now explain the doubtless cryptic comment at the beginning of this
section, indicating that bankruptcy can be a desirable outcome.
Suppose that there is another company, Crewboats Unlimited, in the same eco-
nomic position as Crewboats Limited, except that the shareholders of the company
do not carry limited liability. Here the decline in asset values below promised debt
payments must be made up by shareholders out of their own pockets. The
company cannot then go bankrupt. Clearly, the shareholders of Crewboats Limited
(being able to walk away from the company with no personal liability) are in a better
position than those of Crewboats Unlimited. The option to declare company
bankruptcy and thereby take advantage of limited shareholder liability is a valuable
one.
But as with any event that may be anticipated, we can depend on it that the fi-
nancial markets have priced the shares and bonds of the two companies
appropriately. When Crewboats Limited sold its bonds and shares on the market it
would have received more for its shares and less for its bonds (i.e. paid lower
returns to shareholders and higher interest rates to bondholders) than would
Crewboats Unlimited. There is no ‘free lunch’ in limited liability. The shareholders’
right to walk away from company debt will be priced originally in the company’s
shares and bonds.
This illustration can be used to remind ourselves of another important aspect of
capital structure’s financial economics. Suppose that between Crewboats Limited
and Crewboats Unlimited there were no differences in the costs of bankruptcy, or in
the capacities for shareholders and bondholders to make judgments about the risks
and returns from holding the companies’ capital claims. In that situation, the total
values of the two companies’ capital claims would be the same, with Crewboats
Limited having the higher share (and lower bond) values, and Crewboats Unlimited
having the higher bond (and lower share) values. In the same sense as there is
capital structure irrelevancy between debt and equity in the original M&M illustra-
tion, here, as long as the market sets the prices of securities efficiently, there is
10 One such consideration is probably the high costs of estimating and collecting the personal resources
of shareholders lacking limited liability.
11 This one actually lets the bondholder initially assume that the company will make a decision adverse to
the interests of bondholders, and thus set low prices on the bonds at initial issuance. But then, when
the company does not take such a decision, the bond value increase occurring can be captured by the
company for its shareholders through exercising the call provision on the bonds at a price lower than
the upwardly revised expectations would require.
are only the simplest of the possible set of such complex provisions that can be
designed to avoid the inefficiencies of agency relationships.
uninformed. Very often, not necessarily by any deep analytical process, such rules
have evolved valuable content through the Darwinian nature of competitive
markets. Said more simply, some ‘rules of thumb’ work not because one knows why
they do but because, if they are not used, one goes out of business. Lending and
business risk is probably a rule like this. Out of a complete sense of responsibility,
however, we should let you know why we think the rule works.
Agency costs are probably the reason why lenders and borrowers are less attracted
to risky situations. If you think back to our discussions of these costs, you will see
that the presence of uncertainty in the operations of a company would cause these
to be more heavily felt. (The difficulties in monitoring the actions of the firm, and
the odds of incurring distress costs are obvious examples.) So there seem in fact to
be good reasons why borrowing is less attractive for risky companies: risk is likely to
make agency costs higher.
hands on the assets of the company itself (in times of bankruptcy or severe distress).
In such times, the assets are unlikely to be generating their usual operating cash
flows, and may well be offered for sale in liquidation. Book (historic, depreciated)
values are here probably better indicators of lender expectations of return than are
market values (based upon operating cash flows).
The very nature of tangible assets as collateral for loans may also play a part in
the reliance of finance practitioners upon book values. Consider, for example, the
differences, from a lender’s perspective, between the market values of a computer
software company and a gold mine. The software company is likely to have most of
its value being generated by the future cash flows from its new products across time,
whereas the gold mine’s value is based upon the worth of the gold in the ground.
Now think of the decisions faced by the companies and their creditors as to the best
amount of borrowing for each.
As the software company and its creditors consider the potential for agency
problems in the debt of the company, they recognise that in times of financial
distress, it is not at all unlikely that there would be adverse operational effects upon
the capacity of the company to produce positive cash flows (e.g. concern on the part
of skilled staff as to their future, unwillingness of suppliers or government to
provide services on credit, management time spent on dealing with financial
problems instead of software development, and hence forgone new products).
When such difficulties appear, it is highly probable that the existence of debt itself
will cause a decline in the value of the company on top of the deleterious event that
precipitated the financial distress. The company and its potential creditors must
judge the importance of these value declines in deciding upon optimal company
capital structure. Because these effects are likely to be significant, the decision is to
issue little or no debt.
The gold mine, on the other hand, is unlikely to suffer such distress costs, be-
cause its value depends upon an immutable, tangible asset. When the company finds
itself upon economic hard times, the existence of borrowing in its capital structure
presents few of the same problems experienced by the software company. In times
of low gold prices, the value of the gold mine is about the same regardless of
whether it is equity-financed or (perhaps in bankruptcy) debt-financed.
For the purposes of this discussion, the primary difference between the gold
mine and the software company is in the degree of tangibility of their assets (both
are quite ‘risky’ in the sense of producing rather variable cash flows). Since tangible
assets tend to be those that appear on the balance sheets of companies, we begin to
see why book value ratios seem to be so important in a company’s capital structure
decision. Book values are a proxy for the extent to which a company’s market value
is based upon tangible assets. And tangible assets present fewer problems than do
intangible ones in the presence of borrowing. So it is after all not so surprising that
book value targets and comparisons play such a big role in company borrowing
decisions. This is not because anyone really thinks that book values correctly reflect
the true economic value of the enterprise, but simply that book values are a pretty
good measure of the extent to which values will not be upset by financial distress
write-offs, which can be foreseen due to depletion of their holdings over time.
Simulation indicates that operating cash flows may at times not cover debt service,
but this does not trouble the financial managers of Midlands due to the liquidity and
tangibility of the companies’ resource holdings. What should the company do?
Midlands should probably not borrow. The reason is not so much that there will
be problems with covering the payments of interest and principal, or large distress
or asset liquidation costs unique to borrowing. The underlying assets of the compa-
nies are such that they are excellent candidates for debt collateral. The problem is
that there are few if any benefits that debt can be seen to provide. Since almost all
of borrowing’s attraction is from the perspective of minimising taxes, and since it is
unlikely that any taxes will be paid even if no debt is issued, there is little attraction
to debt.
After some years of rough times, Octopus Enterprises has become profitable by
streamlining management and ‘getting control’ of its far-flung and varied pursuits.
The company now wishes to change its capital structure so as to take advantage of
the tax benefits of borrowing. (Octopus foresees that if it does not do something, its
tax bill will increase dramatically in the near future.) The company’s expectations are
that it will continue to prosper, and there is little reason to think that it will not. It
intends to continue acquiring or starting businesses in which it thinks a good return
is available. It is willing to take risks to do so. Octopus’ simulations indicate that
there should be no serious problem in covering the interest and principal payments
on the amounts of borrowing it intends. In approaching potential lenders (either
through investment banking advice or insurance company ‘private placements’),
Octopus has discovered that its straight debentures will be asked to pay very high
interest rates – much higher than companies of comparable profitability and risk,
but whose lines of business are fewer. What should the company do?
Octopus should consider issuing a form of debt more complex than straightfor-
ward debentures. The relatively high interest rates that Octopus is being asked to
pay are probably caused by lenders’ natural concern that Octopus may alter itself to
be riskier during the life of the loan. If the total value of the company remained
unchanged, its debt values would decline and share values would increase. Creditors
have priced such contingencies into the interest they are charging. Octopus may be
able to reduce its total capital costs by issuing convertible debt securities that carry
the tax benefits of borrowing, but at the same time give lenders a guarantee that any
increases in equity values will be shared. By avoiding this ‘agency cost’, convertibles
should produce borrowing terms more favourable for Octopus.
Learning Summary
The characteristic of company capital structure decision making that creates the
most lasting impression upon finance students is the evolution from what seems at
its earliest introduction to be a very rigorous, quantitative process to what at its
practical application stage is almost exactly the opposite. Witness the capital
structure decision descriptions immediately above, which contain not a single
number! (There are, of course, numbers that would have appeared in the cash flow
and financial statement simulations, but these are simply the grist to be ground by
the decision mill.)
The reasons for the contrast between capital structure’s theoretical rigour and
practical application are a combination of the number and complexity of the
relationships involved (the economics of capital claim hierarchies, the influence of
company and personal taxes, the costs of resolving anticipatable conflicts of interest,
and so forth), and the relatively short period of time that financial thinkers have
devoted to formalising these considerations into real-market decision processes.
Today we know tremendously much more about company capital structure deci-
sions than we did when M&M wrote their groundbreaking work in 1958. But there
is probably just as much yet to be discovered about this most challenging of
financial decisions, particularly in the application of theory to actual decisions taken
by financial managers.
Review Questions
9.1 Companies are concerned about the capital structure decision because of which one of
the following?
A. It is often necessary to borrow money so as to be able to undertake invest-
ments that the company deems desirable.
B. The source of financing may affect the wealth of the capital suppliers of the
company.
C. Borrowing is usually cheaper than issuing equity.
D. Equity tends to produce lower-risk capital structures.
9.2 For any given company, the cash flows promised to debt suppliers are bound to be less
risky than the cash that shareholders expect. This implies that the required returns to
bondholders will always be lower than those of shareholders. However, the more debt
issued, the higher will be the riskiness of the company’s shares, and hence the higher the
required returns to shareholders. This situation implies which one of the following as
the best capital structure for a company?
A. The mix of borrowing and shares that produces the lowest risk to the share-
holders.
B. The mix of borrowing and shares that causes the value of equity to be maxim-
ised.
C. The mix of borrowing and shares that produces a reasonable exposure to risk.
D. None of the above.
9.4 Suppose that in Question 9.3 above there were many Leverage-type companies and only
one Leverageless. Do you think that the value situation described in Question 9.3 is
likely to persist? Why?
A. Yes, because there are probably people out there who would prefer to hold
shares from companies that have not borrowed, and that therefore have lower
risk.
B. No, because the rarity of ungeared firms would cause the value of Leverageless
to increase.
C. Yes, because the value situation currently exists, and there is no reason to
think that anything will happen to change it.
D. No, because the financial market will soon perceive that the shares of Lever-
ageless are too expensive, and their price will fall.
9.5 Your company is considering an investment whose characteristics indicate that it will
provide a 10 per cent rate of return (IRR). Your investment banker advises you that the
company can raise enough money to undertake the entire investment with a straight
debenture debt issue, the after-tax cost of which is 8 per cent. Does this sound like a
good deal?
I. Yes, because it provides a rate of return in excess of the cost of funds necessary to
finance it.
II. No, because nothing has been said about the riskiness of the project.
III. No, because 8 per cent is not the WACC of the investment.
Which of the following is correct?
A. I only.
B. II only.
C. III only.
D. Both II and III.
9.6 Negative considerations important to the company capital structure decision include:
I. The risk of bankruptcy of the company.
II. The chances that some tax benefits may be lost when the company deducts interest
from taxable profits.
III. The chance that the company will not be able to use its assets in the ways it would
have, had it not borrowed.
IV. The chance that shareholders will be forced to make good on the company’s
promised interest and principal payments.
V. Lenders requiring returns that include compensation for the chances that the
company will take actions in conflict with the lenders’ best interests.
Which of the following is correct:
A. I, II and III.
B. II, III and IV.
C. II, III and V.
D. III, IV and V.
9.7 Conglomerate companies (those having several different types of businesses under one
ownership) often issue complex securities in their borrowing. In particular, convertible
bonds seem to be popular financing vehicles for these companies. The reason why
conglomerates sometimes use convertibles is which of the following?
A. Convertible bonds carry lower interest rates than non-convertible bonds,
other considerations held aside.
B. Conglomerates would prefer to issue equity, but because of the high cost they
undertake to issue ‘delayed equity’ in the form of convertibles, which are
subsequently exchanged for shares.
C. Convertible bonds are generally of shorter maturity than non-convertibles.
D. Because convertible bonds diminish the fears of bondholders that the company
will take decisions to increase share value and reduce bond value.
9.8 Intercontinental Shipping Company tends to use borrowing heavily in its capital
structure, while Clever Advertising plc relies mostly upon equity capital. Assuming that
each is representative of the industry in which it operates, which of the following is the
primary reason for the difference in their choices of financing?
A. Much of Intercontinental Shipping’s value is in tangible assets, the operating
characteristics of which would not be much affected by financial distress. The
opposite is true for Clever.
B. Advertising companies are simply riskier than shipping companies, and thus are
capable of sustaining lower amounts of borrowing in their capital structures so
as to avoid bankruptcy in bad times.
C. Since capital structure does not matter to companies, the likely reason for the
difference is that random choices were made by managers, and there has been
no reason to change them.
D. The tax laws favour borrowing for companies such as shippers, which have
large investments in depreciable assets, whilst advertising companies have no
such tax incentive to use debt.
retained earnings. R-D Star expects that the facilities project will add £22 500 000 to its
2009 earnings before interest and taxes, which project to £160 000 000 without the
facilities project. The company pays taxes at a 52 per cent income tax rate.
The first alternative for raising the additional £75 000 000 not supplied by internal
funds is to issue shares to the public. R-D Star’s merchant bankers are of the opinion
that the company could sell shares to net £21 per share (there are currently 20 000 000
shares outstanding, selling at £21.77 per share).
The second alternative is to borrow the necessary cash. R-D Star could borrow
£75 000 000 at 14 per cent interest on a ten-year ‘sinking fund’ arrangement. (A sinking
fund means that the company would pay 1/10 of the principal amount each year, in
addition to interest on the balance outstanding during the year. Assume that the existing
£230 000 000 debt will be repaid on a ten-year sinking fund as well.) Lenders would also
require that R-D Star pay no more than 16 per cent of each year’s after-tax profit as
dividends, maintain net working capital at a minimum of £80 000 000, and issue no other
debt claims of equal or higher priority without the permission of the lenders. Without
actually specifying a number, the lenders have also indicated that a lower interest rate is
available if R-D Star is willing to put forward as collateral for the loan its extensive
library of vintage films, for which it holds international copyrights.
R-D Star’s finance department has provided the following historical data (in £m):
The department has also collected certain information about the industry in which R-
D Star operates:
Current assets/Current liabilities = 2.0
EBIT/Total assets = 0.26
Long‐term debt/Total assets = 0.35
Dividends/Profit after taxes = 0.08
1 Examine this information in the light of what you have learned in Module 9, and advise
R-D Star as to the best choice for financing its facilities project.
Hint: You should be aware that there is no fixed method or template that can be used
to solve a capital structure case of this type. Very often an analysis of a situation such as
presented here is a matter of weighing the various pros and cons of the alternative
financing plans in an unstructured format. Essentially the most important question to be
addressed is whether the company can foresee there being problems arising in its plans
if it chooses the debt financing alternative (assuming that there are probably tax benefits
to be obtained from that type of financing compared with equity financing). Though
there are important quantitative dimensions to these issues, many of them can be solved
almost by inspection (for example, you should not hesitate to use your own approxima-
tions, estimates and forecasts to the extent that they allow the analysis to be efficient
without misstating essential relationships). Reference back to the summary of capital
structure decision making at the end of the module can help to keep you on track to a
decent conclusion of the case. Remember that to think there is a given ‘solution’ to a
case such as this is misleading. A good outcome would be for you to uncover the most
important factors that would influence the decision, and to make an intelligent and
defensible recommendation based upon your appreciation of them.
Learning Objectives
A company’s ‘working capital’ is its short-term investments in cash, marketable
securities, inventories, receivables (debtors) and short-term financings, bank loans
and payables (creditors). Because of the short-term nature of these assets and
financings, an entirely different set of financial management techniques is used in
dealing with them compared with the techniques introduced in the course to this
point. The module begins by discussing why short- and long-term assets and
financings present different risk profiles, and makes the case for ‘maturity matching’
in such decisions. Next, the management of specific asset accounts, such as invest-
ments in cash balances, and receivables or collection policies, are discussed, with
examples of the techniques that companies use to make such decisions. Finally, the
module finishes with an application of working capital management bridging into
cash budgeting, or the array of the company’s detailed and specific expectations as
to what is going to happen to the company’s cash flows in the near term. This
allows the firm to make plans for short-term borrowing or adjustments in other
short-term asset and financing accounts.
10.1 Introduction
The course to this point has been somewhat misleading about the day-to-day
activities of financial managers. We have investigated major episodic financial
decisions in the form of capital budgeting, capital structure and financing decisions.
Making these decisions is a truly important function within the company, but there
is another set of actions regularly undertaken by financial managers that is as
important and often claims more of a typical financial manager’s time. This is the
management of the company’s working capital.
If working capital management is so important, why have we waited until now to
introduce the concepts and techniques of such management? A good question. The
answer has to do with the nature of the assets and financing involved, and the way
in which the company manages this set of concerns.
First we must define specifically what ‘working capital’ is. The most common
definition, and that which we shall adopt, is the set of balance sheet items that
would be included under Current assets and Current liabilities. These include the
assets of cash, marketable securities, accounts receivable (debtors), and inventories
(stocks); and the liabilities of accounts payable (creditors), short-term borrowings,
and other liabilities coming due in less than one year.
This definition makes clear that the distinguishing feature of working capital is
that it is concerned with the short term. In contrast, essentially all of the decisions
we have dealt with in the course up to this point have been, at least potentially, ‘long
term’ in nature. Capital budgeting, capital structure and financing-dividend decisions
deal with assets, product lines and capital sources that can be expected to persist for
at least some years. This is not the case with working capital items, whose very
nature is that they are much shorter-term assets and financings.
This is the first time we have introduced the notion that financial management
may be different for assets and financings that have different terms, or ‘lifetimes’,
and so the issue deserves some elucidation. We shall spend some effort in this
module discussing differences between short-term and long-term financial manage-
ment. As a matter of fact, for these introductory comments, the lifetime of a
decision is a very important attribute, and we shall use the word ‘term’ generally to
indicate the lifetime or duration of the asset or financing in question.
We must here be clear about another aspect of working capital. We have implied
above that managing these assets and financings is different from the finance you
have seen to this point. This is quite true, as we shall soon demonstrate. But keep in
mind that the basic economic principles of financial management will apply as well
to working capital management. That is, management decisions undertaken that
change the amount, timing or riskiness of cash flows to capital suppliers can have
effects upon the wealths of shareholders. Because working capital is short-term in
nature, we are less concerned with discounting far-in-the-future cash flows to
present value, but we are nevertheless as ever concerned with discovering the wealth
effects of decisions taken.
Before continuing with this module we urge you to turn to the appendix to the
module, which deals with financial and ratio analysis of companies, if you have not
encountered those procedures before. To continue with this module it is very
important that you have a good familiarity with that material. We have relegated it to
an appendix for the reasons that (1) many students have studied this or similar
material in other contexts and (2) it is not primarily financial decision oriented, but
is more in the nature of background material for such decisions.
about the differences in investing in short- versus long-term assets, and in using
short- versus long-term finance.
Suppose we were to ask you to categorise a company’s term-specified assets and
financings with respect to their risk and return characteristics. For example, are
investments in short-term assets riskier than long-term assets? How about their
respective return attributes? A complete mapping of the relative risk–return
characteristics of investments and financings of a typical company can be instructive
as to the appropriate mix of these. (If you are impatient to see the answer, you can
look ahead to Figure 10.1, which displays the final result of the discussion immedi-
ately below.)
1 Since we in finance are not limited by accounting conventions as to what can be included in a ‘balance
sheet’, we can mention that long-term assets of the latter type could include managerial expertise,
brand names, market power of a monopolistic nature, and other ‘assets’, all of which definitely have
value but may not appear in the formal accounting records of the firm.
would allow the monopolist to earn higher returns than a firm in competition. If we
extend that argument down to the level of individual assets, the kind of assets in
working capital (cash, marketable securities, accounts receivable and inventory) are
unlikely to be the ones in which a company has monopoly power. These assets are
common (in fact substitutable to a great extent) to all companies, and are thus
‘competitive’ and unlikely to carry high ‘monopolistic’ returns.
On the other hand, long-term assets (plant, equipment, managerial ability, tech-
nological expertise, brand names and so forth) are exactly the assets within which
the firm’s specific and unique ‘line of business’ attributes will lie. These ‘line of
business’ assets are those that can be unique to the firm and that carry its ability to
earn high rates of return. If there are higher rates of return to be earned, we would
thus expect these to be coming from long-term as opposed to short-term assets.
Thus the risk and return characteristic of a company’s asset investment is that
short-term assets exhibit relatively lower expected risk and return, and long-term
assets relatively higher risk and return for a firm.
The rates of return in financing either short- or long-term activities are best
understood by considering the costs of the financing types. (High costs produce low
rates of return, and vice versa.)
First we should understand that the costs of finance from this perspective have
nothing to do with the level of interest rates. This may be surprising, but the
explanation is straightforward. We must first remember that financing markets are
very competitive, so that if short- or long-term financing were to carry higher or
lower interest rates, that would likely reflect simply the costs to lenders of carrying
that type of loan. If a firm got lower interest rates on one type of borrowing, that
rate reflects simply the lower costs that the lender incurs from that kind of finance,
which is commensurate with rates generally available for that risk in financial
markets. Since this risk is something that the firm is unable to offer uniquely, there
would be no gain to the firm from the lower rates. In other words, the firm is
getting paid for exactly what it provides to the market. The fact is, however, that
there is no monolithic tendency for short-term rates to be higher or lower than
long-term rates.2 So interest rates are not the reason for return or cost differences
between short- and long-term finance.
What then causes the costs of short- and long-term finance to differ? Basically
the costs depend upon reversibility differences between the types of finance. When
a company undertakes a particular type of financing, it commits itself to service that
financing for its agreed term. Naturally, short-term sources of capital are of shorter
duration than longer-term ones. In the situations where the company finds itself
with unforeseen reductions in the need for financing, short-term finance is dis-
pensed with (‘reversed’) quickly at the end of its term. Long-term finance is not as
easily cancelled.3 So short-term finance is less costly than long-term finance, because
short-term sources are less costly to discontinue when unnecessary. Because lower
costs mean higher returns, short-term finance exhibits higher returns than long-
term.
Thus the risk and return of the characteristic of a company’s financing is that
short-term financings exhibit relatively higher expected risk and return, and long-
term financings are characterised by lower risk and return. This is exactly the
opposite of the risk return characteristic of its assets.
2 Actually, long-term rates of interest on the average have been higher than short-term rates.
Economists have argued why this is so, without any absolute resolution of the issue. Regardless of this
fact, competition in the financing industry would cause the level of interest rates on short- and long-
term financings to be irrelevant to the choice between them.
3 Long-term debt can be retired prior to its maturity, depending upon ‘call provisions’ in the loans, and
with likely early-retirement penalties. Equity can be repurchased by the firm, but this too carries costs if
forced in an unforeseen manner. Short-term finance has no such extra costs. This reversibility
characteristic is financing’s counterpart to asset liquidity.
ASSETS FINANCINGS
'Maturity matching'
long-term financing
Net lending Total assets
Current assets
0 Time
4 The exception to this may be the costs of a company holding marketable securities. We are assuming
here that the market sets prices of these efficiently enough so that the returns provided are commensu-
rate with the risks felt, and thus such investments must be expected to have no positive or negative
NPV. Why a ‘zero NPV’ characteristic of an investment is a ‘cost’ (given that our previous discussions
of NPV would indicate these investments to be at the point of indifference) may not be clear. Such
investments are probably costly in the sense that such funds could be paid to shareholders and may
thereby avoid some aspects of coming under taxation at both the company and shareholder levels.
anticipated and unanticipated outflows, but does not earn interest.5 It thus has a
‘capital cost’. Accounts receivable (debtors) allow the firm to offer credit to custom-
ers, thus increasing sales, but cause cash receipts to be delayed until the customer
pays, and sometimes create bad debts. Inventories are necessary so as to avoid
stoppages and inefficiencies in the production and selling process, but require that
funds be raised or retained to finance these stocks of materials. Further, each time
inventory is reordered there are record-keeping and other costs of transacting.
5 Some bank accounts may be interest-bearing and be essentially the same as cash.
banks meet reserve requirements). The costs of cash balances are the transactions
costs of switching between higher and lower interest-bearing securities and ac-
counts, and the differential interest rates earned.
Total cost
Total benefit
Total benefit and total cost (£)
Maximum
net benefit
Optimum usage
Maximum
Cash Cash
Minimum
0 Time
6 The formula, called the ‘economic order quantity’, was well known in inventory management and first
applied to cash balances by William Baumol, an economist at Princeton University. The derivation of
formula 10.1 is unnecessary for our purposes, as long as the trade-offs within it are appreciated.
7 This illustration is taken from Miller, M.H. and Orr, D. (1966) ‘A Model of the Demand of Money by
Firms’, Quarterly Journal of Economics, 80 (August), 413–35.
cash balance, and the colours represented in the barrel proportionally to the
probabilities in the distribution (the monkey is trained to replace each ball after it is
examined).
Look at Figure 10.6 – the solution to this cash management problem. As with the
original example, there is a minimum amount of cash (we shall call it £M) below
which the balance is not allowed to fall (due to precautionary or compensating
balance reasons), and this is a ‘lower bound’. You can see that there are two other
levels also indicated in Figure 10.6, namely £U (the ‘upper bound’) and £R (a ‘return
point’).
Cash balance £
0 Time
£U £U
£M £M
0 Time
£ 3 £ /4 £ ﴾10.2﴿
chooses such a return point £R, where £T is again the cost per transaction and i the
interest differential, and here s2 is the variance of the changes in the cash balances,
from the probability distribution mentioned earlier. (As with the prior formula, it is
unnecessary for our purposes to derive the proof herein.) Specifically, if the odds are
as great for an increase as for a decrease in cash balance, and if the amount of
increase or decrease in cash balance is expected, by the probability distribution, to
be £c for each of the number of times t cash balance can change per day,
s2 £c2 t.
A numerical example will help to clarify the point. Suppose that ZYX plc incurs a
transaction cost of £T = £100 per instance of shifting between cash and interest-
bearing securities, has an expected cash-balance change of £c = £4000 per hour,
experiences an interest differential of 10 per cent (per year, which is equivalent to
0.10/365 per day), and has a minimum cash balance of £M of £250 000. Assuming
an eight-hour working day, s2 £4000 2 8 £128 000 000, and:
£ 3 £ /4 £
3 £100 £128 000 000 / 4 0.10/365 £250 000
£282 723.12
Thus if a £250 000 minimum balance is required, when the lower limit is reached,
£32 723.12 of encashing of interest-bearing securities is cost-minimising.
What about reaching the upper limit? As it turns out, the upper limit is part of
the solution itself, and is
£ £ 3 £ £ ﴾10.3﴿
In this example, £U £250 000 3 £32 723.12 , or £348 169.36. ZYX should,
therefore, when its cash balance reaches £348 169.36, purchase 2 £R‐£M or
£65 446.24 of interest-bearing securities (which returns the company to £ R), or,
when its cash balance reaches £250 000, sell £32 723.12 of such securities (also
returning the company’s cash balance to £R). Following the above process will allow
the company to minimise its costs of carrying cash (or deposits with lower interest
rates than other liquid securities) while maintaining the cash balances at the levels
required for running its business lines.
One note of caution should be voiced about the cash-balance decisions described
above. Of all the financial decisions that companies make, the cash-management
decision is the one that has been most heavily influenced by technological change –
and that continues to be so. Eventually, electronic transfer mechanisms for transac-
tions could relegate much of the type of analysis above of little practical significance
(when transfers into and out of interest-earning securities of high liquidity can be
made instantaneously, automatically and at little or no cost).
The two techniques of managing cash balances mentioned above are simply a
sampling of the types of optimisation processes that managers can bring to bear.
There are many others, some more and some less sophisticated than these. As was
mentioned in the introduction to these methods, however, all good techniques will
make an explicit balancing of the benefits and detriments of investing in the asset in
question.
analysis must attempt to trade off between these effects so as to maximise the NPV
of the receivables investment.
Unlike the cash management situation illustrated above, there exist no received
comprehensive techniques in accounts receivable management that promise
simultaneously to combine all relevant costs and benefits. Why this is so is a
function of both the complexity of the decision and the lack of sustained attention
that finance researchers have focused upon this problem. There are, however,
various approaches to solving the problem partially.
One issue that is constantly raised about the investment in receivables is the
deterioration in the quality of customer credit accompanying an increase in the
amounts owed to the company. This has led to methods of discerning customer
creditworthiness. There are many credit-reporting agencies that supply such
information to companies (at some cost), and a company’s own records of customer
payment histories can yield useful information about the likelihood of a customer’s
paying. Further, the credit-granting decision has been raised to a fine art by banks
and other institutions in that line of business. Some companies have found it
beneficial to perform sophisticated statistical analyses, using methods such as
discriminant analysis to examine various customer attributes and thereby make
predictions as to creditworthiness.
Such studies always produce an improvement in the capability of a company to
filter out bad customers. However, the astute financial manager recognises that at
some point rejecting the marginal customer ceases to be worthwhile. That is, the
point at which such efforts should be stopped is when the incremental expenditure
for search and evaluation exceeds the expected gain from discriminating among
good and bad customers. This point will eventually be reached in all such analyses.
Suppose, for example, that a credit-reporting company is willing to offer credit
analyses on prospective customers for £25 apiece. Further, suppose that the odds of
getting a bad customer (figured, perhaps, from historical records) is 5 per cent, that
the average sale generates revenues of £500 and costs of £400, netting £100 profit,
and that the company expects to have 100 potential customers during the period in
question. If the company does not review customers’ creditworthiness and simply
accepts everyone, the expected profit from each sale is given as follows:
Expected profit Number of good customers Profit per customer
Number of bad customers Loss per customer
95 £100 5 £400
£7500
The loss as a result of a bad customer is the company incurring £400 of costs
without any revenues. With 100 customers, the company’s profits are expected to be
£7500 during the period.
If, however, a credit analysis is undertaken on customers at a cost of £25 apiece
(and assuming it is always correct in discerning good from bad), the expected profit
per customer is:
8 See Hill, N.C., Emery, G.W. and Satoris, W.L. (1985) Essentials of Cash Management: A Study Guide,
National Cash Management Association.
9 The entries into this calculation are essentially self-explanatory; you can see the appropriate influences
of bad debts, the number of days of collectibles lengthening, and unit sales effects simply by compar-
ing the numbers in the calculation with those in the text’s explanation of CFS’s situation. The only part
that might be confusing is the working capital adjustment which appears in the last two lines of the
calculation. In it, the change in sales affects the working capital instantaneously (second to last line),
while the eventual recouping of the working capital is recognised in the last line of the calculation.
10 The use of ‘profits’ rather than cash-flow measures in this example is not necessarily inconsistent with
earlier arguments against accounting numbers, given the short-term nature of the investments.
One of the important sources of short-term financing that companies use regu-
larly is credit extended by vendors. This is essentially the mirror-image of a firm’s
own accounts receivable: one company’s receivable is another’s payable.
When a firm takes advantage of credit extended by a vendor, there is usually a set
of payment conditions associated with this ‘trade credit’ as it is called. These
payment conditions almost always include a time when final payment is due, but
also a (shorter) time during which payment would produce a discount from the
market price of what has been bought. In other words, the price quoted implicitly
assumes that payment will not be made until the final due date. (One could argue
that it thereby includes a capital cost of the vendor’s financing of the purchase price
until receipt of the cash from the purchaser.) If payment is made by a particular
(earlier) date, the purchaser gets a ‘discount’ (though an economist might argue that
‘discount’ to be simply a recognition that the vendor has not incurred the capital
cost mentioned above).
Usually these payment terms are described by a phrase such as ‘2/10 net 30’
which signifies that there is a 2 per cent discount which can be taken for payment
within ten days of invoicing, that payment beyond that date is at full market price
(no discount), and that full payment is expected within 30 days. Why is it important
that the process of managing short-term financings deal with this condition of terms
of sale?
Consider the decision as to whether the discount in the above example should be
taken by the firm, that is whether payment should be made on the tenth day or the
thirtieth day.11 If payment is made, the company obtains a 2 per cent price reduc-
tion. Is this good or bad? By what standard should this be judged?
The proper standard is the cost of financing the money that would be used to pay
early, or the interest rate on such short-term borrowing. In essence, by not taking
the discount the company is ‘financing’ an extra twenty days of credit from its
vendor, and by taking the discount the company is financing the twenty days of
credit by borrowing elsewhere for that period. Suppose that the company could
borrow at a short-term rate of 12 per cent so as to take the discount. Should they do
so in this example?
The answer is clearly ‘yes’. Though to the uninitiated a 12 per cent cost may seem
higher than a 2 per cent discount, we must remember that the 2 per cent discount
would be obtained by paying twenty days early whereas the 12 per cent interest cost
is stated on an annual (365 day) basis. The annual equivalent interest cost of a 2 per
cent interest rate for 20 days can be found by the intimidating:
/
0.02
1 1 44.6%
1 0.02
This formula probably looks a bit strange. Some of it is intuitive; Module 1’s logic
would seem to imply that annualising a 2% interest rate for twenty days gives us a
11 Naturally, payment would not be made prior to the tenth if the discount is to be taken, nor prior to the
thirtieth if not, for the obvious reason of money’s time value. Remember also that one transaction may
not be very important in absolute value, but since it is a policy being developed, the cumulative effects
of choosing the wrong policy can be quite large.
Note that row 6 indicates the amounts of cash that CSH expects to take in each
quarter from selling its product. These amounts, along with ‘other’ sources of cash
inflow (not including borrowing or capital infusions, but perhaps tax refunds or
asset sales), are combined to produce the total operating cash inflow on line 11.
Next, CSH estimates its outflows (note that payment of payables is in cash terms;
you can assume that an analysis such as lines 1 to 7 has been performed to generate
these cash-flow estimates). Each of the entries in lines 13 to 18 must be the actual
cash payment expectation and not an accrual number from accounting statements.12
Total cash outflows appear in row 19, with the net of operating inflows and
outflows appearing in row 20.
Finally, rows 21 to 25 deal with the company’s cash situation, and potential short-
falls or surpluses of cash. Row 21 shows the beginning cash balance, which is
combined with row 22, the quarter’s net operating cash flow, to generate row 23,
the end-of-period estimate that would appear if no other action was taken. Row 24
12 For example, companies often report taxes on income statements as accruals of tax liabilities rather
than actual cash taxes that are expected to be paid.
is the minimum cash balance with which CSH is comfortable, and subtracting that
from row 23 indicates in row 25 any need for additional financing. If row 25 is
negative there is a potential cash surplus, which could be invested, or (as in Quarter
2) if row 25 is positive the company must plan to cover the shortfall.
There are several methods by which a shortfall of cash can be covered. It may be
best to plan on some additional short-term borrowing in the second quarter. (You
can see that it could be paid down by the fourth quarter, given the excess cash
amounts shown in the third and fourth quarters.) Or the company might choose to
lengthen its payables, or sell (‘factor’) some receivables, or sell other assets (perhaps
marketable securities). Given that the source of cash shortfall is a large capital-asset
outlay, the company may wish to consider a long-term borrowing or even an equity
issuance.
Which is the best tactic to cover the cash shortfall? This can be decided with the
aid of the techniques introduced in this module, and perhaps with material in the
capital structure module and ratio analysis that you have studied already. But the
importance of a good cash budget is clear: without some detailed knowledge of
when and how much cash the company needs or has in excess, there could be at
best an embarrassing last-minute plea to the bank (not a confidence-producing
exercise), or (worse) other detrimental effects to the business. Excess cash, while
not as immediately pathological, is destined to be a low-earning asset.
Most companies have found it useful to have an on-line computer-based cash-
budgeting process, which is updated daily or even hourly. The advent of relatively
inexpensive computer hardware and ‘off-the-shelf’ software for this purpose
removes any excuse for ignoring such an important dimension of company financial
operations.
Learning Summary
In this module we have surveyed the financial management of companies’ working
capital, its short-term assets and its financings.
Short-term assets and financings are in a sense no different from the longer-term
phenomena that we have studied in other modules. Companies can consider the
optimal amounts and types of such assets and financings from the perspective of
their effects upon shareholders. These short-term decisions are in the abstract no
different from any others. However, the best methods of dealing with these
decisions are unique. Working capital is best considered as being ‘managed’ in an
ongoing process rather than being decided upon in discrete terms.
We discovered that the rule of thumb urging maturity matching of assets and
financings was a good way to see some of the important differences between short-
and long-term commitments that a company makes in its asset portfolios and
financings, and in particular the risk and return differences among them.
Most importantly, we found that working capital management involves two levels
of activity:
1. the ‘hands-on’ application of management techniques to specific asset and
financing decisions (e.g. how much cash to have on hand at any one time, what
credit conditions to set for customers to buy on credit, or whether or not a dis-
count should be taken by paying a supplier before a bill is finally due); and
2. the optimal setting of policies for such decisions, so that each of these small
decisions is almost automatically determined by the company’s well considered
policy.
Finally, working capital management cannot be considered independent of a
company’s cash budget. Without a plan as to the generation and usage of cash
across time, the best management techniques have no data upon which to operate.
An up-to-date or ‘real time’ cash budget is a necessity for effective working capital
management.
companies within the same industrial sector. A common benchmark used for the
purpose of comparison is the data produced by the various ‘clearing houses’ for
industrial statistics. Some of these organisations are operated for profit in the
private sector, while others are run by government departments. Typically, a
company like Trossachs Metals would contribute its own financial and produc-
tion statistics to the organisation of inter-firm comparisons, on a strictly
confidential basis, and receive in return the computed average percentages and
ratios for key performance indicators taken from the information supplied by
many of Trossachs’ competitors in the metals sector. These percentages and
ratios allow Trossachs to gauge its own performance against the industrial sector
as a whole. Unless Trossachs’ operations by themselves dominate the industrial
sector, the average figures (which include Trossachs’ statistics) will be a reasona-
ble guide to Trossachs’ management as to how their company has performed
over the period covered by the industrial averages (normally half-yearly). Note,
too, that most organisations for inter-firm comparisons supply upper- and lower-
quartile figures in each key area; this allows individual companies to position
themselves more accurately than is possible with the overall average percentage
or ratio.
3. A word of warning about ratios: they can be used to suppress poor absolute
figures. For example, a company could report, say, a 300 per cent growth in
turnover, which gives the impression of success and progress. However, the
basis for such a calculation could be a sales figure of £100 when all similarly
positioned competitors are enjoying sales of £1000. In this situation ratios dis-
guise the real picture.
4. Just as no two sets of company financial statements are the same in style and
layout, so too one finds wide diversity in approaches to financial analysis. Ana-
lysts do not argue over their individual definitions of, say, ‘profit’ or ‘capital
employed’ or ‘debt’; they recognise that agreement would be impossible because
no two companies define these items in the same way. They do agree, however,
that they must apply their definitions consistently for the same company for
many accounting periods; otherwise an inter-period comparison would be invali-
dated. The organisations that publish inter-firm comparisons usually define in
detail the key headings to allow subscribing companies to adjust, where neces-
sary, their own figures and percentages.
A financial ratio is a relationship between two quantities, on a company’s set of
financial statements, that is derived by dividing one quantity by the other. The
purpose of using ratios is to reduce the amount of data to workable form and to
make it more meaningful. This goal is defeated if too many ratios are calculated –
for there are hundreds of ratios that could be calculated. The manager must learn
which combination of ratios is most appropriate in a specific situation, and must
remember that ratios seldom provide conclusive answers but rather lead the
manager to ask the right questions and (sometimes) give clues as to possible areas of
strength or weakness.
For the purposes of this appendix the following groups of ratios will be explained
and discussed using the financial statements of Trossachs Metals provided near the
end of this appendix (see Section 10.5.8).
1. Liquidity ratios. These are designed to measure a company’s ability to meet its
maturing short-term obligations.
2. Profitability ratios. These are designed to measure management’s overall
effectiveness: does the company control expenses and earn a reasonable return
on funds committed?
3. Capital structure ratios. These ratios are divided into two groups:
(a) those that examine the asset structure of the company, and
(b) those that analyse the financing arrangements of the company’s total assets,
in particular the extent to which the company relies on debt. This group of
ratios is generally known as the gearing ratios.
4. Efficiency ratios. These give an indication of how effectively a company has
been managing its assets.
Note: In ratio analysis, it is important to study the trend of the ratios calculated
rather than attempting to arrive at sound conclusions based on one accounting
period’s ratios. Readers should calculate Trossachs’ 19X1 ratios for themselves on
the basis of the information given in Section 10.5.8 and check them against the
answers given in the text below.
tically low figure? Or is it because they have less investment in fixed assets and more
in inventory to give them a trading advantage?
Sales are set at 100 per cent and each item is calculated as a percentage of sales. It
can be seen that Company EFG’s percentage savings in cost of sales in 19X2 was
lost in the increases in selling expenses and general and administrative expenses. If
EFG’s management can tighten the controls in this area of spending, then they will
enjoy a percentage increase in net profits, something that has escaped them this
year.
At the conclusion of the sections setting out the liquidity and profitability ratios it
would be helpful to summarise the findings on Trossachs Metals. The liquidity
ratios indicate a possible liquidity problem while the profitability ratios indicate a
healthy profit situation. Thus it can be concluded that Trossachs’ questionable cash
position, as revealed in this ratio analysis, is not due to lack of profits. Further
analysis may reveal the problem, if indeed Trossachs has a real problem of liquidity
at all.
The impact of these structures on the return on owners’ equity can be seen if
three profit-before-interest figures are compared.
The fall in equity earnings is less dramatic under the low-geared structure (from
10 per cent to 3.75 per cent) than it is under the high geared structure (10 per cent
to nil). Conversely, when earnings rise, the equityholders in the highly geared
structure enjoy a higher return (20 per cent compared with 16.25 per cent).
1. Sales occur over the entire year, whereas the inventory figure is taken from the
closing balance sheet. It could be that Trossachs’ pattern of trade dictates a
stocking-up at the end of the year; some adjustment should be made to this in-
ventory figure if sufficient information in that regard is available. However, if
this seasonal pattern is universal in the metals industry, no adjustment is required
in order to compare Trossachs’ stocking level with its competitors.
2. Different industries have significantly different inventory turnovers. Metals and
engineering are slow because of the length of the production process; retail
shops are very rapid, indicating that they need low inventories to service their
sales levels.
poor return. But, again, it must be stressed that the chart will only point managers in
the direction of potential problem areas – charts do not solve the problems them-
selves.
Profit
Total investment
100 = 12.5%
800
Sales Cost of sales Sales 1000 = 2 times Sales 1000 = 3.3 times
1000 900 Fixed assets 500 Net current assets 300
Source of Source of
Revenue 1 Revenue 2
Sales Sales
600 = 60% 400 = 40%
1000 1000
For this purpose cost of production excludes selling, distribution and admin-
istrative costs. If the cost of any category of stock, calculated in this way,
exceeds its net realistic value, it is written down to the lower figure. The
major exception to these principles occurs in the case of one subsidiary,
which carries a base stock of 1600 tonnes of copper that, at 31 December
19X2, was stocked at £950 000 (19X1 £590 000) below the lower of cost
and net realisable value.
The main categories of stock are as follows:
19X2 19X1
Refined and wrought copper and other £72 192 000 £65 445 000
metals
Fabricated products and components 83 974 000 91 444 000
Thousands Thousands
of £s of £s
Notes 19X2 19X1
Turnover 630 906 610 609
Cost of sales 496 247 476 044
Gross Profit 134 659 134 565
Distribution expenses 59 198 56 912
Thousands Thousands
of £s of £s
Administrative expenses 47 221 43 117
106 419 100 029
Profit on ordinary activities
before taxation 1 28 240 34 536
Tax on profit on ordinary activi- 5 693 3 262
ties
Profit on ordinary activities after taxation 22 547 31 274
Minority interests 765 554
21 782 30 720
Profit on metal stocks after 1 647 1 935
taxation
Profit before extraordinary items 22 429 32 655
Extraordinary items after taxation 2 1 063 (3 289)
Profit attributable to members of
Trossachs Metals plc 23 492 29 366
Dividends 9 387 9 169
Retained profit for year 3 14 105 20 197
Earnings per share
excluding extraordinary items 10.8p 15.7p
including extraordinary items 11.3p 14.1p
Movement on Reserves
At 31 December 19X1 (19X0) 168 213 148 016
Retained earnings for year 14 105 20 197
Amounts taken direct to Reserves (3 034) –
At 31 December 19X2 (19X1) 179 284 168 213
Abridged Notes on Consolidated Profit and Loss Account
1. Profit before Taxation
Profit before taxation excludes the effect of changes in the book value of
unsold refined and wrought metal stocks due to fluctuations in the price of
copper: in 19X2 there was a metal profit, after adjustment for taxation, of
£647 000 (19X1 £1 935 000). These items are separately brought into the
profit and loss account in arriving at the earnings applicable to shareholders
in Trossachs Metals plc.
Profit before taxation is after charging £2 672 000 for redundancy and reor-
ganisation costs. In addition, the following have been (charged)/credited in
arriving at profit before taxation.
19X2 19X1
£000s £000s
Share of profits, less losses of major
associated companies 725 1 925
Other investment income 4 992 3 185
Interest payable (14 924) (11 158)
Depreciation (12 450) (11 100)
Audit fees and expens- – UK (459) (406)
es
– Overseas (301) (259)
Directors’ emoluments (360) (290)
Hire of plant and machinery (2 743) (2 844)
2. Extraordinary Items
19X2 19X1
£000s £000s
Decrease in value of net fixed assets of overseas
subsidiaries due to changes in exchange rates – (1 842)
Profit on sale of investments 1 052 –
Other 11 (1 447)
1 063 (3 289)
3. Retained Earnings
19X2 19X1
£000s £000s
By company 4 500 18 900
By subsidiaries 8 750 2 703
By major associates 855 (1 406)
14 105 20 197
Thousands of £s Thousands of £s
19X1 19X2 Notes 19X2 19X1
Capital and Reserves
52 098 52 149 Called up share capital 52 149 52 098
123 579 128 711 Reserves 5 179 284 168 213
175 677 180 860 231 433 220 311
Abridged Notes on Balance Sheets
1. Intangible assets – goodwill
£000s
At 31 December 19X1 31 120
Arising on acquisition of minority interests in subsidiaries 204
At 31 December 19X2 31 324
Goodwill, which is shown at cost less amounts written off, is almost wholly
the excess of the purchase consideration paid for the acquisition of subsidiar-
ies over their net tangible assets at the date of acquisition.
2. Fixed Assets
Notes:
i. The gross book value of land and buildings and plant is made up as fol-
lows:
19X2 19X1
Land and Plant Land and Plant
buildings buildings
£000s £000s £000s £000s
At cost 31 803 149 561 30 016 136 273
Revaluations by certain subsidiaries
principally in 19X0 12 984 12 729 13 707 14 521
44 787 162 290 43 723 150 794
19X2 19X1
£000s £000s
Freehold 38 177 37 475
Long leasehold 5 713 5 344
Short leasehold 897 904
44 787 43 723
Company
19X2 19X1
£000s £000s
Shares at cost 102 292 102 795
Amounts owed by subsidiaries 174 091 145 882
Amounts owed to subsidiaries (70 325) (64 085)
206 058 184 592
4. Current Liabilities
5. Reserves
Company Group
Trossachs Major Total
and Associated Group
Subsidiaries Companies
£000s £000s £000s £000s
At 31 December 19X1 123 579 151 326 16 887 168 213
Retained profit for year 4 451 13 847 258 14 105
Amounts taken direct to reserves:
Share premium 67 67 – 67
Adjustment for exchange movements 614 (1 653) (2 310) (3 963)
Surplus of net assets over cost
of acquiring minorities – 862 – 862
Transfers between subsidiaries
and associates – 1 276 (1 176) –
Company Group
Trossachs Major Total
and Associated Group
Subsidiaries Companies
£000s £000s £000s £000s
At 31 December 19X2 128 711 165 725 13 559 179 284
Review Questions
10.1 ‘Maturity matching’ of asset and financing is an accepted rule of thumb in financial
management. Which of the following best explains the rationale for maturity matching?
A. Maturity matching ensures that the risks and returns of assets and financings
are the same; that is, high risk and return assets are matched with high risk and
return financings and vice versa.
B. Maturity matching generates the highest possible rate return for the amount of
risk undertaken.
C. Maturity matching is a rather crude, but often effective, rule for avoiding
mismatching of risks and returns in assets and financings.
D. Maturity matching associates low risk/return assets with high risk/return
financings and high risk/return assets with low risk/return financings so as to
generate reasonable risk and return levels.
10.2 The Holiday Toys Company is a very seasonal business. It expects to require levels of
fixed and current assets consistent with its business line. The company wishes to
understand the implications of various financing plans available, particularly with respect
to financing as to which is long-term or short-term. Referring to Figure 10.2 and the
related text of Module 10, Holiday Toys can choose to:
I. match assets and financings on the basis of accounting definitions of asset maturity;
II. match assets and financings on the basis of actual asset maturity (the notion of
permanent current assets); or
III. finance as much as possible with long-term finance.
In terms of the risk exposure caused by Holiday Toys’ choices of maturity financing,
which of the following is correct?
A. Strategy I will cause a medium risk exposure, III a low risk and II the highest
risk exposure.
B. Strategy I will cause the highest risk exposure, II the next highest, and III the
least.
C. Strategy I will cause the lowest risk exposure, with II the next lowest, and III
the highest.
D. All strategies will produce the same risk.
10.3 Journeys plc is a travel agent with a need to borrow funds from its bank. The bank
proposes that the one-year loan of £60 000 to be granted should carry a 14 per cent
nominal interest rate and a 10 per cent compensating balance (i.e. Journeys plc must
keep £6000 on deposit at no interest earned for the term of the loan), and that interest
be paid in advance. Which of the following is the effective interest rate of the loan?
A. 18.4%.
B. 15.6%.
C. 14.0%.
D. 16.6%.
10.4 Tookover plc is in the acquisitions business. It expects to use £4 000 000 cash per
annum, evenly spread across the year. To get the cash, the company wishes to use
marketable securities in its possession, upon which it earns 13 per cent per annum. The
cost per transaction is fixed at £42. The approximate combinations of average cash
balance (in excess of the minimum), optimal transaction size and number of transfers
per year are which of the following?
A. £25 420; £50 840; 79.
B. £2540; £5080; 787.
C. £141 000; £141 000; 28.
D. None of the above.
10.5 Up ’n Down plc, a trampoline company, has cash inflows and outflows across time that
generate changes in its cash balances that are as likely to be up as down. The expected
change in either direction is £4000 on any day. Up ’n Down can earn 12 per cent per
annum on marketable securities but nothing on cash, the company incurs a £15 charge
whenever securities are cashed or whenever cash is put back into securities, and it
would be uncomfortable with less than £50 000 cash at any time. Up ’n Down should of
course sell securities when its cash balance falls to £50 000. Which of the following state
the correct approximate amount of securities that it should sell, the upper limit of its
cash balance, and the ‘return point’ or balance resulting immediately after a sale or
purchase of securities?
A. £72; £50 216; £50 072.
B. £25 000; £125 000; £75 000.
C. £8180; £74 540; £58 180.
D. None of the above.
10.6 Henry James, the new financial manager of Daisy Publications, has discovered that
Daisy’s credit terms are ‘net 60’ while its competitors’ are ‘net 30’. James wishes to
consider the effect of coming into line with the industry’s credit terms. Daisy’s market-
ing department has informed James that the new terms would result in a decline in sales
of £75 000 per annum from a base of £650 000, but a reduction in the average collection
period from 65 to 40 days, along with a reduction in bad debts from 4 per cent to 3 per
cent of sales. Daisy’s variable costs are 80 per cent of sales, working capital stays at
about 20 per cent of sales, and Daisy’s interest rate for borrowing is 0.03 per cent per
day. Should Daisy undertake the alteration in credit terms, the NPV of the change
would be which of the following?
A. £0.78
B. £410.18
C. −£0.78
D. None of the above.
10.7 A recent wet summer has caused a decline in Henley’s plc due to the drop-off in sales of
racing sculls. The company, in order to have enough cash to preserve its operations, is
considering two alternatives:
I. give up the discounts on its payables, which are now 2/10 net 30 and which are all
taken on the tenth day, and instead pay on the thirtieth; or
II. borrow the necessary funds from the bank with a ‘discounted’ monthly note – which
means interest in advance – at 16 per cent annual interest.
Henley’s should undertake which of the following?
A. Take the loan because it is cheaper than the new payables policy.
B. Pursue the new payables policy because it is cheaper than the loan.
C. Take the loan even if it is more expensive because of the decline in the
company’s reputation if payables are changed as indicated.
D. Pursue the new payables policy even if the loan is cheaper because a company
in this kind of trouble should not be borrowing money.
10.8 As an exercise, program the data and relationships from the CSH plc cash budget into a
spreadsheet program on a microcomputer to which you have access. Any decent
spreadsheet program will suffice since the problem is straightforward. We suggest that
it would be useful to:
i. Examine the effect upon CSH’s cash budget of its customers changing the way they
pay – say lengthening their payment so that only 75 per cent of this period’s receiva-
bles are paid in cash.
ii. Examine the effect upon CSH’s cash budget of there being a 5 per cent bad-debt
proportion of sales.
iii. Construct within the CSH spreadsheet cash budget a payable cash estimation
process similar to the one for receivables, and test the effects of CSH’s changing its
payable’s policy (whichever one you have invented so as to generate the process).
iv. Include in your spreadsheet (if the program is sophisticated enough) a ‘macro’ which
calculates the equivalent annual interest rate associated with various discounts al-
lowed to customers or taken by CSH.
Once you have constructed the spreadsheet, you will probably wish to do more than
the above, but they are a good start.
KY Ltd Profit and Loss Account for year ended 31 December 20XX
£ £
Sales 5 781 250
Cost of sales
Opening stock 250 000
Purchases 5 975 000
6 225 000
Closing stock 1 600 000 4 625 000
GROSS PROFIT 1 156 250
Expenses 596 250
NET PROFIT before tax 560 000
Tax 200 000
NET PROFIT after tax £360 000
10.19 Using figures relating to the year ended 31 December 19X3 (not given here), the
current ratio of KY is calculated to be 1.3. The average ratio for the same period for
companies in the same industrial sector as KY was 1.6. Three views of this information
can be postulated:
I. KY is in a marginally healthier liquid position than the average of all the other
companies in the same industrial sector.
II. KY is in a marginally worse liquid position than the average of all the other compa-
nies in the same industrial sector.
III. KY is unable to pay its bills since current ratio is smaller than 2.
Which of these statements, if any, are true?
A. I only.
B. II only.
C. III only.
D. Not I or II or III.
10.20 The profit margin for 19X3 for KY was 6.8%. The average figure for other companies in
the same industrial sector for the same period of time was 7.0%. The management of
KY has the following options as courses of action:
I. Examine pricing structure and compare with competitors.
II. Examine costs and overheads.
III. Look for a new product.
Which courses of action could KY’s management be reasonably expected to pursue?
A. I and II only.
B. II and III only.
C. I and III only.
D. Not I or II or III.
10.21 The inventory turnover for KY for 19X3 was 4.8 times. The industrial average was 5.6
times. Three views of these facts can be postulated:
I. KY is possibly over-geared and may find it difficult to borrow additional funds.
II. KY has too much money tied up in stock.
III. KY should improve their credit control procedures.
What conclusions could the management of KY be reasonably expected to draw from
this?
A. I only.
B. II only.
C. III only.
D. Not I or II or III.
Learning Objectives
More and more companies are expanding their operating environments to be
international in scope. These international involvements range from simply export-
ing and importing goods and services, to engaging in operations in foreign
countries, even to raising capital in other countries or in international markets that
do not recognise national borders. Financial managers must be educated in at least
the basics of such markets and transactions. Module 11 begins with a discussion of
foreign exchange, the financial economics of exchange rates, purchasing power
parity, interest rate parity and the influence of inflation upon exchange rates. Next it
illustrates various techniques that are available to companies to hedge the risk of
exchange fluctuations in international transactions. Finally, the module discusses
real-asset investment in foreign countries, and the extent to which hedging exchange
risk (and other international risks such as confiscation) is possible or desirable.
Several exercises in dealing with foreign exchange transactions will help the student
understand this area, which has been the least well appreciated of company financial
decisions.
11.1 Introduction
Companies today are steadily increasing the amounts of business they do with
customers and suppliers in other countries. Sometimes this activity is in the import
and export market, that is with a company operating from a domestic base and
selling to or buying from customers or suppliers in other countries. Ever more
often, however, companies are also choosing to have operations (production
facilities, marketing units, etc.) actually residing in other countries (these are the
ubiquitous ‘multinational firms’). Companies participating internationally in either of
these forms face unique financial decisions not encountered by purely domestic
firms. This module will introduce you to the most important of these situations.
First a disclaimer. The study of international business is a very complex and
challenging subject, only one part of which is financially oriented. International
business as a subject area can include not only financial questions, but also issues of
government regulations, human capital concerns, trade barriers, political analyses,
international accounting differences, and many other topics. We shall deal herein
only in the financial dimensions of international business; the rest of international
business concerns must be left to other sources.
What do we mean by financial dimensions of international business? Consider
the situation of Lookout plc, a UK company selling fabric to customers in Japan.
Lookout has found that it sells more fabric to Japanese customers if it agrees to
prices stated in yen (¥). Those are good customers who pay in 30 days. Lookout,
however, pays its bills and services its capital claims in sterling (£). When Lookout
quotes a yen price to its Japanese customers, it can figure its equivalent sterling price
by examining the exchange rate between yen and sterling. An exchange rate is of
course simply the ratio of how many units of one currency are exchangeable into
another. For example, if Lookout discovered that £1 ¥243.06, fabric that Lookout
wished to price at £10 per metre would be quoted at ¥2430.60 per metre to
Japanese customers.
But Lookout faces a problem here. When it receives the yen in 30 days, there is
no guarantee that the exchange rate will still be 243.06 yen per pound sterling. If
Lookout does nothing about this uncertainty, the company will be adding a risk of
exchange rate fluctuations into the basic business and financial risks it would
undertake as a domestic company.1
The solution to problems such as Lookout’s is one of the important dimensions
of international financial management. We shall indicate how companies can deal
with the risk of exchange-rate fluctuations in this and other types of decisions they
make.
Another instance of a decision that is affected by exchange-rate fluctuations is
the undertaking of a foreign operation. Aside from the uncertainties residing in
the foreign commercial operation itself, since the shareholders of the company will
eventually wish to be rewarded in their own currency, and since the basic returns
will be generated in the foreign currency, there is potentially an exchange risk
problem here also.
In addition to exchange-rate risks there are other international financial concerns
with which managers must contend. These others are primarily involved in financ-
ing decisions associated with foreign operations, specifically in deciding the optimal
location, security and currency in which to borrow. International capital markets
are more varied and complex than purely domestic markets, and financial managers
must be aware of the opportunities and challenges of dealing in those markets.
Finally, there are governmental controls and subsidies existing in international
financial dealings, and these can importantly influence the profitability of transac-
1 If the exchange rate is greater than the rate upon which Lookout based its pricing decision (which is
not necessarily the rate existing on the day an order is taken), Lookout will end up getting less sterling
than it anticipated with the contractual number of yen; this is a loss on exchange. On the other hand, if
the exchange rate is lower than anticipated by the product pricing decision, Lookout would have an
exchange gain. Unless Lookout is a better predictor of exchange rate movements than the market, it
will inevitably face exchange risk.
tions and projects that companies undertake. Financial managers must be aware of
these also.
2 A similar but opposite set of activities on the part of buyers in London, who find things relatively
cheaper to purchase there, would produce the same tendency for the ($)/(£) exchange rate to increase,
or the (£)/($) exchange rate to fall.
3 If, for example, the dollar and sterling prices of diamonds remained the same, the exchange rate would
be forced to $1.70/£ or £0.58824/$. Since one minor commodity alone is unlikely to force exchange
rates to change much, the likelier effect is to lower the $ price or raise the £ price of diamonds. Which
would happen depends upon how the ¥ price of diamonds, the € (euro) price of diamonds, and so
forth compare with the $ and £ prices of diamonds. If the $ price of diamonds is more inconsistent
with all others, it will feel the greatest effect.
If diamonds are only one of many things that are mispriced in dollars relative to the £, the exchange
rate will change and domestic prices may not be as much affected.
Suppose the set of price quotations given in Table 11.1 were those published
nationally for 22 July 2003. The first set of exchange rates is called the spot rate,
which is the basis upon which sterling is exchanged into dollars on that date (or
actually for transactions done the day before). If Hotlady receives some dollars on
22 July 2003, they can be exchanged for 0.6266 as many pounds sterling. The
second set of exchange rates is called a forward rate. A forward exchange rate is the
going price for exchanging between currencies at some future time. In this case the
reported forward exchange rate is between sterling and dollars six months into the
future. By entering into a forward exchange contract, a trader commits to
purchase or sell an amount of currency at a fixed price at a fixed time in the future.
For example, Hotlady may on 22 July 2003 be negotiating a sale of mustard to a
customer in the United States, and will not receive payment in dollars until January
2004. To avoid any uncertainty as to the amount of sterling that those dollars will
buy, Hotlady can ‘buy sterling forward’ at the rate of £0.6337 per dollar. Thus,
regardless of the spot rate of exchange existing between sterling and dollars in
January 2004, Hotlady can guarantee itself an amount of sterling by transacting in
the foreign exchange market.
Obviously such transactions are desirable from the perspective of companies like
Hotlady (as they would be for other UK companies that might have future com-
4 Of course, one can use the forward exchange market to speculate in exchange rates also. If one
commits to purchase sterling with dollars but has no future dollar inflow expectation, one is speculat-
ing that the actual dollar price of sterling will be greater than that forecasted by the present future price
in the market. If the foreign exchange market is efficient, such speculation has a zero expected value,
but a positive variance (and thus risk). For companies like Lookout and Hotlady (which have
expectations of net future cash inflows in foreign currencies) not participating in the foreign exchange
market (or otherwise hedging the foreign cash flow) is foreign exchange speculation.
5 Forward discounts and premiums are defined by the relationship between spot and forward rates in a
pair of currencies. Since the forward dollar price of sterling ($1.5780) is lower than the spot price
($1.5960) there is a forward discount on sterling (often quoted as an annual percentage rate, here
approximately 2 1.5780‐1.5960 /1.5960 or 2.26 per cent) and a forward premium on dollars.
8 Suppose that the forward rate in the Hotlady example was $1.60/£ or £0.625/$. In that situation you
could borrow $1 000 000 at 6.5 per cent (dollar rate) for six months (producing a liability of $1 031 988
six months hence) and today exchange the proceeds of the loan into £626 000 (see the spot rate in
Table 11.1). If you invest £617 961 at 8.94 per cent (sterling rate) for six months it will produce
£644 992, which today can be sold forward at $1.60/£ to cover the $1 031 988 liability exactly.
Undertaking this transaction will be absolutely riskless because all future liabilities are exactly covered
by cash inflows in the same currency, and you can place £626 000 − £617 961 = £8 639 (the difference
between the sterling proceeds of your dollar loan and the cost of your covering sterling lending) into
your pocket and walk away. And you can continue doing this until the market learns its mistake.
What a nice way to make a living! But you should not count on finding such opportunities very often.
Remember that your £8 639 did not appear out of thin air. In order for you to have gained that
amount, someone else has lost it. Financial markets tend not to be populated by people or institutions
who regularly lose money on this type of transaction. Market prices will adjust instantaneously to such
a condition.
9 The simplifying assumption is essentially that inflation in each country is either known for certain or
that market participants are unconcerned about its uncertainty. Neither of these is true, but the
complexities caused by dealing with this rigorously would destroy the pedagogical benefits of any
discussion. More advanced texts will help the curious.
10 There are a few governments, the UK and US among them, that have issued bonds with guaranteed
purchasing power, but these are rare.
11 This is such a famous relationship that you should know who first stated it formally, namely an
economist by the name of Irving Fisher; and it has come to be known as the ‘Fisher Effect’. You may
recall that Module 6 discussed these concepts in another context.
12 Determinants of the levels and differences of real rates of interest across countries is well beyond our
scope and – to be honest – seems to be beyond the understanding of economists themselves! But if
you want to pursue what is known, see any recent international economics text.
13 Remember our assumption of equal real rates in the two countries. If real rates differ, this ratio
relationship will not hold.
relationship between spot and forward exchange in the currencies. This is the
international connection between capital markets, of which we spoke earlier.14
14 Those readers who are international economists may feel uneasy with the simplicity of this conclusion.
Recall, however, that we have, for the very purpose of simplicity, assumed away the complexities of
future spot exchange uncertainty, future inflation uncertainty and the behavioural complexities they
introduce.
One applicable caveat to our discussion is that forward exchange premiums may be influenced by
expectations of exchange restrictions (or, more rigorously, changes in exchange restrictions).
Fortunately, as this is written, such restrictions are not common; and so we would expect that their
expectation is not an important influence upon the forward exchange structure.
15 If a company does not hedge, it is by definition speculating upon future exchange rate movements.
Companies must always remember that to speculate successfully in foreign exchange, it is necessary to
predict correctly not only that rates will increase or decrease, but to predict the amounts and directions
of change better than the set of expected rates implied by the forward exchange structure.
16 Options are in fact much more costly than futures contracts on foreign exchange, and though there is
no single ‘seller’ in the sense of the textual explanation, the ‘market’ in effect performs the same
weighing of odds.
actually creates a position that insures against a bad turn in exchange rates but
retains the advantages of a beneficial movement in exchange. This is not a ‘hedge’ in
the sense that it is defined in financial markets. The option is a sort of ‘super hedge’,
but it is one whose financial benefits are likely to be exactly equal to their cost.
Unless there is some reason for a company to think it can predict future exchange
rates better than the foreign exchange options market, there is no reason for
exchange options to be used in transactions where simple hedging of known future
currency cash flows is desired. Forward exchange contracts (or similar borrowing
and spot exchange transactions) can perform that function quite well.
Module 12 will deal with the theory of options in more detail.
source other than the forward exchange market. Although there are those willing to
make such projections, they are in no sense as reliable as those impounded in the
forward exchange structure (which does not exist for long-term investments like
this). This procedure, by requiring such information, introduces another level of
uncertainty into the analysis.
The best alternative is the second; to use the interest rate structure of the foreign
(€) currency to estimate a risk-adjusted foreign currency discount rate, to find the
foreign currency (€) NPV, and to translate the resulting foreign currency (€) value at
the spot exchange rate to find the domestic (£) value for shareholders.17 This
procedure has the virtue of freeing financial managers from making estimates of
long-term future exchange rates, which is very difficult to do well, and which might
be influenced by extraneous factors (such as a subconscious wish to have a project
look good – or bad).
Another issue in foreign asset investment which should be addressed briefly is
that of adjusting for the extra risks entailed in foreign investments per se. This is a
problematic issue for financial managers, and probably moves to the edge of what is
truly international financial management and what is better left to the larger area of
international business. Two tenets a financial manager should keep in mind about
international investment risks are: first, remember the benefits of diversification;
and, secondly, remember the relative uncertainties of the investment.
What do these tenets imply for foreign investment? First, if a company’s share-
holders are not well diversified across international borders, then a foreign
investment may well deserve a lower risk profile than a purely domestic one if the
foreign investment’s cash flows are not well correlated with a comparable domestic
one. If shareholders are well diversified across countries, this consideration is
neutral.
Second, the kinds of risks inherent in experiencing alterations in foreign tax laws,
exchange restrictions, asset confiscations and frictions in repatriation (of foreign
wealth) can increase the risk of a foreign investment, but there are also comparable
risks in domestic investment. A good analysis of these issues would wish to consider
these relative to comparable domestic risks and add a foreign risk premium only if
truly deserved (there are doubtless instances in which the latter risks are smaller than
domestic ones).
a third location? Our discussions of interest rate parity should lead you easily to the
conclusion that there would be no reason to choose one financing location over
another based upon interest rate differences (of course subsidies offered by gov-
ernments are the exception).
It may also seem that since the value of the investment will first be generated in
the foreign currency, fluctuations in the exchange rate against the shareholder
currency is a risk to be faced. Actually, however, this risk is easy to overestimate if
economic principles underlying exchange rate movements are ignored.
To understand why there may not be much risk of exchange difficulties in for-
eign operations, we must first make certain that you understand the difference
between monetary and real assets. Monetary assets are those whose returns are
expected to be fixed in nominal or money terms in the future, regardless of the
inflation rate in the economy. An example is a receivable (debtor account) held
domestically, but denominated in a foreign currency. With this kind of monetary
asset, a change in the exchange rate (as we have seen in the examples above) affects
the domestic currency value of the asset because the foreign currency value is fixed
whilst the exchange rate varies.
Real assets held in foreign countries are not, however, fixed in foreign currency
value; these real assets increase in value with increases in foreign inflation. Examples
of such assets are primarily plant and equipment and real estate, but managerial
talent, distribution channels, technological expertise and other longer-term produc-
tive assets also qualify as ‘real’.
If an exchange rate changes due to inflation in the foreign country, the domestic
currency value of a real asset held in that foreign currency will not necessarily
change, because the exchange rate effect upon value will be offset by the inflationary
effect upon value. Let us illustrate with an example.
Suppose that Hotlady plc builds its German plant and subsequently experiences a
depreciation of the € relative to sterling. It would seem on the surface that the
sterling value of Hotlady’s German plant would thereby be reduced. (This risk could
be lessened by borrowing in €, so that any decline in asset values due to exchange
would be matched with a decline in liability value.)
However, for the € to fall relative to sterling, it would be very likely that Germa-
ny has experienced an increase in inflation relative to Britain (see Section 11.2.4).
And a German price increase will in turn increase the nominal € costs (and values)
of all €-denominated real assets, including Hotlady’s German plant. So Hotlady’s
British shareholders will experience on the one hand a reduction in the value of
their German plant due to the increase in the €/£ exchange rate, but will also
experience an (offsetting) increase in the €-stated value of the German plant. Both
of these effects are due to the same increase in inflation. Although Hotlady share-
holders’ sterling value per € of German plant has declined, the plant’s increase in €
value compensates, and the plant’s sterling value is unchanged.
This implies that multinational firms need not necessarily finance their real-asset
investments with liabilities in the same currency as the real asset. To generalise a
little, real assets (e.g. plant and equipment) held in other countries tend to experi-
ence increases in value as inflation increases, whereas monetary assets (e.g. accounts
receivable) do not. Only the latter are serious candidates for the hedging of ex-
change risk.
In addition to country locations of financing for foreign investments, we must
also mention that there is a truly international capital market from which
companies can borrow. In this market, long-term funds are usually called ‘Euro-
bonds’ whereas short-term borrowings are described as ‘Eurocurrency’ transactions.
These loans are denominated in a particular country’s currency, but the funds lent
are ‘offshore’ funds in that they have been paid out from the country but not yet
redeposited in a domestic bank in that country. For example, if a US purchaser of
British consulting advice pays in dollars, which are then deposited in a UK bank, the
UK bank can make a ‘Eurodollar’ loan to a third party instead of exchanging those
dollars for sterling.
Since there are several large, efficient national capital markets willing to make
loans to foreigners, why would these ‘Euromarkets’ exist? The answer is the lack of
regulatory oversight in this international capital market relative to domestic markets,
because governments make rules in the latter about bank lending, levy taxes and
occasionally limit the free flow of money across borders. Though comparable-risk
loans will carry very nearly the same interest rates in domestic as in international
markets, there is often a slightly lower borrowing rate in Eurocurrency and Euro-
bond markets than in comparable domestic loans, due to lower regulatory costs.
Finally, companies with international operations should not ignore the potential
for financing to interact with the fund repatriation restrictions that countries’
governments sometimes place upon foreign investment. For example, it is often
easier for a foreign company to export interest payments than it is to pay dividends
to shareholders. Other things being equal, this would indicate that borrowing
instead of equity investment is the better source in such circumstances.
Repatriation of funds in general is often a problem (if nothing more, a public
relations problem) for companies whose shareholders are ‘foreigners’. Companies
must cope with significant measures in many countries that seek to keep within their
borders profits earned by foreign firms. Payments of various types between a parent
firm and a foreign subsidiary are commonly used to ‘facilitate’ the actual flow of
cash from subsidiary to parent. Such things as management fees, royalties, loan
interest and principal repayments, and transfer payments for other services and
goods, are all used (within the legal limitation of the countries involved) to repatriate
funds to the domestic parent company.
words, a foreign country is a law unto itself, and can break promises with impunity
since its only judge is itself. Examples of such risks range from minor non-
compliance with contractual terms to outright confiscation of assets. Any company
that holds assets abroad faces this type of risk to some degree or other.
Management of this risk is possible through invoking certain lessons taught in a
branch of economics called agency theory.18 The trick is to anticipate potential
situations where the host country would be likely to take such action, and build
automatic and irreversible counter-incentives into the original agreement that set up
the foreign investment. Usually this will involve a third party, whom the host
country must appease. For example, participation by the World Bank in a loan that
sets up a manufacturing facility will often ensure that the terms of the original
contract will not be arbitrarily broached by the host country (due of course to the
need for future subsidisations by that institution). Or a contract could specify
automatic ownership transferrals in a third country of host country assets (perhaps
those of a domestic firm in the host country) in the case of a confiscatory action.
It is usually best to assume that all parties to a transaction will act on the basis of
their own best self-interest. It is incumbent upon the good international financial
manager to ensure that the best interests of the sovereign decision maker in the host
country are in accord with those of the foreign investing firm.
Learning Summary
A company operating and transacting across national borders creates additional
‘challenges’ (a business euphemism for ‘problems’) for its financial managers. Most
of these complexities involve dealing with exchange risks and restrictions on profit
repatriation. In this module we have investigated the basic underlying economics of
foreign exchange markets and why exchange rates behave the way they do.
We have learned about hedging against exchange risk on monetary assets, and
why real assets are less likely to require such hedging.
We have outlined the preferred technique of analysing foreign real-asset invest-
ments, and discussed the primary arguments about choosing a foreign as opposed to
a domestic source of international real-asset investment. In the process, we intro-
duced Eurobond and Eurocurrency markets, and their reason for existing.
There are a couple of additional points about international financial management
that are important but do not fit easily into one of the structured sections of the
module, and so we shall discuss them here.
First, one large problem faced by all multinational firms is the artificiality that can
be introduced into financial performance results of foreign subsidiaries by exchange
rate fluctuations. As is clear from our investigations in the module to this point,
such fluctuations can be hedged if they threaten to affect monetary assets and are
18 Agency theory is a very important set of concepts that deal with situations where parties to a
transaction or contract have conflicts of interest, and with the economics of how those conflicts are
best and most efficiently resolved. Module 9 on capital structure introduced a few such concepts and
Module 12 will discuss them in more detail.
probably irrelevant for real assets. Nevertheless many companies either do not
hedge all of their net monetary exposures and/or do not account for inflationary
value increases in real assets in their financial reporting. Both are dangerous biases
(the former a ‘real’ value effect, the latter a reporting distortion) that can and do
affect the way managers and foreign operations are judged. Companies should
ensure that exchange biases do not cloud the real performance (both good and bad)
of foreign subsidiaries.
Secondly, in firms that transact internationally there is often a tendency to be
self-congratulatory when exchange rate changes cause monetary returns to be
greater than they would otherwise have been, and to blame an unpredictable
exchange market when the opposite occurs. We must always keep in mind that there
is no need for a company to expose itself to such risks, and little evidence that any
company, country, individual or institution is likely to be a long-term winner in
betting against the predictions existing in the forward exchange market or in interest
rate differentials among countries.
Review Questions
11.1 Dental Specialities plc, with headquarters in the UK, also operates in Incaland, a country
in South America. The Incaland subsidiary requires an injection of funds in the amount
of Æ60 000 for a year (Æ, or ‘eyes’ as they are called, are the Incaland currency units).
(Yes, the company could not resist an Incaland advertising campaign using ‘An eye for a
tooth!’) At present the exchange rate between sterling and eyes is Æ5 = £1 (i.e. it takes
five eyes to a pound sterling). To acquire the funds, Dental Specialities can either
borrow sterling for a year at 10.24 per cent or eyes for the same period at 13.36 per
cent. If the foreign exchange market is functioning efficiently, which of the following
should Dental Specialities do?
A. Not worry which currency it chooses to borrow, because shareholder
(sterling) wealth will be the same in either case.
B. Borrow sterling and exchange £ for Æ because the interest rate is lower in the
UK.
C. Borrow eyes because the implied inflation rate in Incaland will cause the eyes
necessary to repay the loan to be less expensive than the implied lower-
inflation sterling.
D. Borrow eyes because the Æ liabilities thereby generated will be repaid from
future eyes, thus removing exchange risk from the transaction.
11.2 The one-year forward exchange rate between eyes and sterling in Question 11.1 must
be which of the following?
A. Æ5/£.
B. Æ5.5120/£.
C. Æ5.6680/£.
D. Æ5.1415/£.
11.3 Using the data from Question 11.1, suppose that the real rates of interest in the UK and
Incaland are identical at 4 per cent per annum. The implied inflation rates in the two
countries are thus which of the following?
A. UK inflation = 6.24 per cent, Incaland inflation = 9.36 per cent.
B. UK inflation = 6.00 per cent, Incaland inflation = 9.00 per cent.
C. UK inflation = 10.24 per cent, Incaland inflation = 13.36 per cent.
D. UK inflation = 9.00 per cent, Incaland inflation = 6.00 per cent.
11.4 With the data for the UK and Incaland that you generated in the above questions,
assume that you are a UK resident and have £1000 to lend for a year, at which point
you wish to spend the resulting cash for consumption. You would be well advised to:
I. Lend sterling because of the exchange risk of coming back into sterling from eyes a
year hence.
II. Lend eyes because of the higher eye interest rate, and the fact that since you will be
consuming in sterling, the higher eye inflation will not affect your sterling purchasing
power.
III. Lend eyes as long as you hedge the future exchange of eyes for sterling.
Which of the following is correct?
A. Both I and III.
B. III only.
C. II only.
D. None of the above.
11.5 Suppose that Dental Specialities foresees an inflow of Æ100 000 one year hence from its
Incaland operation. Further, suppose that the company is concerned that the spot rate
of exchange of eyes for sterling one year hence will be worse than now. Dental
Specialities could:
I. Hedge by borrowing eyes now in an amount requiring a Æ100 000 payment one
year hence, and exchange the eye loan proceeds for sterling now.
II. Hedge by selling one-year Æ100 000 in the forward exchange market.
III. Do nothing in the exchange market because the company’s financial manager is
convinced that in a year the eye/sterling exchange rate will be lower than it is now.
Which of the following is correct?
A. I only.
B. II only.
C. Either I or II.
D. III only.
11.6 If Dental Specialities wishes to pursue strategy (I) in the preceding question, it should do
which of the following?
A. Borrow Æ90 711 and exchange to £18 142.
B. Borrow Æ100 000 and exchange to £20 000.
C. Borrow £17 643.
D. Borrow Æ88 215 and exchange for £17 643.
11.7 Dental Specialities should expect to get present value of sterling from the correct
transaction(s) in Question 11.5 of how much?
A. £18 142.
B. £17 643.
C. £20 000.
D. £17 157.
11.8 Dental Specialities’ marketing programme has been so successful that it is thinking of
building a denture manufacturing plant in Incaland. The company’s project analysts have
generated cash-flow estimates in eyes for the Incaland plant for each year it is expected
to operate and have estimated the riskiness of those cash flows. The question now is
the most appropriate method to evaluate the project’s cash flows. Dental Specialities
uses NPV analysis for such decisions. Which of the following should it do?
A. Discount the Æ-stated cash flows with risk-adjusted Æ-based discount rate(s),
and convert the resulting NPV to sterling at the current spot rate.
B. Discount the Æ-stated cash flows with risk-adjusted sterling-based discount
rate(s), which accomplishes both discounting and exchange in one step.
C. Convert the Æ-stated cash flows in each future period to sterling using
forward exchange rates observed or estimated for those periods, and find NPV
by discounting with sterling-based risk-adjusted discount rate(s).
D. Convert the Æ-stated cash flows in each future period to sterling using
forward exchange rates observed or estimated for those periods, and find NPV
by discounting with Æ-based risk adjusted discount rate(s).
11.9 You have been hired as a consultant by Dental Specialities to advise it about hedging the
exchange risks of their Incaland operations. The company is particularly concerned
about the extreme volatility of eyes, the Incaland currency, and its effect upon the
wealth of the company’s sterling shareholders. Dental Specialities’ operations in Incaland
comprise some minor importing and exporting of raw materials and finished product,
but now are primarily devoted to manufacturing and selling dentures in the country.
Your advice to the company should be to do which of the following?
A. Hedge all Incaland assets by undertaking either forward exchange transactions
or foreign borrowing, because changes in exchange rates will affect all asset
values.
B. Hedge only the company’s monetary assets held in Incaland, because only these
are likely to feel a net effect of inflation and exchange rate changes.
C. Hedge the important real assets in Incaland, but not the less important
monetary assets, because the import/export side of the business is trivial.
D. Do not hedge.
11.10 Dental Specialities is writing a contract with a joint venturer in Incaland to start up a
quite profitable hygienist training school in that country. Dental Specialities is concerned
that if the school is a success, the joint venturer will begin a competing school and
gradually take away the business. Dental Specialities intends, of course, to include
wording in the contract that aims to prohibit such an action, but it is concerned that the
Incaland courts would be biased toward a domestic company in any resultant litigation.
Which of the following would you suggest to Dental Specialities that it also write into
the contract?
A. That the joint venturer’s adherence to the contract be collateralised with
assets held in either Dental Specialities’ home country or an uninterested third
country.
B. That the joint venturer’s adherence to the contract be collateralised with
assets held in the joint venturer’s home country.
C. That the son of the joint venturer’s managing director be sent to live with the
family of Dental Specialities’ managing director.
D. That the adjudication of disputes between Dental Specialities and its joint
venturer be settled in Dental Specialities’ home country’s courts.
Learning Objectives
This final module of the course explores topics that are at the cutting edge of
financial management practice. To say that these are ‘new’ ideas or procedures
would not be purely correct, because some of the concepts have been studied
academically for decades and have taken that long to begin appearing in financial
management practice. Other ideas and financial instruments we examine in this
module have actually been known to practising financial managers for quite a while,
but only recently have their conceptual bases and importance been recognised.
Options are properly termed ‘contingent claims’ because their value depends
upon the value outcome of some other underlying asset. For example, a call option
allows its owner to buy an underlying asset at a fixed price for a fixed time. Aside
from the importance that these financial assets have taken in their own right (on
options exchanges) in the recent past, option characteristics are contained in other
not-so-obviously option-oriented securities. The module describes how the com-
mon shares of a geared company is such an option. From a simple model of option
valuation (the binomial model), it portrays the development of very powerful option
valuation mechanisms that are used every day in actual options markets.
Even more important, these option models allow us to understand certain securi-
ty market phenomena that would be hard to explain without such insights. To
investigate these, a more detailed description of a subject briefly mentioned in
several earlier modules, that of agency relationships is presented. Agency prob-
lems arise when a decision maker (an agent, e.g. the managers of a company) acting
12.1 Introduction
As does any active subject area, the field of finance possesses researchers who
continually push forward the boundaries of understanding about the field. This
module will introduce you to options valuation, agency relationships, deriva-
tives and financial engineering, ideas that have relatively recently made their way
from the realm of pure financial research theory to important applications in
financial decision making.
This is a good way to finish your finance course, for two reasons. First, these
topics, having recently attracted the attention of sophisticated financial practitioners,
allow students to offer excellent signals to the ‘marketplace for managers’ as to the
depth and freshness of the financial education they have received. Secondly, though
the topics are different in their essential nature (options, derivatives and financial
engineering are primarily a set of quantitative techniques with important current
applications, whereas agency concerns are more conceptual), they are interrelated
and indicate much about the way finance as a subject has evolved and is evolving.
This should help you to evaluate critically new ideas in finance that will appear
during your professional career.
12.2 Options
This section will introduce the notion of options, how they are valued, and the
extent to which options are much more common in financial markets and decision
making than you might have expected. As a matter of fact, viewing some very
familiar decisions and securities as options (even though you do not usually think of
them as such) will prove to be a very important way to increase the depth of your
understanding about finance.
But first we must learn about the basics of options.1
1 Option markets have been around a long time, and have until fairly recently been on the fringe of the
more formal markets such as the large stock exchanges. Options markets, probably because of their
comparative institutional insularity over the years (and also due to the peculiar individuals who
populated these markets), have developed a colourful terminology all their own. To the extent that
these terms are acceptably printable in a publicly circulated text, we shall indicate them in bold face
below.
2 With respect to when options can be exercised, there are two types: European (which can be exercised
only at expiration) and American (which can be exercised at any time before or at expiration). As you
would expect, whether an option is European or American has nothing to do with its geographic
location. How those names arose is, we suppose, another story.
3 Contrast this with the forward or futures contracts that we discussed in Module 1. In those contracts
the purchaser of a forward contract to buy a security is forced to complete the transaction even if
completion is to the detriment of the purchaser (e.g. the security could be purchased more cheaply in
the open market). An option holder would not be required to complete the transaction.
4 Remember a few other things about the transaction. First, as we mentioned earlier, the option writer
has no option to avoid the transaction. If you write an option you are allowing someone else the
choice to transact or not. Secondly, few options on formal options markets are ever really exercised in
the sense of shares changing hands; transaction costs would be minimised by having the option writer
in this example simply give the option holder the £25 differential. In well-developed options markets,
the writer and holder never even meet, and all transactions are cleared in a central computer, with the
option exchange standing by the validity of the contracts as written. Finally, if you have written a call
option and actually own the underlying securities (e.g. here you own 100 shares of 3rd Rate), you are
described as having written a covered option or a covered call. If you have written the option but do
not own the underlying securities, you have written a naked option or naked call.
5 Henceforth we shall discuss option values on a ‘per share’ basis (e.g. £0.25 rather than £25) for
simplicity.
6 You may sometimes encounter the term ‘theoretical value’ as a synonym for exercise value. This is a
misnomer because, as we shall see very soon, exercise value is not market value, and thus ‘theoretical’
would also be synonymous with ‘wrong’, which makes the alternative term of no use.
45%
7 Since the rest of this module will concentrate on the valuation of call options, you should assume that
– unless otherwise indicated – ‘option’ means ‘call option’.
8 All of the illustrations in this module are simplistic in that transaction costs (brokerage fees, etc.) are
ignored, as they are in this example.
9 If no ‘at the money’ option existed, you could write one and sell it for some positive amount of money.
money’ options since the underlying securities must increase more in value before
the option has exercise value.
‘In the money’ options also sell for more than their exercise values, for essentially
the same but somewhat more complex reasons than those above. Though the full
reasons will later become more clear, the simplest intuitive reasoning for market
value to exceed exercise value for ‘in the money’ options is that the option holder
stands to gain at exercise the benefit of possible interim increases in underlying
asset or security value, but is not at risk for all possible reductions in underlying
asset value.
Look again at Figure 12.1, and consider an only slightly ‘in the money’ option
(say, a price of £1.30 for 3rd Rate’s shares). Here, if 3rd Rate goes up in price, you
can see that exercise value goes up with it, whereas reductions in value of 3rd Rate
could not cause the option to decline in value below zero. So there is a kind of
positive asymmetry to the possible returns from holding the ‘in the money’ option
that does not exist in holding the underlying security itself. This is why ‘in the
money’ options sell for more than their exercise values.10
Market and exercise values
Market value
Exercise value
10 Exercise of an ‘in the money’ option would net the holder shares of the underlying security worth the
exercise value. These shares would not have the same ‘lower boundary’ benefits as the option did; this
is another way of seeing why the option is worth more than its exercise value.
for a price above £1.25, your option to purchase will sell for its exercise value plus a
premium.11 For example, with 3rd Rate selling for £1.50, your option would sell for
a premium above its £0.25 exercise value. How much above? The next section will
begin developing the ideas that can answer that question.
S0 = £1.50
1–
q=
0.4
dS0 = 0.5 x £1.50
= £0.75
11 One other thing about ‘in the money’ option values: the further ‘in the money’ an option is, the less
will be the excess of market over exercise value. The reason is that zero minimum exercise value is less
and less comfortable, the higher is exercise value. And the further ‘in the money’ is an option, the
higher is exercise value. For example, if 3rd Rate were selling for £20.00, the £1.25 exercise price
option with an exercise value of £18.75 would have little downward value protection from reductions
in 3rd Rate’s share values. Of course, the mirror-image holds for well ‘out of the money’ options, for
the reason that eventual increases in underlying security value to generate positive exercise values are
less likely the further ‘out of the money’ is the option. It should not surprise you to hear that the 3rd
Rate option to purchase at £1.25 would not sell for much more than zero if 3rd Rate’s shares were
selling for only £0.10.
12 This idea was first formalised in 1979 by Cox, Ross and Rubenstine in the Journal of Financial Economics
(September 1979), 229-63 as ‘Option Pricing: A Simplified Approach’, and by Rendleman and Bartter
in the December 1979 Journal of Finance as ‘Two-State Option Pricing’.
13 This option pricing model will work as well with ‘at the money’ and ‘out of the money’ options, as you
shall see soon. We are also assuming a ‘European’ option (exercise only at expiration), though the
discrete time nature of this market (in other words prices quoted only ‘now’ and ‘later’) makes such an
assumption implicit.
Of course these potential changes are uncertain; we do not know which of the
two prices will actually happen. Let us assume that there is a 60 per cent chance of
the increase and a 40 per cent chance of the decline in the share price of 3rd Rate.
Thus we have a simple (it is called a ‘binomial’) probability distribution of potential
price changes in the underlying security.
Figure 12.3 also uses some notation that will eventually serve us well in describ-
ing option value:
the current price of the underlying security £1.50
the likelihood of the underlying security price increase 0.6
the underlying security increased price result £3.00
the underlying security reduced price result £0.75
We can actually regard the u and d notations as ‘multipliers’ for original share price,
such that:
upward multiplier for underlying security price 2.0
downward multiplier for underlying security price 0.5
You can check to see that the multipliers applied to £1.50 will produce either
£3.00 or £0.75 as the price of 3rd Rate’s underlying shares at option expiration.14
Now let us consider the value of your ‘in the money’ call option on 3rd Rate.
First, we shall specify the final payoffs to you as the holder of the option, depending
(contingent) upon what happens to the shares of 3rd Rate. But first, some more
notation:
current market value of the option
option payoff at expiration if underlying security price is up
option payoff at expiration if underlying security price is down
exercise or striking price of the option
14 There are some additional technical assumptions we must make to be rigorous. Markets must be
‘efficient’ in impounding information, and there can be no taxes nor brokerage fees; interest rates must
not change during the period in question; 3rd Rate can pay no dividends before option expiration, you
must be able to ‘sell short’ securities without restriction, and there must be a ‘stable stochastic process’
generating the price changes in 3rd Rate’s shares. In addition, this ‘binomial’ model of option valuation
requires that the option extends for ‘one period’ of time, measured discretely from ‘now’ until
‘expiration’.
None of these assumptions affects very much the real application of this pricing model, so we shall not
do more than mention them here. If you wish to pursue actual transactions in options markets, you
should be aware of these considerations because they can affect the net profitability of such transac-
tions.
Cu = max (0,uS0 – X)
0.6 = max (0,3.00 – 1.25)
q= = 1.75
C0
1– Cd = max (0,dS0 – X)
q=
0.4 = max (0,0.75 – 1.25)
=0
15 Not exercising is the ‘zero’ alternative in the specification of potential payoffs to holding a call option.
When you have written a call option, ‘zero’ means the holder would not choose to exercise. The same
is true for put options.
16 This investment is sometimes termed a ‘call-equivalent portfolio’.
by forming a portfolio of the shares of 3rd Rate itself, in combination with borrow-
ing or lending money.
The portfolio must have payoffs equal to Cu and Cd, with the same likelihoods as
those of your option. Let us define three more pieces of notation:
the risk‐ free interest rate for either borrowing or lending
for the period of the option; assume this is 10 per cent
the number of underlying shares to purchase or if negative, to sell short
the amount lending if positive or borrowing if negative at the risk‐ free
rate
For the portfolio to duplicate the option’s payoffs, it must be true that the final
payoffs from the shares in the portfolio (Y times either uS0 or dS0) and lend-
ing/borrowing Z times 1 rf t be the same payoffs that the option would
produce, given either the increase or decline in 3rd Rate’s shares. This can be
written formally as:
The payoff if 3rd Rate increases: YuS 1
The payoff if 3rd Rate decreases: YdS 1
You can recognise from elementary algebra that the above is a set of simultane-
ous equations. Leaving out the tedious intermediary steps, we can solve the above
for Y and Z, to get:
£1.75 £0.00 ﴾12.1﴿
0.7778
£1.50 2.0 0.5
and:
2.0 £0.00 0.5 £1.75 ﴾12.2﴿
£0.5303
1 2.0 0.5 1 0.10
This tells us that if you purchase 0.7778 of a share of 3rd Rate, and simultaneous-
ly borrow £0.5303, you will have the same future cash-flow expectations that would
be produced by holding the call option.
t
+ YuS0 + Z(1 + rf)
= 0.7778...(2.0)(1.50) – 0.5303...(1.10)
0.6
q= = 1.75
Portfolio
+ YS0 + Z
1– t
q= + YdS0 + Z(1 + rf)
0.4
= 0.7778...(0.5)(1.50) – 0.5303...(1.10)
=0
Figure 12.5 Call option payoff diagram from call equivalent portfolio
Let us see whether that is true. Figure 12.5 will help us to visualise the workings
of this investment. Note that if the shares of 3rd Rate increase, the 0.7778 shares in
this portfolio will be worth 0.7778 £3.00 £2.33, but the interest and principal
on the borrowing will produce a liability of £0.5303 1.10 £0.58, so the net
value is £2.33–£0.58 £1.75, the same as the option. Note that this has the same
(60 per cent) likelihood as the option’s payoff, because both this and the option
payoff are contingent upon the same event, namely the increase in 3rd Rate’s share
price.
If 3rd Rate’s shares go down, this portfolio returns 0.7778 £0.75 £0.58 mi-
nus the £0.58 interest and principal, or £0.00 – also the same as your option, and
with the same likelihood. So this call-equivalent portfolio duplicates exactly the call
option’s expectations of the cash flow’s amounts, timing and riskiness.
‘Very clever’, you say. ‘But what good does that do us? The above implies only
that I can get the same payoffs as the call option from another source. That is nice,
but is not the purpose for which we began the exercise. You were supposed to show
me what my call option is currently worth in the market.’ Believe it or not, we have
shown you.
Take another look at Figure 12.5. It shows that the call-equivalent portfolio du-
plicates the payoffs of the option, and it also shows how to calculate the cost of the
call-equivalent portfolio. What good is that information? Remember our ‘law of one
price’: two investments offering identical expectations of future cash flows cannot
sell for different prices in an efficient market. This clearly tells us that the cost of the
call-equivalent portfolio is in fact the market value of your call option. They must
have the same value if they have the same cash-flow expectations.17 What is the
value of your option? Its value is the cost of the call-equivalent portfolio:
0.7778 £1.50 £0.5303 £0.6364 ﴾12.3﴿
Market value of call option:
Cost of call‐ equivalent portfolio £0.6364
Your call option is currently worth a bit less than £0.64. Notice that the option’s
exercise value is £1.50‐£1.25 £0.25, and so the option is selling at a ‘premium’ of
about £0.64‐£0.25 £0.39 over its exercise value.
This technique of calculating the market value of an option is perfectly general
for ‘binomial’ cases like this one. All you need do is use Equation 12.1, Equation
12.2 and Equation 12.3. For example, suppose that the price of 3rd Rate’s shares
were £1.25 so that your option was selling ‘at the money’, and all other characteris-
tics of the above were the same. The option’s value would be:18
17 If they did not, arbitrage opportunities would exist. For example, if the call-equivalent portfolio were
cheaper than the option, you could simultaneously write the option and purchase the call-equivalent
portfolio, be perfectly hedged at option expiration (because the gains or losses on the option would
exactly offset the losses or gains on the call-equivalent portfolio), and pocket the difference in price
between the two investments right now.
Such arbitrage opportunities cannot exist for any appreciable time in an efficient market. (Who would
be silly enough to purchase the higher-priced option which you write, if they could do the same
arithmetic as you?) This is the driving force behind the ‘law of one price’.
18 Remember to calculate: Cu max 0,uS0‐X max 0, 2.0 £1.25 ‐£1.25 £1.25
and Cd max 0,dS0‐X max 0, 0.5 1.25 ‐£1.25 £0.00
adjusted discount rate, and then adjust it for the gearing of the company so as to
come to an independent estimate of share value.
To value this call option, however, we dealt only with characteristics of the un-
derlying security, the option’s contractual provisions, and the risk-free rate. This
is substantially less data – and easier-to-estimate data – than any valuation model
we have seen. The reason for this is primarily because the option is a contingent
claim. In a sense, the market has already done a good part of our valuation task
for us by setting the price of the underlying security. By understanding that we
are dealing with an option security whose value is purely based upon another
value outcome, we have an unprecedentedly efficient valuation mechanism.
3. Finally, it is also important to remember that the basic valuation process we have
here relied upon is a ‘no arbitrage’ one. Recall that we discovered the value of the
option by finding the cost of obtaining identical future cash flows with another
investment. This is a very simple yet powerful approach to financial valuation, in
that it requires that we make very few assumptions about how people and mar-
kets behave, other than that mispriced securities cannot long exist (or that
arbitrage profits will quickly disappear, which is the same thing).
This straightforwardness is a very desirable characteristic of option valuation,
and we shall see that applications of option valuation are broader than this ex-
ample has implied. But we must also remember that the elegant simplicity of
option valuation does require that an underlying security or asset already be
valued. Thus there must be instances where option models will not work, and
where our original valuation mechanisms will be required.
4. We assumed that the option was a ‘European’ option in that it could not be
exercised before maturity. Actually, most options on active markets are ‘Ameri-
can’ options, which can be exercised at any time up to or at maturity. But notice
something else: the market value of an option is always greater than its exercise
value (see Figure 12.2). This means that options are not in practice exercised prior
to expiration, and that European and American options will therefore have the
same value.19
5. The illustration of option valuation we have studied above is for a call option.
Exactly the same valuation process is applicable to finding the value of a put
option, with respect to specifying the contingent payoffs and their ‘put equiva-
lent’ portfolio value. Generally, any type of contingent claim is subject to the
type of valuation process we have illustrated here; the concepts are applicable as
well to more complex contingent claims such as convertible and callable bonds,
as we shall soon see.
19 For some options based upon underlying securities that are expected to pay significant cash dividends
prior to option expiration, this may not be the case. Options may also be exercised before maturity if
there is any risk that the terms of the option may change.
Time 0 1 2
q Cuu
Cu 1–
q q
C0 Cud
q
1–
q Cd
1–
q Cdd
‘Wait just a minute,’ you say. ‘Not so fast. We admit that you have cleverly got
the model to have many potential outcomes. But in order to get them, you used so
many periods that we would be great-grandparents before the option expired. How
“realistic” is that?’
A good, but not unanticipated question. Recall that we never told you anything
about the calendar length of the periods in the model. As there is nothing to stop
us from having a lot of periods, there is also nothing to stop us from making those
periods very short. Suppose that we told you that each period was 30 seconds long
– or even less than a second. Now the model can have lots of outcomes in a very
short calendar period.
Envisage Figure 12.6 with hundreds or thousands of periods, each of which was
almost infinitesimally short. That kind of binomial market is not a bad representa-
tion of what we experience in actual options markets. We could get such a model to
produce results. It would still be conceptually simple, but its numerical complexi-
ty would probably render it inefficient for calculations. Fortunately there is an
alternative.
Two researchers in 1973 published a groundbreaking option valuation model
(this was actually invented before the simple binomial version we just investigated).20
To make a very complex story as short as possible, you can regard Black and
Scholes’s option pricing model as essentially the same as the ‘many short period’
binomial model, except that the time periods are so short that they are ‘continu-
ous’.21 Instead of discrete time periods, time in ‘Black–Scholes’ (as this model has
come to be known) is a continuous process, and security prices can therefore
change continuously and thus have no unusual restrictions.22
We shall not attempt here to derive the Black–Scholes option model (our work in
the binomial method and the appendix to this module is enough of that), but rather
portray and interpret its manifestation:
﴾12.7﴿
where:
½ ½
ln / rfT / 0.5 ﴾12.8﴿
and:
½
﴾12.9﴿
At first blush, these equations seem overwhelmingly complex. But take heart,
they are actually more familiar than they appear. Consider Equation 12.7, which
produces the C0 option value. It is actually doing the same thing that our binomial
option valuation process did: calculate the value of a ‘call equivalent’ portfolio. The
20 See Fisher Black and Myron Scholes’s ‘The Pricing of Options and Corporate Liabilities’ in the May–
June 1973 issue of the Journal of Political Economy.
21 Recall that Module 1 introduced the concept of continuous time in discounting.
22 Technically, underlying security prices must be generated by a ‘stable’ process with a ‘constant
variance’. But you need not concern yourself with that unless you are attracted to such concepts.
first part of the formula, S0N d1 is the number of shares of the underlying security
to be purchased, where N d1 is the equivalent of our Y in the binomial process, and
the second part of the formula ‐Xe‐rfTN d2 is the amount of borrowing, our Z in the
binomial process.23 The N d1 and N d2 are actually instructions to take the d1 and
d2 numbers calculated in Equation 12.8 and Equation 12.9 and find the value of the
‘cumulative normal unit distribution.’
What does this mean? It is an instruction to take the value of the ‘cumulative
normal unit distribution’ at the point d1, where d1 is calculated in Equation 12.8.
The cumulative normal unit distribution is a standard statistical measure, tables for
which appear in all statistical reference books.24 Next is the exercise price X
multiplied by e‐rfT. This latter is a continuously compounded interest or discount rate
with e being (recall Module 1) the base of the natural system of logarithms, rf being
our familiar risk-free rate of interest and T a new variable representing the time
remaining until expiration of the option.25 N d2 is another cumulative normal
distribution value, this time for the variable d2 calculated in Equation 12.9.
Equation 12.8 describes the calculation of d1, with ‘ln’ being an instruction to
find the natural logarithm of the bracketed expression, and the variables within the
expression being already familiar, namely the ratio of underlying stock price (S0) to
exercise price X . The remaining parts of Equation 12.8 are also familiar, with the
exception of σ, which is the instantaneous standard deviation of the price of the
underlying security. Equation 12.9 has no variables that have not already been
mentioned.
Though admittedly intimidating, the above system of formulas is performing a
calculation very similar to that which we did in the binomial model of option
valuation. The only important difference is that time is now continuous instead of
discrete, and thus there are effectively an infinite number of price changes that
could occur in the underlying security.
Performing calculations with the Black–Scholes formula is not difficult, particu-
larly if you have access to a typical spreadsheet computer program. Let us go
through a Black–Scholes calculation using the 3rd Rate company from the binomial
option example. Using the data for the ‘in the money’ call option:
£1.50,
£1.25,
10%,
1,
23 The N d1 component is typically called an ‘option delta’ by professionals who deal in such markets,
and is closely related to the option’s ‘hedge ratio’ as explained in the appendix to this module.
24 We do not include such a table here because it is extraneous. These formulas have been widely
programmed into computers and even personal calculators, which are readily available to those
interested.
25 The time until expiration of the option must of course be calculated in the same units as the interest
rate is quoted. For example if T is stated in months, the interest rate must be a monthly rate.
and we shall assume that the standard deviation of 3rd Rate’s shares, σ .69.26
From Equation 12.8 we get:
½ ½
ln / rfT / 0.5
½
ln 1.50/1.25 0.10 1 /0.69 1 0.5 0.69 1½ 0.7542
and from Equation 12.9:
½
.7542 0.69 1½ 0.0642
We now go to our spreadsheet program and have it look up the cumulative nor-
mal unit distribution figures for d1 and d2:27
N d1 0.7746
and
N d2 0.5256
to which we then apply Equation 12.7 to get:
.
1.50 0.7746 1.25 2.71828 0.5256 0.567
The Black–Scholes option value of 3rd Rate’s call option is approximately £0.57,
given our assumptions as to standard deviation of 3rd Rate’s share value.
The above system of equations is important to the realm of options for two
reasons. The first is historical. This Black–Scholes option valuation model is the
original idea that has fathered a whole industry of practical application and academic
theorising (which we shall review briefly below). The model, which began as a
theoretical exercise, has proved itself (with some slight modification) to be very
robust in predicting the actual price behaviour of options.
The second reason the Black–Scholes model is important to students of options
is that the model makes obvious the importance of specific option characteristics in
determining option value. Look again at the system of Equation 12.7, Equation 12.8
and Equation 12.9. There are five variables for determining option value:
26 This standard deviation figure is actually consistent with the u and d assumptions we made for 3rd
Rate in the binomial valuation method, through the relationship that u eσ/ where I is the time
interval as a proportion of a year. As we shall see, this actually produces a lower value for the call
option than the binomial method. The reason this happens has to do with the contrast between the
binomial implication of one ‘up or down’ per period, as opposed to the Black–Scholes’ continuous
application of the ‘up or down’ through the standard deviation.
27 The Excel© spreadsheet uses the function NORMSDIST for this process; other spreadsheets have
similar functions.
28 Those who choose to pursue more advanced study of options will discover that the more complex and
esoteric types of options are actually just combinations of various call and put options.
29 The illustration that follows assumes throughout that put and call options will have the same exercise
prices and will be held to expiration. More advanced treatments of option valuation can relax those
assumptions, but they do not cause the essential relationships to be much different from those shown.
Exercise price
45%
0
Exercise price
Value of underlying asset
30 Keep in mind, however, that the put option’s market value would not be zero ‘at the money’ or ‘out of
the money’ unless there was no chance that the underlying asset would decline in price to be below the
put’s exercise price before expiration.
31 This is in contrast to the call’s exercise value maximum, which is conceptually unlimited (see
Figure 12.1).
Exercise price
Exercise value of riskless asset
0
Value of underlying asset
found by combining Figure 12.7 and Figure 12.8, which produces the payoffs
shown in Figure 12.10.
Exercise
price
0
Exercise
price Value of underlying asset
0
Exercise
price Value of underlying asset
‘Very clever,’ you say. ‘It is interesting to see that the equity of a geared firm is
really a call option. But what good is that? How does that recognition push forward
our understanding of sophisticated financial ideas?’
The answer to that question lies in adapting the discussion of the variables in the
Black–Scholes option model in the previous section to the equity of a geared firm.
Recall that there are five variables of interest in the option equations, and we
investigated how each affects option value. Those variables have familiar counter-
parts in the leveraged firm. And if we ‘experiment’ with changes in those variables,
we can predict effects upon equity and debt value, given the option formula. These
‘changes in variables’ in the option formula have quite straightforward translations
to financial decisions which we have discussed in other modules. In the process of
this experiment we shall gain additional insight into the effects of various financial
decisions upon shareholder and bondholder wealth, and the reasons for certain
complex contractual provisions of bond issues.
Consider the importance of the ratio of underlying asset value to exercise price
S0/X . For the option held as equity by shareholders of the company, underlying
asset value is the total value of the company’s assets, and exercise price is the
interest and principal promises to creditors. If this ratio is high, the option is well ‘in
the money’, which implies a low proportion of debt in the company’s capital
structure. If the ratio is low, the company has lots of debt. A ratio of 1.0 implies that
all of the current value of the company is promised to creditors. We learned that the
premium above exercise value for an option tends to be greatest when an option is
‘at the money’ (see Figure 12.2). In this context, though the absolute (exercise) value
of the equity of the firm is highest when there is no debt (a fully ‘in the money’
option), equity has its most advantageous option characteristics when the company
is very highly geared (‘at the money’).34
One very important influence upon equity value as an option is given by the
standard deviation of underlying asset value (σ). Recall that option, and therefore
equity, value increases as σ increases. The interpretation of an increase in σ is
straightforward: it means that the operating assets of the firm are more risky. For
example, by shifting its operations into a new, more risky line of business, the firm
can increase the value of its shares.
Other option value variables have similar interpretations. For example, the longer
the maturity of the debt, the higher is equity value, other things being held the same.
‘That is terrific!’ you say. ‘We have discovered a new way to increase shareholder
wealth. All that is necessary is to examine the company decisions that would
increase equity value from the option perspective, take those actions, and our
shareholders will be better-off. Their appreciation will certainly be reflected in high
salaries for us as financial managers.’
34 Remember that equity value falls when debt is issued, but the cash proceeds of the debt issue are
available to shareholders, so a move from a well ‘in the money’ to an ‘at the money’ position would
entail the shareholders being compensated for the decline in share price at least to the extent of the
exercise value of the option. A ‘well out of the money’ option in this sense could be a firm in
bankruptcy, where the debt claims far exceed the value of the company’s assets.
Not so fast. We have not yet repealed the ‘law of conservation of investment
value’, which has served us well to this point. To paraphrase that law: increases in
value have to come from somewhere. From where do the equity value increases
outlined above come? The answer is that they come from the pockets of debt
holders.
Think of the situation this way. Suppose that a leveraged company wishes to
increase its riskiness so as to increase equity value. It does this by shifting assets
from less risky to more risky activities.35 Equity value increases. But unless the value
of the company’s assets increases, and if asset value is the sum of equity and debt
value, the only possible source of increases in equity value is reductions in debt
value.
This is not difficult to understand. Recall that debt has a fixed claim upon the
future cash flows (value) of the firm. Even if the value of the underlying assets
remains the same, the riskier operations create greater likelihoods of debt not
receiving its promised payments (in bad times). This causes debt value to decline.
There will also now, with higher variance company activities, be greater likelihoods
of equity receiving very high returns (in good times). Thus equity value increases
and debt value decreases, in like amounts.
‘Well, so what?’ you ask. ‘We may feel sorry for debt suppliers, but their wealth
loss is their problem. If we can take such an action, we are bound to do so in the
interest of our shareholders.’
Exactly. But creditors are not stupid. They can anticipate your feeling exactly as
you do. So when they lend money to the company, they insist upon certain provi-
sions in the bond contract that prohibit such actions on behalf of shareholders.
Examples of such ‘covenants’ are prohibitions or restrictions upon asset sales,
mergers, dividend payments, seniority rules and a wealth of other actions by firms.
In other words, though you may wish to increase equity wealth by undertaking such
activities, you will probably be contractually prohibited from doing so. Or if your
bond contract allows such activities, you were probably charged ‘in advance’ by
agreeing to a much higher interest rate than you would otherwise have paid.
The above paragraph is an important perspective on option theory applied to
business finance, and it is also an introduction to the next topic we shall discuss in
this module: the theory of agency. Agency relationships are important when there
are potential conflicts of interest in decision situations. It is easy to see that complex
company capital structures can cause conflicts of interest in financial decision
situations.
Our lesson from this application of option valuation theory is that specific ac-
tions taken by geared companies, as these appear in the option-value models, can
shift wealth from debtholders to shareholders (and of course vice versa, though
financial managers would not last long in their jobs if they shifted wealth from
35 To make the illustrations as simple as possible, assume that the risk increase is unsystematic only, and
does not change the value of the assets.
36 One of the most difficult aspects of such provisions is that, although they constrain shareholders from
capturing creditors wealth, they may also constrain the firm from undertaking activities that could
increase underlying asset value to the benefit of all capital suppliers. This trade-off is central to the
capital structure decision of the firm.
Time t0 t1 t2 t3 t4
FCF* −£2 400 +£45 +£1 470 +£1 725 −£30
With a required 18% rv* discount rate, and an NPV of −£272, the WalkPhone
project was (with a negative NPV) not acceptable. But suppose that the company’s
accepting the WalkPhone project will position it to enter the market for cellular
Internet devices in a few years, whereas without the experience in the WalkPhone
market the cellular Internet devices (hereafter cId) market will be unavailable to the
company.
‘Well fine,’ you say. ‘If the WalkPhone project is itself a loser, but will let the
company enter a subsequently very profitable market, all we must do is attach the
cId cash flow projections to those of the WalkPhone project and evaluate the NPV
of both together.’
Sounds reasonable.
Now suppose that the cash flows that will result from the cId project are quite
uncertain, and given what is now known, could be very good or very bad. Much of
this uncertainty has to do with the project’s not actually beginning for several
periods into the future. Further suppose, as is often the case, that as time actually
passes the nature of the project’s cash flows will become more evident, but the
apparent desirability of the project will fluctuate up and down as evidence and
opinions evolve. Nevertheless, the best guess currently available as to the cash flows
associated with the cId project’s cash flows is:
Time t4 t5 t6 t7 t8
FCF* −£9 075 +£170 +£5 558 +£6 523 −£113
Further suppose that the 18% rv* discount rate is also applicable to this project.
Applying this discount, the NPV of the cId project is −£1 027 at t4, and therefore
−£530 at t0. So combining the cash flows of this project with those of WalkPhone
would produce a total NPV of −£530 (cId) and −£272 (WalkPhone), or −£802.
‘Terrific,’ you say. ‘What a nice example. Not only should the company avoid the
WalkPhone, it should also forget about entering the market for cId. I cannot really
say that I am very impressed by your ability to illustrate the importance of real
options.’
Careful here; we have not yet dealt with all of the information we were provided
about this project. The negative NPV of the cId project is based on the set of cash
flow expectations as they exist right now ( t0). But the information we have is that
these cash flow expectations could change dramatically and be much better (or
much worse) by the time the project actually would begin (at t4). How can we
properly evaluate this more complicated situation? We can do so by regarding the
situation as an option valuation problem.
Consider again the implications of accepting the WalkPhone project. Its own
cash flows have an undisputed NPV of −£272. But by accepting the WalkPhone
project the company will also be purchasing an option of pursuing the cId project
four periods hence. Today the cId project does not look so good, but that could
change between now and the time the project would be undertaken. Specifically,
undertaking the WalkPhone project carries with it a −£272 NPV, plus the value (if
any) of the real option to undertake the cId project.
Now, let us recall one important thing about options: an option cannot have a negative
value. In other words, the currently negative NPV of the cId project would only be
applicable if the company were committing itself to undertaking that project with its
currently expected poor cash flows. And accepting the WalkPhone project does not
necessarily imply accepting the cId project. Accepting WalkPhone gives the compa-
ny the ability to decide (four periods hence) whether or not to pursue the cId
project. If the cId project still looks bad at that point, the company can simply
choose not to accept it, thereby avoiding its poor results. So the currently negative
NPV of the cId project, though not completely irrelevant is certainly not a proper
basis for thinking of the value of this option. The absolute worst result of having
this option must be simply to leave the NPV of WalkPhone as it is without the
option.
But how then can we think about valuing this option inherent in accepting
WalkPhone? To be honest, rigorous valuation of such real options is still to some
extent in its intellectual infancy.37 Nevertheless, even our introductory exploration
of option valuation to this point can help us conceive how we might estimate the
value of this option.
For example, this option sounds like a call option (the right to buy something in
the future). Call options have (amongst other attributes) an exercise price, a time until
expiration, and an underlying asset value that are important parameters in option
valuation. Here the exercise price is the outlay to undertake the cId project, or
£9 075 at t4. And the time until expiration is the four periods into the future when
the above outlay for the project would be made. So far, so good. But what about the
underlying asset value of the option? That is simply the value of the project’s cash
flows from beyond the date of exercise, discounted to the present t0 . With a PV of
£8 048 at t4, the underlying asset value at t0 (using the 18% rv* is £4 151.
So we can describe the real option imbedded in the WalkPhone project as being a
four period option with an exercise price of £9 075 and an underlying asset value of
£4 151. Clearly (and consistent with the NPV analysis we have done) the option is
‘out of the money’ (its exercise price is higher than the underlying asset value). But
37 This would doubtless be hotly disputed by the several corporate financial consulting firms that are
actively marketing advice as to how this is done.
that does not mean the option is worthless. Even ‘well-out-of-the-money’ options
can be valuable. What would cause this option to have value? Of course it would be
that there is some chance the underlying asset value will increase to be above the
exercise price between now and the exercise date (i.e. of the option going into the
money, or equivalently, the cId project’s expectations producing a positive NPV).
And what creates such a chance? The underlying asset value will increase if the
currently existing cash flow expectations for the cId project improve (due perhaps
to upward revisions in revenues or downward revisions in costs or operating risk,
etc.).
It may have occurred to you already that the option valuation parameter that
captures this likelihood of the option going ‘into the money’ is the standard deviation
of underlying asset value.38 Recall from those discussions of call option valuation that,
other things held the same, an option is more valuable the higher is the standard
deviation of underlying asset value. That should make some sense here. Remember
that the company buys a call option on the cId project if it undertakes WalkPhone.
And that option is one that can be left unexercised (i.e. reject the cId project) if at
the decision point four periods hence the cId project is undesirable. But if the cId
project ‘turns good’ between now and then (and stays that way until t4) the company
would undertake it. And the higher is the standard deviation of the project’s underlying asset
value (the more it ‘bounces up and down’ across time), the higher are the odds that the cId project
will ‘turn good’ between now and the expiration date. In other words, risky prospective
projects like the cId project (with high standard deviations of underlying asset value
prior to their being undertaken) are the kind that could produce high option values.
How high must the standard deviation of underlying asset value be in order for
the option to have a positive value? Not very high. As a matter of fact, as long as
there is any chance at all, no matter how tiny, that the option could go ‘into the
money’ it will have a value greater than zero. This implies that the −£272 NPV of
the WalkPhone project understates the true value of the project to the
company because it ignores the almost certainly positive value of the real
option to undertake the cId project.
‘O.K,’ you say. That makes some sense. But should the company undertake the
WalkPhone project or not? What you have shown so far is that the WalkPhone’s
NPV is not as bad as −£272, but you have not shown what it is. At this point we
still seem to be in the non-quantitative ‘strategy’ mode where we began.’
True, but we are not quite finished with the example. We have earlier illustrated
specific techniques of valuing options, one of which is the binomial method. That
method can be extended to deal with multiple period examples. With information as
to the market rate of interest and the u and d parameters of cId underlying asset
value for each year we could apply the process and value the cId option implicit in
the WalkPhone project. We have also investigated the Black–Scholes call option
valuation formula based on the continuous passage of time (a more realistic
38 This is the set of u and d parameters in the binomial illustrations, or the σ parameter in the Black–
Scholes option valuation formula.
portrayal of actual option experience). Let us try to apply the Black–Scholes model
to this cId real option.
The text’s earlier explanation Black–Scholes option valuation indicates that we
currently have three of its five necessary parameters (exercise price is £9 075,
underlying asset value is £4 151, and time to expiration is 4 periods). If we assume
that the observable riskless interest rate is 10%, we have four of the five. What
remains to be specified is the standard deviation of the underlying asset value. But
here, rather than simply assume a value, it is more informative to return to the
decision facing the company: Should WalkPhone (with the cId option) be undertak-
en? To answer ‘Yes’ requires that the NPV of the combination must be greater than
zero. Because we know that WalkPhone by itself has an NPV of −£272, another
way of asking the same question is: does the option have a value at least as high as
+£272? If so, WalkPhone should be undertaken, if not, both WalkPhone and its cId
option should be rejected. Since the only parameter now missing from the option
valuation is the underlying asset standard deviation, a final way of asking the
question is: How risky must the underlying asset be, in order for the option to have
a value of at least +£272?
Here one can simply begin working the Black–Scholes process until the critical
standard deviation is found (the one that causes option value to equal £272), or
more efficiently if you have access to a typical spreadsheet computer program,
invoke a ‘goal seeking’ instruction to find that standard deviation after having set the
Black–Scholes process into the spreadsheet as described earlier in the text. Either
way, if done correctly, you will see that an annual standard deviation of 24.1% is the
critical value for the option to be worth at least £272.
With the above parameters, the Black–Scholes process from Equation 12.8 and
Equation 12.9 is:
0.5924
1.0737
and therefore by taking the cumulative standard normal of each:
0.2768
0.1415
and by the final Black–Scholes Equation (Equation 12.7):
£272.00
So it is possible to estimate the value of the cId option. If the variability of the
underlying asset value of the cId project is approximately 25% or more for the next
four periods, the company should undertake the WalkPhone project, because the
option to pursue the cId project would at least offset the negative NPV of
WalkPhone. (Parenthetically, a 24.1% standard deviation is not a terribly high rate,
with many average securities on stock exchanges having annual rates of about 20%.)
company to pursue a subsequent investment. There are other types of real asset
decisions with embedded options. Consider the situation where a project can either
be operated or liquidated at various points during its projected lifetime. At each of
the potential liquidation points (which would be chosen if the project has turned out
badly) there is effectively a put option that allows the company to sell the project
instead of operating it for its remaining lifetime (or at least until the next liquidation
option point). The company would choose to liquidate the project if the exercise
price (the liquidation value) is greater than the present value of the remaining
project cash flows plus the value of the additional options to liquidate further in the
future. And therefore the NPV’s associated with operating the project through its
lifetime must be augmented with the put option values inherent in the ability to
abandon or liquidate the project. Ignoring abandonment or liquidation options can
produce misleading investment NPVs.
More advanced treatments of financial decision-making (such the advanced fi-
nancial courses in this program) are necessary to examine in any detail the other
specific types of options embedded in real asset decisions, but there are many, and
more are being discovered with great regularity. As a matter of fact, some financial
thinkers have expressed the sentiment that just about all real asset decisions have
some kind of embedded option. Think of a run-of-the-mill investment project. How
many of these actually lack abandonment alternatives at future times? For what
proportion is it impossible to react to altered information across time by changing
some characteristic of the investment? Precious few. And only those precious few
are purely good candidates for straightforward NPV analysis; the rest ought to be
judged by criteria that include their option values.
12.3 Agency
Finance is a fairly complex subject. But the complexity of finance as a set of ideas
and techniques is as nothing compared with the complexity of actual financial
markets and securities. In fact, students (and skilled practitioners) of finance have
often and correctly observed that actual financial markets and securities seem to
have a complexity that is unexpected given the more straightforward nature of pure
financial economics (e.g. discounting and valuation processes, taxes, risk and claim
hierarchies). What is the cause of this complexity? The odds are that it has some-
thing to do with the topic of this section: agency relationships.
What do we mean by ‘agency’ as it applies to financial phenomena? The answer
itself is a fairly complex one, but is well worthwhile pursuing, and we shall provide
that answer soon. But first, a few words of encouragement.
Learning something about agency relationships is worthwhile because of the
impressive ability this concept has shown for explaining why certain financial
situations require complexity, and also why that complexity is intellectually pene-
trable. In other words, we shall see that agency concepts can explain real market
actions which at first seem either irrationally complex or outside the scope of
traditional financial economics.
In the discussion below we shall not attempt to describe the area of agency in
finance in all of its elaborate detail. Rather, our intent is to familiarise you with the
basic concerns that can be profitably addressed with this approach, and show a few
examples of its application.
The primary engines driving agency situations are conflicts of interest. Recall the
situation of shareholders and bondholders of a firm, which we addressed in Section
12.2.6. We were at that point interested in the option characteristics of geared
equity, and pointed out that financial managers can shift wealth from bondholders
to shareholders by causing the firm to undertake unanticipatedly risky asset invest-
ments. Obviously this is a situation where shareholder and bondholder interests
diverge. It is a strong candidate for analysis as an agency problem.
Now let us define an agency situation.39 First, to be a problem in agency, there
must be an ‘agent’. An agent is an individual, group or organisation to whom a
principal has designated decision-making authority. In the context of the share-
holder–bondholder conflict, shareholders (or their proxies, the financial managers
of the firm) are the agents, and the bondholders are the ‘principals’. Principals are
those who feel the ultimate effects of the decisions taken by agents.
In order for an agency situation to be interesting (to be a ‘problem’), it must
contain the potential for a conflict of interest between principal and agent. The
shareholder–bondholder conflict of interest is an important one, but by no means
the only agency problem experienced in company finance.
Other important agency problems arise in the relationship between a firms’s
managers and its shareholders.40 Consider that managers can be assumed to
maximise their personal wealth as a function of the decisions that they undertake for
the firm. Depending upon the type of remuneration paid to managers, the efficien-
cies of the ‘market for managerial talent’ and the corporate ‘takeover’ market,
managers may find instances of decision making where their personal best interests
conflict with those of shareholders. Managers may consume company resources by
spending on ‘perks’ such as thick carpets, private jets and attractive secretaries, all of
which return less in improved company performance than they provide as con-
sumption to managers. Managers can also ‘consume’ company-paid time by shirking
duties. The resulting lower value of the firm is an ‘agency cost’.
For managers to discontinue such activities, their personal interests must be
melded with those of shareholders. To accomplish this, it is not generally enough to
give some shares to managers. As long as managers own less than 100 per cent of
the firm’s shares, managers will gain £1 in personal benefit from consuming £1 of
company resources, while losing only £1 multiplied by their percentage holding of
all company shares due to the resulting lower value of shares.
If managers are discovered undertaking such company consumption, an efficient
market in managerial talent could drive down their salaries. But it is not clear, in a
firm with widely dispersed shareholdings, that this kind of compensation pressure
can be brought to bear upon managers (who probably control the board of direc-
tors). Thus many economists think that an efficient market for company takeovers
is an important solution to the agency problem of manager–shareholder conflict.
In a takeover, existing shareholders are offered a premium above existing market
price to sell their shares, usually by an outside group of managers. The outsiders are
effectively telling the shareholders that existing management is inefficient, and thus
the increase in company value that will appear upon more efficient new managers
taking over (and somehow avoiding the prior agency difficulties) can be allocated
between old and new shareholders.41
There are some important general lessons about agency problems in the above
discussion. First, note that there is an incentive to solve the agency problem. The
overall incentive is that the increase in the value of the company if managers
perform without unproductive personal consumption exceeds the value to the
managers of their personal consumption. So, in general, managers would lose less
than the company would gain if such activities were curtailed. But unless managers
are provided personal incentives to change their behaviour, this value is permanent-
ly lost (an agency cost). The ‘solutions’ illustrated above include buyouts by outside
management groups.42
There are other possible solutions to this agency problem, from complex man-
agement employment contracts, to managerial stock options, to auditing by
independent directors, and many more. Sometimes good solutions to conflicts of
interest can be derived from such devices. At other times a takeover or buyout is the
ultimate solution.
But any solution to an agency problem requires that (1) there be an overall gain
to solving the problem, which is allocated in such a way that (2) the agent has an
incentive to participate in the solution. This holds, as we shall see, for bondholder–
shareholder conflicts as well as shareholder–manager conflicts.
Agency problems can also arise when ‘information asymmetries’ exist. An
information asymmetry occurs when a buyer and seller in a transaction may have
different amounts or quality of information about the decision at hand.
Consider the situation of a company attempting to sell bonds so as to undertake
a positive NPV project. Suppose that the managers inside the firm have specific
information as to the NPV of the project, but that the potential bondholders have
only a general notion of its NPV (as is often the case, due to legal and competitive
constraints upon information release).
If there is a range of possible NPVs, bondholders likely assume that NPVs lower
than those known by the firm are possible. Therefore the firm will not be able to
41 When a company ‘goes private’, by the managers buying the firm from shareholders, similar
premiums above prior market values are also paid. Managers evidently themselves recognise that
holding 100 per cent of the shares will cause different behaviour on their own parts.
42 These outside management groups are often described as ‘hostile’. Though they are usually hostile to
existing managers, they are not usually so to existing shareholders. As a matter of fact, entrenched
management groups often go to great lengths (with publicity, lawsuits, delaying actions, and so forth)
to discourage potential takeovers. Their incentives for doing so are obvious.
sell bonds at prices commensurate with the actual higher NPVs of the project. The
lower bond prices are the agency cost of this situation.
Is there a ‘solution’ to this agency problem? It would seem that the basic condi-
tions are there (the incentive to increase the value of the bonds by revealing the true
NPV of the project). A simple solution would require that the bondholders must
somehow be assured of the project’s NPV before the bonds are sold, and this is
usually impossible. Eventually, of course, if the project is undertaken, its NPV will
be revealed by its outcome, and bond prices will increase. The problem with this
situation is that bondholders instead of shareholders will receive at least part of the
NPV of the project, as bond prices increase. With that foreseeable, the shareholders
may not allow the project to be undertaken in the first place.43
One solution to this agency problem may be to issue a security more complex
than a simple bond. Suppose that the company issued a bond with a call provision.
A call provision, you remember, allows the company to repurchase the bond from
the bondholders at a set price for a given period of time.44 Call provisions in effect
allow the shareholders to buy the bond at a fixed price no matter what it would be
worth without the call provision. How does this solve the agency problem caused by
information asymmetry about the investment?
The agency problem is that the bond is not as valuable as it should be (thus the
firm does not take in enough cash from selling it originally). The call provision,
being a call option on bond value, must also therefore be undervalued. The bond-
holders, by undervaluing the NPV of the project and thus the bond value, are also
undervaluing the call provision that shareholders have upon the bond. So when the
project finally proves itself, the company can call the bonds and gain their increased
value.45 Thus the agency problem caused by information asymmetry is solved by this
complex security, the call-provisioned bond.
There are innumerable such agency situations that arise in finance. (One of the
other important ones we have already discussed in Module 9 on capital structure:
the bankruptcy costs of borrowing.) Interestingly, the ‘solutions’ to these problems
were invented by financial markets well before researchers appreciated the agency
problem itself. Actually, agency theories in finance have been most impressive in
explaining financial conditions and securities that already existed but that seemed to
be irrational.46
43 In a more rigorous sense, the NPV of the project to shareholders will be lowered by the present value
of the expected eventual amount of bond value increase.
44 Yes, the call provision is in fact a call option on the bond.
45 The actual transaction would be for the firm to set a call price for the bond such that the bondholders,
because of their relative pessimism about the project, would not assign much likelihood to the bond’s
price increasing so as to make the call provision attractive. The firm, of course, is aware that because of
the higher NPV project, the call provision is more likely to be exercised than the bondholders assume.
Actually, all of this will be accomplished in present-value terms as soon as the bond with call provision
is originally sold.
46 The call provision is a good example of this. Simplistically, it is possible to think of a bond’s call
provision as a bet between the bondholder and shareholder about their respective capacities to predict
changes in future interest rates. (Of course, if interest rates go up, bond prices go down, and vice versa.
12.4 Derivatives
12.4.1 What They Are and Are Not, and the Reasons for their Reputation
During the past few years, the popular press has produced a number of dramatic
headlines describing spectacular financial market losses incurred by companies and
other institutions that have participated in investments called derivatives. Well
known names such as Procter & Gamble, Barings Bank, Orange County in Califor-
nia and Sumitomo Bank are but several examples of the many organisations that
have lost spectacularly, in the hundreds of millions of dollars, in very brief periods
of time.
But what exactly are derivatives? Are they the frighteningly complex securities
that are used only by those unfortunate enough to be led (or misled) into these
markets by their own ignorance or avarice? Are they designed to accomplish
nothing more than quickly emptying the pockets of participants (and perhaps their
shareholders or citizens)?
Actually, derivatives can be very dangerous financial instruments, so complex
that those who buy them and sell them (and even at times those who invent them)
do not understand enough to make responsible decisions about them. But that is
not the inherent nature of derivatives. A ‘derivative’ is simply any financial security
whose return or outcome set is derived from some other asset’s value or return
outcome. By this definition we have already studied a number of derivatives in the
course.
Recall, for example, our review of forward and futures contracts in Module 1.
The prices and returns on such securities are purely derived from the prices of
underlying (sometimes called ‘primitive’) assets such as grain, metals, foreign
exchange or even other financial securities such as government bonds. Thus these
contracts properly fit the definition of ‘derivatives’, but in no sense are they terribly
difficult to understand or necessarily financially dangerous. The same would hold
true for other examples of derivatives we have seen, such as options and other
contingent claim securities. Our study of these securities has more often than not
indicated uses that tend to reduce risk (through hedging activity) rather than expose
buyers and sellers to large risks of loss.
Why then have derivatives acquired this popular reputation as being very compli-
cated and dangerous? Well, one reason is that they can be. There has evidently been
A call provision, which makes a bond cheaper for the bondholder to buy, will produce a gain to
shareholders if interest rates turn out to be lower than the price of the original call provision implied,
and vice versa.)
But why in the world would shareholders and bondholders be interested in betting against each other
on the behaviour of interest rates in the future by attaching a call provision onto a bond? All of the
best evidence is that neither group would be expected to be better than the other in predicting changes
in interest rates. The answer is that shareholders and bondholders are not in the least interested in
participating in such a lottery (it is a ‘fair’ lottery in that the expected value is equal to its cost). What
they are doing is avoiding the agency cost of asymmetric information. Only agency theory could have
offered that explanation (though the market discovered the call provision solution well before we
academics were able to explain why it existed).
a tendency on the part of some financial security issuers to offer some frighteningly
complicated derivative securities, the risks of which even they do not understand.
And even simple derivatives can be very dangerous when misapplied.
Consider for example a simple futures contract in copper. Our study of financial
securities such as those in Module 1 implies that the participants who would
logically buy and sell these securities would be producers or users of copper seeking
to hedge the uncertainties of future price movements in that metal. But there is
nothing (save the size of one’s initial wealth) that would stop anyone from taking
positions, either long or short, in copper futures. If you took a large position in
copper futures, and the price unexpectedly changed in a direction adverse to your
position, you could lose a lot of money very quickly, particularly if you bought on
margin (leveraging), as is typically done. But engaging in such a transaction would be
simply taking a bet that you have better anticipated changes better in the price of
copper than has the futures market in the metal, an obviously naive assumption on
your part.
This latter scenario has been an unfortunate characteristic of many of the large
losses sustained by participants in derivatives markets, including the Barings,
Sumitomo and Orange County fiascos. In each of these, there were either irrespon-
sible or unsophisticated participants that led their institutions into holding derivative
positions that had immense – if not bizarre – exposures to changes in metals prices,
interest rates or foreign stock market indexes. Each of these stories is great fun to
read, to review the embarrassing outcomes and to puzzle over how they could have
happened. But the causes seem to be mostly organisational shortcomings at the
institutions in question rather than any real problem in the markets for the deriva-
tives that were the proximate cause of the losses.
There is absolutely no reason for Orange County, California, to be taking large
leveraged bets on whether interest rates in the United States are going to increase or
decrease. US Treasury securities markets are populated by tremendously well
informed traders who spend all their time seeking to anticipate and arbitrage very
small mispricings of such securities. The notion that a political subdivision in
California would have any consistent ability to beat such traders at their own game is
ludicrous; yet it tried.
The other typical scenario of large losses in derivatives markets is more compli-
cated than the Orange County or Barings examples. In this alternative scenario
(which we can call the Procter & Gamble type), a company with a legitimate reason
for participating in derivatives markets (usually to hedge interest-rate risks stemming
from other financings or working capital) bought into (or was led into, depending
upon whom you believe) positions in very complex, even one-of-a-kind securities.
Rather than producing hedges, the positions had exposures to changes in interest
rates likely not to be comprehended by either the holder or the issuer. And interest
rates did change in ways that caused huge unanticipated losses to the holders. What
were thought to be hedged positions turned out to be very risky bets on particular
types of changes in interest rates.
47 This not to say that one can only hedge in these markets. Obviously the positions of Barings and
Orange County were not hedges. There are potentially non-hedged and non-pathological positions that
could be taken in derivative markets, but such transactions essentially require that the participant has
monopolistic information about future movement in prices of the assets that underlie derivatives.
Consistently obtaining such information is most unlikely.
Stock Market
Futures Contracts on Market Indexes
Options on Market Indexes
Options on Individual Securities
Foreign Exchange
Forward Contracts
Futures Contracts
Options
Swaps
Mortgage
Complex Derivatives48
Real Asset
Forward Contracts
Futures Contracts
Options Contracts
Hybrids and Exotics
As you peruse the list, you will doubtless notice that many of the securities listed
are familiar. Actually, the only ones that you may not be able to identify are likely to
be the ones named ‘swaps,’ ‘hybrids and exotics’ and perhaps the mortgage-based
derivatives. We shall relegate the study of the latter to those of you who are interest-
ed in pursuing those specialised securities beyond the depth permitted in this text.
Swaps, however, are commonly enough encountered so that at least a passing
familiarity with them is appropriate for a student of finance at your advancing level.
12.4.4 Swaps
‘Swaps’ are derivatives designed to allow hedging the risks of interest rate and
foreign exchange-rate movements. A swap is a conceptually straightforward
transaction where one party to the transaction exchanges one stream of cash flow
with a ‘counterparty’, who provides the other stream of cash flow to be exchanged.
There are often ‘side payments’ of some type to or from one or the other party or a
third-party facilitator to the swap.
With that general definition of a swap, an example is in order. To illustrate an
interest rate swap, suppose that a firm had issued a long-term bond with a set of
promised interest and principal payments extending for some period of time. And
suppose that this firm was in a line of business that caused its operating cash flows
to be volatile, such that there was the chance of operational constraints due to the
need to cover the periodic required interest and principal payments. If the volatility
of the firm’s operating cash flows was due to a type of risk that appeared in some
other type of interest rate-based security (say, due to raw materials costs or credit
48 There are many different types of mortgage-based derivatives, more than a listing here could credit.
Mortgage derivatives differ from interest-based derivatives primarily in that mortgage-based derivatives
have risks contingent upon changes in the timing of principal repayment by borrowers as interest rates
fluctuate. Texts have been written on this subject alone.
terms that fluctuated in response to the same things that caused short-term interest
rates to fluctuate), the firm could undertake an interest rate swap to reduce the risk
caused by the fixed-interest payments of its long-term bonds.
Investment bankers or other financial institutions would be happy to ‘facilitate’
such an ‘interest rate swap,’ where the firm exchanges its responsibility for making
fixed long-term interest payments for a responsibility to make interest payments that
fluctuate up and down in concert with the firm’s operating fortunes (thus in effect
hedging the firm’s risk of operational disruption due to financial distress). To do
that, the firm transacts with a counterparty to make the firm’s fixed interest pay-
ments in exchange for the firm making variable interest payments on behalf of the
counterparty.49 Thus the firm is relieved of the risk of its operating cash flows being
disrupted by the necessity of making fixed interest payments.
‘Wait,’ you say. ‘We already know how to do that another way. The firm could
have bought or sold some kind of interest-rate futures. Why do we need another
hedging instrument?’ A good question. There really is no risk of interest rate or
exchange rate movement that could not be hedged by forward or futures transac-
tions instead of swap transactions, at least in concept. The reason for these kinds of
swaps to exist is that they are cheaper to undertake than futures or forward interest-
rate transactions (in the sense of avoiding a long stream of commissions to brokers
on these transactions), or because those markets are not ‘complete’ enough (in other
words, they may not offer exactly the type of contract that would hedge the risk in
question), or both.
Hedging risks of adverse interest or exchange rate movements is probably the
most important use of swaps, but there are others. Swaps have been used in
complex combinations in exchange and interest-rate markets – to avoid certain
kinds of costly national securities regulations or adverse taxation consequences, for
example.
That swap transactions are important is without question. Within the last few
years there has been a huge increase in the number and size of swaps, both in terms
of interest-based and exchange-based risks and opportunities.
12.4.5 Exotics
‘Exotics’ are true derivatives, but ones formulated from combinations or mixtures
of other types of derivatives. These are conceptually important derivatives. They are
tailored to the very specific risk exposures of a single firm, and can be very compli-
cated – so complicated, in fact, that there have been instances where there is at least
an allegation that not even the inventor of the security understood the potential
49 This transaction is typically done through a third-party investment banking house, who may or may not
be the counterparty. And the firm is not at all released from its legal liability to make its long-term
interest payments, but rather will use the payments it receives from the counterparty to make those
payments. Obviously, the quality of the counterparty’s promise is important, and should be no less
good than the firm’s, or the firm will be less well off. On occasion, the investment banking house will
offer some type of guarantee, though there may well be a counterparty who would have exactly the
opposite type of need.
range of outcomes that could be experienced by the firm entering into the transac-
tion (the Procter & Gamble scenario is an example).
A detailed example of this type of derivative is difficult for this text, not only
because of its complexity but also because of its length. Think, nevertheless, of a
security that paid off on the contingency that some type of interest rate increased
above a particular ‘cap’ rate, or declined below a particular ‘floor’ rate (the distance
between the two rates sometimes being termed, appropriately enough, a ‘wedding
band’). Pricing such a security and shaping the risks it can hedge is formidable
indeed. And further think of a security that paid off on the ‘inverse’ contingency of
‘X% minus’ some interest rate standard.
You begin to see the reason for the term ‘exotics’, and the above are not particu-
larly complicated examples of the types of transactions being offered in these
markets.
Are such transactions useful? Do they accomplish any reasonable economic
purpose? Apparently so, or we should not expect to see anything like the volume
that we do. They are important because of the very sophisticated type of ‘financial
engineering’ they exemplify.
will exhibit particular kinds of cash-flow patterns of size and volatility, and that the
project will have complex interactions with national regulations and taxation
regimes. Financial engineering can be employed to design purchases and sales of
various financial securities (complex combinations of futures and forward contracts
in interest and exchange, swap commitments, options and other participations) that
serve to reduce the risks of adverse movement in each of the risk exposures implied
by the project (changes in interest rates, exchange rates, real asset prices, etc.), and
to reduce the costs of national regulations and taxation.
Of course, designing such complex and specific shapings of risk, contingencies,
timings and locations of present and future cash flows is not quantitatively a simple
task. The semi-pejorative term ‘rocket scientists’ is often employed to describe the
individuals with training and expertise in producing these financially engineered
exposures. And there have been some rather spectacular failures due to poorly
designed packages of contracts, as we have discussed above. But the writing on the
wall is clear: financial engineering is not a mere passing fad, and will be an increas-
ingly important set of tools for the use of financial managers.
Most true financial engineering activities are ‘one-off’ analyses requiring great
technical expertise and familiarity with not only the financial instruments that can be
used but also the company or organisation demanding the service. And because very
few operating companies have the expertise on their staff to perform the necessary
analyses, most financial engineering services are purchased from second-party
advisors or investment bankers. But the technology of financial engineering is
becoming more widely available, in both professional training and automated
computational contexts.
50 See, for example Smith and Smithson’s The Handbook of Financial Engineering, Harper & Row, New York,
1990.
The conceptual contribution of financial engineering is the idea that all familiar
financial securities can be regarded as comprising some combination of these three
aforementioned (and highlighted) characteristics, in greater or lesser proportion; and
that inventing new or ‘hybrid’ financial securities to fit exactly some requirement of
an individual financial market participant is a matter of combining these essential
elements into a package of claims that produces a profile of cash-flow expectations
meeting this specific need.
Recall the complicated situation of the firm facing the project described in Sec-
tion 12.5.1. A financial engineer would dissect or decompose the elemental nature of
the exposures and opportunities of that firm’s position with respect to the project,
and design securities that would appropriately hedge the interest, exchange and real
asset risks and exploit the regulatory and taxation opportunities. These elemental
exposures and opportunities would have the essential characteristics of credit
extension (borrowing and lending), price fixing and price insurance.
Learning Summary
The purpose of this module has been to introduce you to more advanced topics in
finance, from two perspectives. First, well-educated finance students should be
aware of the general direction of advanced thinking in the field, even if they are not
deeply involved at those levels. This is important because finance is a growing,
active field of research and new application, with an almost constant flow of
innovative concepts. Even practitioners beginning in the field must have some
capacity to appreciate and make judgments about these new applications and ideas.
Our second purpose has been to show you two specific advances in financial
thinking: options and agency theory. The module’s presentations of these contained
ideas that were unknown only a short time ago. Yet the applications of options and
agency concepts are growing very quickly in financial markets, and have even greater
promise. You can rely upon encountering many instances where these concepts are
invoked to explain or solve financial problem.
51 Over and above generating a very specific and complex set of derivative cash flows, these contracts
must of course be priced by the sellers (typically an investment house or commercial bank), who then
have the resulting problem of hedging their own exposures on the other sides of the contracts.
uS0 – mCu
S0 – mC0
dS0 – mCd
On the right-hand side of Figure 12.12 are the payoffs to the hedge portfolio.
The expression uS0‐mCu is the amount of money that the portfolio would get if the
underlying security increases in price, whereas dS0‐mCd is the portfolio’s payoff if
the share price goes down. Notice that the payoffs comprise both the share price
change and the payoff to having written the call option.
Also in Figure 12.12 is the condition necessary for the portfolio to be perfectly
hedged:
﴾12.11﴿
In other words, the portfolio payoffs must be the same no matter what happens to
underlying security price or option payoff.
We can solve Equation 12.11 for m so as to find out the only thing we do not
know: how many call options to write:
﴾12.12﴿
The variable m actually has a special name: it is called the hedge ratio because it
tells us how many options to write against each share of the underlying security so
as to achieve the perfectly hedged portfolio:52, 53
Let us try out the above process on 3rd Rate’s shares and call options. Recall
from the module that:
the current price of the underlying security £1.50
the likelihood of the underlying security price increase 0.6
the underlying security increased price result £3.00
the underlying security reduced price result £0.75
Further recall that the exercise price of the call option is £1.25, creating:
£1.75
and:
£0.00
We can now solve for the hedge ratio:
52 This is obviously an important concept to those who are interested in using options markets to alter
the riskiness of their security holdings. One caution is that hedge ratios in real options markets (i.e.
non-binomial markets) are somewhat more complex to calculate (though by no means are they
inaccessibly so).
A more serious caution is that – as Equation 12.12 indicates – the hedge ratio itself is a function of the
underlying stock price and option payoffs, so that any change in these over time will cause the hedge
ratio to change, requiring revisions in the hedge portfolio. Many options holders have learned an
expensive lesson that such actions must be taken very fast in actual markets, and sometimes are
impossible to execute before the damage has been done (see the major market moves of October 1987,
and the financial press’s reporting of ‘portfolio insurance’ – which did not work as well as its
promoters had hoped during that period).
53 The astute reader may have noticed that the hedge ratio m is also the inverse of Y from the ‘call
equivalent’ valuation process. The economic significance of this is better left to more advanced
treatments of option issues.
54 This illustration is taken from a working paper by John Hannum, titled ‘Convertible Debt and the
Incentive to Shift Risks’.
55 Note that the bonds need to be converted into shares to accomplish this recapture. Just like American
call options (which are not exercised before expiration because market value always exceeds exercise
value), the conversion attribute of a bond causes the bond’s market value to be higher due to the
simple ability to convert.
following period, the implied asset values at t2 are (uu) £3920, (ud) £1680 and (dd)
£720. This set of value expectations is shown in the top panel of Figure 12.13.56
Straight Convertible
u = 1.4, d = 0.6
debt debt
Firm value Values Values
at t = 2 at t = 2
1290.18 1606.60
3920
2800 1290.18 1026.50
2000 1680
1200 720 720
720
Value Value
at t = 0 at t = 0
1000 1000
Straight Convertible
u = 1.8, d = 0.2 debt debt
Firm value Values Values
at t = 2 at t = 2
56 Note its similarity to Figure 12.6. The only difference is that Figure 12.7 leaves out the irrelevant
probabilities.
57 We have not before characterised a straight bond as being an option. But a bond is a ‘contingent claim’
in the same sense that a share is: the final payoffs to a bondholder are contingent upon the asset
values that the firm actually obtains.
the bondholders end up with the entire asset value of the firm).58 Thus at t2,
Cuu Cud Cdu £1290.18, and Cdd £720. Assuming an interest rate of 10 per cent
per period, we can calculate the possible values of the bond at t1. If u occurs during
the first period, bond value at t1 calculated using Equation 12.1 is:
£1290.18 £1290.18
0.00
£2800 1.4 0.6
and, applying Equation 12.2:
1.4 £1290.18 0.6 £1290.18
£1172.89
1 1.4 0.6 1 0.10
Therefore (from Equation 12.3):
C1 YS1 Z 0 £2800 £1172.89 £1172.89
If d occurs during the first period, using Equation 12.1, the bond value will be
(at t1):
£1290.18 £720
0.59394
£1200 1.4 0.6
and, applying Equation 12.2:
1.4 £720 0.6 £1290.18
£265.79
1 1.4 0.6 1 0.10
Therefore (from Equation 12.3):
C1 YS1 Z 0.59394 £1200 £265.79 £978.52
Taking the two possible values of the bond at t1 as the outcomes for the bond at
that time, we can now find the value of the bond at t0:
£1172.89 £978.52
0.12148
£2000 1.4 0.6
and:
1.4 £978.52 0.6 £1172.89
£757.04
1 1.4 0.6 1 0.10
Therefore:
C0 YS0 Z 0.12148 £2000 £757.04 £1000
So the bond will be worth £1000 (see the top panel of Figure 12.13) and there-
fore, with the total value of the firm at £2000, the shares must also be worth £1000.
The above assumes that the bondholders will believe the company’s assessment
of the risk of the project. Suppose, however, that the bondholders are concerned
that the firm will take their money and put it in a higher-risk project than the one
implied by the company values above. Specifically, let us assume that the bondhold-
ers fear that the firm will choose an investment yielding the asset values in the
bottom panel of Figure 12.13. You can see the higher risk evident with u 1.8 and
58 The shareholders will not take up their option to purchase the assets of the firm from the bondholders
at a price equal to the promised payment, because the promised payment exceeds the value of the
assets.
d 0.2. The company is still worth £2000, but this value is generated by a greater
spread of final values at t2, as Figure 12.13 indicates (the highest is now £6480
instead of £3920, and the lowest is only £80 instead of £720).
The effect of this fear on the part of bondholders is that they are not willing to
pay £1000 for the bond, even though the bond carries the same promised payment
of £1290.18 at t2. The higher risk of final asset values changes the possible final
payoffs to bondholders to those shown in the lower panel of Figure 12.13. With this
riskier project, the bondholders are looking at payoffs of either £1290.18, £720 or
£80 as opposed to the £1290.18, £1290.18 or £720 implied by the lower-risk
project.59
The lower panel of Figure 12.13 indicates that bond value will fall to £642.90.60
Bondholders will only be willing to provide the firm with this much cash, since they
are aware that, given the chance, the firm will take the higher risk project so as to
benefit shareholders.
The problem here is that the firm must under this situation take the riskier in-
vestment, because if it does not, the bond would then increase in value, and shares
decline by the same amount. This is an agency problem. But it has a nice solution:
the firm can issue a convertible bond instead of the simple one used in the example
thus far.
Suppose that the company, instead of issuing a straight bond, issues a convertible
bond with a promised payment of £1026.50 along with an option for the bondhold-
er to convert the bond into 40.985 per cent of the company’s common stock at any
time.61
The payoffs to such a bond are illustrated in the right-hand columns of the top
and bottom panels of Figure 12.13. If the firm issues this bond and subsequently
takes the lower-risk investment (top panel), the bond’s payoffs are either
Cuu £1606.60 (the bondholders convert their bonds into 40.985 per cent of the
shares, 0.40985 £3920 £1606.60, because this is greater than the bond’s
promised payment of £1026.50), Cud £1026.50 (the bondholders do not convert
because 40.985 per cent of £1680 is obviously less than the promised payment of
59 The higher variance of asset value increases the value of the option that shareholders have to
repurchase the assets of the firm from the bondholders for the promised payment. In simpler terms,
the higher variance benefits the shareholders by increasing the size of their potential highest payment
without hurting the size of their potential lowest payment. This increases equity value. And since total
company value is unchanged, the value shareholders gain comes from the pockets of bondholders.
60 We shall assume either (1) that you are willing to take our word for the new bond value or (2) you can
substitute the new payoffs in the sample calculations from the top panel of Figure 12.7 (which we just
presented) to validate the figure.
61 Two points about this convertible bond:
a. First, the 40.985 per cent is obviously not chosen randomly; we have chosen that specific number
for a reason that will become clear.
b. Secondly, the terms by which a convertible bond can be changed into common stock are specified
in the bond contract, and are usually quoted in terms of a ‘conversion ratio’ (number of shares per
bond) or ‘conversion price’ (price of shares per £ of bond face value at conversion) both of which
amount to simply a fixed percentage of shares.
£1026.50) or Cdd £720 (the bond is in default of its promised payment, and the
bondholders take the entire assets of the firm).
Valuing the convertible bond under the assumption that the firm accepts the
lower-risk investment, if u occurs at t1:
£1606.60 £1026.50
0.25897
£2800 1.4 0.6
and:
1.4 £1026.50 0.6 £1606.60
£537.66
1 1.4 0.6 1 0.10
Therefore:
0.25897 £2800 £537.66 £1262.78
If d occurs during the first period, bond value will be (at t1):
£1026.50 £720
0.31927
£1200 1.4 0.6
and:
1.4 £720 0.6 £1026.50
£445.57
1 1.4 0.6 1 0.10
Therefore:
0.31927 £1200 £445.57 £828.69
Taking the two possible values of the convertible bond at t1 as the outcomes for
the bond at that time, we can now find its value at t0:
£1262.78 £828.69
0.27131
£2000 1.4 0.6
and:
1.4 £828.69 0.6 £1262.78
£457.38
1 1.4 0.6 1 0.10
Therefore:
0.27131 £2000 £457.38 £1000
The convertible bond has a value of £1000 at t0 if the firm adopts the lower-risk
investment.
Recall that the agency problem was that straight bondholders feared that the firm
would not take the lower-risk investment, but would instead adopt the higher-risk
investment so as to shift wealth from bondholders to shareholders. The lower panel
of Figure 12.13 shows (right-hand column) that the convertibility characteristic of
the bond changes its payoffs substantially relative to the straight bond when the
firm takes the higher-risk investment. Notice particularly that the upper limit of
payoff to the convertible bond increases to £2655.80 (bondholders here obviously
choose to convert to shareholders); this recognises that the convertibility provision
of the bond is like an insurance policy for bondholders. If the firm switches from
the lower risk to the riskier investment, the reduction in size of the minimum
62 Again, to save space we do not show the calculation for the high-risk bond, though it proceeds
identically to that of the low-risk.
63 Suppose that the low-risk investment had an NPV slightly higher than the high-risk investment. In that
situation, a straight bond issue would force the firm to choose the lower NPV investment, whereas the
convertible bond issue would allow the firm to choose the higher NPV investment. We did not design
this into the example because the numbers were complicated enough already.
Review Questions
12.1 One of the important variables influencing option value is the variance of return on the
underlying security or asset. As the variance of the underlying asset increases, a call
option on that asset becomes which of the following?
A. More valuable because in a call option higher underlying asset variance means
positive returns are accentuated and negative returns are dampened.
B. Less valuable because the call option is thereby more risky.
C. More valuable because Equation 12.7, Equation 12.8 and Equation 12.9 say so.
D. Less valuable because the chances of the option ever going ‘in the money’ have
decreased.
12.2 Another of the important variables influencing option value is the market interest rate.
As interest rates increase, which of the following does the value of the option become?
A. Lower, because the discount rates applied to the option’s future cash flows are
now higher.
B. Higher, because the present value of the amount of money necessary to
exercise the option is lower, and option value varies inversely with exercise
price.
C. Indeterminate, because although discount rates are higher, the present value of
exercise price is less.
D. None of the above.
12.3 The equity of a firm that has borrowed money is correctly considered a ‘call option’
because of which of the following?
A. The shareholders can repurchase the bonds if there is a call provision on them.
B. The shareholders can repurchase the assets of the firm by paying the interest
and principal on the bonds.
C. The bondholders can keep the assets if they wish.
D. The bondholders hold a put option to resell the assets of the firm to the
shareholders.
12.4 Using the binomial option model, calculate the value of a call option due to expire in
one period, with an exercise price of £22, underlying share price of £25, u = 1.2, d = 0.7
and rf = 10 per cent.
12.5 If a company has bonds outstanding, the bonds are almost certain to have contractual
provisions that restrict the ability of the company to undertake activities such as a
merger, dividend payments, entering a new line of business and others. The net effect of
these provisions is that they:
I. Probably hurt the value of the firm as a whole because they inhibit the ability of the
firm to pursue otherwise optimal operating and financial activities.
II. Are necessary so as to get the bondholders to agree to lend money to the firm, so
are a required part of having debt in the company’s capital structure.
III. Are good for the firm in that they hold managerial decision making to some kind of
an external, easily measurable standard.
Which of the following is correct?
A. Both I and II.
B. II only.
C. I, II and III.
D. I only.
12.6 The reason why convertible bonds exist in financial markets is which of the following?
A. They are a mechanism to merge the conflicting interests of shareholders and
bondholders.
B. They will increase shareholder wealth if share prices turn out to be lower than
what was expected at the time that the conversion characteristics of the bond
were priced by the market.
C. Interest rates are lower on convertible bonds than on non-convertible bonds.
D. When the company issues convertible bonds, with other provisions the same
as a non-convertible bond, the company takes in more money.
Statistical Tables
Table A1.1 Present value of £1 to be received at time point t
1
1
Period
t 1% 2% 3% 4% 5% 6% 7% 8% 9% 10% 11% 12%
1 .9901 .9804 .9709 .9615 .9524 .9434 .9346 .9259 .9174 .9091 .9009 .8929
2 .9803 .9612 .9426 .9246 .9070 .8900 .8734 .8573 .8417 .8264 .8116 .7972
3 .9706 .9423 .9151 .8890 .8638 .8396 .8163 .7938 .7722 .7513 .7312 .7118
4 .9610 .9238 .8885 .8548 .8227 .7921 .7629 .7350 .7084 .6830 .6587 .6355
5 .9515 .9057 .8626 .8219 .7835 .7473 .7130 .6806 .6499 .6209 .5935 .5674
6 .9420 .8880 .8375 .7903 .7462 .7050 .6663 .6302 .5963 .5645 .5346 .5066
7 .9327 .8706 .8131 .7599 .7107 .6651 .6227 .5835 .5470 .5132 .4817 .4523
8 .9235 .8535 .7894 .7307 .6768 .6274 .5820 .5403 .5019 .4665 .4339 .4039
9 .9143 .8368 .7664 .7026 .6446 .5919 .5439 .5002 .4604 .4241 .3909 .3606
10 .9053 .8203 .7441 .6756 .6139 .5584 .5083 .4632 .4224 .3855 .3522 .3220
11 .8963 .8043 .7224 .6496 .5847 .5268 .4751 .4289 .3875 .3505 .3173 .2875
12 .8874 .7885 .7014 .6246 .5568 .4970 .4440 .3971 .3555 .3186 .2858 .2567
13 .8787 .7730 .6810 .6006 .5303 .4688 .4150 .3677 .3262 .2897 .2575 .2292
14 .8700 .7579 .6611 .5775 .5051 .4423 .3878 .3405 .2992 .2633 .2320 .2046
15 .8613 .7430 .6419 .5553 .4810 .4173 .3624 .3152 .2745 .2394 .2090 .1827
16 .8528 .7284 .6232 .5339 .4581 .3936 .3387 .2919 .2519 .2176 .1883 .1631
17 .8444 .7142 .6050 .5134 .4363 .3714 .3166 .2703 .2311 .1978 .1696 .1456
18 .8360 .7002 .5874 .4936 .4155 .3503 .2959 .2502 .2120 .1799 .1528 .1300
19 .8277 .6864 .5703 .4746 .3957 .3305 .2765 .2317 .1945 .1635 .1377 .1161
20 .8195 .6730 .5537 .4564 .3769 .3118 .2584 .2145 .1784 .1486 .1240 .1037
21 .8114 .6598 .5375 .4388 .3589 .2942 .2415 .1987 .1637 .1351 .1117 .0926
22 .8034 .6468 .5219 .4220 .3418 .2775 .2257 .1839 .1502 .1228 .1007 .0826
23 .7954 .6342 .5067 .4057 .3256 .2618 .2109 .1703 .1378 .1117 .0907 .0738
24 .7876 .6217 .4919 .3901 .3101 .2470 .1971 .1577 .1264 .1015 .0817 .0659
25 .7798 .6095 .4776 .3751 .2953 .2330 .1842 .1460 .1160 .0923 .0736 .0588
26 .7720 .5976 .4637 .3607 .2812 .2198 .1722 .1352 .1064 .0839 .0663 .0525
27 .7644 .5859 .4502 .3468 .2678 .2074 .1609 .1252 .0976 .0763 .0597 .0469
28 .7568 .5744 .4371 .3335 .2551 .1956 .1504 .1159 .0895 .0693 .0538 .0419
29 .7493 .5631 .4243 .3207 .2429 .1846 .1406 .1073 .0822 .0630 .0485 .0374
30 .7419 .5521 .4120 .3083 .2314 .1741 .1314 .0994 .0754 .0573 .0437 .0334
35 .7059 .5000 .3554 .2534 .1813 .1301 .0937 .0676 .0490 .0356 .0259 .0189
40 .6717 .4529 .3066 .2083 .1420 .0972 .0668 .0460 .0318 .0221 .0154 .0107
45 .6391 .4102 .2644 .1712 .1113 .0727 .0476 .0313 .0207 .0137 .0091 .0061
50 .6080 .3715 .2281 .1407 .0872 .0543 .0339 .0213 .0134 .0085 .0054 .0035
13% 14% 15% 16% 17% 18% 19% 20% 25% 30% 35% 40% 50%
.8850 .8772 .8696 .8621 .8547 .8475 .8403 .8333 .8000 .7692 .7407 .7143 .6667
.7831 .7695 .7561 .7432 .7305 .7182 .7062 .6944 .6400 .5917 .5487 .5102 .4444
.6931 .6750 .6575 .6407 .6244 .6086 .5934 .5787 .5120 .4552 .4064 .3644 .2963
.6133 .5921 .5718 .5523 .5337 .5158 .4987 .4823 .4096 .3501 .3011 .2603 .1975
.5428 .5194 .4972 .4761 .4561 .4371 .4190 .4019 .3277 .2693 .2230 .1859 .1317
.4803 .4556 .4323 .4104 .3898 .3704 .3521 .3349 .2621 .2072 .1652 .1328 .0878
.4251 .3996 .3759 .3538 .3332 .3139 .2959 .2791 .2097 .1594 .1224 .0949 .0585
.3762 .3506 .3269 .3050 .2848 .2660 .2487 .2326 .1678 .1226 .0906 .0678 .0390
.3329 .3075 .2843 .2630 .2434 .2255 .2090 .1938 .1342 .0943 .0671 .0484 .0260
.2946 .2697 .2472 .2267 .2080 .1911 .1756 .1615 .1074 .0725 .0497 .0346 .0173
.2607 .2366 .2149 .1954 .1778 .1619 .1476 .1346 .0859 .0558 .0368 .0247 .0116
.2307 .2076 .1869 .1685 .1520 .1372 .1240 .1122 .0687 .0429 .0273 .0176 .0077
.2042 .1821 .1625 .1452 .1299 .1163 .1042 .0935 .0550 .0330 .0202 .0126 .0051
.1807 .1597 .1413 .1252 .1110 .0985 .0876 .0779 .0440 .0254 .0150 .0090 .0034
.1599 .1401 .1229 .1079 .0949 .0835 .0736 .0649 .0352 .0195 .0111 .0064 .0023
.1415 .1229 .1069 .0930 .0811 .0708 .0618 .0541 .0281 .0150 .0082 .0046 .0015
.1252 .1078 .0929 .0802 .0693 .0600 .0520 .0451 .0225 .0116 .0061 .0033 .0010
.1108 .0946 .0808 .0691 .0592 .0508 .0437 .0376 .0180 .0089 .0045 .0023 .0007
.0981 .0829 .0703 .0596 .0506 .0431 .0367 .0313 .0144 .0068 .0033 .0017 .0005
.0868 .0728 .0611 .0514 .0443 .0365 .0308 .0261 .0115 .0053 .0025 .0012 .0003
.0768 .0638 .0531 .0443 .0370 .0309 .0259 .0217 .0092 .0040 .0018 .0009 .0002
.0680 .0560 .0462 .0382 .0316 .0262 .0218 .0181 .0074 .0031 .0014 .0006 .0001
.0601 .0491 .0402 .0329 .0270 .0222 .0183 .0151 .0059 .0024 .0010 .0004 .0001
.0532 .0431 .0349 .0284 .0231 .0188 .0154 .0126 .0047 .0018 .0007 .0003 .0001
.0471 .0378 .0304 .0245 .0197 .0160 .0129 .0105 .0038 .0014 .0006 .0002 .0000
.0417 .0331 .0264 .0211 .0169 .0135 .0109 .0087 .0030 .0011 .0004 .0002 .0000
.0369 .0291 .0230 .0182 .0144 .0115 .0091 .0073 .0024 .0008 .0003 .0001 .0000
.0326 .0255 .0200 .0157 .0123 .0097 .0077 .0061 .0019 .0006 .0002 .0001 .0000
.0289 .0224 .0174 .0135 .0105 .0082 .0064 .0051 .0015 .0005 .0002 .0001 .0000
.0256 .0196 .0151 .0116 .0090 .0070 .0054 .0042 .0012 .0004 .0001 .0000 .0000
.0139 .0102 .0075 .0055 .0041 .0030 .0023 .0017 .0004 .0001 .0000 .0000 .0000
.0075 .0053 .0037 .0026 .0019 .0013 .0010 .0007 .0001 .0000 .0000 .0000 .0000
.0041 .0027 .0019 .0013 .0009 .0006 .0004 .0003 .0000 .0000 .0000 .0000 .0000
.0022 .0014 .0009 .0006 .0004 .0003 .0002 .0001 .0000 .0000 .0000 .0000 .0000
Period
t 1% 2% 3% 4% 5% 6% 7% 8% 9% 10% 11% 12%
1 0.9901 0.9804 0.9709 0.9615 0.9524 0.9434 0.9346 0.9259 0.9174 0.9091 0.9009 0.8929
2 1.9704 1.9416 1.9135 1.8861 1.8594 1.8334 1.8080 1.7833 1.7591 1.7355 1.7125 1.6901
3 2.9410 2.8839 2.8286 2.7751 2.7232 2.6730 2.6243 2.5771 2.5313 2.4869 2.4437 2.4018
4 3.9020 3.8077 3.7171 3.6299 3.5460 3.4651 3.3872 3.3121 3.2397 3.1699 3.1024 3.0373
5 4.8534 4.7135 4.5797 4.4518 4.3295 4.2124 4.1002 3.9927 3.8897 3.7908 3.6959 3.6048
6 5.7955 5.6014 5.4172 5.2421 5.0757 4.9173 4.7665 4.6229 4.4859 4.3553 4.2305 4.1114
7 6.7282 6.4720 6.2303 6.0021 5.7864 5.5824 5.3893 5.2064 5.0330 4.8684 4.7122 4.5638
8 7.6517 7.3255 7.0197 6.7327 6.4632 6.2098 5.9713 5.7466 5.5348 5.3349 5.1461 4.9676
9 8.5660 8.1622 7.7861 7.4353 7.1078 6.8017 6.5152 6.2469 5.9952 5.7590 5.5370 5.3282
10 9.4713 8.9826 8.5302 8.1109 7.7217 7.3601 7.0236 6.7101 6.4177 6.1446 5.8892 5.6502
11 10.368 9.7868 9.2526 8.7605 8.3064 7.8869 7.4987 7.1390 6.8052 6.4951 6.2065 5.9377
12 11.255 10.575 9.9540 9.3851 8.8633 8.3838 7.9427 7.5361 7.1607 6.8137 6.4924 6.1944
13 12.134 11.348 10.635 9.9856 9.3936 8.8527 8.3577 7.9038 7.4869 7.1034 6.7499 6.4235
14 13.004 12.106 11.296 10.563 9.8986 9.2950 8.7455 8.2442 7.7862 7.3667 6.9819 6.6282
15 13.865 12.849 11.938 11.118 10.380 9.7122 9.1079 8.5595 8.0607 7.6061 7.1909 6.8109
16 14.718 13.578 12.561 11.652 10.838 10.106 9.4466 8.8514 8.3126 7.8237 7.3792 6.9740
17 15.562 14.292 13.166 12.166 11.274 10.477 9.7632 9.1216 8.5436 8.0216 7.5488 7.1196
18 16.398 14.992 13.754 12.659 11.690 10.828 10.059 9.3719 8.7556 8.2014 7.7016 7.2497
19 17.226 15.678 14.324 13.134 12.085 11.158 10.336 9.6036 8.9501 8.3649 7.8393 7.3658
20 18.046 16.351 14.877 13.590 12.462 11.470 10.594 9.8181 9.1285 8.5136 7.9633 7.4694
21 18.857 17.011 15.415 14.029 12.821 11.764 10.836 10.017 9.2922 8.6487 8.0751 7.5620
22 19.660 17.658 15.937 14.451 13.163 12.042 11.061 10.201 9.4424 8.7715 8.1757 7.6446
23 20.456 18.292 16.444 14.857 13.489 12.303 11.272 10.371 9.5802 8.8832 8.2664 7.7184
24 21.243 18.914 16.936 15.247 13.799 12.550 11.469 10.529 9.7066 8.9847 8.3481 7.7843
25 22.023 19.523 17.413 15.622 14.094 12.783 11.654 10.675 9.8226 9.0770 8.4217 7.8431
26 22.795 20.121 17.887 15.983 14.375 13.003 11.826 10.810 9.9290 9.1609 8.4881 7.8957
27 23.560 20.707 18.327 16.330 14.643 13.211 11.987 10.935 10.027 9.2372 8.5478 7.9426
28 24.316 21.281 18.764 16.663 14.898 13.406 12.137 11.051 10.116 9.3066 8.6016 7.9844
29 25.066 21.844 19.188 16.984 15.141 13.591 12.278 11.158 10.198 9.3696 8.6501 8.0218
30 25.808 22.396 19.600 17.292 15.372 13.765 12.409 11.258 10.274 9.4269 8.6938 8.0552
35 29.409 24.999 21.487 18.665 16.374 14.498 12.948 11.655 10.567 9.6442 8.8552 8.1755
40 32.835 27.355 23.115 19.793 17.159 15.046 13.332 11.925 10.757 9.7791 8.9511 8.2438
45 36.095 29.490 24.519 20.720 17.774 15.456 13.606 12.108 10.881 9.8628 9.0079 8.2825
50 39.196 31.424 25.730 21.482 18.256 15.762 13.801 12.233 10.962 9.9148 9.0417 8.3045
13% 14% 15% 16% 17% 18% 19% 20% 25% 30% 35% 40% 50%
0.8850 0.8772 0.8696 0.8621 0.8547 0.8475 0.8403 0.8333 0.8000 0.7692 0.7407 0.7143 0.6667
1.6681 1.6467 1.6257 1.6052 1.5852 1.5656 1.5465 1.5278 1.4400 1.3609 1.2894 1.2245 1.1111
2.3612 2.3216 2.2832 2.2459 2.2096 2.1743 2.1399 2.1065 1.9520 1.8161 1.6959 1.5889 1.4074
2.9745 2.9137 2.8550 2.7982 2.7432 2.6901 2.6386 2.5887 2.3616 2.1662 1.9969 1.8492 1.6049
3.5172 3.4331 3.3522 3.2743 3.1993 3.1272 3.0576 2.9906 2.6893 2.4356 2.2200 2.0352 1.7366
3.9975 3.8887 3.7845 3.6847 3.5892 3.4976 3.4098 3.3255 2.9514 2.6427 2.3852 2.1680 1.8244
4.4226 4.2883 4.1604 4.0386 3.9224 3.8115 3.7057 3.6046 3.1611 2.8021 2.5075 2.2628 1.8829
4.7988 4.6389 4.4873 4.3436 4.2072 4.0776 3.9544 3.8372 3.3289 2.9247 2.5982 2.3306 1.9220
5.1317 4.9464 4.7716 4.6065 4.4506 4.3030 4.1633 4.0310 3.4631 3.0190 2.6653 2.3790 1.9480
5.4262 5.2161 5.0188 4.8332 4.6586 4.4941 4.3389 4.1925 3.5705 3.0915 2.7150 2.4136 1.9653
5.6869 5.4527 5.2337 5.0286 4.8364 4.6560 4.4865 4.3271 3.6564 3.1473 2.7519 2.4383 1.9769
5.9176 5.6603 5.4206 5.1971 4.9884 4.7932 4.6105 4.4392 3.7251 3.1903 2.7792 2.4559 1.9846
6.1218 5.8424 5.5831 5.3423 5.1183 4.9095 4.7147 4.5327 3.7801 3.2233 2.7994 2.4685 1.9897
6.3025 6.0021 5.7245 5.4675 5.2293 5.0081 4.8023 4.6106 3.8241 3.2487 2.8144 2.4775 1.9931
6.4624 6.1422 5.8474 5.5755 5.3242 5.0916 4.8759 4.6755 3.8593 3.2682 2.8255 2.4839 1.9954
6.6039 6.2651 5.9542 5.6685 5.4053 5.1624 4.9377 4.7296 3.8874 3.2832 2.8337 2.4885 1.9970
6.7291 6.3729 6.0472 5.7487 5.4746 5.2223 4.9897 4.7746 3.9099 3.2948 2.8398 2.4918 1.9980
6.8399 6.4674 6.1280 5.8178 5.5339 5.2732 5.0333 4.8122 3.9279 3.3037 2.8443 2.4941 1.9986
6.9380 6.5504 6.1982 5.8775 5.5845 5.3162 5.0700 4.8435 3.9424 3.3105 2.8476 2.4958 1.9991
7.0248 6.6231 6.2593 5.9288 5.6278 5.3527 5.1009 4.8696 3.9539 3.3158 2.8501 2.4970 1.9994
7.1016 6.6870 6.3125 5.9731 5.6648 5.3837 5.1268 4.8913 3.9631 3.3198 2.8519 2.4979 1.9996
7.1695 6.7429 6.3587 6.0113 5.6964 5.4099 5.1486 4.9094 3.9705 3.3230 2.8533 2.4985 1.9997
7.2297 6.7921 6.3988 6.0442 5.7234 5.4321 5.1668 4.9245 3.9764 3.3254 2.8543 2.4989 1.9998
7.2829 6.8351 6.4338 6.0726 5.7465 5.4509 5.1822 4.9371 3.9811 3.3272 2.8550 2.4992 1.9999
7.3300 6.8729 6.4641 6.0971 5.7662 5.4669 5.1951 4.9476 3.9849 3.3286 2.8556 2.4994 1.9999
7.3717 6.9061 6.4906 6.1182 5.7831 5.4804 5.2060 4.9563 3.9879 3.3297 2.8560 2.4996 1.9999
7.4086 6.9352 6.5135 6.1364 5.7975 5.4919 5.2151 4.9636 3.9903 3.3305 2.8563 2.4997 2.0000
7.4412 6.9607 6.5335 6.1520 5.8099 5.5016 5.2228 4.9697 3.9923 3.3312 2.8565 2.4998 2.0000
7.4701 6.9830 6.5509 6.1656 5.8204 5.5098 5.2292 4.9747 3.9938 3.3317 2.8567 2.4999 2.0000
7.4957 7.0027 6.5660 6.1772 5.8294 5.5168 5.2347 4.9789 3.9950 3.3321 2.8568 2.4999 2.0000
7.5856 7.0700 6.6166 6.2153 5.8582 5.5386 5.2512 4.9915 3.9964 3.3330 2.8571 2.5000 2.0000
7.6344 7.1050 6.6418 6.2335 5.8713 5.5482 5.2582 4.9966 3.9995 3.3332 2.8571 2.5000 2.0000
7.6609 7.1232 6.6543 6.2421 5.8773 5.5523 5.2611 4.9986 3.9998 3.3333 2.8571 2.5000 2.0000
7.6752 7.1327 6.6605 6.2463 5.8801 5.5541 5.2623 4.9995 3.9999 3.3333 2.8571 2.5000 2.0000
13% 14% 15% 16% 17% 18% 19% 20% 25% 30% 35% 40% 50%
1.1300 1.1400 1.1500 1.1600 1.1700 1.1800 1.1900 1.2000 1.2500 1.3000 1.3500 1.4000 1.5000
1.2769 1.2996 1.3225 1.3456 1.3689 1.3924 1.4161 1.4400 1.5625 1.6900 1.8225 1.9600 2.2500
1.4429 1.4815 1.5209 1.5609 1.6016 1.6430 1.6852 1.7280 1.9531 2.1970 2.4604 2.7440 3.3750
1.6305 1.6890 1.7490 1.8106 1.8739 1.9388 2.0053 2.0736 2.4414 2.8561 3.3215 3.8416 5.0625
1.8424 1.9254 2.0114 2.1003 2.1924 2.2878 2.3864 2.4883 3.0518 3.7129 4.4840 5.3782 7.5938
2.0820 2.1950 2.3131 2.4364 2.5652 2.6996 2.8398 2.9860 3.8147 4.8268 6.0534 7.5295 11.391
2.3526 2.5023 2.6600 2.8262 3.0012 3.1855 3.3793 3.5832 4.7684 6.2749 8.1722 10.541 17.086
2.6584 2.8526 3.0590 3.2784 3.5115 3.7589 4.0214 4.2998 5.9605 8.1573 11.032 14.758 25.629
3.0040 3.2519 3.5179 3.8030 4.1084 4.4355 5.7854 5.1598 7.4506 10.604 14.894 20.661 38.443
3.3946 3.7072 4.0456 4.4114 4.8086 5.2338 5.6947 6.1917 9.3132 13.786 20.107 28.925 57.665
3.8359 4.2262 4.6524 5.1173 5.6240 6.1759 6.7767 7.4301 11.642 17.922 27.144 40.496 86.498
4.3345 4.8179 5.3503 5.9360 6.5801 7.2876 8.0642 8.9161 14.552 23.298 36.644 56.694 129.75
4.8980 5.4924 6.1528 6.8858 7.6987 8.5994 9.5964 10.699 18.190 30.288 49.470 79.371 194.62
5.5348 6.2613 7.0757 7.9875 9.0075 10.147 11.420 12.839 22.737 39.374 66.784 111.12 291.93
6.2543 7.1379 8.1371 9.2655 10.539 11.974 13.590 15.407 28.422 51.186 90.158 155.57 437.89
7.0673 8.1372 9.3576 10.748 12.330 14.129 16.172 18.488 35.527 66.542 121.71 217.80 656.84
7.9861 9.2765 10.761 12.468 14.426 16.672 19.244 22.186 44.409 86.504 164.31 304.91 985.26
9.0243 10.575 12.375 14.463 16.879 19.673 22.901 26.623 55.511 112.46 221.82 426.88 1477.9
10.197 12.056 14.232 16.777 19.748 23.214 27.252 31.948 69.389 146.19 299.46 597.63 2216.8
11.523 13.743 16.367 19.461 23.106 27.393 32.429 38.338 86.736 190.05 404.27 836.68 3325.3
13.021 15.668 18.822 22.574 27.034 32.324 38.591 46.005 108.42 247.06 545.77 1171.4 4987.9
14.714 17.861 21.645 26.186 31.629 38.142 45.923 55.206 135.53 321.18 736.79 1639.9 7481.8
16.627 20.362 24.891 30.376 37.006 45.008 54.649 66.247 169.41 417.54 994.66 2295.9 11223.
18.788 23.212 28.625 35.236 43.297 53.109 65.032 79.497 211.76 542.80 1342.8 3214.2 16834.
21.231 26.462 32.919 40.874 50.658 62.669 77.388 95.396 264.70 705.64 1812.8 4499.9 25251.
23.991 30.167 37.857 47.414 59.270 73.949 92.092 114.48 330.87 917.33 2447.2 6299.8 37877.
27.109 34.390 43.535 55.000 69.345 87.260 109.59 137.37 413.59 1192.5 3308.8 8819.8 56815.
30.633 39.204 50.066 63.800 81.134 102.97 130.41 164.84 516.99 1550.3 4460.1 12348. 85223.
34.616 44.693 57.575 74.009 94.927 121.50 155.19 197.81 646.23 2015.4 6021.1 17287. ∗
39.116 50.950 66.212 85.850 111.06 143.37 184.68 237.38 807.79 2620.0 8128.5 24201. ∗
72.069 98.100 133.18 180.31 243.50 328.00 440.70 590.67 2465.2 9727.9 36449 ∗ ∗
132.78 188.88 267.86 378.72 533.87 750.38 1051.7 1469.8 7523.2 36119. ∗ ∗ ∗
244.64 363.68 538.77 795.44 1170.5 1716.7 2509.7 3657.3 22959. ∗ ∗ ∗ ∗
450.74 700.23 1083.7 1670.7 2566.2 3927.4 5988.9 9100.4 70065. ∗ ∗ ∗ ∗
∗ Interest factors exceed 99999.
Table A1.4 £
1 1
Period
t 1% 2% 3% 4% 5% 6% 7% 8% 9% 10% 11% 12%
1 1.0000 1.0000 1.0000 1.0000 1.0000 1.0000 1.0000 1.0000 1.0000 1.0000 1.0000 1.0000
2 2.0100 2.0200 2.0300 2.0400 2.0500 2.0600 2.0700 2.0800 2.0900 2.1000 2.1100 2.1200
3 3.0301 3.0604 3.0909 3.1216 3.1525 3.1836 3.2149 3.2464 3.2781 3.3100 3.3421 3.3744
4 4.0604 4.1216 4.1836 4.2465 4.3101 4.3746 4.4399 4.5061 4.5731 4.6410 4.7097 4.7793
5 5.1010 5.2040 5.3091 5.4163 5.5256 5.6371 5.7507 5.8666 5.9847 6.1051 6.2278 6.3528
6 6.1520 6.3081 6.4684 6.6330 6.8019 6.9753 7.1533 7.3359 7.5233 7.7156 7.9129 8.1152
7 7.2135 7.4343 7.6625 7.8983 8.1420 8.3938 8.6540 8.9228 9.2004 9.4872 9.7833 10.089
8 8.2857 8.5830 8.8923 9.2142 9.5491 9.8975 10.260 10.637 11.028 11.436 11.859 12.300
9 9.3685 9.7546 10.159 10.583 11.027 11.491 11.978 12.488 13.021 13.579 14.164 14.776
10 10.462 10.950 11.464 12.006 12.578 13.181 13.816 14.487 15.193 15.937 16.722 17.549
11 11.567 12.169 12.808 13.486 14.207 14.972 15.784 16.645 17.560 18.531 19.561 20.655
12 12.683 13.412 14.192 15.026 15.917 16.870 17.888 18.977 20.141 21.384 22.713 24.133
13 13.809 14.680 15.618 16.627 17.713 18.882 20.141 21.495 22.953 24.523 26.212 28.029
14 14.947 15.974 17.086 18.292 19.599 21.015 22.550 24.215 26.019 27.975 30.095 32.393
15 16.097 17.293 18.599 20.024 21.579 23.276 25.129 27.152 29.361 31.772 34.405 37.280
16 17.258 18.639 20.157 21.825 23.657 25.673 27.888 30.324 33.003 35.950 39.190 42.753
17 18.430 20.012 21.762 23.698 25.840 28.213 30.840 33.750 36.974 40.545 44.501 48.884
18 19.615 21.412 23.414 25.645 28.132 30.906 33.999 37.450 41.301 45.599 50.396 55.750
19 20.811 22.841 25.117 27.671 30.539 33.760 37.379 41.446 46.018 51.159 56.939 63.440
20 22.019 24.297 26.870 29.778 33.066 36.786 40.995 45.762 51.160 57.275 64.203 72.052
21 23.239 25.783 28.676 31.969 35.719 39.993 44.865 50.423 56.765 64.002 72.265 81.699
22 24.472 27.299 30.537 34.248 38.505 43.392 49.006 55.457 62.873 71.403 81.214 92.503
23 25.716 28.845 32.453 36.618 41.430 46.996 53.436 60.893 69.532 79.543 91.148 104.60
24 26.973 30.422 34.426 39.083 44.502 50.816 58.177 66.765 76.790 88.497 102.17 118.16
25 28.243 32.030 36.459 41.646 47.727 54.865 63.249 73.106 84.701 98.347 114.41 133.33
26 29.526 33.671 38.553 44.312 51.113 59.156 68.676 79.954 93.324 109.18 128.00 150.33
27 30.821 35.344 40.710 47.084 54.669 63.706 74.484 87.351 102.72 121.10 143.08 169.37
28 32.129 37.051 42.931 49.968 58.403 68.528 80.698 95.339 112.97 134.21 159.82 190.70
29 33.450 38.792 45.219 52.966 62.323 73.640 87.347 103.97 124.14 148.63 178.40 214.58
30 34.785 40.568 47.575 56.085 66.439 79.058 94.461 113.28 136.31 164.49 199.02 241.33
35 41.660 49.994 60.462 73.652 90.320 111.43 138.24 172.32 215.71 271.02 341.59 431.66
40 48.886 60.402 75.401 95.026 120.80 154.76 199.64 259.06 337.88 442.59 581.83 767.09
45 56.481 71.893 92.720 121.03 159.70 212.74 285.75 386.51 525.86 718.90 986.64 1358.2
50 64.463 84.579 112.80 152.67 209.35 290.34 406.53 573.77 815.08 1163.9 1668.8 2400.0
13% 14% 15% 16% 17% 18% 19% 20% 25% 30% 35% 40% 50%
1.0000 1.0000 1.0000 1.0000 1.0000 1.0000 1.0000 1.0000 1.0000 1.0000 1.0000 1.0000 1.0000
2.1300 2.1400 2.1500 2.1600 2.1700 2.1800 2.1900 2.2000 2.2500 2.3000 2.3500 2.4000 2.5000
3.4069 3.4396 3.4725 3.5056 3.5389 3.5724 3.6061 3.6400 3.8125 3.9900 4.1725 4.3600 4.7500
4.8498 4.9211 4.9934 5.0665 5.1405 5.2154 5.2913 5.3680 5.7656 6.1870 6.6329 7.1040 8.1250
6.4803 6.6101 6.7424 6.8771 7.0144 7.1542 7.2966 7.4416 8.2070 9.0431 9.9544 10.946 13.188
8.3227 8.5355 8.7537 8.9775 9.2068 9.4420 9.6830 9.9299 11.259 12.756 14.438 16.324 20.781
10.405 10.730 11.067 11.414 11.772 12.142 12.523 12.916 15.073 17.583 20.492 23.853 32.172
12.757 13.233 13.727 14.240 14.773 15.327 15.902 16.499 19.842 23.858 28.664 34.395 49.258
15.416 16.085 16.786 17.519 18.285 19.086 19.923 20.799 25.802 32.015 39.696 49.153 74.887
18.420 19.337 20.304 21.321 22.393 23.521 24.709 25.959 33.253 42.619 54.590 69.814 113.33
21.814 23.045 24.349 25.733 27.200 28.755 30.404 32.150 42.566 56.405 74.697 98.739 171.00
25.650 27.271 29.002 30.850 32.824 34.931 37.180 39.581 54.208 74.327 101.84 139.23 257.49
29.985 32.089 34.352 36.786 39.404 42.219 45.244 48.497 68.760 97.625 138.48 195.93 387.24
34.883 37.581 40.505 43.672 47.103 50.818 54.841 59.196 86.949 127.91 187.95 275.30 581.86
40.417 43.842 47.580 51.660 56.110 60.965 66.261 72.035 109.69 167.29 254.74 386.42 873.79
46.672 50.980 55.717 60.925 66.649 72.939 79.850 87.442 138.11 218.47 344.90 541.99 1311.7
53.739 59.118 65.075 71.673 78.979 87.068 96.022 105.93 173.64 285.01 466.61 759.78 1968.5
61.725 68.394 75.836 84.141 93.406 103.74 115.27 128.12 218.04 371.52 630.92 1064.7 2953.8
70.749 78.969 88.212 98.603 110.28 123.41 138.17 154.74 273.56 483.97 852.75 1491.6 4431.7
80.947 91.025 102.44 115.38 130.03 146.63 165.42 186.69 342.94 630.17 1152.2 2089.2 6648.5
92.470 104.77 118.81 134.84 153.14 174.02 197.85 225.03 429.68 820.22 1556.5 2925.9 9973.8
105.49 120.44 137.63 157.41 180.17 206.34 236.44 271.03 538.10 1067.3 2102.3 4097.2 14962.
120.20 138.30 159.28 183.60 211.80 244.49 282.36 326.24 673.63 1388.5 2839.0 5737.1 22443.
136.83 158.66 184.17 213.98 248.81 289.49 337.01 392.48 843.03 1806.0 3833.7 8033.0 33666.
155.62 181.87 212.79 249.21 292.10 342.60 402.04 471.98 1054.8 2348.8 5176.5 11247. 50500.
176.85 208.33 245.71 290.09 342.76 405.27 479.43 567.38 1319.5 3054.4 6989.3 15757. 75752.
200.84 238.50 283.57 337.50 402.03 479.22 571.52 681.85 1650.4 3971.8 9436.5 22057. *
227.95 272.89 327.10 392.50 471.38 566.48 681.11 819.22 2064.0 5164.3 12740. 30867. *
258.58 312.09 377.17 456.30 552.51 669.45 811.52 984.07 2580.9 6714.6 17200. 43214. *
293.20 356.79 434.75 530.31 647.44 790.95 966.71 1181.9 3227.2 8730.0 23222. 60501. *
546.68 693.57 881.17 1120.7 1426.5 1816.7 2314.2 2948.3 9856.8 32423. * * *
1013.7 1342.0 1779.1 2360.8 3134.5 4163.2 5529.8 7343.9 30089. * * * *
1874.2 2590.6 3585.1 4965.3 6879.3 9531.6 13203. 18281. 91831. * * * *
3459.5 4994.5 7217.7 10436. 15090. 21813. 31515. 45497. * * * * *
* Interest factors exceed 99999.
Inter-period compounding
1 i/m mt – 1
YTM/IRR (Interpolation)
YTM 1 2 1
Standard deviation
Beta coefficient
Standard deviation of return Correlation of with market
Beta
Standard deviation of market return
This can be expressed as:
Ungeared beta
E D
V V
Revenue-adjusted beta
Project revenue volatility
Revenue‐ adjusted
Company revenue volatility
Certainty equivalent
E
Covariance CF,
Variance
Total company value
FCF ∗
V ∗
Binomial option
1
Black–Scholes
where:
½ ½
ln / rfT / 0.5
and:
½
Put–call parity
Put Underlying Asset Call Riskless PV of Exercise Price
This section contains two practice final examinations, each of which is indicative of
the type and level of material that appears in the Heriot-Watt University final MBA
degree examination in Finance.
The duration of the examination is 3 hours. The marks value of each section is
shown. Within the total time of 3 hours, students may allocate their time among
sections as they see fit. The pass mark is 50%.
There is no choice in the selection of questions to be answered.
In the multiple choice questions, there are no marks deducted for wrong an-
swers.
For each question, a solution is provided, which will allow students to assess
their performance. The examination serves two purposes: to test understanding of
the course and to provide information on standards required to pass the university
final degree examination.
The rationale for providing two examinations is that students who have worked
through the course, have taken the first practice examination and, on the basis of
their performance in that examination, are not satisfied that they have attained
mastery of the material, will be able to study the course again and have a second
opportunity to test themselves. Where the first examination is satisfactory, the
second may be used for additional practice.
1 The financial market asks and pays interest of 9 per cent per year, and you have
currently the expectation of receiving £2000 per year, paid now and at the end of the
next two years (three cash flows). What is the maximum amount of money you can
spend from this source (including that from transacting with the financial market) at the
end of the first year?
A. £2180.
B. £5985.
C. £6000.
D. £6015.
2 You expect to receive £1000 per annum for 50 years and wish to value that expectation
at a 10 per cent discount rate, but your present-value tables do not have enough
periods and your pocket calculator cannot exponentiate. So you are facing a quite
tedious calculation to find the exact value. By how much would you err in using a simple
perpetuity valuation?
A. £10 000/ 1.10
B. A huge amount, since an infinite amount of money will be ignored.
C. £1000/ 1.10
D. 50 £1000/1.10
4 What do free cash flows of an investment project include when they are appropriate to
be discounted with the project’s WACC?
A. All expected payments between the firm and its capital suppliers.
B. All changes in the payments between the firm and its capital suppliers induced
by the project.
C. All changes in the payments between the firm and its capital suppliers induced
by the project, were the project to be financed only with equity.
D. All changes in the payments between the firm and its capital suppliers induced
by the project, were the project to be financed only with equity, but incre-
mented with the expected interest tax shields.
7 IRR is more likely to give incorrect indications of investments ranking among a group of
mutually exclusive investments when the investments
I. are of significantly different magnitude.
II. are of significantly different duration.
III. have very different NPVs.
Which of the following is correct?
A. I only.
B. II only.
C. I and II.
D. I, II and III.
9 Suppose that your firm is considering two alternative roofing materials for a new plant it
is building. The plant is expected to last indefinitely, but the roofing material will of
course require occasional refurbishment. Fibreglass shingles will last an expected 25
years, whereas slate’s expected lifetime is 32 years. Fibreglass costs £130 000 to install,
and has annual maintenance costs incurred at year ends that is expected to average
£11 000, whereas slate costs £216 000 and has expected annual maintenance costs of
only £2000. Because of differing underlayment structures, once the company decides on
a roofing material, it must stick with it or else incur additional large costs of renovation.
If a 10% discount is appropriate, what should the company do? (Hint: You may be forced
to calculate a numerical result from the formula at the top of page A1/3, since Table 2
does not have a specific listing for 32 years, or you may be fortunate enough to have a
calculator or spreadsheet that has that formula built in.)
A. Choose neither because they both have negative NPVs.
B. Choose slate because its equivalent annual cost is less than £25 000, compared
with fibreglass’s in excess of £25 000.
C. Choose fibreglass because the present value of its costs is less than £230 000
compared with slate’s in excess of £230 000.
D. Choose fibreglass because its equivalent annual cost is less than £25 000
compared with slate’s in excess of £25 000.
10 The reason why a portfolio’s risk cannot be reduced to zero by diversification is that
I. mathematical techniques of combining securities into optimal portfolios are not well
enough understood.
II. there is a common influence among most investments that causes their returns to
be positively correlated
III. many investments are difficult to discover, especially the ones with negative
correlations to others.
Which of the following is correct?
A. II only.
B. II and III only.
C. III only.
D. I, II and III.
11 Investment A has an expected return of 14% and a standard deviation of 5%, while
investment B has an expected return of 10% and a standard deviation of 3%. Which of
the following is true if you were to put half of your wealth into each investment?
A. The risk of the portfolio will be 4% and the expected return 12%.
B. The risk of the portfolio will be at least 4% and the expected return will be
12%.
C. The risk of the portfolio will be no greater than 4% and the expected return
12%.
D. The question does not provide enough information to make any of the
comments about risk in A, B or C.
12 The β coefficient you have decided is appropriate for your firm’s equity investment in a
new manufacturing facility for fertiliser is 1.16. If the risk-free rate is 9% and the excess
of the market’s expected return above the risk-free rate is 8%, what return will you
require on the equity investment?
A. Approximately 17.0%.
B. Approximately 17.4%.
C. Approximately 18.0%.
D. Approximately 18.3%.
16 Debt in a company’s capital structure may alter the company’s cost of capital and
shareholder wealth, relative to what these would have been without debt. Among the
sources of such alterations are
I. equity’s increased risk due to debt’s senior claim on company cash flows.
II. debt’s capital cost being less than equity’s due to debt’s senior claim.
III. interest deductibility.
Which of the following is correct?
A. I only.
B. III only.
C. I and II only.
D. II and III only.
17 Suppose that the only tax in an economy was company income tax, and that there were
no other ‘frictions’ (such as transactions costs, bankruptcy costs, or other agency costs).
Which of the following is correct?
Optimal capital structure for companies in this economy would likely be
A. very little debt, since shareholders would not be paying personal income taxes.
B. a reasonable balance of debt and equity, such that in difficult economic times
the odds of defaulting on interest and principal promises was acceptable.
C. as much debt as they could issue with deductible interest.
D. zero debt because interest would not be tax deductible.
18 Suppose that Intelligent Micro Designs plc (IMD) is seeking your advice about financing a
proposed research and development project. IMD’s business is designing computer
motherboards, accomplished through a very talented group of design engineers. IMD
can raise either equity or debt capital. IMD currently has a very low proportion of debt
in its capital structure, and has shown an impressive history of earnings increases.
Interest and principal payments on the company’s proposed borrowings seem to be
modest relative to the company’s net cash flows. What would your recommendation
be?
A. That IMD should issue equity rather than debt because of very high bankruptcy
or financial-distress costs.
B. That IMD should issue debt because it has not exploited sufficiently debt’s tax
benefits.
C. That IMD can issue either debt or equity without differentially affecting its
shareholder wealth.
D. That IMD should issue a complex debt instrument like a convertible bond.
19 What is the primary trade-off that occurs in managing Accounts Receivable (Debtors)?
A. In order to get creditworthy customers, you must give them especially good
credit terms.
B. Good customers sometimes take longer to pay than is best for the company.
C. The costs of finding out who will be good customers can be more than the
money saved by avoiding poor credit risks.
D. Increasing revenues by reducing credit standards versus having secure receiva-
bles with lower sales revenues.
20 Inventories
I. are costly to hold, but allow revenues to be higher.
II. should be kept at the absolute minimum that allows the firm to operate.
III. can be made cheaper to hold if the firm reorders less often.
Which of the following is correct?
A. I only.
B. II only.
C. I and III only.
D. II and III only.
21 Suppose that the current one-year interest rate in the US is 9.4% and the UK rate is
11%. Further, suppose that the spot exchange rate of dollars for pounds is 1.5 $/£.
What should your prediction of the one-year forward rate for $/£ be?
A. 1.351
B. 1.50
C. 1.478
D. 1.665
22 Your firm is building a plant and beginning to sell on credit terms in South Aerok, and is
concerned with the potential risks arising from the indications of growing inflation in
that country. Your suggestion about hedging this risk should be
I. hedge the receivables.
II. hedge the plant.
Which of the following is correct?
A. I only.
B. II only.
C. Both I and II.
D. Neither I nor II.
23 A call option has a striking price of £4.50 and the shares upon which it is based are
selling for £4.65. There are several months until the option expires. What will the
premium of the call option be?
A. Zero, because share price is above option value and exercise would simply
produce the ‘in the money’ gain of £0.15.
B. Negative, because ‘in the money’ options may go ‘out of the money’ between
now and expiration.
C. Positive, because ‘in the money’ options stand to benefit more than they lose
from price volatility.
D. Positive, negative or zero depending upon the values that appear for the
parameters of the Black–Scholes option-valuation formula.
Case Study 1
You are given the following information about the market for risk-free government
bonds (all of which pay interest at the end of the year):
There is a 9% coupon bond maturing in two years (it has remaining payments at the
end of this and the next year), and it is currently selling for £1017.07.
There is a three-year 10% coupon bond the price of which you do not know, but
the coupon from the first interest payment (the end of this year) is selling in the
market for £91.74.
The three-year spot interest rate is 7%.
There is a four-year 11% coupon bond currently selling for £1164.24.
Please answer the following questions about the bond market (and in answering, you
should round to the nearest £0.01 and to the nearest 0.01%):
6 Suppose that you owned the four-year 11% coupon bond and you were approached to
sell the third coupon payment (the £110 that is expected for the end of year 3). What
price would you agree to sell this coupon for two years from now?
(4 marks)
7 Suppose that there is another four-year risk-free bond in the market, which promises to
pay as follows:
8 Calculate the Yield to Maturity for both four-year bonds, and explain in either case the
reason why the bonds’ YTMs are such as they are.
(6 marks)
TOTAL = 30 marks
Case Study 2
You are considering ‘going short’ (issuing, or ‘writing’) a call option on one share of UK
Aero plc. The call option has an exercise price of £18, and one share of UK Aero is
currently selling for £17. Your opinion is that at the end of the single period until option
expiration, only two UK Aero share prices have any probability of occurring – either
£13.60 (probability of 0.35) or £22.10 (probability of 0.65). The risk-free market
interest rate is 10%, and all other market conditions are as specified in the section in
Module 12 on the binomial model of option valuation.
1 How much money would you collect for issuing the option right now? Construct an
explanation to an intelligent but uninitiated student as to why the market is willing to
give you money for issuing an option that lets the holder buy something for £18 that is
currently selling for £17.
(8 marks)
2 Suppose that you were in fact seeking the risk–return characteristic exhibited by the
issuance of this call option, but the option was not available. Illustrate the transaction(s)
you could undertake so as to produce the same risk and return exposure as you would
have derived from issuing the option.
(7 marks)
3 You are interest in another share, this time in Okri plc. The share price is 190p (pence)
and you want to sell a call option on the call options with an exercise price of 200p.
The option has three months to expiry and currently is priced at 25p. Draw the payoff
diagram for the seller of a call option, with the axes labelled and the option and share
values shown. What happens if the share price is 220p at expiry?
(5 marks)
TOTAL = 20 marks
1 A bank pays an interest rate of 15%. How much must be placed in an account at the
bank on 1.1.20 if the following amounts are to be withdrawn from the account and the
balance after the last withdrawal is to be zero?
A. £7230.
B. £8314.
C. £11 511.
D. £13 238.
‘Make seven annual payments of £1000 to a bank with the first payment on
1.1.18 and the last on 1.1.24. Receive from the bank £1000 annually in perpetu-
ity with the first payment on 1.1.25.
Ignoring taxes, what interest rate does an investor receive if he takes up this offer?
A. 7.05%.
B. 10.41%.
C. 11.51%.
D. 20.51%.
3 Suppose you have a project having a positive NPV. Now, if this project is funded by debt
and equity, the resulting increase in company NPV will go to:
A. both bondholders and old shareholders.
B. both bondholders and new shareholders.
C. both old shareholders and new shareholders.
D. old shareholders only.
4 Rite Bite Enterprises sells toothpicks. Gross revenues last year were £350 000 and total
costs were £50 000. Rite Bite is an all-equity firm with one million shares outstanding.
Gross sales and costs are expected to grow at 5% per year. The appropriate discount
rate is 15% and all cash flows are received at a year’s end. There is no corporate tax;
and all cash flows are paid out in dividends at the end of the year. The dividend for last
year has just been paid. What is the current price per share of Rite Bite stock?
A. £0.50
B. £3.15
C. £3.50
D. £4.00
5 If the one-year spot rate is 7% and the two-year spot rate is 12%, what is the one-
period forward rate commencing one year from now?
A. 4.67%.
B. 9.56%.
C. 17.23%.
D. 25.44%.
6 The book value of the debt of the Burp Gas Company is £10 million. Currently, the
market value of the debt is 90% of book value and is priced to yield 12%. The one
million outstanding shares of Burp stock are selling for £20 per share. The required
return on Burp stock is 20%. The tax rate is 34%. Given the above information, what is
the WACC for the company?
A. 12.25%.
B. 13.25%.
C. 16.25%.
D. 17.25%.
7 Missile, Inc is considering a £20 million modernisation project in the power systems
division. The project’s after-tax cash flow will be £8 million in perpetuity. Missile’s debt
rate is 10% and its cost of equity is 20%. The target debt–equity ratio is 2 (i.e. value of
debt is twice the value of equity), and the firm is in the 34% tax bracket. What is the
WACC and NPV? Should the firm undertake this project?
A. 0.1000; £60 000 000; yes.
B. 0.1107; −£52 267 389; no.
C. 0.1107; £52 267 389; yes.
D. 0.1507; £32 267 389; yes.
9 Company management believes that inflation is going to fall steeply and stay at a low
level. Market prices do not reflect this expectation. If management decides to act on its
belief, it will immediately
A. replace long-term debt with short-term debt.
B. replace short-term debt with long-term debt.
C. raise the debt–equity ratio.
D. not worry about inflation and make no changes in its financial plan.
10 Consider the following projects being looked at by the Nitrex company. Nitrex has a
required rate of return of 15% for this type of project, but temporarily has no access to
the capital market. It has a maximum of £500 000 available for investments. The projects
cannot be postponed.
11 Mrs T. Potts of Ideal China has a problem. The company has ordered a new kiln for
£400 000. Of this sum, £50 000 is estimated to be the installation cost. Mrs Potts does
not know whether this will be classified by the tax authorities as current expense or
capital investment. Assume a five-year straight-line depreciation schedule, no salvage
value, a tax rate of 34%, and the opportunity cost of capital of 5%. What is the present
value of the tax shield of the installation cost if it is classified as current expense and
there is a one-year delay in making tax payments?
A. £14 190.
B. £14 850.
C. £16 190.
D. £16 850.
12 Using the data from the previous problem, what is the present value of the tax shield if
the installation cost is classified as capital investment?
A. £14 718.
B. £15 850.
C. £16 190.
D. £16 850.
13 The probability that the economy will experience moderate growth next year is 0.6, and
the probability of a recession is 0.2. The other possibility is rapid expansion. If the
economy falls into recession, you can expect to receive a return on your portfolio of
5%; with moderate growth your return would be 8%; and under rapid expansion your
portfolio will return 15%. What is the standard deviation of the portfolio?
A. 0.0331
B. 0.0533
C. 0.0880
D. 0.1000
14 Portfolio risk (standard deviation) is equal to the weighted average (linear combination)
of the risks of individual assets when the individual asset risks are which of the follow-
ing?
A. positively correlated.
B. negatively correlated.
C. perfectly correlated.
D. not correlated at all.
15 Bald plc is the most similar market-traded firm to the investment you are intending to
make in a hair-care centre. Bald is financed with 25% debt and 75% equity, has a
reported equity β coefficient of 1.5 and a debt β coefficient of 0.8. You intend to finance
your project with equity. The β coefficient you should use in setting the required rate of
return for your project is which of the following?
A. 0.714
B. 1.325
C. 1.565
D. 2.333
If the market risk premium is 8.5% and the expected risk-free return is 6.6%, what is the
expected return of Gamma plc?
A. 7.5%.
B. 15.1%.
C. 16.0%.
D. 16.4%.
21 XYZ plc has traded at £12 for the last three years, whereas ABC plc has moved from
£50 to £100. Over this time the market has risen by 100%. Which of the following is
correct?
A. XYZ has a low β coefficient, where ABC has a high β coefficient.
B. XYZ has a market β coefficient.
C. ABC has a market β coefficient.
D. Price volatility does not indicate the values of β coefficients.
22 The Allen Company has monthly credit sales of £600 000. The average collection period
is 90 days. The cost of production is 70% of the selling price. What is the Allen Compa-
ny’s average investment in accounts receivable?
A. £42 000.
B. £540 000.
C. £1 260 000.
D. £5 400 000.
23 The spot rate for the dollar per sterling pound is $1.50/£. If the interest rate is 7% in
the US and 4% in the UK, what is the appropriate one-year forward rate if no arbitrage
opportunities exist?
A. $1.458/£
B. $1.543/£
C. $1.560/£
D. $1.605/£
24 Storm Equipment Inc has come across an investment in Germany. The project costs
€10 million and is expected to produce a cash flow of €4 million in year one, and flows
of €3 million in years two and three. The current spot exchange rate is $1.50/€1 and
the current risk-free rate in the US is 7%, while it is 4.5% in Germany. The appropriate
discount rate for the project is estimated to be 11%, the US cost of capital for the
company. The company could be sold for €2.1 million at the end of three years. Given
the above information, the NPV is:
A. +$368 614
B. –$232 453
C. –$348 680
D. –$1 030 000
25 If the exercise price of a call option you own is £2.45 and the stock price is currently
trading at £2.40, then:
A. the call is ‘in the money’ and its value is positive.
B. the call is ‘out of the money’ and its value is positive.
C. the call is ‘out of the money’ and its value is zero.
D. the call is ‘out of the money’ and its value is negative.
Case Study 1
Heritage Leisure plc has a business that involves purchasing historic houses, castles etc.
from their owners and developing the property so as to attract visitors. This usually
involves building restaurants and bars, providing car parking, improving the property
itself and often adding subsidiary attractions such as a small zoo or a miniature steam
railway.
Heritage has been expanding and aims to continue to expand. Several other compa-
nies have noticed this business opportunity, partly as a result of Heritage’s early success,
and competition to buy suitable properties is now intense. Heritage is aware of the
danger that the market will become oversupplied but believes there will be several
more years before this happens. Heritage is aware that leisure spending is very income-
sensitive. In a year of recession, revenue would fall by 25%, while operating costs could
only fall by 5%. Although there has been no recession recently, Heritage knows that,
based on past experience, one year in five is a recession year and recessions occur
irregularly and unpredictably.
Heritage now has a total of £25 million invested in its seven properties. This amount
is made up of both the initial purchase price and the subsequent capital expenditure on
improvements. This investment is funded by £10 million of long-term debt (at a fixed
interest rate of 11%), £5 million of short-term debt (current interest rate 8%) and
equity of £10 million. There are 10 million shares outstanding, of which 5.5 million are
owned by Mr Taylor, the founder and Chief Executive of the company. The remainder
are held by outside investors, with no single investor holding more than 3% of the
shares. The current market price of Heritage’s shares is £2.00.
The income statement for the year just ended (2010) is as follows:
£ (million)
Revenue 20.0
Operating costs 11.0
Depreciation 3.0
EBIT 6.0
Interest 1.5
Earning before tax 4.5
Tax at 33⅓% 1.5
Earnings after tax 3.0
Dividend 15p/share 1.5
Retained earnings 1.5
Growth in earnings has taken place because the number of properties operated by
Heritage has grown. The revenue from a property once it is operated shows no
particular long-term trend. Inflation has been negligible.
Heritage has recently had the opportunity to purchase Manderley Castle. The price
would be £2 million and an additional £1 million would be needed to improve the
property. The incremental financial figures from Manderley, in a non-recession year, are
forecast to be:
£ (million)
Revenues 2.0
Operating costs 1.1
Depreciation 0.3
EBIT 0.6
The general expectation of financial forecasters is that interest rates, will remain
stable. Mr Taylor’s private forecast is that interest rates, both long- and short-term, are
likely to fall by about 1% over the coming year and will thereafter remain stable.
Heritage has decided to buy Manderley and now has to choose how to finance the
purchase. The retained earnings are being used for a marketing programme, and so the
full £3 million must be raised. The alternatives are:
A. To sell 1.7 million new shares at a price of 177p per share to outside investors
(current shareholders would not be given a prior right to buy the shares). The com-
pany believes this is the highest price at which these new shares could be sold.
B. To borrow £3 million on a long-term fixed interest rate basis at 11%.
C. To borrow £3 million for one year at 8%. At the end of the year Heritage would
require to renegotiate the interest rate and other conditions of the loan.
If the money is borrowed, the bankers would require Heritage to offer the Mander-
ley property as primary collateral. They would also require that Heritage agree not to
pay dividends for any year greater than the amount of after-tax earnings in that year.
There is no such condition regarding dividends attached to the present borrowings.
Required
Note: Questions 1 and 2 of this problem are general and do not require reference to
the information about Heritage.
1 According to the classic Miller–Modigliani analysis, and ignoring taxes, how does the
choice between debt and equity affect the wealth of current shareholders?
(2 marks)
(3 marks)
4 How does the probability of a recession in Heritage’s market affect the choice of
finance?
(3 marks)
6 How does the dividend constraint imposed by potential leaders affect the decision?
(3 marks)
7 How should Mr Taylor’s expectation of future interest rate changes affect the decision?
(3 marks)
8 What conclusion do you draw on the balance of advantages and disadvantages of using
long-term rather than short-term debt in this cases?
(4 marks)
9 What conclusion do you draw on the balance of advantages and disadvantages of using
equity rather than debt in this case?
(4 marks)
TOTAL = 30 marks
Case Study 2
You are given the following information about three bonds, each with a face value of
£100 and interest payable annually. The next interest payment will be in one year’s time.
1 What is the current price of a zero coupon bond maturing in one year; two years; three
years?
(6 marks)
(Note: The following questions are quite independent of B in the table above. The
information given in B in the table above is irrelevant.)
The nominal one-, two- and three-year spot rates are 7%, 9% and 8.5% respectively.
2 What are the prices of zero coupon bonds, face value £100 with one, two and three
years to maturity?
(3 marks)
3 What are the nominal forward rates one year and two years hence?
(4 marks)
4 What is the price of the three-year bond expected to be in one year’s time?
(4 marks)
1st year 1%
2nd year 2%
3rd year 2.5%
What are the expected inflation rates in each of the next three years?
(3 marks)
TOTAL = 20 marks
Examination Answers
7 The correct answer is C. The problems which arise with IRR are in divergent
patterning of mutually exclusive project cash flows. Both different sizes and timings
of cash flows fit that definition; thus answers A and B are incorrect. Answer D is
incorrect because IRR’s confusion is not generally related to the size of NPVs of
competing projects.
8 The correct answer is D. Capital rationing is not having enough money to pursue all
desirable projects. Answer A is incorrect because capital markets have more money
than any single firm could ever find projects (if capital markets will not provide
enough money, it is because they disagree as to the desirability of projects, not
because of any shortage of funds). Answer B is incorrect because financial markets
stand ready at any time to provide funding for good projects. Answer C is incorrect
for the same reason. The text indicates that capital rationing is a reaction to an
information shortfall in either the market or the company, and thus answer D is
consistent with the latter.
9 The correct answer is B. This is a problem in which the investments are ‘repeatable’
or ‘renewable’. The correct technique (other than specifying all cash flows into the
future until the two alternatives have a common terminal point, an unrealistic
calculation in this case, since the first common terminal point is 800 years in the
future!) is to find each alternative’s ‘equivalent annual cost’. To do this, first you
find each alternative’s NPV in its first cycle, and divide the result by the annuity
factor for that particular duration:
For fibreglass:
NPV 25 year annuity factor £11 000 £130 000
9.077 £11 000 £130 000
£229 847
For slate:
NPV 32 year annuity factor £2000 £216 000
9.526 £2000 £216 000
£235 052
The equivalent annual costs are respectively:
EAC NPV /25 year annuity factor £229 847/9.077
EAC £25 322
EAC NPV /32 year annuity factor £235 052/9.526
EAC £24 675
Thus slate is cheaper as measured by equivalent annual cost, because its higher
installation cost is offset by the longer duration between installations and the lower
annual maintenance. Answer A is incorrect because the negative NPVs are appro-
priate due to these being ‘cost-minimising’ investments.
10 The correct answer is A. As the text described, most investments are influenced by
a ‘common factor’, which is usually described as the ‘market’. This common factor
produces a situation where investments have some lower limit on the extent to
which diversification reduces risk. Some newer models of market equilibrium (such
as the APT) have seemed to uncover more than one ‘common factor’. I is incorrect
18 The correct answer is A. Most of IMD’s value is in intangible assets, made up of its
engineering skill. In times of financial distress (generated by declines in the comput-
er industry, which do happen) such assets quickly lose a good bit of their value as
employees leave for greener pastures. Debt suppliers, foreseeing this, would require
much higher interest rates as compensation for this asset intangibility. Answer B is
incorrect because, though IMD could perhaps gain interest tax shields, such tax
benefits may be otherwise available (through R&D credits, for example), which do
not carry bankruptcy costs. Answers C and D propose tactics that would be
applicable to firms in other situations: C is an irrelevancy suggestion, which IMD
does not represent; D would be applicable if bondholders felt that IMD could ‘shift
risks’ of investments after borrowing.
19 The correct answer is D. Receivables management is primarily a trade-off between
acquiring more sales by easing credit terms and thereby delving more deeply into
the pool of questionable customers, versus having lower sales but with faster and
more reliable payments. There is no empirical evidence that would support answers
A or B, and answer C, though correct, is not the primary trade-off of receivable
management.
20 The correct answer is A. Physical inventories allow the firm to respond to the
requirements of markets more flexibly. However, inventories, including cash, are
assets and require financing, and thus cannot be expanded without eventually
reaching uneconomic size. Option II is wrong because it offers too stringent a
standard by the above description of the role of inventories. Option III is opposite
to the truth.
21 The correct answer is C. As Module 11 points out, the ratio between interest factors
(1.11/1.094) = 1.01463 must be the same as the ratio between spot and forward
exchange rates. Thus (1.5/One-year forward rate) must also equal 1.01463; there-
fore the one-year forward rate must equal 1.5 ÷ 1.01463 or 1.478. The other rates
shown are inconsistent with this required economic relationship.
22 The correct answer is A. Real assets tend to increase in value along with inflation, so
there is no need to hedge the plant. Financial assets (such as receivables), however,
have future payoffs stated in nominal currency terms, and if inflation is unexpected-
ly high, future receipts in both the foreign and domestic economy will be less
valuable than anticipated when received. Thus answer A is the only correct one of
those listed.
23 The correct answer is C. An option allows the holder to benefit from positive
events (in this case increases in price of the underlying security) in greater degree
than to lose from negative events. Specifically, increases in stock price above £4.65
will produce gains in current £0.15 exercise value on a £-for-£ basis, while reduc-
tions in stock price below £4.64 cannot reduce exercise value below £0.00. Thus the
premium (the amount by which option value exceeds exercise value) must be
positive; the option must be selling for more than £0.15. Answers A and B are
incorrect because they predict non-positive values, and answer D is incorrect
because premiums must be either positive or zero (and will only be zero if there is
no time until expiration).
24 The correct answer is D. When a company borrows money by issuing bonds, it (in
economic effect) is selling the assets of the firm to the bondholders (the bondhold-
ers have first claim on those assets in the case of default) in exchange for the money
borrowed. Shareholders, however, (in effect) retain an option to repurchase the
assets of the firm from bondholders upon the firm paying the bonds’ contractual
interest and principal (and otherwise abiding by the terms of the bond indenture).
Answer A is incorrect because the option characteristic of common stock has
nothing to do with the shares’ marketability. Answer B is incorrect, because
shareholders may not in fact be able to buy back the bonds (and even if they could
it would not be the characteristic that allows shares to be considered options).
Answer C is closer to being correct than A or B, and in some contexts may be
another way of looking at share value; but in the sense explained by Module 12, is
not as good an answer as D.
25 The correct answer is C. Both of conditions I and II are required. If there is no
overall gain, there is no value increase to be allocated between principal and agent,
and thus no prospect of inducing a change in behaviour. Even with such an increase
in overall value, however, the agent must somehow be allocated a part of the gain,
or there is no incentive for the (decision-making) agent to change behaviour.
Case Study 1
1
The problem implies the following price and cash-flow information, along with
the 7% three year spot rate:
The price of the one-year 8% coupon bond depends upon the receipt of £1080 one
year hence, discounted at the one-year spot rate. The one-year spot rate is not given,
but can be deduced from the price of the end-of-first-year coupon of the three-year
bond. Since the £1080 cash flow from the one-year bond and the £100 cash flow
from the three-year bond have the same risk and timing, they must be subject to the
same discount rate:
£100
£91.74
1
£100
1 1.09
£91.74
9%
With a 9% one-year spot rate, the current price of the one-year 8% coupon bond is
£1080/1.09 £990.83.
2 The price of the three-year 10% coupon bond is a function of its known cash flows
discounted with the one-, two- and three-year spot rates. From Question 1’s answer
we now have the one-year spot rate, and the three-year spot rate is given in the
problem as 7%. So the remaining unknown is the two-year spot rate. It can be
deduced by analysing the two-year 9% coupon bond:
£90 £1090
£1017.07
1 1
£90 £1090
1 0.09 1
£1090
£82.57
1
£1090
£934.50
1
£1090
1 1.1664
£934.50
1 1.08
8%
With a two-year spot rate of 8%, the price of the three-year 10% bond is:
£100 £100 £1100
£?
1 1 1
£100 £100 £1100
£? £1075.40
1 0.09 1 0.08 1 0.07
(This question can also be solved using forward interest rates.)
3 If the current price of the four-year bond is known, and if the first three spot rates
are known, it is simple algebra to find the four-year spot rate:
£110 £110 £110 £1110
£1164.24
1.09 1.08 1.07 1
£1110
£100.92 £94.31 £89.79
1
£1110
1 1.26248
£879.22
1 1.06
6%
4 The one-year forward rate one year hence, , can be found most easily by
1 2
working with the cash flows and prices of the two-year bond:
£90 £1090
£1017.07
1 1 1
£90 £1090
1.09 1.09 1
£1090
1.07
£934.50 1.09
7%
5 This question is asking for a ‘forward price’ of the year-three £110 coupon,
expected to exist as of the end of the second year. To find this value, you must
discount the year three cash flow by the forward interest rate for that year, .
2 3
This latter forward rate has not yet been calculated (unless you solved one of the
earlier answers with forward rates instead of spot rates). The simplest way to
calculate 3 is to recall that:
2
1 1 1
1 1.07 / 1.08 1.0503
5.03%
Therefore the end-of-year-two forward price of the end-of-year-three £110 coupon
of the four-year bond is:
£? £110/ 1 £110/1.0503 £104.73
6 Pricing this bond requires only that the specified cash flows be discounted at the
appropriate spot or forward rates:
£973.38 £110 £110 £110
£?
1.09 1.08 1.07 1.06
£? £1164.24
Note that this price is the same as that of the four-year 11% coupon bond.
7 The four-year 11% coupon bond’s YTM is 6.23%, whereas the bond in 7 has a
YTM of 7.97%. (These YTMs are, of course, the constant discount rates that would
produce the correct market prices of the bonds.) If you did not have access to a
calculator that could find the YTMs easily, you should have said that the YTM of
the 11% coupon bond must be expected to be less than the YTM of the other four-
year bond. The reason why the yield of the coupon bond is less than the yield of the
other bond is that the pattern of cash flows presented by the coupon bond is such
that much of that bond’s present value is generated by its final cash flow (£879.22
of its £1164.24 value), whereas the other bonds’ value is generated almost equally
across time. Since the coupon bond’s value comes from a cash flow being operated
upon by a much lower discount rate than the average (note that, in this market, rates
are lower farther into the future), the YTM, which is a funny ‘value weighted
average’ of the spot rates, must be less for the coupon bond because of the im-
portance to its value of the lower-rate-generated portion compared with the other
bond. Both bonds are, of course, subject to exactly the same set of spot and
forward market interest rates. The difference in YTMs is caused solely by differ-
ences in cash-flow patterns.
Case Study 2
1 The call option that you issue will bring you (assuming no brokerage fees) cash
equal to the value of the option. To value the option, first you must find uS0 and
dS0. With share price increasing from £17 to £22.10 the former is 1.3, and the latter
is 0.8 with a decline from £17 to £13.60.
max 0, max 0,1.3 £17 £18 £4.10
max 0, max 0,0.8 £17 £18 £0
£4.10 £0
0.48235
£17 1.3 0.8
and:
1.3 £0 0.8 £4.10
£5.96364
1 1.3 0.8 1 0.10
0.48235 £17 £5.96364 £2.24
The value of the call option is £2.24. This is the amount of cash that you would
receive from writing the option. Note that the exercise value is zero, yet you are
paid a positive amount for issuing the option. The reason you are given this money
for issuing this option even though it is ‘out of the money’ is that you have prom-
ised to sell something for a fixed price (£18), and it has at least some probability
between now and option expiration of being worth £22.10. The buyer of the option
is purchasing the chance of gaining the spread between exercise price (£18) and
£22.10 (netting £4.10) or, if share price instead goes down, simply walking away and
not exercising the option. The market values this ‘lottery ticket’ at £2.24.
2 You have already actually done the calculations for the answer to this part of the
problem. Remembering that Y is the number of underlying shares to issue so as to
form the ‘call equivalent’ portfolio, and Z is the amount of lending (if positive), the
transaction that would duplicate the issuance of the option would be to sell Y shares
and borrow Z (if positive):
£4.10 £0
0.48235
£17 1.3 0.8
and
1.3 £0 0.8 £4.10
£5.96364
1 1.3 0.8 1 0.10
You can sell 0.48235 shares and lend (since negative) £5.96364 at the risk-free rate.
Your payoff is then, if share price increases:
1 0.48325 £22.10 £5.96364 1.10 £4.10 ﴾A3.1﴿
and if share price declines:
1 0.48325 £13.60 £5.96364 1.1 £0.00 ﴾A3.2﴿
Note that these payoffs are exactly the same (if you have not rounded the results) as
those for the option payoffs.
3 With the sale of the call option, the seller receives the premium and will keep that if
the share price stays below the exercise price. If the share price rises, the call seller
will start to lose out.
If the share price is 220p at expiry, it means the option is worth 20p (S0 – X, 220 –
200). The call option seller received 25p originally, but will have to pay out this 20p
at the expiry of the option. The net position is a gain of 5p to the call option seller.
The share price has gone up by 30p over the three months, but the call option ends
up worth less than at the start of the period. The reason for this is that the time
value contained in the option (which was all 25p of the option price as it was out of
the money) is progressively eroded as the time to expiry nears.
Gain
+
25p
X
S0
200p
–
Loss
1 1
and the present value of an annuity at that point is, of course, simply:
1
If you are giving up the seven-year payment stream to get the perpetuity at that
point, your interest rate on the whole set of cash flows is found by finding the rate
that causes the two to be the same, i.e.:
1 1 1
which reduces to
1 2
You can solve this for the answer, 10.41%, if you have a computer or powerful
enough calculator, or you can simply look in Table A1.3 in Appendix 1 for the
future value seven years hence of a single payment (the obvious form of the
reduced solution), which the Table indicates to have a value of 2 approximately
half-way between the interest rates of 10% and 11% for a seven-year period.
3 The correct answer is D. Both new debt and new equity will be sold at market
prices; that is, on the basis that the PV of the prospective cash inflows, calculated at
the risk-adjusted rate established in the market, just equals the price at which new
debt or new equity is sold. So the purchasers of new debt and new equity get no
NPV. All the gain from the positive-NPV project goes to the old shareholders.
4 The correct answer is B. If dividends are expected to grow at a constant rate, the
formula D1/ r‐g can be used to value the shares of the company, where D1 is the
next dividend, r the required rate of return and g the growth rate of dividends. Next
year’s dividend will equal next year’s gross revenue (£350 000 × 1.05) less next
year’s total costs (£50 000 × 1.05). This dividend of £315 000 will be distributed
amongst one million shares. Therefore the value of each share will be
£0.315
£3.15
0.15 0.05
5 The correct answer is C. Using the formula:
1 1 1
we obtain
1 / 1
1.12 /1.07
1.1723
Thus the implied one-year forward rate is 17.23%.
6 The correct answer is C. The total value of Burp Gas Company is the value of its
debt (£9 million) plus the market value of its equity (£20 million), totalling £29
million. The Weighted Average Cost of Capital (WACC) is determined by the
product of the cost of debt and the proportion of the debt to the value of the firm
(adjusted for the tax shield) plus the product of the required rate of return for equity
and the proportion of equity to the market value of the firm. Thus:
WACC 1 0.34 0.12 9/29 0.2 20/29
0.1625
7 The correct answer is C. Using the above formula:
WACC 1 0.34 0.1 2/3 0.2 1/3
0.1107
Using the WACC as the discount rate, the NPV of an investment of £20 million
yielding £8 million in perpetuity is
£8 000 000/0.1107 £20 000 000 £52 267 289
A positive NPV means the firm should undertake the project.
8 The correct answer is B. Nominal cash flows discounted with nominal discount
rates is the superior technique due to the difficulties of estimating real discount
rates. Though answer A could be made to work in concept, in reality its application
is prohibitively difficult. Answer C is incorrect because it amounts to something like
using nominal discount rates on real cash flows, which will bias NPVs downward.
9 The correct answer is A. If the management’s expectation is correct, interest rates
will tend to fall with inflation as lenders require less compensation for the loss in
their purchasing power resulting from their decisions to postpone consumption.
Choosing option A will allow the firm to reduce its borrowing costs. Option B
would commit the firm to higher real and nominal interest rates. Option C might be
considered if the management expected high and persistent inflation that was not
reflected in market prices.
10 The correct answer is B. Projects W, Y, and Z give the highest total NPV for the
initial outlay of £500 000.
11 The correct answer is C. If the installation cost is a current expense, it can all be
deducted from taxable income this year and there will be a cash saving of
£50 000×0.34 in one year’s time. The present value of this saving is:
£50 000 0.34
£16 190
1.05
12 The correct answer is A. If the installation cost is treated as a capital investment, the
deduction from taxable income is £10 000 per year for 5 years. The tax savings are
34% of these amounts delayed for one year.
£10 000 0.34 £10 000 0.34 £10 000 0.34
1.05 1.05 1.05
£10 000 0.34 £10 000 0.34
£14 718
1.05 1.05
13 The correct answer is A. The average expected return of the portfolio is 8.8% (the
weighted average of all possible outcomes). The standard deviation of the portfolio
is simply the weighted average of the square root of the weighted average of the
difference between the possible outcomes and the average return, that is
0.2 0.05 0.088 0.6 0.08 0.088 0.2 0.15 0.088 0.0331
14 The correct answer is C. Look again at Figure 7.3. It is only when the assets are
perfectly correlated (that is, when the correlation coefficient is plus one) that the
risk of the portfolio can be a weighted average of its components. The risk of a
portfolio with assets that are not perfectly correlated (that is, the correlation
coefficient is less than one) is a non-linear function of asset risk and correlation with
each other, and this is far more complex than the simple weighted average.
15 The correct answer is B. The β coefficient of the required rate of return is the
weighted average of the β coefficient of debt and the β coefficient of equity,
namely:
0.25 0.8 0.75 1.5 1.325
16 The correct answer is D. Once again, the Capital Asset Pricing Model should be
used to find the expected return for Gamma plc. To do this, we have to calculate
Gamma’s β coefficient using the data on covariances and variances provided. Thus:
Covariance of Gamma’s return with the market
Variance of the market
0.0607172/0.052851
1.1488373
E R 6.6 1.1488373 8.5
16.365
17 The correct answer is D. Statement I is incorrect as dividends are not relevant:
shareholders will be compensated through higher capital gains. Statement II is
incorrect as dividends are not shareholders’ wages: the wage to shareholders is the
return they receive through dividends and capital gains.
18 The correct answer is D. Statement I is incorrect as it does not take the required
return for equity into account. Similarly, statement II is incorrect as higher gearing
does not necessarily imply greater risk.
19 The correct answer is C. Stock prices fall when bad news is announced and rise
when good news is announced. The ability to raise debt is generally seen as good
news as it indicates lenders’ confidence in the company. Conversely, issuing equity
is often seen as bad news as it is often a sign that the firm is short of capital and
cannot raise funds elsewhere.
20 The correct answer is D. Statement I is incorrect: indentures give bondholders the
right to any assets in the case of bankruptcy; shareholders only get what is left after
the bondholders get what is owed to them. Statement II is incorrect. Indentures
reduce the return required by bondholders by reducing their risk.
21 The correct answer is D. The β coefficient for each company is determined by the
covariance and variances of their returns (dividends and capital gains) with the
market. Prices alone are not enough to work with.
22 The correct answer is C. The cost of each month’s sales is
£600 000 0.70 £420 000
On average, 3 months’ sales are outstanding at any time, so the amount invested in
accounts receivable is
£420 000 3 £1 260 000
Case Study 1
1 The debt–equity decision has no effect on the wealth of current shareholders,
according to M&M.
2 If interest is tax-deductible but dividends are not, then the use of debt results in a
tax saving. The wealth of shareholders is increased by the present value of those
future tax savings. If an incremental amount of debt D is taken on permanently, the
tax saving is TD (where T is the tax rate).
3 An amount of 45% of Heritage’s shares are held by outside investors and the shares
are quoted on the market. A failure to maintain the dividend would probably lead to
a loss of confidence in the company and a sharp fall in the price of the shares.
Keeping the share price up is important for Heritage’s long-term growth plans,
since it may want to fund future expansion by share issues.
With Manderley, the income statement in a recession year would be as follows:
With share funding, the dividend can be maintained using the cash flow from
depreciation and the small amount of after-tax earnings. With debt funding, the
dividend would have to be cut.
4 The possibility of a recession makes Heritage’s business more risky. The chance of
disaster in the form of bankruptcy depends partly on the risk of the business and
partly on the level of debt. Heritage can offset an increased operating risk in its
business by reducing the amount of financial risk (debt) in its balance sheet.
5 Mr Taylor has 55% of the shares at present, which gives him unquestioned control.
If the Manderley development is funded by a share issue to outside shareholders, his
share will go down to 47%. This is still normally enough to control the business,
since it is unlikely that all the other shareholders would combine to vote against Mr
Taylor. Nevertheless, Mr Taylor may prefer to maintain a majority of votes in his
hand and this would suggest the use of debt finance.
6 See Question 3 above. In a recession year the dividend could be maintained out of
current cash flow but not out of after-tax earnings. If debt is used to finance
Manderley, Heritage will lose the ability to maintain the dividend in a bad year.
7 If Mr Taylor believes that interest rates will fall, it would be a mistake to make a
long-term commitment to borrow at rates that are temporarily high. This factor
would argue against the use of long-term debt.
8
Long-term interest rates are generally higher, and so profits will look better in
the short run if short-term debt is used.
According to the expectations hypothesis, the fact that short-term debt is
cheaper now suggests that it is expected to be more expensive in the future, and
there is no clear long-term benefit from using one type of debt rather than the
other.
Case Study 2
1 Bond A maturing in one year gives a final cash flow of £105. The value of bond A
paying £100 is thus:
100
£99.0566 £94.340
105
The one-year interest rate is therefore:
100
1 6.00%
94.340
Bond B maturing in two years gives a cash flow of £10.5 (after 1 year)
£110.5 (after 2 years).
100 1 7.98%
.
.
The value of 100 after three years is 100 78.68
£100
2 One year to maturity = £93.458
1.07
£100
Two years to maturity = £84.168
1.09 2
£100
Three years to maturity = £78.291
1.085 3
1.09 2
3 One-year nominal forward rate = 1 11.037%
1.07
1.085 3
Two-year nominal forward rate = 1 7.507%
1.09 2
Module 1
Review Questions
1.1 The correct answer is B. The ‘maximum you can consume’ implies that all of your
consumption will be at t0, so in addition to the £3000 of cash flow already at that
time, you must shift your future cash flow to the present. Since your future cash
flow is at t1, it must be discounted to t0 using the 11% rate given. Thus:
PV maximum
1
£5328
£3000
1 0.11
£3000 £4800
£7800
This calculation is the final arithmetic of an actual financial market transaction that
would result in that amount of t0 cash flow being available. For example, a lender in
the financial market would be willing to provide the £4800 of t0 cash to you, if you
provided in return a claim upon your t1 cash flow. The £4800 would require an
interest payment of £4800(0.11) = £528, plus the return of the original £4800 lent,
to total your £5328 at t1. The lender of course must concur in your expectation that
the £5328 will actually appear at t1.
Questions 1.1 to 1.11 refer to Module 1, Section 1.3. The graphical solution to these
questions is given in Figure A4.1.
t1
£8658
£7548
A
C
B
£5328
£3108
£7548 at t1, an additional £2220 in excess of your £5328 initial cash flow at that
time point, you would necessarily give up (lend) £2000 at t0. The £2000 given up
serves to reduce your original t0 £3000 to £1000.
1.5 The correct answer is D. The present value of all of the consumption combinations
in the questions above is £7800. This is apparent in Figure A4.1 as all of these are
points along the same exchange line emanating from the t0 intercept of £7800. The
reason the present values are all the same is because, with £7800 of t0 resources, all
of the combinations are exactly attainable by various borrowing and lending
activities at the market interest rate; no other t0 amount can do that. Since total
resources shifted to t0 is by definition present wealth, and £7800 is that amount for
you, the (B) answer is in that sense tautological.
1.6 The correct answer is C. The easiest way to see this is to notice that, in Figure A4.1,
any financial market transaction for the purpose of altering consumption patterns
simply serves to move the participant up or down the same exchange line, and that
exchange line has a single t0 intercept, present wealth. The economic importance of
the observation that wealth is unchanged by financial market transactions is that
such transactions are best used by participants to reallocate their resources across
time so as to consume a fixed amount of wealth according to their preferences. But
financial markets, if they efficiently price the securities in them, do not allow
participants to change their wealth by transacting in those markets. The timing of
consumption does not affect wealth, nor does the fact that with a positive interest
rate, the amounts consumable at various times may not be the same. You may
prefer one pattern of consumption to another, but that does not affect your wealth,
defined as the present value of all your future cash flows or resources.
1.7 The correct answer is B.
£1250
NPV 1 £1000 £126.13
1.11
£650
NPV 2 £500 £85.59
1.11
£1650
NPV 3 £1500 £13.51
1.11
Investments 1 and 2, in producing positive NPVs, are expected to have future
returns that exceed the returns that investments in the financial market would
provide. Investment 3, in producing a return of £1650 at t1 in exchange for a £1500
investment at t0, is not providing a return as high as the financial market would for a
similar investment ( £1500 1.11 £1665). Positive NPVs are associated with
returns in excess of market rates; negative NPVs come with investments whose
returns are less than what the market would provide.
1.8 The correct answer is B.
£1250
0 £1000
1 IRR 1
IRR 1 25%
£650
0 £500
1 IRR 2
IRR 2 30%
£1650
0 £1500
1 IRR 3
IRR 3 10%
Investments 1 and 2 offer average per-period rates of return (IRRs) in excess of
those generally available on similar investments in the financial market, so are
acceptable. Investment 3’s 10 per cent return is less than the 11 per cent return the
market would provide, so it is rejected. Note that the two criteria, NPV and IRR,
have in this instance indicated the same ‘correct’ decision in each case.
1.9 The correct answer is A.
£1
Investment 3
Investment 1
Investment 2
0 £7800 t0
£8011.92
Another way to see this is given in Figure A4.2. In this graph we have plotted the
investments (we have them emanating from the original present wealth rather than
the original cash-flow amounts, which has no effect upon the results but is the more
common convention in finance), the new exchange lines that would be associated
with them, and the resulting present wealth. You can see that if investments 1 and 2
are undertaken, wealth will be maximised, and the quantitative increase is from
£7800 to £8011.72, an increase of £211.72.
1.10 The correct answer is D. In the financial market the way we have designed it to this
point, it would make no difference to your wealth (as long as the investments were
somehow or other accepted) whether you provided the money or not, whether the
money was derived from your borrowing or the company borrowing, or by the
company issuing new shares. Since the investments’ cash flows and discount rates
are known for certain, and since there are no frictions like taxes and brokerage fees
in the market, money will be provided at an opportunity cost of 11 per cent, and the
resulting values will obtain. You should not reject the company’s request for funds
simply because your preference is to consume an amount equal to all of your
income at t0, because if you give money to the company you can always borrow
what you need from the market. Since the investments for which your money will
be used will earn more than it costs you to borrow the money, providing the money
the company requests is worthwhile (but no more nor less worthwhile than the
company borrowing the money or issuing new shares).
The only caution we would point out is that whenever the financial market is
approached for funds, either for borrowing or for issuing new ownership shares, the
market must be made aware of and concur in the expectations associated with the
investments that will claim the funds. (It is particularly important that share prices
increase to reflect the NPVs before the new shares are sold, or else the new
shareholders will capture part of your NPV.) If that concurrence is not the case, the
market may not provide the money for investments at the same opportunity rate
that you used as the discount rate to evaluate the investments. The resulting wealth
effects of the investments would not then be the same as you expected, and could
actually produce wealth decreases. This applies to both borrowing and raising new
ownership funds.
1.11 The correct answer is A. You should accept the investment with the highest NPV,
because it increases your wealth the most. (Incidentally, though we did not ask it,
the investment that does that is investment 1.) You should not accept one invest-
ment over another on the basis of the rates of return they earn, because rates of
return are not the same as potential consumption amounts, and it is the opportuni-
ties to consume wealth that motivate participants in financial markets. To see this,
notice that investment 2 has a higher IRR than investment 1, even though investing
in 2 would produce a smaller excess cash return than would investing in 1. If you
put your money in investment 2, after opportunity costs are subtracted, you end up
with £650 – £500 1.11 £95 at t1, whereas investing in 1 gives you £1250 –
£1000 1.11 £140 at t1. So, regardless of the fact that investment 2 promises a
return of 30 per cent to investment 1’s 25 per cent, investment 2 does not produce
as much wealth. (Note that the present ( t0) values of the excess amounts calculated
above are in fact the investment NPVs.)
Students often here raise the point that investment 2 would be preferred to invest-
ment 1 if we were allowed to invest more than the £500 outlay in 2 at the 30 per
cent return. That is indisputably true. If you invest £1000, for example, at a 30 per
cent return you would receive £1300 at t1, exceeding the return from investing in 1.
But the odds are that you could not do that, because the investments are real assets
like machines, manufacturing plants, inventory or even education. Just because the
first machine earns 30 per cent is no reason to think that the next one will, and even
less so the third! If more money poured into it and yet investment 2 continued to
earn 30 per cent per pound invested, we would argue that the investment analyst
was at fault for not specifying in the original analysis that investment 2 had that
capacity. The truth is that securities in financial markets are the only investments
that generally have the characteristic of offering the same IRR regardless of the
amounts invested (because the supply of such assets is essentially unlimited relative
to our individual capacities to invest in them). And we have already discussed the
chances of increasing wealth by investing in such financial assets.
1.12 The correct answer is A. You should choose location 1 because it has the higher
NPV:
£1250 £1300 £1450
NPV 1 £2500 £754.70
1.10 1.10 1.10
£1300 £1300 £1300
NPV 2 £2500 £732.91
1.10 1.10 1.10
1.13 The correct answer is A. You should choose location 2 if your opportunity costs are
25 per cent per period, as indicated by a recalculation of NPVs:
£1200 £1300 £1450
NPV 1 £2500 £34.40
1.25 1.25 1.25
£1300 £1300 £1300
NPV 2 £2500 £37.60
1.25 1.25 1.25
The interesting point about Questions 1.12 and 1.13 is that they show how changing
the discount rate alters not only your wealth (note that the NPVs of both alterna-
tives are lower with the 25 per cent opportunity cost than with the 10 per cent), but
also the relative desirability of the alternatives (location 1 is preferred at the lower
discount rate, and location 2 at the higher). The reason for that result appears when
we sketch a graph of the relationship between discount rates and NPVs for these
two alternatives, as we have done in Figure A4.3. Notice that as the discount rate
increases, the NPV of location 1 declines more quickly than location 2’s, such that
even though location 1 was preferred at the lower discount rate, eventually – as the
discount rate increases – location 2 becomes the preferred alternative. The arithme-
tic reason for this is not difficult to see. Look at the patterns of cash flows that the
two alternatives exhibit: location 1 has its largest cash flows further into the future
and is thus more subject to the vagaries of discount rate changes than location 2
whose cash flows are more evenly spread across time. So changes in discount rates
will tend to cause greater changes in the NPV of 1 than 2, and that is exactly the
characteristic of which we have taken advantage in designing this example.
NPV
Location 2 Location 1
£754.70
£732.91
IRR1
£37.60
IRR2
£34.40
10% 25%
Location 1 Location 2
£800/0.10 £8000. To be equally desirable, the first investment must also have a
present value of £8000, assuming they cost the same. This means that the first
investment, with a per-period cash flow of £1000, must have a ratio between cash
flow and value of eight times so as to produce a present value of £8000. Since we
know that this is an annuity, and we know the discount rate (10 per cent), we can go
to the annuity tables in Appendix 1 and look in the 10 per cent discount rate
column for a number close to 8. The table indicates that an annuity running for 17
years has a present value equal to 8.0214 times its per-period cash flow.
1.16 The correct answer is C. You would necessarily put aside £63.67 per month for a
year, with 10 per cent annual interest compounded monthly so as to end up with
£800 at the end of the year. The following steps will arrive at that result:
1. First find the present value of the £800. This is £800/ 1 0.10/12 12
£724.17.
2. Next, find the present value of £1 per month for one year at the interest offered.
This is 11.3745, an annuity value that is the sum of the series 1/ 1 0.10/12 1
1/ 1 0.10/12 2 … 1/ 1 0.10/12 12.
3. Finally, divide the present value of what you want to end up with (£724.17) by
the present value per pound of what you will put in (11.3745), and the result
(£63.67) is the number of pounds per compounding period that you must put in
so as to end up with the £800 at the end of the year.
There is a somewhat easier way to solve this problem, which requires the use of the
formulas for future and present values of annuities that we have not presented.
Because of the many variants that these can assume, and the limited usefulness of
the full range of these formulas, we recommend that you refer to a good reference
book of mathematical tables to get the one that applies to any particular problem
you may be facing.
1.17 The correct answer is B. It would be necessary for you to deposit £723.87 in the
bank at the beginning of the year at a continuously compounded rate of 10 per cent
so as to finish with £800 at the end of the year. This is nothing more than the
present value of £800 one year hence at a continuously compounded discount rate
of 10 per cent: PV CF/eit £800/e0.10 £723.87.
1.18 The correct answer is C. You should undertake the investment because it has a
positive NPV:
£7000 £11 000
NPV £15 000
1.10 1.10
£454.54
But since there is risk that will change, you can simultaneously hedge the t2
1 2
investment cash flow’s present value by selling an interest rate future for the second
period.
If should increase to 20 per cent, as the question indicates, your NPV of the
1 2
investment will become:
£7000 £11 000
NPV £15 000
1.10 1.10 1.20
£303.03
This would cause you to regret having made the investment. But if you had at t0
sold an interest rate future based upon an £11 000 t2 cash flow, the result would
differ. That interest rate future, when you sold it under the original term structure
(with an implied of 10 per cent), would have a fixed t1 price of £11 000/(1.10)
1 2
= £10 000. So you would own a contract that commits you to sell an £11 000 t2 cash
flow for £10 000 at t1. When the rate increases to 20 per cent, the t1 value of a
1 2
t2 £11 000 becomes £11 000/(1.20) = £9166.67. So you own the right to sell
something worth £9166.67 for £10 000, a gain to you of £833.33. But this gain is as
of t1. At t0 (with an unchanged i1 of 10 per cent), this has a value of £833.33/(1.10)
= £757.57. So the interest rate future contract that you own will increase in value
£757.57 at t0. On the other hand, your investment NPV has decreased from
+£454.54 to −£303.03, a reduction of £757.57. Thus the change in value of your
(sold) interest rate future exactly hedges you against the risk of investment NPV
diminution due to interest rate changes.
In addition you are provided with the information that the current one-period spot
rate is 10 per cent. (This latter information is not literally necessary, as the very
astute student may have discovered: the one-period spot rate can be deduced from
the simultaneous equation solution to the information about bonds A and B, the
two-period bonds. At this stage of the text, however, it would be unfair to think that
everyone would see this relationship.)
The current price of bond B can be found by remembering that the YTM is the
discount rate that equates the bond’s price to its cash-flow promises:
£100 £1100
PV B £1025.49
1.085595 1.085595
2 The current two-period spot rate of interest can be found with the information
about the price of either of the two-period bonds and the one-period spot rate. For
example, using bond A, the present value of its t1 cash flow is £40/1.10 £36.36.
Since its total present value is £919.97, the present value of its t2 cash flow is
£919.97 – £36.36 £883.61. That value can be regarded as being produced by the
two-period spot rate in the equation £883.61 £1040/ 1 i2 2. Solving for i2 we
get 8.489 per cent as the two-period spot rate.
An interesting check on this result is to see if it works on bond B. We should be
able to reproduce B’s present value using the spot rates for the first two periods,
since they are supposed to apply to all bonds of equal risk. Using the spot rates, the
present value of B’s t2 cash flow is £1100/ 1.08489 2 £934.59, and the present
value of its t1 cash flow is £100/ 1.10 £90.91. These sum to £1025.50, the price
of bond B (rounded) that we found initially by using its YTM as the discount rate.
3 The one-period forward rate of interest for the second period is the rate of
exchange for shifting resources from t1 to t2. The easiest way to calculate that rate is
to use the relationship between spot rates and forward rates: 1 plus the spot rate of
interest raised to the nth power (where n is the number of periods that the spot rate
covers) is equal to the product of {1 plus each of the intervening forward rates}.
That rule translates in this situation to:
1 1 1
Remembering that the one-period spot rate is the same thing as the first period’s
forward rate , we know every rate in the above except the forward rate
for the second period, 2 . Solving for it, we have:
1
1 1 / 1
1 1.08489 / 1.10
1 1.07
7%
4 The one-period forward rate of interest for the third period can be calculated by the
same technique as we used for ; that is, if we know the spot rate for the entire
1 2
span of time between now and the end of the period from which we wish to extract
the forward rate, we need merely to multiply together {1 plus all of the forward
rates but the one we wish to find}, and divide that product into the spot rate raised
to the nth power. The problem that we face here in doing that is that we do not
known the three-period spot rate.
The three-period spot rate requires that we find the rate of exchange between t0 and
t3. This information is somehow contained in the cash flows and price of the three-
period bond. If that bond were a ‘pure discount’ bond (a bond that had no interim
interest payments) we could find the spot rate by dividing the current price into the
t3 cash flow, then taking the cube root and subtracting one. But that tactic will not
work with our three-period bond because its current price also includes the values
of its interim t1 and t2 coupon payments. Our first step must be to find the present
value of the t3 payment alone. That obviously requires that we subtract the present
values of the t1 and t2 cash flows from the current market price of bond C. We
know that the interim cash flows can be valued with the spot or forward rates for
the appropriate periods:
times that the cash flows are to be received. It is more difficult to say that than to do
it:
V B / 1
£1100/ 1.07
£1028.04
8 You have already calculated the current price of this security (based on a claim upon
the t2 cash flow of bond C). In Question 4 of the case you discovered that PV CF2
for bond C is £67.97. That is, in fact, under the conditions of the question, the t0
price of a claim on the £80 interest payment of C at t2.
9 By using futures markets in financial securities you can, if faced with the type of
uncertainty posed in this case, ensure that a particular NPV will actually result. Here,
though the amounts of cash that are expected to appear across the future are known
with certainty, the interest rates that will apply between future time points are not.
The existing set of spot and forward rates is known (and was used, we assume, to
calculate the NPV).
The problem can be illustrated by supposing that, immediately after the commit-
ment to undertake the project at t0, interest rates rise. Now, there is nothing we can
do to make interest rates go back to where they were. Therefore the NPV of the
investment itself is irrevocably lower. But there is something that we should have
done to have our wealth remain intact. We should, at the same time we commit to
the investment, also engage in another transaction that has exactly the opposite
exposure to the risk of interest rate changes that the investment has. One such
transaction is to sell forward contracts in financial securities at the same time that
the investment commitment is made.
A forward contract is a commitment to purchase a financial security at a fixed price
at a set future time point. When you sell one of those, you are committing yourself
to sell the security on the terms of the contract. As interest rates (discount rates)
increase, security prices decline. So as interest rates rise at t0 and security prices fall,
the price of the contract to purchase a security in the future at a fixed price also
falls, but your negative holding of the contract (selling it) increases in value.
In other words the commitment that you own (to sell the security at a fixed price)
becomes more valuable as the security’s price itself declines, and the security’s price
declines as interest rates increase. Finally, since the NPV of the real asset declines
with the increase in interest rates, that decline will tend to be offset by the increase
in value of the contract to sell a financial security. Naturally, for the hedge (as this
tactic is called) to be perfect, great care must be taken in deciding exactly what kind
of a contract (or, more realistically, contracts) to sell. But that analysis is well beyond
us at this point, and not really necessary to the basic lesson that such activities are
possible.
There is one other point that should be made in this discussion of hedging against
discount rate uncertainty in real-asset investments. The astute student may have
already asked the question, ‘Suppose that you had borrowed money to undertake the
investment. If you had made a commitment to pay a fixed interest rate for the
duration of the real-asset investment, how does all this business of changing interest
rates apply? It would seem that since you are certain of the cash you are going to
take in and the cash you are going to pay out, interest rate changes are irrelevant.’
A good point. The essential intuition of this view is satisfactory, but needs a bit of
refinement. It is true that the original example, in order to be an accurate reflection
of the risk of interest rate changes, would necessarily assume that however the
investment was financed, its interest rates (or other required returns to capital
suppliers) must be able to vary with changes in market rates. If they do not, as in the
question just posed, the prices of the securities you have issued to finance the
investment will fluctuate with changes in market interest rates. In our example,
when rates go up, the amount you would have to pay to creditors would not
increase if the interest payments were fixed, and therefore the values of those claims
would decline. This is economically the same effect as hedging in futures markets,
but is done here by making a fixed commitment to pay interest amounts rather than
to sell a security. It is possible to hedge your wealth against changes in future
interest rates by financing a project with commitments that would vary in value
exactly opposite to your investment’s cash flows as discount rates change.
10 The actual hedging transaction in this marketplace would be to sign a futures
contract to sell at t2 a security that claims a t3 cash flow of £1000. The contract price
for t2 will be £1000/ 1 £1000/ 1.05 £952.38. Should change at
2 3
any time, increasing or decreasing the NPV of your investment by changing the
present value of its £1000 cash inflow at t3, your futures contract’s present value will
also change in the opposite direction, in exactly the same amount. You are thus
hedged against the risk of a change in 3 upon your investment NPV.
2
Naturally these, along with the £12 000 present cash flow, sum to equal the total
present value calculated in Question 1.
3 Given the expectations that exist now, you would expect to be able to sell your t2
cash flow at t1 for:
£14 000
V1 CF £12 962.96
1.08
The t2 cash flow must be discounted one period to t1.
4 Similarly, the value currently expected for the t3 cash flow at t1 is:
£15 000
V1 CF £12 860.08
1.08
The t3 cash flow must be discounted two periods to t1.
5 The solution to this question, of how much you can consume as a maximum
constant amount at each time point, requires an annuity calculation. But instead of
finding the present value of the annuity, we must find the annuity cash-flow
amounts that produce an already known present value. The reason that the solution
is structured in this way is that we know the present wealth to be allocated across
the future but we do not know the constant consumption amounts that would
discount to equal that present value.
There will be four equal consumption amounts, one now and one at each of the
next three time points. Taking ‘CC’ to stand for the constant consumption amount
at each time point, the amount of present wealth to be allocated across the future is
equal to total present wealth minus present-time-point consumption (PW − CC). If
we divide that amount by the annuity present-value factor for the correct number of
periods (three) and the correct discount rate (8 per cent), the result will be CC, the
cash flow per period that will produce the known present value of future consump-
tion. That amount is given thus:
Annuity factor
1 Annuity factor
£47 947.26
1 2.5771
£13 403.95
With an initial set of cash-flow expectations as given, and a constant discount rate of
8 per cent per period, you can consume £13 403.95 per period.
There are an infinite number of ways that you can arrive at that consumption
pattern by borrowing and lending in the financial market. We present one such
below, but any of the others would also be correct. One can be reasonably certain of
a correct solution if the result is the constant consumption amount at each time
point with no residue at the end.
t0 t1 t2 t3
Initial resources £12 000.00 £13 000.00 £14 000.00 £15 000.00
Borrowing £1 403.95 £516.27
Interest paid £112.32 £153.62 £118.22
Repayment of loan £442.43 £1 477.83
Lending
Interest received
Net (consumption) £13 403.95 £13 403.95 £13 403.95 £13 403.95
The amounts of borrowing shown are the new borrowings at each time point,
whereas the interest amounts are calculated on the basis of the total borrowings
outstanding for each period. For example, at t2 the interest is calculated on a
borrowed amount of £1920.22, which is the sum of the two borrowings at t0 and t1
(£1403.95 + £516.27). The slight discrepancy at t3 is due to rounding at earlier time
points.
Note that this solution has no lending. There is nothing that would have prevented
you from borrowing more than was necessary at any time point (except t3) and
lending the extra. There would have been no wealth effect of that, since the
borrowing and lending rates are the same and there are no costs incurred on a ‘per
transaction’ basis.
6 In this question we have changed the market interest rates to a one-period spot rate
of 6 per cent, a two-period spot rate of 8 per cent, and a three-period spot rate of 9
per cent. Otherwise the situation is the same. Your present wealth is now:
£13 000 £14 000 £15 000
£12 000 £47 849.64
1.06 1.08 1.09
Your wealth with the constant 8 per cent market rate was £47 947.26, and so the
change in market interest rates has caused your wealth to decrease marginally.
7 The separate cash-flow present values under the two interest rate regimes are:
Note that in present-value terms the t1 cash flow has increased, the t2 cash flow
has stayed the same, and the t3 cash flow has decreased. The straightforward reason
is that the opportunity costs of acquiring those amounts (the market interest rates)
have changed in exactly the opposite directions as the cash-flow present values.
(The amounts necessary to invest at t0 in order to end up with the initial resource
amounts have changed because market interest rates have changed.) One of the
present values has changed in the same direction as the overall wealth change, one
in the opposite direction, and the third has not changed at all. The net effect of
those three changes is the overall wealth decrease.
8 The correct answer to this question depends upon an interpretation of exactly what
is being asked. If the question is asking whether you can consume exactly the
amounts that appeared in the answer to Question 5, the response must be ‘No, you
cannot.’ The easiest way to discover this is to find the present value of the preferred
consumption pattern and see whether your present wealth under the new interest
rate regime is as large. If present wealth is as large as the present value of the
consumption pattern, there will be some set of financial market transactions using
your initial resources that will allow you to reach the desired pattern:
£13 403.95 £13 403.95 £13 403.95
PV CC £13 403.95
1.06 1.08 1.09
£47 891.22
The present value of the consumption pattern is £47 891.22, whereas your present
wealth is £47 849.64. This means that you cannot consume £13 403.95 now and at
each of the next three time points with the altered interest rates. If you do not
believe this, try to find a set of borrowing and lending transactions with your
resources that would produce that consumption pattern.
On the other hand, if the question is interpreted to be asking whether you can
consume a maximum constant amount per period under the new interest rate
structure, the answer is of course that you can. Be cautioned, however, that the
exact constant amount that can be consumed at each time point is more difficult to
calculate than it was when interest rates were constant across the future, because
annuity tables are not usable unless rates are constant. The most straightforward
means of arriving at an answer would probably be some type of search process.
9 Yes, you would accept the investment (using the NPV criterion) under the constant
8 per cent rate structure:
£1500 £2000 £2480
NPV £5060 £12.27
1.08 1.08 1.08
10 No, under the non-constant rate structure the NPV is negative:
£1500 £2000 £2480
NPV £5060 £15.21
1.06 1.08 1.09
11 Reversing the t1 and t3 cash flows of the investment causes its relative desirability
under the two rate structures to change:
£2480 £2000 £1500
NPV £5060 £141.72
1.08 1.08 1.08
£2480 £2000 £1500
NPV £5060 £152.58
1.06 1.08 1.09
The investment now is acceptable under the non-constant rate structure. As a
matter of fact, the investment generates a greater NPV under this structure than it
does under the constant-rate structure. The reason for this becomes apparent when
we examine the interaction of the cash-flow pattern change with the alternative rate
structures. The switch of t1 and t3 cash flows has substituted a higher for a lower
cash flow nearer to the present, and vice versa. The effect of that change is to cause
both NPVs to increase (as you can see by comparing the NPVs immediately above
to those originally calculated). But notice also that the constant-rate structure has its
highest discount rate at the period furthest into the future and its lowest at the
period nearest the present. The original cash flows had the same pattern, which
resulted in the largest discount on the highest cash flow. The reversal of cash flows
caused the opposite association under the non-constant structure, namely the
smallest discount on the highest cash flow. This is sufficient not only to cause the
investment to be acceptable under this rate regime, but also to make it more
desirable than it would be under the constant-rate regime.
Module 2
Review Questions
2.1 The correct answer is A. Financial managers should attempt to maximise the wealth
of currently existing shareholders because that capital supplier group is unprotected
by specific contractual performance dimensions, whereas all other entities with
which the company transacts are protected. If the decision processes of managers
were at odds with the wealth of shareholders, equity capital would be much more
expensive or even impossible to raise.
In addition, the other interests mentioned in (C) (the poor, customers, the environ-
ment) are themselves protected in their transactions with the company, either by
government or some other entity. Further, there should be little confidence that
company managers could efficiently and fairly adjudicate among the conflicting
interests of such groups with respect to the various allocations of company-
generated wealth that is under the managers’ control. Such decisions are probably
best left to the elected officials of the country, who usually have taxation, wealth-
transfer and other allocative powers, and are answerable to the groups through the
election process.
2.2 The correct answer is C. The method of arriving at this result is to recognise that the
market price of Perpetual Payments plc at any time is correctly considered to be
either the discounted value of its entire set of future dividends or the present value
of {the final share price at which it can be sold at some future time plus the interim
dividends}. Thus if you are convinced that you can sell the shares for £210 a year
hence, and given the growth perpetuity nature of the company’s dividends, you may
use the formula:
Dividend Share price
Share price
1
£10 £210
1 0.10
£200
So the price you would now be paying for the shares of Perpetual Payments plc is
£200. To find the implicit growth rate, we have:
Dividend
Share price
Dividend
Share price
£10
0.10
£200
0.05
It is also possible to solve this problem by using the price at t1 (£210) and the
dividend at t2 (which would be £10 multiplied by 1.05, or £10.50). Either way, the
implied growth rate is 5 per cent.
The lesson intended for this question is to reiterate that the market prices of capital
claims are the discounted values of the amounts of cash that holders of those claims
expect to receive in the future. It makes no difference whether these amounts of
cash are paid by the company to the holder as a dividend or paid by another outside
purchaser of the shares to the holder as share price. Such purchasers must, in the
end, be looking to the eventual dividends paid by the company, as would the holder
were the shares not sold.
2.3 The correct answer is D. Both II and III are correct. Indeed, II must be correct if
III is; the source of financing for the project is irrelevant as long as the value and
outlay amounts are as shown.
If the project is financed by a dividend reduction, old shareholders lose the amount
of the dividend reduction (which is equal to the outlay for the project), but gain the
full value increase, the net of which is NPV = £1 000 000. If the project is financed
by new equity, old shareholders lose no dividend, but gain only £1 000 000 of the
£3 000 000 value increase because the company would have necessarily given
£2 000 000 in value to new shareholders for them to have contributed that amount
for the project’s outlay.
If the project is financed with debt, the same argument as for new equity applies, as
would similar ones for any combination of dividend reduction, new equity or debt.
2.4 The correct answer is D. When attempting to interpret the information implicit in a
company’s price/earnings ratio, an analyst must be careful to recognise that the P/E
ratio is a type of very crude valuation formula, which relates earnings (a proxy for
the first period’s cash flow) to the market price of the shares. Given that an accurate
value formula must deal with size, timings and riskiness (through the discount rate)
of all of the expected cash flows of the company, the P/E ratio can only give
unambiguous information when all but one of these several factors that can differ
among companies are held constant in the comparison. This is not the case when
comparing a company’s P/E ratio to an average, which would include companies
with different risks, cash-flow timings and amounts expected for the future.
Valuable comparisons are likely to be made amongst only very similar firms, usually
within a single industry.
Module 3
Review Questions
3.1 The correct answer is D. Both I and II are correct, because the numbers recorded in
the income statements of companies are in some cases not cash flows at all, or omit
cash flows that actually occur, and can be mislocated in time with respect to when
the cash flows are actually expected to occur. III is incorrect in that the accounting
numbers are not ‘wrong’ (in the sense of being incorrect) but are merely developed
and used for purposes other than the types of decisions comprehended by financial
cash flows.
3.2 The correct answer is B. It is true that finance is concerned only with cash flows,
but it is not true, as the other answers imply, that finance is unconcerned with
keeping track of the amounts that are owed the company by customers. Answers
(A) and (C) are obviously incorrect in this dimension. Answer (D) is correct in the
implication that the cash flows across time will capture the actual or expected
payments of cash by customers, but is incorrect in the assertion that owed amounts
can therefore be ignored – amounts owed the company by customers are as much
assets as machines and equipment, and must be managed as carefully.
3.3 The correct answer is B. Option II is correct because, consistent with our insistence
upon ‘cash flows, only cash flows, and all cash flows’, the inclusion of the cash
amounts taken in when the asset is eventually expected to be sold is the correct
perspective from which to compare the information validity of depreciation
expenses with financial cash flows. In other words, the cash flow generated upon
the eventual disposal of the asset will in effect accomplish the same result as
recording the deterioration of its usefulness across time by depreciation expenses,
and record this more correctly in terms of the amounts and timings of associated
cash flows.
Answer (A) is wrong because it implies that cash flows would not include those
from the eventual sale of the asset, and answer (C) is wrong because it implies that
the company can be unconcerned with the period-by-period value changes or
deterioration in the asset. The company must be concerned about all of these things,
but the free cash flows thereby expected to be generated are the things that will
cause the capital claims of the company to have value.
3.4 The correct answer is D. Because we are concerned with the changes that will take
place in the cash flows of the company were it to accept the given project, and since
taxes are cash flows, we must account for the changes in the company’s taxes that
will be induced by the new product. If the new product will reduce the company’s
taxes because of negative accounting income, those cash-flow changes are legiti-
mately included for the periods during which they are expected to occur. This, of
course, implicitly assumes that the company has taxable income from other sources
against which such losses can be written.
The other answers are incorrect because they either ignore the legitimate tax cash
flows in question or they make unsupportable assertions about tax cash flows
offsetting each other across time. Even if tax increases and decreases were identical
and in opposite directions, the fact that they are expected to occur at different times
would affect their present values, so as to produce little likelihood of exactly
offsetting effects.
3.5 The correct answer is A. ‘Free cash flow’ is the amounts of cash that the company is
expected to be able to pay to (or must raise from) capital suppliers in order to
validate its plans for the future. It is neither necessarily the maximum that could be
paid at any time (the company may plan to retain cash for investment purposes
which could have been paid as a dividend to shareholders), nor is it simply the sum
of all dividends, interest and principal amounts expected to be paid to capital
suppliers (the company may, for example, both pay dividends and raise new share
capital, whence only the net amount of those is ‘free’ in the financial sense).
‘Free cash flow’ is not strictly determined by any particular legal encumbrances on a
company’s cash flow, though such legal constraints may affect the free cash flow in
any period. The correct interpretation of free cash flow is the residue after all plans
and expectations of the company are considered. Some of these may be legally
required, while others may be entirely voluntary.
Module 4
Review Questions
4.1 The WACC–NPV of the robot reel fabricator project is calculated as follows:
FCF ∗
NPV ∑ ∗
1
where the WACC is:
∗ ∗
The necessary data are all either given in the question (ru) or are available from the
answer to Question 4.1 (FCF* and rd), excepting the interest tax shields. To find
these, we merely multiply the interest expectations by the tax rate (50 per cent):
(£s)
Now Yr 1 Yr 2 Yr 3
Interest – 244.94 191.68 130.44
Interest tax shield – 122.47 95.84 65.22
Module 5
Review Questions
t1 t2 t3 t4 t5
Vans 15 000 15 000 15 000 0 9 000
Phones 1 000 1 000 1 000 1 000 0
Total depreciation 16 000 16 000 16 000 1 000 9 000
The depreciation schedule includes the ‘write-off’ of the vans, which is allowed
upon their sale at the end of year five.
At this point we are ready to take the next step, which is to calculate the changes in
taxes that PC Problems will experience if it accepts the project. To do so we must
calculate the changes in taxable income implied by the project’s acceptance (in £s):
t1 t2 t3 t4 t5
Sales revenue 75 000 150 000 250 000 275 000 302 500
Other revenue 15 000
Total revenue 75 000 150 000 250 000 275 000 317 500
Direct expense:
Labour 20 000 40 000 46 000 52 900 60 835
Other 11 250 22 500 37 500 41 250 45 375
Management 10 000 11 000 12 100 13 310 14 641
Marketing 15 000 10 000 10 000 10 000 10 000
Total direct expense 56 250 83 500 105 600 117 460 130 851
Depreciation 16 000 16 000 16 000 1 000 9 000
Total expenses 72 250 99 500 121 600 118 460 139 851
Profit before tax 2 750 50 500 128 400 156 540 177 649
Taxes (52%) 1 430 26 260 66 768 81 401 92 377
There are several interpretations of the original information provided that may cause
your answers to differ from those above. For example, note that we have included
the revenue from selling the vans at t5, and that we have included the new manager’s
compensation as opposed to that of the person who will be working on this project
(£10 000 instead of £15 000, at t1). The reason for the latter is that £10 000 is the
change that will take place in the company’s cash flows were the project to be
accepted.
One interpretation of the data not subject to debate is that the expenditure for the
market survey is irrelevant to the issue of the cash flows of this project. The market
survey is a classic ‘sunk cost’. No decision taken by the company at this point can
have an effect upon the cash flow associated with the market survey, so the market
survey cash flow can be ignored.
Further, note that we have ignored the overhead allocation of managerial compensa-
tion in general to the project, because the information did not indicate that any
other compensation would change as a result of the project. Note also that we have
not charged the project’s profits with the interest that PC Problems intends to pay.
The reason is that the WACC–NPV technique specifically adjusts for such interest
and its tax effects in the discount rate used.
The final calculations necessary before we can deduce the relevant cash flows of the
on-site service proposal are those necessary to recognise the investments that the
company is making in what is called ‘working capital’. You recall from the module
that the easiest way to recognise these for tax purposes is to find the increase or
decrease each year in the company’s net working capital accounts. This can be used
as an adjustment to the accounting profit figures to produce a set of cash-flow
estimates. When done, all of the cash-flow effects of changing inventory, cash on
hand, accounts receivable and accounts payable will be included. Using the infor-
mation provided, the levels of the accounts are (£s):
t0 t1 t2 t3 t4 t5
Accounts receivable 0 15 000 30 000 50 000 55 000 0
Accounts payable 0 7 500 15 000 25 000 27 500 0
Cash and inventories 7 500 15 000 25 000 27 500 30 250 0
The indications at t5 that the amounts are zero is consistent with the liquidation
potential mentioned in the information given initially. Though the company may not
be seriously intending to liquidate the project at that point, its reluctance to deal
with estimates beyond year five would require you to estimate present values of
remaining net assets at that point, and this process is as good as any. Failure to
‘liquidate’ in this or similar fashion would bias the analysis, and ignore value that is
expected to exist. Note that we are specifically assuming that the company will not
‘work down’ the inventory during the year, but will sell the intact inventory at the
end of the year; thus instantaneously before the liquidation the cash and inventory is
£30 250 at t5. (Receivables and payables are higher than their t4 amounts at t5, being
£60 500 and £30 250 respectively.)
Thus the changes in the accounts from the prior year, and thus the changes in net
working capital (remembering that changes in accounts payable are subtracted from
the changes in receivables, cash and inventories) are (£s):
t0 t1 t2 t3 t4 t5
Accounts receivable 0 +15 000 +15 000 +20 000 +5 000 −55 000
Accounts payable 0 +7 500 +7 500 +10 000 +2 500 −27 500
Cash and inventories +7 500 +7 500 +10 000 +2 500 +2 750 −30 250
Change in net working capital +7 500 +15 000 +17 500 +12 500 +5 250 −57 750
t0 t1 t2 t3 t4 t5
Sales revenue +75 000 +150 000 +250 000 +275 000 +302 500
Total direct expense −56 250 −83 500 −105 600 −117 460 −130 851
Change in net working capital −7 500 −15 000 −17 500 −12 500 −5 250 +57 750
Assets −58 000 +15 000
Taxes −1 430 −26 260 −66 768 −81 401 −92 377
FCF* −65 500 +2 320 +22 740 +65 132 +70 889 +152 022
4 The appropriate response to this question is that such erosion in related products
should indeed be included as a reduction to the effect upon the company’s revenues
of accepting the proposal, but only if such a reduction would not have been
expected to occur because of competitive conditions in the service market regard-
less of the introduction of the on-site service by PC Problems plc.
Module 6
Review Questions
6.1 The correct answer is D. Payback is not generally a preferred technique because it
ignores the discounting process and the cash flows beyond the payback point. But
under a few limited instances (constant cash flows and equal-risk projects, for
example), a company could design a payback criterion that produced correct
answers.
6.2 The correct answer is D. AROI cannot generally or even specifically be made to
produce correct answers to investment analyses (i.e. those identical to NPVs). The
problem with AROI is that it relies upon the accounting profit numbers developed
for purposes entirely different from cash-flow estimation. Further, AROI being an
average of profit rates upon depreciated investment across time, does not duplicate
the discounting process whereby financial markets set values upon capital claims,
and therefore does not give a good measure of the effects of shareholder wealth that
are caused by company investment activities.
6.3 The NPVs and IRRs of the investments are thus:
NPV IRR
Investment 1 £107 12%
Investment 2 £89 13%
t0 t1 t2
Investment 1 −£1500 +£180 +£1680
Investment 2 −£1000 +£130 +£1130
Incremental investment −£500 +£50 +£550
Next, by trial and error, we discover that the IRR of these incremental cash flows is
10 per cent. We compare this rate to the appropriate discount rate for the projects,
which is 8 per cent. The incremental IRR exceeds the rv* for the projects, so the
‘defender’ or Investment 1 is the choice, since its IRR also exceeds the hurdle rate
of 8 per cent.
The explanation of this technique’s capacity to uncover the correct investment
alternative is that, by using the incremental cash flows, it finds the IRR of the extra
£s invested in the defender given an investment in the challenger. If the IRR of the
extra investment in the defender is larger than the discount rate, the extra invest-
ment must be worthwhile relative to investing only in the challenger. In order to
undertake this extra investment, we reject the challenger altogether, and by accept-
ing the defender we get the cash flows of both the challenger and the incremental or
extra investment, which together are of course simply the cash flows of the defend-
er.
6.5 The correct answer is B. The cost–benefit ratio does in fact have almost the same
drawbacks as the IRR in choosing among investment alternatives, because a ratio
suffers the same limitations as a rate of return when comparisons as to wealth
implications are required.
6.6 The correct answer is B. Everything said in the answer to Question 6.5 about the
cost–benefit ratio can also be said about the profitability index.
6.7 The correct answer is D. The other criteria, as above, do not necessarily order
projects correctly (i.e. put them in the order of their wealth-increasing potential),
and so they cannot generally be depended upon to produce correct orderings of
projects to be considered. Further, even if they did, accepting projects in that order
under a budget constraint could result in a total package or portfolio of projects
being accepted that was not optimal in the sense of being the one with the greatest
total NPV within the budget constraint (see the illustrative example within the main
text of Module 6).
The only entirely correct prescription is to examine all possible combinations of
projects that are within the budget constraint and choose the combination with the
highest NPV. There are several programming software packages available that can
help to make that search more efficient.
6.8 The correct answer is that Happy Pooch can choose either {S and F} or {S, H and
F} and expect to get the same NPV. Both of these combinations have NPVs of
£165 000 and none other has as high. The display of project combinations and their
interactions are (in £000s):
6.9 The correct answer is C. Icky should purchase the Ugh because the present value of
the future costs of the Ugh is less than the Echh, considering future replacements
for both. The Echh has a single-cycle NPV of −£84 824 (its three future operating
outlays discounted at 12 per cent and added to its cost), whilst the Ugh’s single-cycle
NPV is −£98 496. But the equivalent annual cost of the two, found by dividing their
single-cycle NPVs by the annuity present-value factor for the discount rate and
number of periods appropriate to each, is:
£84 824
Echh £35 314
2.402
£98 496
Ugh £32 432
3.037
Thus the Ugh is the preferred aerator. The equivalent annual cost has correctly
captured the effect on the present-value costs of the machines of their replacement
cycles, their relative initial costs, and their relative annual operating costs.
6.10 The correct answer is D. The cardinal rule for treatment of inflation is to discount
inflated cash flows with inflated discount rates, and real (uninflated) cash flows with
real discount rates. Further, the approach of dealing with inflation by using inflated
cash flows and actual market-based discount rates (which are already inflated) is the
preferred method because it does not require that company financial managers
estimate what the market is expecting the effect of future inflation to be. The latter
would be necessary if one were to attempt to deal with ‘real’ discount rates and cash
flows to arrive at correct NPVs.
The implication of answers (B) and (C) that this approach is either wrong or no
better than the real discounting process is wrong. For the detailed discussions of
why, see the text of the module.
Note that answer (A) is also not correct, it being an unqualified approval of the
existing technique. The cautions voiced in answer (D) are appropriate in that they
recognise that the technique as described did not ensure that cash-flow estimates
coming from operating parts of the company were non-inflated to begin with. (If
they included inflation, and if the finance department again increased them for
inflation, they would be double-counting inflation.) The additional suggestion that
operating units may well be better inflation estimators in their own cash flows than
the finance department also seems valid.
Module 7
Review Questions
7.1 The correct answer is D. Standard deviation is indeed an appropriate risk measure
for the entire portfolio of assets held by someone. But when an individual asset is
added to or removed from that portfolio, the effect upon the portfolio’s risk – and
thus upon the risk felt by the individual – is only the asset’s systematic or undiversi-
fiable risk. All of the remainder of the standard deviation of the security’s return
disappears through the phenomenon of diversification in the portfolio.
There is also some truth to the answer in (C). Some legitimate reservations about
the nature of the testing being done upon the ‘market model’ have been voiced by
responsible investigators. But this is not the main reason for the result mentioned in
the question.
7.2 The correct answer is A. You would consider investing only in securities X and Y.
The reason can be seen by using the SML as represented by Equation 7.1:
E
where rf 0.10 and E rm ‐rf 0.08, as given, for each of the β coefficients of the
securities. The resulting returns indicate what investments of comparable systematic
risk would be expected to return in the market. Applying this formula to each
security, we have:
Only in the cases of securities X and Y are the returns that the securities expected to
provide as high as those generally available in the market for investments of
comparable systematic risk (listed as ‘Required return’ and calculated with the SML
formula).
No reasonable investor would have chosen answers (B) or (D), but someone who
did not diversify (for some perhaps irrational reason) could choose answer (C),
which implies holding only a single security. This of course is not a correct answer
when you are already holding a well diversified portfolio, as implied by the question.
7.3 The correct answer is C. As was argued in the Learning Summary of Module 7 (see
Learning Summary, the market will set prices on securities under the assumption
that the shareholders have invested in well diversified portfolios. Thus any corpo-
rate-level diversification of the type proposed by Anglo-Auto plc is destined to be at
best useless and at worst costly in the sense of transactions costs paid by Anglo-
Auto to acquire the mine. If that type of investment was desirable for shareholders,
they could have simply bought shares in a gold mine (and may well have already
done so, in which case Anglo’s actions could cause them to incur costly portfolio
rebalancing transactions to get rid of their gold stocks).
Answer (D) is incorrect because, even though shareholders may be less than well
diversified, the market will price the shares as if they were, and thus there would be
no price increase in reaction to the acquisition.
Finally, remember that the investment was expected to be of no operational use to
Anglo-Auto. This is a necessity for the above arguments to be correct. Were there
to be any real economic interaction between the companies’ cash flows (e.g. if
Anglo-Auto used gold in the cars, and thus avoided the gold market by buying the
mine, or if the gold mine used cars and it was cheaper for the merged firm to get
cars to the gold mine than to acquire them on the market, minus any profits for
Anglo-Auto), then the merger may or may not be profitable for Anglo-Auto,
depending upon the price paid for the mine and the economic gains to be made. An
analysis of that question would involve treating the acquisition as any other pro-
posed investment by Anglo-Auto, and evaluating it with a WACC based upon the
systematic risks of the implied free cash flows of the project.
7.4 The correct answer is D. This question is designed to test your intuitive
understanding of what diversification is, and when it is relevant. To answer the
question, you must posit that auto and steel companies are likely to react very
positively to overall economic conditions (good when the economy is good and vice
versa), pawnbrokers very negatively, and bakeries not much at all. Thus General
Automobile Manufacturers and General Steel Company would likely have correla-
tions with overall economic activity that are positive and strong. General
Pawnbrokers plc, on the other hand is likely to have a negative correlation with
overall economic activity. Thus combinations of these companies would provide
large diversification effects.
The argument in answer (B) concerns real economic interactions between merging
companies, not those effects that would accrue in portfolio combinations; further,
the effects listed are not ‘diversification’ in the sense of stochastic risk reduction.
Answer (C) is wrong because diversification effects, though likely to exist, would be
less between General Bakeries and any of the other companies, because the bakery
has a lower overall volatility with respect to economic conditions.
7.5 The β coefficient is the slope of the line of best fit in Figure 7.10 for each security.
Thus the highest-sloped lines are the highest β coefficients, and vice versa. Specifi-
cally, the ranking by β coefficient is:
Security A
Security C
Security D
Security B.
The total risk of a security can be visualised as the spread of dots with respect to the
vertical axis (the rate of return for the individual security). Thus the securities with
the greatest spread of dots viewed from the vertical axis are those with the highest
total risk. Specifically, the ranking by total risk is:
Security B
Security C
Security A
Security D.
7.6 The transformation of total risk into systematic risk in Figure 7.10 can be visualised
as the dots themselves disappearing (being diversified away), with the only remain-
ing uncertainty being that associated with the securities’ relationship with the
market, as represented by the slope of the line of best fit, or the β coefficient. Thus
as the overall market return varies along the horizontal axis, securities’ returns are
varying along the vertical axis as dictated by the line of best fit. This reduction of
risk from total to systematic is contingent upon these securities being held in well
diversified portfolios, for otherwise the ‘off-line’ dots would still be present and the
market would not fully explain the risk of the securities.
7.7 The correct manner in which to analyse this situation is to separate the two
different-risk components of the project. For example, the successful development
would imply market value of the investment as of three years hence of:
£3 000 000
Value £5 000 000 £25 000 000
0.10
(which is exactly what the unnamed French company would be willing to pay). But
the odds are only 25 per cent that such an outcome will occur, creating an expected
NPV at that point of 0.25 × Value3 or £6 250 000. This, however, is only the
expected NPV three years hence, and must be compared on a correct basis with the
£3 000 000 present outlay for research. To be acceptable, the certainty equivalent of
the risky £6 250 000 must be at least equal to £3 000 000 1.08 3, i.e. about
£3 780 000 at that future time point. Alternatively, a risk-adjusted discount rate of
about 27.7 per cent (sufficient to discount £6 250 000 three years hence to equal
£3 000 000 now) is the maximum that would still generate a non-negative NPV for
the project. The calculation of the 27.7 per cent rate is:
Maximum discount £6 250 000/£3 000 000 1 27.7%
So one correct comparison is to decide whether exchanging a certain £3 780 000 for
a 25 per cent chance of £25 000 000 and 75 per cent chance of nothing (an expected
value of £6 250 000) three years hence is desirable. Another correct comparison is
whether a 27.7 per cent three-period IRR is acceptable for the current outlay of
£3 000 000 in this project, whose risk resolution, but not final cash flow, will be
three years hence. There is not enough information in the problem to decide, but
there is enough to see how the problem should be approached.
Note that we can for no purpose use the 25 per cent discount rate mentioned in the
problem, for that is inclusive of all risks, including those that would continue to
exist after the technical success of the project had been resolved. It would also
obviously imply that the project is undesirable, given that the 25 per cent discount
rate would be applied to all of the project’s cash flows, even those after the major
risk had been resolved.
Cyrano’s equity required return (31 per cent) is substantially higher than NOSE’s of
21 per cent. Note that this is caused by the revenue, fixed cost (operational gearing)
and financial gearing differences between NOSE and Cyrano. The same SML
relationships apply to each, however.
This now allows us finally to calculate Cyrano’s WACC:
∗
D ∗
E
V V
0.30 0.15 1 0.50 0.70 0.31
0.24 24%
Thus Cyrano’s free cash flows (FCF*) should be discounted at a WACC of 24 per
cent per period. To contrast this with NOSE’s WACC, we require one additional
piece of information about NOSE, its rd. We know NOSE’s βd is 0.20, so it is a
simple matter to find rd, again via our Equation 7.1:
E
E 0.10 0.091 0.2
0.12 12%
Now we are ready to find NOSE’s WACC:
∗
D ∗
E
V V
0.35 0.12 1 0.50 0.65 0.21
0.16 16%
Cyrano needs a 24 per cent WACC while the company undertaking the project
requires only a 16 per cent WACC. Were NOSE to have evaluated Cyrano with the
company’s WACC, the project would appear to be more attractive than it is.
Module 8
Review Questions
8.1 The correct answer is B. The shares will not change in price, because the growth
rate in the future dividend to the existing shareholders will decline by the exact
amount necessary to offset the cash dividend increase. To see this, remember that
ABT must now plan to raise £10 more per share in new equity each period, also
growing at the 5 per cent per annum rate. These new shares will claim a steadily
increasing portion of the future dividends of the company. The effect of this
steadily increasing claim of new shareholders is to depress the growth rate of
dividends for existing shareholders. Specifically, the £10 per share dividend increase
in the first year is 10 per cent of the value of the equity of the company, and thus
that percentage must now be raised from new shareholders. Therefore, 10 per cent
of the next year’s dividend will go to new shareholders. This is effectively a reduc-
tion of 10 per cent in the growth rate of the future dividend. This process continues
ad infinitum, so that:
Dividend
Price per share
10%
£20
0.15 0.05 0.10
£20
0.20
£100
Current shareholders have effectively traded lower growth rates in dividends for
higher near-term cash dividends. They will find that they are progressively giving up
their proportional claim on the company to whomsoever buys the new shares.
Answer (C) is thus incorrect, because the company is not liquidating itself (it is still
making the same investments that it always has, but now is raising the money from
new shareholders instead of paying a lower dividend and retaining the cash); existing
shareholders are liquidating their claim.
Answer (A) is incorrect because, although dividends are in fact doubled, the net rate
of future dividend increase to the currently existing shares exactly offsets that in a
value sense. Answer (D) is wrong because there are no taxes.
8.2 The correct answer is B. The best answer to this question is (B), even though there
are elements of truth in all but (D).
Answer (B) is the most ‘modern’ depiction of the dividend irrelevancy argument
because it would hold even in realistic markets, whereas the statements in answers
(A) and (C) would be true only in the restrictive cases of frictionless markets, not
really the most ‘modern’ explanation of irrelevancy. Note the distinction between
‘dividends’ as cash flows (which are not at all irrelevant, and form the basis for
share value), and dividend policy (i.e. how the residual cash is split between
dividends and retention, which can be irrelevant).
Answer (D) is wrong because it ignores the arguments in answer (B). One should, of
course, also mention that the issues raised by dividend irrelevancy arguments in this
context also ignore questions of signalling, agency and so forth, which could argue
against irrelevancy.
8.3 The correct answer is D. The best answer is (D).
One is usually at first attracted to the logic of answer (A), because it is correct that
the company would probably be able to reduce its capital-raising costs by lowering
its dividend payout and retaining more cash. But this argument ignores the fact that
the policy the company has been pursuing has probably attracted a set of sharehold-
ers who prefer that policy. Shifting to a lower payout would not be likely to raise the
share price, because that would require another group to pay a premium for the new
pattern of dividends, and there are plenty of companies offering each possible
pattern. The gain would be the transaction costs saved, and the loss would be the
costs to shareholders of rearranging their portfolios to find a different company
with a dividend policy they prefer. It is not at all clear that the net result would be
positive.
The ‘signalling’ rationale is also reasonable.
8.4 The correct answer is A. By simply paying out the excess cash as a regular dividend,
the company is in danger of giving the market a misimpression about future ‘good
news’ – hence the ‘extra dividend’ (a very common practice in real markets).
Retaining the cash to invest in government securities is only worthwhile if there are
foreseeable future profitable real investment needs that will require the money, or if
there are some unusual tax characteristics of the decision, neither of which are
specified in the question. Finally, the retirement of debt is not on the face of it a
desirable choice, there being no special reason to think that the company is present-
ly using the wrong amount of debt capital. (There is a natural bias to think that it is
always good to retire debt when one can. This is not at all true for companies. We
explore this issue in Module 9 on capital structure.)
Module 9
Review Questions
9.1 The correct answer is B. The module has attempted to show that the choice
between equity or borrowing can affect the wealth of shareholders (and bondhold-
ers) of companies. To the extent that managerial decision makers are concerned
about the interests of these groups, the decision is important to the company.
Answer (A) is incorrect because a company is never forced to borrow; it can instead
issue shares, though perhaps at higher cost. Answer (C) is incorrect as a general
principle, which is demonstrated in the several arguments in the module about the
pros and cons of borrowing. Answer (D) is correct to some extent but, without the
recognition that there are costs to low-risk capital structures (such as forgone tax
benefits), the argument is incomplete.
9.2 The correct answer is D. None of the other answers is truly correct. Answer (A) is
obviously wrong, because a company achieves minimum risk exposure simply by
having no borrowing. There are often benefits to borrowing that offset to some
extent the increased risk. Answer (B) is incorrect because the highest equity value is
probably in an ungeared firm. In such a situation, equity value is high because
shareholders have so much invested, not because there has been a good decision
about capital structure. Though difficult to argue with as a homily, answer (C) is not
destined to be useful as a decision criterion for financial managers.
If there is a positive but general answer to the question, it is as follows. ‘The issue of
risk from the perspective asked is irrelevant. Of course, increased borrowing causes
increased risk. But if shareholders are concerned about such risk, it is easily neutral-
ised (for example, by altering their personal portfolios to include fewer shares and
more bonds of the company). Capital structure alterations of the type implied by the
question serve simply to reallocate the operating risk of the company among capital
claims. Shareholders can undo this at will, and so such concern is unwarranted
(unless there would be high transactions costs in doing so).’ The foregoing state-
ment is simply another way of saying the same thing that M&M did in their first
capital-structure ‘irrelevancy’ argument.
9.3 The correct answer is A. By purchasing all of the securities of either company, you
will experience the full business risk of the tool industry because you will thereby
claim all of the operating cash flows of the firm (and the companies have identical
operating cash flows). The reason that Leverage is better than Leverageless is simply
that the cost of Leverage’s shares and bonds, £96 000 000, is less than Leverageless’s
£100 000 000 shares. Since either purchase would claim the same future cash flows,
one naturally chooses the less expensive. With this information, all answers other
than (A) are clearly wrong.
9.4 The correct answer is D. As Question 9.3 indicates, anyone interested in the
business risk of the tool industry would do better by buying the bonds and shares of
Leverage rather than just the shares of Leverageless. The same general aversion to
Leverageless will also apply to anyone thinking of purchasing the shares of that
company. It simply is offering nothing that cannot be more cheaply purchased by
buying Leverage’s capital claims. That being the situation, the price of Leverageless’s
shares must fall to be worth £96 000 000, the value of Leverage’s claims.
Answer (A) is incorrect not because there are no such people with proclivities to
obtain the risk and return of an ungeared firm, but because Leverageless is simply
too expensive. Leverageless’s risk can be obtained for £4 000 000 less by buying all
of the bonds and shares of Leverage.
Answer (B) is incorrect because rarity is irrelevant in the sense used. The risk and
return that a correctly priced Leverageless would offer is widely available, not only
from Leverage as described above, but also from any number of other investments
(other companies in other industries, real estate, artwork, and so forth) if correctly
combined with personal borrowing or lending. The only thing rare about Leverage-
less, as far as the market is concerned, is that its price is wrong. Arbitrageurs will
quickly ensure that this mispricing disappears.
Answer (C) is wrong because there is an incentive to cause the price of Leverageless
to decline. For example, those who hold the shares of Leverageless could sell those
shares for £100 000 000 and purchase the bonds and shares of Leverage for
£96 000 000, obtaining the same future cash-flow expectations as with Leverageless
but with £4 000 000 in their pockets.
The only interesting point implied by answer (C) is how the price of Leverageless
got to be wrong in the first place. The answer to that question is, of course, that we
designed it that way so as to make the question interesting. In other words, we
arbitrarily caused the financial market to make a mistake, and expected you to spot
it.
9.5 The correct answer is D. Option II is correct because we cannot be certain how to
evaluate the 10 per cent IRR being predicted for the investment. If the company’s
assets in general are to be pledged as collateral for the loan (a straight debenture),
the 8 per cent rate asked may be reflecting the risks of the company in general rather
than the investment in particular. Comparing the investment’s return with the
interest on the debt used to ‘finance’ it is thus wrong, since the company itself – not
the investment – is being ‘financed’.
Option III is also correct, because the 8 per cent cost of borrowing ignores the
effect of the borrowing upon the shares of the company. Even though the cash
outlay for the investment is fully funded by the borrowing, that borrowing compris-
es the ‘financing’ of the investment only if the debt collateral is limited to the
investment, and if the investment has no NPV. (With a positive NPV, part of the
‘financing’ is the equity value created by the investment’s NPV, in the sense that the
future cash flows of the project must cover the expectations of the creditors and
the shareholders assigning the positive NPV and attendant share price increase.)
With a straight debenture, the company’s assets in general have collateralised the
loan, and thus are at risk. The WACC can capture such considerations; the debt cost
cannot. Option I is obviously wrong because it implies that the investment is
financed solely by the borrowing.
9.6 The correct answer is C. Option II is correct because one of the important reasons
that companies borrow is to reduce their taxes. If a company has weighed the pros
and cons of borrowing and decided that there are net benefits to using debt rather
than equity, but has ignored the chances that tax benefits through debt are merely
replacing other deductions from income, the net benefit of borrowing may be
incorrect.
Option III is correct because even when the company is not in violation of borrow-
ing covenants, it may be constrained to forego profitable activities either
involuntarily (e.g. because an indenture covenant precludes mergers, or constrains
dividend payments) or voluntarily (e.g. because the positive NPV of a project would
accrue to bondholders rather than shareholders). Furthermore, the company may be
forced to undertake actions it would rather not – for example, maintaining more
current liquid assets, or even liquidating operating assets to pay interest and princi-
pal.
Option V is correct because lenders will anticipate that, given the chance, the
company will take any opportunity to shift wealth from bondholders to sharehold-
ers (by undertaking risky investments, by funnelling resources to shareholders and
managers instead of bondholders, and so forth). These contingencies will be priced
in advance by lenders, and cause borrowing costs to be higher than they would
otherwise be. To some extent, such ‘agency’ costs may be avoided by activities such
as writing constraining contracts that do not materially affect the optimal asset
operations of the company and have low monitoring costs, or issuing complex
securities such as convertible or callable bonds that reduce the costs or chances of
such conflicts of interest.
Option I is wrong because bankruptcy in and of itself is merely the shifting of asset
ownership from shareholders to bondholders. This is in fact a benefit, not a
detriment, because in the absence of the capacity to declare bankruptcy, companies’
shareholders, in their role as residual owners, would be forced to make good on the
company’s interest and principal promise. This also explains why option IV is
incorrect. The only problem with bankruptcy is that there are likely to be both direct
costs incurred in litigation, and indirect operational costs and inefficiencies associat-
ed with this process of shifting asset ownership.
9.7 The correct answer is D. Conglomerates engage in so many unrelated activities that
it is difficult for a lender to know with any great certainty the mix of riskinesses that
will appear in the company over time. If a company succeeds in convincing the
lender that a particular amount of risk will be experienced, and then causes the
actual risk to be higher, the lender’s bond value will be lower and, given an un-
changed total company value, shares will be more valuable. There will have been a
wealth shift from bondholders to shareholders. Bondholders judge the likelihoods
of this when they originally purchase bonds (thereby setting the required returns).
Thus, for a conglomerate the costs of borrowing would be higher due to the lenders
charging in advance for the wealth shift likelihood. Convertible securities do away
with this charge, because the conversion characteristic causes any increases in share
value to be reflected in the bond values. Wealth shifts are no longer a problem,
lenders can charge lower interest rates, and the company has avoided the costs of
potential conflicts of interest.
Answer (A) is incorrect because, though interest costs are indeed lower for convert-
ible bonds, it is not the lower rate per se that causes the bonds to be desirable
financing vehicles. Even if there were no potential conflicts of interest, if the
conversion terms of the bond are set so as to be valuable, the bonds will carry a
lower interest rate than non-convertible bonds. (For example, a company’s shares
are selling for £50 apiece, and a £1000 face-value bond is convertible into 25 shares;
such a bond would obviously be worth more than a straight debenture of the same
face value and coupon interest rate, even if the bond were issued by a firm with no
chances to shift wealth to shareholders from bondholders.) The lower interest rate
on this convertible, however, is ‘paid for’ in full by the shareholders when the bonds
are exercised at advantageous conversion rates, and the new shareholders claim their
pro rata portion of company cash flow and value.
Answer (B) is incorrect because convertible bonds are not at all necessarily ‘delayed
equity issuances’. As a matter of fact, there is no particular reason why bondholders
– unless forced to do so – would ever choose to convert their bonds into shares.
With convertible bonds, they maintain their higher-priority claim upon interest and
principal payments, whilst at the same time enjoying the benefits of equity share-
value increases.
Answer (C) is wrong because there is no such general relationship between maturi-
ties of convertible and non-convertible bonds.
9.8 The correct answer is A. From the perspective of both lenders and borrowers, it is
important that the operational cash flows arising from a firm not be heavily dis-
turbed by the existence of borrowing when the company comes upon hard times.
With ships and other tangible assets as the primary basis of value for the company,
Intercontinental Shipping and its creditors can feel secure that there will be few
additional operational problems with the firm’s assets should the shipping compa-
ny or its industry come upon hard times. The ownership and collateralisation of the
assets is clear, and there is a ready market in ships.
Most of Clever Advertising’s value, on the other hand, likely derives from expecta-
tions of future cash flows generated by the skill of its copy writers, and
‘brainstorming’ of account executives. This is precious little collateral for a lender to
look toward in bad times. It is likely that such resources would simply seek other
employment rather than be operated or ‘sold’ by creditors should Clever turn out to
be dull. Any adverse economic conditions would be thus exacerbated by the
existence of borrowing.
Answer (B) is wrong because risk as a unique consideration (the variability of future
cash flows) is not a final determinant of company capital-structure decisions.
Shipping companies are, in fact, at least as risky in this sense as are advertising
companies. The real test is: given the risk, is borrowing a further detriment? For
Intercontinental Shipping, it is not; for Clever Advertising, it is.
Answer (C) is wrong because capital structure is probably not irrelevant. The world
of the original M&M theory is not the world in which real companies operate. And
Miller’s arguments about real market irrelevancy, though interesting, were probably
incomplete.
Answer (D) is wrong because there is nothing in the tax situations of the companies
that would indicate that borrowing is better for Intercontinental Shipping than for
Clever Advertising. As a matter of fact, with the capacity that tangible assets have
for shielding income from taxes, Intercontinental’s tax situation in terms of borrow-
ing may be worse than Clever’s.
which of the two capital types is cheaper? If that can be answered without reference
to the particular costs of each, there is no reason to pursue the latter issue.
The first test of debt’s desirability is to see whether its tax benefits are likely to be
realised. This in turn depends upon whether the company will have income subject
to taxation. With acceptance of the facilities project, R-D Star is expected to have
EBIT of £182 500 000 in 2009. Interest on the new borrowing is 14 per cent of
£75 000 000, or £10 500 000. Thus the best guess is that the company will report
taxable income of £139 800 000 in 2009 if it finances itself with debt. (This is the
£182 500 000 minus the £10 500 000 of interest on the new debt and minus
£32 200 000 interest on the £230 000 000 of old debt, assuming the same interest
rate on old and new borrowing). Though there is significant variability in the
company’s past income stream, the odds that operating results would deteriorate
sufficiently during the life of the loan to cause the tax benefits of interest deductabil-
ity to disappear are low. This information, and the risk attributes of gearing as
discussed in the module, are depicted in the EBIT-EPS chart of Figure A4.4. (The
EPSs are after tax, assuming deductible interest, and based on either the 20 000 000
shares outstanding if debt is issued, or 23 571 429 shares if equity is issued.)
EPS
£3.35
£3.05
0 32 43 £182.5 EBIT
Old interest Old + New interest (£m)
Projected EBIT
tions for the company and its shareholders. Having now decided that borrowing will
likely carry tax benefits, we must see if there are any significant negative considera-
tions in R-D Star’s borrowing alternative.
What can go wrong when a company borrows? The most obvious danger in
borrowing is that interest and principal payments may not be met, and the company
may go into default from that breach of the debt contract. A glance at Figure A4.4
should be enough to convince you that there is little likelihood that R-D Star will
not be able to meet its contractual payments to creditors. (Note also, however, that
interest and principal on total borrowing – not merely on new debt – must be
covered, and that the interest alone may be 14 per cent of £305 000 000, or
£42 700 000.)
The more significant considerations are probably in the other covenants of the debt
contract. Recall that there are constraints upon dividend payments, net working
capital, and other borrowing that will apply to R-D Star if it enters into the debt
contract.
Consider the first constraint. The company’s dividend has been stable in absolute
terms for the past few years, even in the face of declining and variable income. In
the history presented, there has not been a reduction of dividend to R-D Star’s
shareholders. This is an indication that a forced reduction in dividends might be
costly to the shareholder clientele of the company. With projected EBIT of
£182 500 000, and interest at £42 700 000, after-tax profits are 48 per cent of the
difference, or £67 104 000. Dividends are constrained to be no more than 16 per
cent of this, or approximately £10 700 000. This is slightly above last year’s figure,
but not by much. It would take only a small shortfall in operating income to force
R-D Star to cut its dividend to shareholders if borrowing is used.
Net working capital must be kept to at least £80 000 000. In 2008 the difference
between Current assets and Current liabilities is only £72 700 000, but the facilities
project will use £20 000 000 of its funding for an increase in net working capital, and
so this constraint is not likely to be onerous.
The limited information available in the case makes it more difficult to judge the
importance of the bond contract’s constraint upon future borrowing. Such con-
straints, when binding, most often have the effect of forcing the company to use
some other type of (more costly) financing for future projects. Whether this is a
problem for R-D Star is not clear.
What has this review of the contractual constraints upon the company indicated
about the desirability of borrowing? At this point it seems that the major problem is
a capacity to maintain the company’s dividend payment. Dividends, however, are
only one of a number of things intended to be paid from R-D Star’s EBIT. Re-
member that the company plans to use £63 000 000 of retained earnings as part of
the internally generated funding for the facilities project. The other uses for EBIT
comprise dividends, interest and principal. We can arrive at the expected claims
upon projected EBIT by listing the plans of R-D Star, and the amounts of EBIT
they would require. Remembering that taxes must be paid upon everything save
interest, we have:
Interest:
New borrowing £10 500 000
Old borrowing £32 200 000
Principal:
New borrowing £15 600 000
Old borrowing £47 920 000
Facilities project:
EBIT required £131 250 000
Dividends: £21 450 000
Total EBIT required: £258 920 000
(These calculations assume that a 52 per cent tax must be paid upon non-deductible
EBIT amounts in order to arrive at after-tax cash. For example, the £63 000 000 of
facilities investment requires £63 000 000/0.48 £131 250 000 of EBIT. Principal
payments and interest rates on old borrowing are assumed to be on the same terms
as the new borrowing.)
The obvious conclusion is that R-D Star cannot hope to do all that it expects to do
with a projected EBIT of only £182 500 000. The shortfall is nearly £80 000 000. If
the company wishes to pursue the facilities project with borrowing, it will at the
same time be forced to alter its expected usage of EBIT. This may require refunding
the old debt issue (thereby effectively foregoing its principal payments this year),
cutting its dividend, or some combination of the two.
Another valuable exercise in evaluating the financing choice is to see whether R-D
Star will deviate significantly from the industry norms if it accepts the debt alterna-
tive. Taking all of these intentions of the company, we can construct a set of results
for 2009:
(These figures assume that 10 per cent of old borrowing is paid off, and that the net
increase in working capital is due to the addition of current assets, as opposed to the
addition of both assets and liabilities.)
The projected results can be translated into a set of operating and financial ratios,
which can be compared to the industry averages:
Industry R-D
Star
Current assets/Current liabilities 2.00 2.42
EBIT/Total assets 0.26 0.26
Long-term debt/Total assets 0.35 0.40
Dividends/Profits after taxes 0.08 0.15
The facilities project financed with borrowing would cause R-D Star to be some-
what more liquid, about as profitable, more highly geared and to continue paying a
higher rate of dividends than other comparable companies.
Actually, as we have seen, R-D Star cannot finance all it wishes to do in 2003
without changing some of the decisions implied by the above ratios. Re-examining
the ratios in this light can be instructive as to the alternatives available to R-D Star.
For example, with liquidity high, perhaps some current assets can be reduced, thus
reducing financing needs and therefore borrowing necessities. Or R-D Star, with
debt as projected being above-average in gearing, perhaps should abandon plans to
borrow. On the other hand, dividends are higher than what comparable companies
pay to shareholders, and thus these could perhaps be cut without major problems.
There are additional considerations that must also not be neglected. For example, it
is not clear how important the restriction upon future borrowing is for R-D Star. If
the company has additional expansion plans, it may be better to issue equity now so
as to avoid the restrictions upon future borrowing; or perhaps it would be best to
bargain with lenders to loosen somewhat the restriction on future borrowing in
exchange for a higher interest rate. The proposal to negotiate new terms of the loan
based upon the tangible value of R-D Star’s inventory of old films may also be
important.
What, then, is the ‘solution’ to this case? In truth, as with most interesting real-
market problems, there is both no solution and an infinite number of solutions. Any
plan that can be financed with expected EBIT and borrowing or equity is a ‘solu-
tion’ in the limited sense of allowing the company to carry out its expansion and
probably survive. But a plan that aspires to optimality must balance the positive and
negative effects of its financing choices. All things considered, in the current case
many financial managers would choose to issue equity instead of borrowing. The
reasons cited would be the high gearing ratio that would be produced by the debt
issue, and the likely necessity to reduce dividends.
On the other hand, reasonable managers might also argue that borrowing is best,
and the liquidity and dividend cutting is a good tactic to gain the tax benefits of
debt. They would further argue that using more of EBIT than is available in 2009 is
not significant, because that phenomenon is for one year only, and disappears in the
year 2010 with no need for a large expansion claim on available cash. The 2009
shortfall can be easily covered by one year’s rolling-over in principal payment on old
borrowing, or by a short-term loan.
Finance managers might further argue that the benefits of the tangibility of R-D
Star’s film inventory is already included in the interest rate on the proposed debt
issue, and though a lower rate could be achieved by a specific mortgage, there would
be no net gain because other lenders would thereby be forced to charge the compa-
ny higher rates because these lenders would now have less collateral for their loans.
As with most cases, there is not enough information provided in the case study to
come down with great authority on one side or the other without making additional
assumptions about the company or its environment. A large, planned, future
expansion might argue for equity now and borrowing later. Concern about control-
ling interest in shareholdings might cause there to be reticence to issue equity.
Module 10
Review Questions
10.1 The correct answer is D. This is exactly the description that occurs in the text (see
Section 10.2.3). Answer (C) is also a decent answer, but not as detailed as (D).
Answer (A) is wrong because it mixes up the matching order of this technique, and
answer (B) is wrong because it endows maturity matching with more power than it
can justifiably claim.
10.2 The correct answer is B. When a company matches asset and financing terms on the
basis of strategy I accounting definitions of term, it will finance ‘current assets’ with
short-term finance and ‘fixed assets’ with long-term finance. Since current assets
overstate the true short-term assets of a company due to the permanent nature of
some short-term assets, this strategy will produce less long-term (low-risk) finance
and more short-term (high-risk) finance than will strategy II, which uses the real
categorisations, or III, which maximises long-term finance. Answers (A), (C) and
(D) are inconsistent with (B), and are wrong.
10.3 The correct answer is A. Interest of £8400 per annum will be paid, based upon a
£60 000 loan at 14 per cent interest. To find the correct effective interest rate,
interest must be divided by the amount borrowed (on a one-year loan with repay-
ment of principal at the end of the year) – the calculation is just that simple. The
amount borrowed is net of the £8400 interest paid in advance and the 10 per cent
(£6000) compensating balance, making a net borrowing of £60 000‐£6000‐£8400
£45 600. Dividing that into the £8400 interest gives a rate of 18.4 per cent.
10.4 The correct answer is A. Substituting correctly in Equation 10.1 from the module
(reproduced below) produces the combination shown in answer (A). If your answer
was (B), you divided by 13 instead of 0.13 in the formula. If you answered (C) it was
likely nothing but a wild guess. Since there is a correct answer, (D) is wrong.
½
£ 2 £ £ / ﴾A4.1﴿
10.5 The correct answer is C. This is found by substituting correctly into Equation 10.2
from the module (reproduced below). Incorrect answers may have been generated
by failing to square the £4000 change or failing to approximate the daily interest rate
as 0.12/365. The other answers are wrong, and could have been arrived at by errors
such as the above.
£ 3 £ /4 £ ﴾A4.2﴿
10.6 The correct answer is A. You can review your understanding of this answer by
referring to Section 10.3.3 of the module. If Daisy Publications undertakes the
alteration in its credit policy as proposed, it will be better off by £0.78 (per day) in
present-value terms. The calculation that produces this result is taken from the Hill
et al. work mentioned in footnote 8 of the text of the module, and it proceeds as set
out hereafter.
The reduction in sales will cost the firm £166.72 as the discounted value of receipts,
net of bad debts. This is found by discounting the new daily sales of £1 575.34 × (1
− 0.03) at 1.0003 40 and subtracting from that the original £1 780.82 × (1 − 0.04)
discounted at 1.0003 65. The remainder of the calculation is analogous in this
respect with that of the text:
NPV Change in PV of sales receipts
Change in costs
Change in working capital investment
£1 575.34 1 0.03 £1 780.82 1 0.04
1 0.0003 1 0.0003
0.8 £1 575.34 £1 780.82
£1 575.34 £1 780.82
0.20 £1 575.34 £1 780.82
1 0.0003 1 0.0003
£0.78
10.7 The correct answer is A. The loan is much cheaper than the giving-up of discounts.
As Section 10.3.4 of the module indicates, the annual interest rate equivalent of
foregoing the discount is:
0.02
1 1 44.58%
1 0.02
The cost of the loan can be found by:
0.16
19.05%
1 0.16
The reason that this gives us the cost of the loan is that the interest cost is 0.16
times the ‘principal amount’ of a discount loan, but the actual amount that the bank
gives you at the beginning of the loan period is only the ‘discounted’ amount, or 1‐
0.16 0.84 times the ‘principal amount’. Because you do not get the entire
‘principal amount’, you are paying interest at a rate higher than the stated rate, and
the ratio 0.16/ 1‐0.16 0.1905 displays the effective interest rate on the discount
loan.
Thus the loan is clearly less expensive. Answer (C) is not entirely unreasonable but,
since the loan is cheaper, it is debatable. Actually, simply giving up discounts is not
nearly as bad as being a late (or non-) payer. Answer (D) is wrong because it forgets
that payables are simply another form of borrowing, and this is well known to
capital markets.
10.8 There is no answer provided for this exercise.
prices in line with its competitors. A new product, by itself, is not necessarily the
answer to its problems.
10.21 The correct answer is B. A relatively slow inventory turnover indicates too much
inventory, and therefore B is the correct option. The other two options are not
relevant to inventory.
Module 11
Review Questions
11.1 The correct answer is A. If Dental Specialities borrows sterling, it must repay
(Æ60 000)/5 × 1.1024 pounds sterling, or £13 228.80 at the end of the year. If the
company instead borrows eyes, it must repay (Æ60 000) × 1.1336 eyes, or
Æ68 016.00. But the latter is denominated in eyes, not sterling, the shareholder-
wealth currency. The question is thus how much sterling must in effect be paid at
the end of the year if the loan is now taken out in eyes. The answer depends upon
the exchange rate of sterling for eyes at the end of the year. If, as the question
specifies, the forward exchange market is working efficiently (and otherwise has the
characteristics assumed in the text of the module), the forward exchange rate will be
given by:
Forward exchange rate 1.1336
5 1.1024
Forward exchange rate 5.1415
Thus the sterling necessary to pay off the loan is Æ68 016/5.1415 £13 228.80,
exactly the same amount as if the loan were made in sterling originally.
One implicit assumption made in the above explanation is that the actual future
exchange rate that will exist between sterling and eyes is well predicted by the
existing forward exchange rate. Though this is empirically valid as an expectation,
there is uncertainty about the actual rate. This uncertainty can be hedged by
engaging in a forward exchange transaction if funds are borrowed in sterling but will
be repaid from eye revenues (by selling eyes ahead), or the reverse if funds are
borrowed in eyes but will be repaid by sterling revenues (by selling sterling ahead).
Answer (D) is a correct answer only if in fact the loan liability will be liquidated with
eye revenues.
Answer (B) is wrong because interest rates are not really ‘lower’ in the UK, given
that present and future exchanges of currencies must be made, and the exchange
rates imply interest rate differentials, as the above calculations show.
Answer (C) is wrong because, though the implied inflation rate in Incaland is higher
than the UK (assuming real interest rates are the same), the higher Incaland interest
rates will compensate exactly for the ‘cheaper’ eyes used to repay the loan.
11.2 The correct answer is D. As the answer to Question 11.1 indicated, the correct
response is (D).
This is the only result consistent with ‘interest rate parity’ between the UK and
Incaland.
(Forward exchange markets generally do not extend as far into the future as real-
asset investments of companies). Incaland inflation predictions are accessible from
the comparison of interest rates in the UK and Incaland, as per the text.
11.9 The correct answer is B. As the text maintains, only monetary assets feel the full
brunt of unexpected exchange rate changes, because only their payoffs are denomi-
nated in fixed amounts of the foreign currency. Real assets, on the other hand, have
values that are likely to fluctuate positively with inflation, and thus adverse move-
ments in exchange rates would tend to be offset by this phenomenon. Thus answer
(A) encourages higher transactions costs and may in fact unbalance the ‘natural’
hedging of real assets. Answer (C) is simply perverse, and answer (D) ignores the
fact that monetary assets, though admittedly not great for this operation, may still
require hedging.
11.10 The correct answer is A. The correct answer is (A) (though (C) may be effective
depending upon the extent to which the joint venturer is family-oriented).
The key to this answer is that sanctions brought to bear upon the joint venturer
must be invoked by powers outside Incaland but must also be enforceable on the
joint venturer inside of that country. Answer (B) obviously does not fit this descrip-
tion, but answer (D) also falls short since UK courts probably have no jurisdiction
inside Incaland. Answer (A) has both attributes necessary to enforce the contract.
Whether or not the joint venturer will accept such terms depends upon the profits
foreseeable, and other sources of financing and expertise to undertake the venture.
Module 12
Review Questions
12.1 The correct answer is A. As we discussed in the module, the very nature of option-
contingent payoffs dictates that underlying security variances are positively related to
option values. The reason is that an option holder is allowed to take advantage of
upswings in underlying asset prices but does not as fully experience downswings.
Answer (C) is technically correct, but is trivial. Answers (B) and (D) go in the wrong
direction, and so must be wrong also.
12.2 The correct answer is B. As interest rates increase, in effect the less money must be
put aside now so as to take advantage of (exercise) the option at expiration. The
reduction in the present value of exercise price is an increase in option value. (This
is true even though few options are ever truly ‘exercised’.) Answer (A) is wrong
because the future cash flows in the question are negative, not positive. And a
reduction in the value of negative cash flows increases (positive) option value. The
above illustrates why answers (C) and (D) are also wrong.
12.3 The correct answer is B. One proper manner of considering the value of geared
equity is to regard it as a call option on the assets of the firm, which can be exer-
cised by paying interest and principal to bondholders who bought the assets by
purchasing bonds, but at the same time have issued a call on those assets to share-
holders.
Answer (A) is true, but not a correctly general answer to the question. Answers (C)
and (D) are incorrect descriptions of the contractual rights of bondholders.
12.4
max 0, max 0,1.2 £25 £22 £8
max 0, max 0,0.7 £25 £22 £0
Using Equation 12.1:
£8 £0
0.64
£25 1.2 0.7
and (based on Equation 12.2 and Equation 12.3):
1.2 £0 0.7 £8
1 1.2 0.7 1 0.10
£10.1818 …
0.64 £25 £10.1818 … £5.82
The value of the call option is £5.82. Note that its exercise value is £25‐£22 £3, so
it is selling at a ‘premium’ of £2.82.
12.5 The correct answer is C. All three descriptions of the role of constraining indenture
provisions are probably correct. The module text discusses the first two in some
detail, showing them to be consistent with the agency relationship between share-
holders and bondholders, and one mechanism (though not necessarily the most
efficient one) of insuring bondholders against actions by the firm to shift wealth
from them to shareholders. Option III is a new concept, but consistent with the
type of agency problem that the module discussed as existing between managers and
all outside capital suppliers. With interest and principal payments (and other specific
performance-type bond indenture provisions) existing on debt claims, managers’
degrees of freedom to consume corporate resources is inhibited. This is one more
reason why companies may borrow.
12.6 The correct answer is A. The last example in the text of the module illustrated that
the best interpretation of the reason for a convertible bond to exist is to guarantee
bondholders that they will participate with shareholders in any increase in share
value, including those that would otherwise be shifts of wealth from bondholders to
shareholders.
All of the other ‘reasons’ listed are correct descriptions of what happens when a
company issues convertible bonds instead of non-convertible bonds, but they are
not reasons for convertible bonds to exist. Reasonably efficient securities markets
can set appropriate prices on all of those phenomena at the time of the bonds’
issuance, and there is no reason to think that shareholders would either benefit or
lose relative to bondholders due to them. In order to have ‘survival value’ in such a
market, a security must be able to do something better (or at least as well) as any
other competing security. The ‘benefit’ of convertibles is their insurance characteris-
tic for bondholders, which (as the module illustrates) allows a firm to make other
decisions that are value-maximising.